Kritik an der speziellen Relativitätstheorie (SRT) gibt es hier in den Kommentaren ja relativ häufig. Meist wird dieser Kritik1 eher spöttisch begegnet, Physiker halten die SRT also anscheinend für über jeden Zweifel erhaben. Warum ist das so? Sind wir alle einsteingläubige Dogmatiker?

1Ich beziehe mich hier zunächst auf Kritik, die die gesamte SRT in Frage stellt und stattdessen die Gültigkeit der Newtonschen Physik behauptet. Andere Kritiken an der SRT diskutiere ich am Ende.

Schaut man einmal bei Wikipedia, welche Belege dort für die SRT angeführt werden, so gibt es da Messungen der Zeitdilatation in Teilchenbeschleunigern oder an Myonen der Höhenstrahlung, ähnliche Messungen mit Uhren, die in Flugzeugen bewegt werden, dann eine Erklärung der Lichtablenkung (Aberration) und des Dopplereffektes, sowie schließlich die Erklärung der Lorentzkraft, also der magnetischen Kraft auf ein bewegtes Teilchen.

Für sich genommen sind diese Effekte und ihre Erklärung durch die SRT sicherlich schon gute Argumente und jeder Kritiker der SRT täte gut daran, sie alle ernst zu nehmen, aber sind das nicht eher sehr spezielle Effekte? Könnte man die nicht auch wirklich anders erklären.

Diese Effekte, die als direkte Demonstrationen der SRT angesehen werden können, allein erklären wirklich nicht ausreichend, warum eine Rückkehr zur Newtonschen Physik von Physikern für absolut absurd gehalten wird. Um das zu verstehen, muss man sich ansehen, welche Rolle die SRT in anderen Bereichen der Physik spielt. Ich gebe hier nur ein paar Beispiele – wer “relativistic effects” bei google scholar eintippt, kann sich jede Menge weitere Anregungen holen.

Anmerkung: Ja, mir ist klar, dass ich die argumentresistenten Kritiker, die hier in den Kommentaren oft auftauchen, mit diesem Artikel nicht überzeugen werde –
“You cannot reason people out of a position that they did not reason themselves into.” Aber vielleicht hilft dieser Artikel ja denen, die sich über die heftigen Reaktionen der Physiker wundern, diese besser zu verstehen.

Atomphysik, Festkörperphysik, Chemie
Die Quantenmechanik feierte Anfang des letzten Jahrhunderts große Erfolge. Eins der Hauptziele der QM war es, die Atomspektren erklären zu können. Das Bohrsche Atommodell machte den Anfang, Sommerfeld verfeinerte das Modell weiter. Schon dabei verwendete er die SRT, indem er die relativistische Formel für die kinetische Energie anwandte und so die Feinstruktur des Wasserstoffspektrums berechnete. Eine detailliertere Analyse gelang mit der Dirac-Gleichung, zu der ich später noch etwas sage. Ohne relativistische Korrekturen sind aber die beobachteten Atomspektren nicht zu erklären – wer die SRT für ungültig erklärt, muss also eine andere (quantitative!) Erklärung der beobachteten Atomspektren liefern.

Wie jeder weiß, hat Gold eine gelbliche Farbe. Warum eigentlich? Die Farbe eines Metalls wird durch die Energieniveaus der Elektronen bestimmt, die sogenannte Bandstruktur. Diese Bandstrukturen zu berechnen, ist eine der wichtigsten Aufgaben der Festkörperphysik. Berechnet man nun die Bandstruktur und die Absorption von Gold mit den Mitteln der Quantenmechanik, so stellt zunächst man fest, dass Gold ähnlich wie Silber alle Farben reflektieren sollte. Dass dies nicht so ist, liegt am relativistischen Massezuwachs der Elektronen in den Orbitalen. Eine kurze Erklärung findet man am Fermilab. Ein ähnliches Phänomen ist verantwortlich dafür, dass Quecksilber flüssig ist.

“Na und”, könnte jetzt jemand einwenden. “Dann liegt die Farbe von Gold halt an etwas anderem.” Tja, aber woran? Wenn es nämlich nicht die relativistischen Effekte sind, welche Effekte sind es dann? Die Folgerung wäre, dass unsere aktuelle Quantenmechanik ebenfalls falsch oder zumindest lückenhaft ist – angesichts der Präzision, mit der man die Quantenmechanik und daraus abgeleitet die Festkörpertheorie verwenden kann, um beispielsweise Halbleiterbauteile zu berechnen (ohne die kein Computer funktionieren würde),scheint das sehr unwahrscheinlich. Auch hier gilt wieder dasselbe: Wer die SRT ablehnt, ist in der Pflicht, auch diese Phänomene zu erklären.

Apropos Halbleiter: Auch die Bandstrukturen von Halbleitern wie Germanium und Galliumarsenid können nur durch relativistische Korrekturen korrekt berechnet werden, siehe beispielsweise diesen Artikel.

Auch anderen Effekte in der Chemie werden auf relativistische Korrekturen zurückgeführt. Einen Überblick findet man hier. Hier die schöne Übersicht über das Periodensystem der Elemente mit eingezeichneten Effekten (zum Vergrößern anklicken):

i-64525884f4f79ce5eb616016249b380a-relativityPeriodicTable-thumb-550x266.jpg

Nicht in diesem Bild eingezeichnet sind relativistische Effekte bei chemischen Verbindungen, z.B. in der Bindungslänge – ohne diese Korrekturen liegen berechnete Werte für die Bindungslängen beispielsweise von Quecksilberverbindungen deutlich neben den experimentell gemessenen. (Hierzu muss allerdings gesagt werden, dass im zitierten Artikel auch mit relativistischer Korrektur die Bindungslängen nicht allzu präzise vorhergesagt werden können. Neuere Veröffentlichungen liefern allerdings bessere Ergebnisse, beispielsweise hier oder hier.) Weitere Beispiele für relativistische Effekte in der Chemie findet man bei der englischen Wikipedia.

Falls also die Relativitätstheorie (insbesondere der relativistische Massenzuwachs) für Elektronen nicht gelten würde, dann wären auch viele chemische Effekte nicht erklärbar. Das wiederum würde bedeuten, dass auch die Quantenmechanik lückenhaft ist – das kann man natürlich glauben, aber die experimentellen Belege für die Quantenmechanik sind (insbesondere eben in der Atom- und Festkörperphysik) überwältigend.

Kernphysik
Kernphysik habe ich im Studium nie gemocht, deswegen habe ich davon nicht so schrecklich viel (ehrlich gesagt: nahezu keine) Ahnung. Die SRT spielt aber natürlich auch in der Kernphysik eine wichtige Rolle, das sieht man zum Beispiel in diesem paper, aus dem der folgende Satz stammt:

This is no doubt that relativistic effects explain important
aspects of nuclear structure and scattering.
[ Es gibt keinen Zweifel, dass relativistische Effekte wichtige Aspekte der Kernstruktur und -streuung erklären.]

Ein Beispiel zeigt dieses Bild (wieder zum Vergrößern anklicken) – hier ist der Vergleich zwischen relativistischer (durchgezogene Linie) und nicht-relativistischer (gestrichelte Linie) Lösung und dem Experiment direkt eingezeichnet:

i-6ce7457b7f59dcd2d66fe1dd201903c4-relativisticNuclear-thumb-550x614.jpg

Auch hier erkennt man, dass die nicht-relativistische Rechnung deutlich schlechter ist. Einen weiteren (und sehr neuen) Überblick über relativistische Effekte in der Kernphysik gibt es hier.

Quantenfeldtheorien
Noch wesentlich drastischer sieht die Sache aus, wenn wir uns der Elementarteilchenphysik zuwenden. Beginnend mit der Dirac-Gleichung wird hier seit jeher rein relativistisch gerechnet – anders als in der Chemie, wo die Effekte der SRT oft als relativistische Korrekturen eingebaut werden, sind hier die Gleichungen selbst schon so strukturiert, dass sie nur im Rahmen der SRT einen Sinn ergeben. Die Erweiterung der Dirac-Gleichung führt zu den sogenannten Quantenfeldtheorien – alle heutigen Theorien der Elementarteilchen sind solche QFTs. Sie alle sind von ihrer mathematischen Struktur her relativistisch – die Grundgleichungen der SRT sind quasi implizit in die QFTs eingebaut.

Alle Ergebnisse der Quantenfeldtheorien sind damit implizit immer auch Bestätigungen der SRT. Ein Beispiel ist die Vorhersage des magnetischen Moments des Myons (eine schwere “Variante” des Elektrons). Der experimentelle Wert beträgt
ae = 11659208.0 10-10
ein aktuell berechneter theoretischer Wert ist
at = 11659180.4 10-10
Die Übereinstimmung ist nicht perfekt, weil die theoretische Berechnung (mittels Feynmandiagrammen) immer eine Näherung enthält, aber sie ist schon verdammt gut. Wenn die SRT für Elementarteilchen nicht gelten würde, dann müsste diese Übereinstimmung (immerhin auf 6 zählende Stellen1) auf Zufall beruhen.

1Übrigens ist hier das magnetische Moment schon so normiert, dass nur die Abweichung des Wertes, der aus der Dirac-Gleichung folgen würde, aufgeschrieben ist. Absolut betrachtet wäre die Genauigkeit noch um 2 Größenordnungen besser.

Auch die elektroschwache Theorie funktioniert ohne SRT nicht. Die korrekte Vorhersage der berühmten “Eichbosonen” in den siebziger Jahren wäre also ebenfalls Zufall gewesen. Genau genommen wäre jede korrekte Vorhersage der Elementarteilchenphysik der letzten 60 Jahre reiner Zufall.1 Alles, was wir über Quarks, Gluonen, Leptonen etc. wissen, wäre ohne SRT falsch.

1O.k., es könnte natürlich auch eine gigantische Verschwörung sein. Jedes Jahr beginnen allein in Deutschland so etwa 5000 Menschen ein Physikstudium – die alle müssten entweder erfolgreich getäuscht werden oder sie müssten alle Mitglied der Verschwörung sein. (Weltweit wären es noch ein paar mehr.) Im Geheimen würden dann alle Physiker vielleicht mit den Newton-Formeln rechnen, aber wenn sie veröffentlichen schnell alles auf die SRT umschreiben. Und niemand hat in den letzten 50 Jahren geredet (obwohl das ein sicherer Weg wäre, berühmt zu werden). Und all dieser Aufwand, um … ja, was eigentlich? Was würde man gewinnen? Am besten, wir vergessen die Verschwörungstheorie wieder, die ist einfach zu absurd.

“Na und? Die Wissenschaft hat sich doch schon früher geirrt!”

Schon richtig – aber hier wäre eben nicht nur jede Theorie über Elementarteilchen falsch, sondern auch jede Messung, die am CERN oder DESY oder jedem anderen Teilchenbeschleuniger jemals gemacht wurde, wäre erstens falsch ausgewertet und dann fehlerhaft theoretisch interpretiert worden. Als Beispiel hier mal ein einziges Messergebnis vom DESY, stellvertretend für Tausende andere:

i-a5235421b56453cf4d6f743d537c0317-schmuesermyon2.jpg

Dargestellt ist die Wahrscheinlichkeit, dass sich ein Elektron und ein Positron vernichten und dabei ein Myon-Antomyon-Paar erzeugen – die Datenpunkte sind die Messwerte, die Linie ist die theoretische Vorhersage nach der Quantenelektrodynamik. Wäre die SRT falsch, müsste die Übereinstimmung hier rein zufällig zustande kommen. Wie gesagt, Messungen dieser Art gibt es seeeehr viele – und bisher wurden die verwendeten QFTs dabei exzellent in ihren Vorhersagen bestätigt.1 Alles nur Zufall?

1 Das ist ja gerade ein “Problem” der heutigen Elementarteilchenphysiker – es gibt eigentlich keine Phänomene, die nicht mir dem “Standardmodell” der Elementarteilchentheorie erklärt werden können. Auf der anderen Seite enthält die Theorie zu viele freie Parameter, als dass man sie wirklich für fundamental halten würde.

Astronomie
Gut, in Astronomie kenne ich micht nicht so gut aus – Florian oder Ludmila können sicher ohne Luft zu holen hundert Beispiele für das Wirken der SRT in der Astronomie aus dem Hut zaubern. Aber ein nettes Beispiel will ich hier anführen: Die Supernova-Explosion 1987A. Bei einer solchen Explosion werden massenhaft Neutrinos freigesetzt. Neutrinos sind elektrisch nicht geladen und haben deshalb mit Licht zunächst nichts zu tun. Wenn also die Lichtgeschwindigkeit nur für Licht relevant ist und keine absolute Schranke darstellt, dann sollten die Neutrinos – die bei so einer Explosion mit extrem hoher Energie freigesetzt werden – mit irgendeiner Geschwindigkeit fliegen. Tatsächlich erreichten die Neutrinos dieser Supernova die Erde etwa 3 Stunden vor dem Licht. Bei einer Entfernung von etwa 157000 Lichtjahren heißt das, dass sie (zunächst rechnerisch) mit einer Geschwindigkeit flogen, die um ein fünfhundert Millionstel von der Lichtgeschwindigkeit abweicht. So ganz irrelevant scheint die Lichtgeschwindigkeit für Neutrinos also nicht zu sein.

Die Neutrinos flogen übrigens nicht mit Überlichtgeschwindigkeit – anders als das sichtbare Licht kommen sie sofort aus dem Kern des Sterns heraus, während die Photonen und die Druckwelle selbst ein bisschen brauchen, bis sie die Sternoberfläche erreichten. Tatsächlich flogen die Neutrinos (die ja Masse haben) etwas langsamer als das Licht. Ohne die Lichtgeschwindigkeit als Obergrenze ist das Verhalten der Neutrinos aber nicht zu erklären, denn mit den riesigen Energien bei einer Supernova-Explosion müssten dann ja auch entsprechend riesige Geschwindigkeiten verbunden sein – stattdessen flogen alle Neutrinos ziemlich genau gleich schnell, nämlich ziemlich genau mit Lichtgeschwindigkeit. Schneller geht’s eben nicht.

Kann die SRT falsch sein?
Das klingt jetzt so, als wäre die SRT das, wovon eigentlich alle Naturwissenschaftler sagen, dass es nicht existiert: Eine bewiesene Theorie. Ist es also unmöglich, dass die SRT falsch ist?

Um das zu beantworten, muss man sich zunächst fragen, was “falsch” eigentlich bedeutet. Die Newtonsche Mechanik ist sicherlich in dem Sinne “falsch”, dass sie unter bestimmten Bedingungen (nämlich bei hohen Teilchengeschwindigkeiten) falsche Vorhersagen macht. Für niedrige Geschwindigkeiten ist sie aber dennoch anwendbar, wenn auch mit (sehr kleinen) Fehlern. Die Newtonsche Mechanik hat einen Anwendungsbereich, sie gilt nur für kleine Geschwindigkeiten. Streng genommen ist sie falsch, aber trotzdem kann man sie in vielen Fällen verwenden.

Das gleiche gilt auch für die SRT: Sie ist, wie wir oben gesehen haben, für viele Fälle geprüft und macht korrekte Vorhersagen. Aber natürlich haben Experimente eine endliche Messgenauigkeit – es kann also sein, dass es winzige Abweichungen gibt, die wir nicht beobachten können. Bei sehr kleinen Abständen und extrem hohen Energien wird die SRT zumindest in der Theorie problematisch, und die meisten Physiker gehen wohl davon aus, dass sie in diesen Fällen nicht gilt (über eine mögliche Lösung habe ich vor einiger Zeit schon mal etwas geschrieben).

Die Abweichungen, die durch eine Erweiterung oder Änderung der SRT zu Stande kommen, müssen allerdings so klein sein, dass sie mit bisherigen Beobachtungen im Einklang stehen, und das setzt die Grenzen schon ziemlich eng. Das ist das Problem mit jedem ernsthaften Versuch, die Physik zu revolutionieren: Man muss sich etwas Neues ausdenken, aber dieses Neue darf nicht im Widerspruch zu bereits bekanntem stehen. Für die SRT bedeutet dies: Jede Theorie, die die SRT ersetzen soll, wird diese als Grenzfall (beispielsweise für hinreichend große Längen) enthalten müssen, so wie die Newtonsche Mechanik als Grenzfall in der SRT (und in anderer Weise auch in der Quantenmechanik) enthalten ist. Daran führt kein Weg vorbei – wer etwas anderes behauptet, hat eine Menge zu erklären.

Kommentare (1.068)

  1. #1 WeiterGen
    27. Dezember 2010

    Danke, sehr gut zu lesender Artikel.

  2. #2 KommentarAbo
    28. Dezember 2010

  3. #3 Dr. Webbaer
    28. Dezember 2010

    Eine Theorie gilt dann als “falsch”, wenn sie empirisch nicht adäquat ist, wie es etwas aufgeblasen gerne genannt wird. Empirisch adäquate Theorien konkurrieren beim Humankapital (vs Bärenkapital) um den Nutzen und die nutzenreichste Theorie gewinnt. – Zwischen den Theorien gibt es also eine Art Wettstreit und die nutzenbringenste gewinnt, dabei wird auch mit Brimborium gearbeitet, wie bspw. mit dem (Schein-)Argument “besonders gut belegt”.

    Es spricht nichts dagegen anzunehmen, dass auch die SRT beizeiten, “streng genommen” wie der Blogautor schreibt, falsch sein wird. – Warum sollten Humantheorien auch richtig sein, bedürfen Sie doch dem Systemzugriff (das absolute Verstehen der physikalischen Vorgänge) um wirklich richtig sein zu können?

    An der modernen Wissenschaftlichkeit sich messende Theorien sind eine Art Werkzeug und zivilisatorische Leistung.

    Welche Theorie welche Theorie “als Grenzfall” oder anders begründet “enthalten wird müssen”, nunja, schaun mer mal.

    HTH
    Dr. Wb

  4. #4 perk
    28. Dezember 2010

    Welche Theorie welche Theorie “als Grenzfall” oder anders begründet “enthalten wird müssen”, nunja, schaun mer mal.

    welchen teil der dargelegten argumentation lehnen sie denn ab, dass sie den zweifel streuen? gibts dafür auch gründe oder machen sie das nur weils schön klingt so als abschluss des eigenen beitrags?

  5. #5 Sascha Vongehr
    28. Dezember 2010

    “Aber vielleicht hilft dieser Artikel ja denen, die sich über die heftigen Reaktionen der Physiker wundern, diese besser zu verstehen.”

    Nein, weil die Polarisierung der Debatte (heftige Reaktionen) mit falschen Argumenten daherkommt (die Du hier nicht gebracht hasst, dafuer meinen herzlichen Dank), zum Beispiel das der Aether widerlegt sei (obwohl stringtheory ihn laengst wieder belebt hat (universe on a membrane)) oder das jegliche ueberlicht Geschwindigkeit v>c sofort Kausalitaet verletzt (was es nur tut wenn sie nicht an ein specifisches inertial System gebunden ist (e.g. Aether)). Die Physiker die heftig reagieren liegen meistens voellig daneben und das unterstuetzt die Cranks nur, insbesondere weil diese ja oft an einen Aether denken und es in jenem ja sofort klar ist das v>c die Aether Anregungen (pseudo particles) nicht in die Vergangenheit zurueckschicken kann – Kausalitaet ist im Aether ja trivial gesichert. Die “heftigen Reaktionen Physiker” die sich aus Borniertheit nie auf den Aether Standpunkt “herablassen” haben natuerlich mehr Schwierigkeiten das zu begreifen. Fuer die Cranks ist dann sofort klar das die Physiker irgendwie anscheinend zu doof sind, und da haben die ja auch recht soweit es die “heftigen Reaktionen Physiker” betrifft: “Wenn die etwas so einfaches nicht begreifen, ja warum soll ich mich denn dann auf kompliziertere Argumente einlassen – alles nur Schall und Rauch!”

  6. #6 Dr. Webbaer
    28. Dezember 2010

    @perk
    Der Artikel war erstklassig! – Lob kommt von Dr. Wb eben nur selten, weil der Dissens interessanter ist.

    Die Annahme (oder Meta-Theorie :), dass eine Theorie Teile einer abgelösten Theorie beinhalten muss, ist dagegen einfach falsch. Herr Bäker ist hier nahe an der SRT geblieben, d.h. er hat seine Gründe, aber grundsätzlich ist die Annahme falsch. – Auch wenn es gelegentlich einiger Fantasie bedarf…

    So wie die Theoretisierung nie die pol. Folgerung bestimmt (Hume’s Law), so bestimmt die Datenlage nie die Theoretisierung.

    MFG
    Wb

  7. #7 Ireneusz Cwirko
    28. Dezember 2010

    Sind wir alle einsteingläubige Dogmatiker?

    Ja. Ihr seit alle Deopen

  8. #8 rolak
    28. Dezember 2010

    ^^Ist das jetzt eine Mischung aus ‘Deisten’ und ‘Popen’ – oder sind die Finger vor lauter Frust angeschwollen?
    Falls letzteres…

  9. #9 roel
    28. Dezember 2010

    @MartinB “Die Newtonsche Mechanik ist sicherlich in dem Sinne “falsch”” Warum schreiben Sie nicht: Die Newtonsche Mechanik ist sicherlich nur mit Einschränkungen richtig? Vieleicht habe ich auch nur eine zu harte Definition von falsch?!

  10. #10 Frank Wappler
    28. Dezember 2010

    Martin Bäker schrieb (im Artikel):
    > Kann die SRT falsch sein? […] Um das zu beantworten, muss man sich zunächst fragen, was “falsch” eigentlich bedeutet.

    So weit, so gut.

    > Wie jeder weiß, hat Gold eine gelbliche Farbe. [D.h. Gold kann nicht …] ähnlich wie Silber alle Farben reflektieren […]
    > Warum eigentlich? Die Farbe eines Metalls wird durch die Energieniveaus der Elektronen bestimmt […]

    Um diese Frage zu beantworten, müsste man sich doch zunächst fragen, was “Farbe” und “Energieniveaus” eigentlich bedeuten — das leistet die (S)RT.

    Und ebenso müsste man sich doch fragen was “Gold” usw. bedeutet — das leistet die (S)RT, unter Einsatz von Begriffen der Teilchenphysik (im bekannt anmaßendst-weitesten Sinne &).

    So wie die meisten “Warum?”-Fragen ist demnach auch die betrachtete Frage (Warum eigentlich reflektieren Gold und Silber ungleich?? eher abwegig bzw. rhetorisch.

    Direkt beantwortbar dagegen wäre eine Frage wie:
    “Ist das Gegebene (Hergezeigte und als wiedererkennbar Betrachtete) ein Stück Gold, oder kein Stück Gold, oder weder der eine noch der andere Eigenzustand des relevanten Stück-Gold-Messoperators?”

    Die Nachvollziehbarkeit des Messoperators und der RT, auf dem dieser beruht, bleibt davon unberührt, ob und welcher Messwert dabei in einem bestimmten Versuch erhalten würde.
    Man mag allenfalls unter Benutzung eventueller anderer Theorien noch andere Messoperatoren erstellen, um aus den (durch Beobachtung des Hergezeigten) gegebenen Beobachtungsdaten noch weitere Messwerte zu gewinnen.

    (Die entsprechenden Messwerte oder Erwartungen, insbesondere zusammen mit gemessenen oder erwarteten geometrischen Beziehungen zwischen den hergezeigten Beteiligten, lassen sich als Modelle zusammenfassen — z.B. (Standard-)Modelle der Geologie, der Astrophysik, der Teilchenphysik, des Goldumlaufes, des Bankwesens usw.)

    > Wer die SRT ablehnt, ist in der Pflicht, auch diese Phänomene zu erklären.

    Wer die SRT (insgesamt) ablehnt, der beraubt sich dadurch bestimmter Messoperatoren sowie (schlimmstenfalls) der dadurch schon erhaltenen Messwerte und Unterscheidungen.

    Wer (lediglich) das Einsteinsche Axiom ablehnt, dass man zumindest im gedanken-experimentellen Prinzip in jedem Fall knallhart und glasklar beurteilen könne, ob man
    Das Zeigen des kleinen Zeigers meiner Uhr auf 7 und das Ankommen des Zuges ” koinzident beobachtete, oder in bestimmter Beobachtungsreihenfolge, der ist MBMN zwar bewundernswert realistisch, hat aber trotzdem die Messoperatoren der RT zunächst einmal verworfen und steht vor ziemlichen Schwierigkeiten bei der Erstellung irgendwelcher anderer Messoperatoren.

    Wer aber schlicht meint, die SRT experimentell zu testen, der verwirft deren Messoperatoren (und gewonnenen Messwerte) offenbar sogar ohne ggf. irgendeinen nachvollziehbaren Ersatz anzubieten zu wollen.

    Zusammenfassend:
    Ein bestimmtes Modell (einschließlich bestimmter Erwartungen zu weiteren Messwerten) kann falsch sein bzw. experimentell getestet werden;
    Theorien (wie z.B. die SRT) dagegen werden gebraucht und vorausgesetzt, um bestimmte Messoperatoren und bestimmte Modelle überhaupt zu erstellen.

    p.s.
    Dieses ist bitte auch als Beantwortung deines Kommentars anderswo zu verstehen (vgl. den vor dir dort eingeführten und hier wieder benutzten Begriff des “Herzeigens“).

    Von da her hätte ich noch die Frage nach “Masse” als “Lagrangeschem Multiplikator” zu beantworten:
    An passender Stelle hab ich wohl mal den Term
    “m Sqrt[ 1 – beta^2 ]”
    bemerkt, und mich (rhetorisch) gefragt: “Warum eigentlich (dieser Term, da)?” …

  11. #11 schlappohr
    28. Dezember 2010

    @MartinB

    (Ich hoffe, Dir war klar, was Du da für ein Fass aufmachst…)

    Ich versuche mal, Deine Definitionen des “Falsch”-Begriffs zu verfeinern (Konstruktive Kritik ist willkommen):

    – Eine Theorie ist ungenau, wenn Sie aufgrund nicht hinreichend genau bestimmter Konstanten (z.B.Lichtgeschwindigkeit) Ergebnisse liefert, die immer ein wenig “daneben” liegen.

    – Eine Theorie ist unvollständig, wenn sie in bestimmten bekannten Grenzsituationen (und nur dort) Vorhersagen liefert, deren Ungenauigkeit signifikant groß ist.

    – Eine Theorie ist falsch, wenn Sie nicht oder höchstens zufällig hinreichend genaue Vorhersagen macht.

    (Die Schwammigkeit dieser Definitionen ist mir durchaus bewusst.)

    Wir können niemals eine Theorie als richtig bezeichnen, aber wir können sie als “nicht-falsch” (im Sinne obiger Definition) bezeichnen, wenn sie in der überwältigenden Mehrzahl der Fälle Vorhersagen liefert, die wir im Rahmen der Genauigkeit der Konstanten und Messungen akzeptieren können, und wir die Grenzfälle, in denen unsere Theorie versagt, genau identifizieren können. Im diesen Sinne ist die SRT also “nicht-falsch”.

    Schaut man sich die Argumente der Kritiker (beispielhaft der SRT) an, so findet man in den meisten Fällen folgende Aussagen:

    – Die SRT ist ungenau und unvollständig.
    Sicher ist sie das, aber daran krankt prinzipbedingt jede Theorie, also insbesondere auch die “Gegentheorien” der Kritiker. Erst wenn jemand die Weltformel aufgestellt hat und alle Nachkommastellen der Lichtgeschwindigkeit kennt, wird es interessant.

    – Es gibt eine andere Theorie, die die gleichen Ergebnisse liefert.
    Nehmen wir einmal an, das stimmt. Wenn diese Ersatztheorie also die gleichen Ergebnisse liefert und die gleichen Grenzfälle besitzt, ist sie sinnlos. Wenn sie andere Vorhersagen liefert, ist sie falsch, da die Experimente nun mal SRT gehorchen (Gold bleibt immer goldfarben). Erst wenn sie die gleichen Vorhersagen liefert und weniger oder zumindest andere Grenzfälle besitzt, wird es interessant.

    Leider stellen die meisten Kritker keine derartigen Überlegungen an. Ihr Wunsch nach einer Gegentheorie ist im Allgemeinen von weltanschaulichen Gründen getrieben (weil nicht sein kann, was nicht sein darf), oder von einer generellen Unzufriedenheit über ein in seiner Komplexität kaum noch überschaubares Theoriengebäude der modernen Physik (Stichwort einsteingläubige Dogmatiker).

    Es geht den Physikern nicht darum, um jeden Preis eine “anbetungswürdige” Theorie zu erhalten, sondern darum, etwas Besseres zu finden. Solange das nicht der Fall ist, bleibt die SRT das Maß aller Dinge.

  12. #12 MartinB
    28. Dezember 2010

    @roel
    Das meinte ich ja genau mit “in diesem Sinne” – sie ist halt streng genommen falsch, aber oft anwendbar.

    @FrankWappler
    Ich habe den Kommentar nicht wirklich verstanden – wieso beantwortet die SRT die Frage, was Energieniveaus oder Farben sind? Die gäbe es doch auch in einer Welt mit Schrödingergleichung und Newton-Mechanik.

    Warum-Fragen sind natürlich in der Physik immer ein problem, da habe ich sicher etwas schlampig formuliert (siehe dieser Post https://www.scienceblogs.de/hier-wohnen-drachen/2010/08/kann-die-physik-die-welt-erklaren.php)

    Den Begriff des Messoperators im Zusammenhang mit der SRT kann ich auch nicht ganz nachvollziehen – ist das einfach ein anderes Wort für “Messgröße”?

    Der Unterschied zwischen “Modell” und “Theorie” ist mir auch nicht so ganz klar.

  13. #13 MartinB
    28. Dezember 2010

    @Schlappohr
    Auch wenn ich die Begriffe etwas anders verwendet habe, stimme ich dem zu – genau so sollte man das sehen (und wie man nun genau “falsch” definiert, ist ja ziemlich egal, wenn man sagt, wie man es meint.)
    Inwieweit ich da ein Fass aufmache, mal sehen – die rumpöbelnden Cwirkos und noannas können wir einfach ignorieren, die haben ja inhaltlich nichts beizutragen.
    Und hier ging es mir auch nicht so sehr um eine Diskussion (darf aber gern sein 😉 ), sondern darum, vielleicht mitlesenden Nicht-Physikern klarzumachen, warum die RT-Leugner einem Physiker so absurd erscheinen.

  14. #14 cydonia
    28. Dezember 2010

    Ich bedanke mich für die gute Argumentation!
    Natürlich wird es dennoch notorische Nicht-Wissen-Woller nicht daran hindern können, aus purer Gewohnheit rumzupöbeln. Möge sie der Blitz(der Vernunft!) treffen.

  15. #15 Dr. Webbaer
    28. Dezember 2010

    @Frank Wrappler
    Sie haben jetzt den 192 gefundenen Messoperatoren 8 weitere hinzugefügt, d.h. der Begriff ist nicht gängig. Unter Theorien hat der Wb zudem bisher Sichten verstanden und unter Modellen Nachbauten. Kurzum, vielleicht noch mal ein wenig erläutern?!

    MFG
    Wb

  16. #16 Frank Wappler
    28. Dezember 2010

    MartinB schrieb (28.12.10 · 11:43 Uhr):
    > wieso beantwortet die SRT die Frage, was Energieniveaus oder Farben sind?

    Die SRT beinhaltet eine gedanken-experimentelle Definition, wie “Dauer” (eines gegebenen Beteiligten, von einer bestimmten seiner Anzeigen, bis zu einer weiteren bestimmten seiner Anzeigen) zu messen bzw. zu vergleichen ist;
    nämlich (recht vereinfacht): unter Berücksichtigung des Integrals über die Zahl
    “Sqrt[ 1 – beta^2 ]” des Anzeigenden, von Anfangs- bis Endanzeige.

    In Anwendung dieser Messdefinition der SRT kann u.a. beurteilt werden, ob oder innerhalb welcher Genauigkeit ein gegebener Oszillator Perioden gleicher Dauer anzeigte,
    und ob bzw. wie fern diesem Oszillator folglich ein bestimmter “(Oszillations-)Frequenz”-Wert zuzuordnen ist
    (sowie ggf. welches Verhältnis die Oszillationsfrequenzen zweier gegebener geeigneter Oszillatoren hatten).

    Der Begriff “Energieniveau(-Unterschied)” bezieht sich auf solche Frequenzwerte an sich; der Begriff “Farbe” auf Frequenzwerte bzw. auf Frequenzverhältnisse auch im Zusammenhang mit “Intensität”. (Darüber hinausgehende Diskussion von “Physiologie” erspar ich uns …)

    Wie sonst wäre die Frage nach den Bedeutungen dieser beiden Begriffen aus dem Artikel nachvollziehbar zu beantworten??

    > Die gäbe es doch auch in einer Welt mit Schrödingergleichung und Newton-Mechanik.

    Woher soll man wissen, was (wenn überhaupt) Schrödinger oder Newton oder Wer-auch-immer gemeint (hergezeigt, beobachtet, beurteilt) haben könnten, wenn nicht im Bezug auf die genannten nachvollziehbaren Gedankenexperimente der RT?

    > Den Begriff des Messoperators im Zusammenhang mit der SRT kann ich auch nicht ganz nachvollziehen – ist das einfach ein anderes Wort für “Messgröße”?

    Versteht sich der Begriff “Messgröße” als einschließlich einer bestimmten gedanken-experimentellen Definition wie ein bestimmter Messwert ggf. aus geeigneten gegebenen Beobachtungsdaten zu gewinnen wäre
    (und gegenüber dem man z.B. “systematische Unsicherheit” abschätzen würde, falls man der Messdefinition nicht folgeleisten würde)?
    Falls so, dann ja: diese beiden Worte erscheinen austauschbar.

    > Der Unterschied zwischen “Modell” und “Theorie” ist mir auch nicht so ganz klar.

    Versuch doch (noch) einmal in einem mögichst konkreten Beispiel (oder auch mehreren) die folgende hypothetische Aussage zu vervollständigen:
    “Die SRT ist experimentell falsifiziert falls folgender Befund erhalten wurde: …”.

    Was auch immer da eingefügt würde, ist sicherlich mit einer der folgenden Reaktionen zu beantworten; entweder:

    1. Falls dieser hypothetische Befund nicht unter Anwendung der SRT erhalten wurde — was bedeuten die darin auftretenden Begriffe?
    (Und was immer sie bedeuten mögen, die Nachvollziehbarkeit der SRT ist davon unberührt.) Oder

    2. Dieser hypothetische Befund wurde in Anwendung der SRT erhalten — er falsifiziert nicht die SRT an sich, sondern bestimmte Modellannahmen oder -erwartungen (die ohnehin schon unter Anwendung der SRT formuliert wurden), z.B. die Modellerwartungen
    – dass ein gegebener Oszillator die gleiche Periodendauer behielt, auch während (noch) nicht gemessen wurde, ob dieser Oszillator dabei die gleiche Periodendauer behielt,
    – dass eine bestimmte Startmarkierung und eine bestimmte Zielmarkierung zueinander ruhten, auch während (noch) nicht gemessen wurde, ob diese beiden zueinander ruhten,
    – dass zwei gegebene Beteiligte gleiche elektr. Ladung und/oder gleiches magnetisches Dipolmoment hatten, auch während (noch) nicht gemessen wurde, ob …
    – dass …, auch während (noch) nicht gemessen wurde, ob …
    .

    Das illustriert den Unterschied zwischen Theorie und Modell.
    Eine Theorie (die Messdefinitionen betrifft) sollte von vornherein festgelegt werden und nachvollziehbar bleiben.
    Ein Modell (sofern es Erwartungen einschließt) kann experimentell getestet und ggf. falsifiziert werden.
    Die RT einerseits und die bekannten Standardmodelle (der Teilchenphysik, der Geologie, usw. usf.) sind entsprechend richtig benannt.

  17. #17 H.M.Voynich
    28. Dezember 2010

    Es hat schon was, wenn man an so eine Kurve “QED” dranschreiben kann … ,)

    @schlappohr:
    Danke, schön zusammengefaßt.

  18. #18 MartinB
    28. Dezember 2010

    @FrankWappler
    Das mit der Dauer finde ich zunächst problematisch – Einstein hat doch gerade andersherum gesagt “Zeit ist das, was eine Uhr misst”.

    Und ich sehe nicht, warum ich zwangsläufig eine Frequenz zur Energiemessung brauchen sollte – ich kann ja z.B. eine Spektralzerlegung machen, dann übersetze ich Energien bzw. Frequenzen in Ortskoordinaten. Oder ich kann einen Photoeffekt verwenden, auch da messe ich Photonenenergien ohne eine Zeitdauer.

    Den Unterschied zwischen Theorie und Modell habe ich nach wie vor nicht verstanden. Ich ergänze mal den Satz:
    “Die SRT ist falsifiziert, wenn” (alle folgenden Aussagen sind im ungekrümmten Raum zu verstehen, sonst gibt’s Komplikationen mit der ART)
    – eine informationstragende Wirkung beobachtet wird, die sich in einem Bezugssystem mit Überlichtgeschwindigkeit bewegt
    – wenn die beobachtete Energiezunahme eines Teilchens mit der Geschwindigkeit der Einsteinformel widerspricht
    – E=mc2 verletzt wäre
    – usw.

    Was soll denn genau bedeuten, einen Befund unter Anwendung der SRT zu erhalten? Das hängt vermutlich irgendwie mit deinem Frequenz/Zeitbegriff zusammen, den du ja auch auf die SRT zurückführst, aber das ist mir ehrlich gesagt nicht klar. Würde das nicht heißen, dass man die SRT nicht widerlegen kann? Sorry, aber ich verstehe es immer noch nicht.

  19. #19 Dr. Webbaer
    28. Dezember 2010

    Herr Wappler, Dr. Wb nicht folgen können, wählen Sie doch einmal eine einfache verständliche Sprache. Sie zitieren in Ihrem Erstkommentar ja subversiv Popper, der immer gut verständlich war.

    MFG
    Dr. Wb

  20. #20 Thomas J
    28. Dezember 2010

    @Webbaer

    “Herr Wappler, Dr. Wb nicht folgen können, wählen Sie doch einmal eine einfache verständliche Sprache”

    Scherzkeks 🙂

    BTW…. Weihnachten schon vorbei? Rudolf nicht mehr spitz?

  21. #21 Dr. Webbaer
    28. Dezember 2010

    Weihnachten ist vorbei, Bruder – Rudy abgesteift, korrekt!
    Haben Sie folgen können?

    MFG
    Wb

  22. #22 Thomas J
    28. Dezember 2010

    nö… denke aber, dass man nichts Relevantes verpasst hat. Wollte ja nur den Spiegel vorhalten, Sie verstehen.

  23. #23 Dr. Webbaer
    28. Dezember 2010

    denke aber, dass man nichts Relevantes verpasst hat

    Das sieht Dr. Wb aber gar nicht so. Wb immer schon gute Nase gehabt haben für Werthaltiges. – Wer wie “Dr.” Wappler vorträgt, könnte etwas zu sagen haben. Insofern ist es womöglich nicht gut, wenn es beim Erkenntnissubjekt “Klonk” macht, höhö.

    MFG
    Wb

  24. #24 threepoints...
    28. Dezember 2010

    Aus dem beitrag von oben:

    “Ein Beispiel ist die Vorhersage des magnetischen Moments des Myons (eine schwere “Variante” des Elektrons). Der experimentelle Wert beträgt
    ae = 11659208.0 10-10
    ein aktuell berechneter theoretischer Wert ist
    at = 11659180.4 10-10
    Die Übereinstimmung ist nicht perfekt, weil …”

    -> Wieviel Prozent Abweichung/Unterschied ergeben diese beiden Zahlen?

  25. #25 MartinB
    28. Dezember 2010

    @threepoints
    Steht doch da: “immerhin auf 6 zählende Stellen”, also etwa 1 zu einer Million – wenn man die Anmerkung in der “Fussnote” oben berücksichtigt, ist es etwa 1:100Millionen.

  26. #26 MartinB
    28. Dezember 2010

    Entschuldigung, hab mich vertan – so wie die Zahlen da stehen, ist es 1:500000, der zweite Faktor wäre etwa 1:300Millionen.
    Nochmal zur Erläuterung: Die gemessene Zahl ist der g-Faktor. Der ist laut Diracgleichung gleich 2, aber es kommen Korrekturen durch jede Menge virtuelle Prozesse hinzu. Deshalb ist er etwas größer als 2. Die oben angegebene Zahl a ist a=(g-2)/2

  27. #27 Frank Wappler
    28. Dezember 2010

    MartinB schrieb (28.12.10 · 16:15 Uhr):

    > Das mit der Dauer finde ich zunächst problematisch – Einstein hat doch gerade andersherum gesagt “Zeit ist das, was eine Uhr misst”.

    Nicht dass ich wüßte. Eine bekannte und wichtige Aussage Einsteins (die die Worte “Zeit” und “Uhr” gebraucht) ist dagegen

    […] daß ich an Stelle der „Zeit“ die „Stellung des kleinen Zeigers meiner Uhr“ setze

    usw.; vgl. z.B. https://wikilivres.info/wiki/Zur_Elektrodynamik_bewegter_K%C3%B6rper

    (“Weitere Festsetzungen” beschäftigen sich dort anschließend mit Beziehungen zwischen den Anzeigen der einzelnen Beteiligten, insbesondere mit “Gleichzeitigkeit”; und zumindest mittelbar mit “Dauer”, “Distanz”, “Geschwindigkeit” etc.)

    > Was soll denn genau bedeuten, einen Befund unter Anwendung der SRT zu erhalten?

    Ein naheliegendes Beispiel ist sicher der Vergleich der Lebensdauern von (instabilen) Teilchen; oder z.B. auch von (sterblichen) Individuen.

    > Ich ergänze mal den Satz:
    > “Die SRT ist falsifiziert, wenn” (alle folgenden Aussagen sind im ungekrümmten Raum zu verstehen, sonst gibt’s Komplikationen mit der ART)

    Diese Bedingung ist für die Diskussion im Rahmen der SRT sicher sinnvoll. Die Messdefinitionen, wie zu entscheiden wäre, ob gegebene Beteiligte zueinander ruhen (d.h. zueinander starr sind, und zueinander Euklidisch-flach liegen), oder nicht, und ob der Brechungsindex einer gegebene Region gleich 1 ist, oder nicht, gehören wohl zum Aufgabenbereich der ART.

    > – eine informationstragende Wirkung beobachtet wird, die sich in einem Bezugssystem mit Überlichtgeschwindigkeit bewegt

    Das ist natürlich noch nicht annähernd eine so konkrete gedanken-experimentelle Beschreibung, wie es für die Diskussion wünschenswert wäre.

    Soll es dabei um zwei bestimmte Beteiligte gehen, z.B. “Wirk-Signal-Geber A” und “Wirk-Signal-Empfänger B”?

    Sollten A und B (u.a.) während aller zu betrachtenden Versuche zueinander geruht haben?,
    und sollte es ihnen insbesondere gelungen sein, ständig einen weiteren Beteiligten (“M”) als Mitte zwischen einander zu identifizieren und damit zu jeder Anzeige As eine bestimmte Anzeige Bs als gleichzeitig festzustellen?

    Sollten A und B ihre Distanz untereinander Versuch für Versuch einvernehmlich chrono-geometrisch aus der halben (Wirk)-Signal-Pingdauer ermitteln?

    Sollte die Geschwindigkeit des Wirk-Signal-Austausches zwischen A und B aus dem Verhältnis der Distanz zwischen A und B und As Dauer von seiner Anzeige des Wirk-Signals bis zu seiner Anzeige gleichzeitig zu Bs Anzeige des Wirk-Signal-Empfangs ermittelt werden?

    Falls so, dann lässt sich beweisen, dass die Geschwindigkeit des Wirk-Signal-Austausches zwischen A und B höchstens so groß ist, wie die Geschwindigkeit des Ping-Signal-Austausches zwischen A und B, das der Distanzbestimmung zugrundeliegt;
    Über“-Signalgeschwindigkeit gehört nicht zum Wertebereich des Messoperators.

    Also benutzt deine Befund-Vorgabe offenbar Begriffe, die außerhalb der SRT-Definitionen liegen …

    > – wenn die beobachtete Energiezunahme […]

    Es geht doch sicherlich um Messung, also die Feststellung (reeller oder Boolscher) Werte; nicht bloße Beobachtung von Signalen (oder bloße Beurteilung der Koinzidenz oder Reihenfolge von Beobachtungsanzeigen).

    > Und ich sehe nicht, warum ich zwangsläufig eine Frequenz zur Energiemessung brauchen sollte
    (… aber immerhin zur Messung von “Farbe“? …)
    > – ich kann ja z.B. eine Spektralzerlegung machen, dann übersetze ich Energien bzw. Frequenzen in Ortskoordinaten.
    Wie wolltest du denn nachweisen (nicht zuletzt dir selbst), dass was immer du da machen wolltest irgendetwas bestimmtes mit “Energie” zu tun hätte?? …

    Deine beiden vorgeschlagenen Ergänzungen meiner Satzvorgabe haben auch wirklich nicht viel mit Modellen zu tun.
    Aufgrund der im Artikel diskutierten Beispiele hätte man ja stadessen eher mit (denkbaren) Ergänzungen rechnen können wie z.B.

    “Die SRT ist falsifiziert, wenn”
    – “ein Stück vorgefunden wird, das alle Farben außer Gelblich reflektiert”, oder
    – “ein Teilchen vorgefunden wird, dessen elektrische Ladung 1/4 der eines Elektrons ist”, oder
    – “ein Paar ein-eiiger Zwillinge vorgefunden wird, die ungleiche Lebensdauern hatten”.

    > Würde das nicht heißen, dass man die SRT nicht widerlegen kann?

    Jedenfalls nicht experimentell.
    Genauso wenig, wie man z.B. Zahlentheorie durch Nachrechnen widerlegen könnte, oder Musiktheorie durch Musizieren.
    Aus- bzw. Abwahlkriterien für Theorien sind stattdessen ihre Nachvollziehbarkeit und (für Theorien der experimentellen Wissenschaften) die Prognose-Zuverlässigkeit der Modelle, die ihre Begriffe einsetzen.

  28. #28 threepoints...
    29. Dezember 2010

    Eine Uhr ist kein Messgerät. Sie misst nicht die Zeit – sie zeigt sie nur an. Zumindest gilt diese Aussage für die rein mechanische Uhr.
    Das eine Uhr die Zeit nicht misst, knn man daran erkennen, das sie vor oder nach geht….würde sie nämlich die Zeit selbst messen, dann täte sie immer die “richtige” zeit anzeigen …!

  29. #29 MartinB
    29. Dezember 2010

    @FrankWappler
    Ich fange am Ende an:

    “Würde das nicht heißen, dass man die SRT nicht widerlegen kann?
    Jedenfalls nicht experimentell.”

    Das halte ich für absolut falsch. Wenn die gemessenen Lebensdauern von teilchen, die mit 99% c fliegen, sich nicht gegenüber ruhenden Teilchen verändern, dann wäre das eine Verletzung der SRT. Wenn Die Teilchen im CERN mit Überlichgeschwindigkeit durch den Beschleuniger fliegen, wäre das eine Widerlegung der SRT. Meine Beispiele oben passen entsprechend.

    Der Vergleich mit Zahlentheorie oder Musiktheorie ist vollkommen falsch, weil beides keine empirischen Wissenschaften sind. Und der Nachsatz mit den Aus- und Abwahlkriterien macht die Sache dann wieder unverständlich: Ich kann die SRT nicht experimentell widerlegen, aber ich kann Modelle, die aus ihr abgeleitet werden, widerlegen? Wieso widerlegt das nicht die SRT? Der Unterschied Modell-Theorie ist mir immer noch nicht klar geworden.

    Die Beispiele aus dem Artikel (SRT widerlegt, wenn Gold nicht gelb ist usw) sind deswegen nicht so geeignet, weil nicht-gelbes Gold ja auch eine Widerlegung der Quantenmechanik sein könnte.

    Wie wolltest du denn nachweisen (nicht zuletzt dir selbst), dass was immer du da machen wolltest irgendetwas bestimmtes mit “Energie” zu tun hätte?? …

    Z.B. mit dem Photoeffekt in Verbindung mit einer bias-Spannung – bias-Spannung für Licht unterschiedlicher Wellenlängen (an unterschiedlichen Orten im Spektrum) messen, dann werde ich eine schöne lineare Beziehung zwischen Ort im Spektrum und bias-Spannung (sprich Energie) bekommen, ganz ohne Frequenz. Farbe sehe ich übrigens mit dem Auge, da spielt auch die Absorptionsenergie der entsprechenden Moleküle eine Rolle.

    Die Behauptung, Energien seien nur im Rahmen der SRT sinnvoll definierbar (so verstehe ich dich im Moment) wird auch dadurch widerlegt, dass der Energiebegriff älter ist als die SRT und dass die Newtonsche Mechanik und die QM nicht-relativistisch sinnvoll mit dem Energiebegriff operieren können.

    Auch das mit der Messung der Überlichtgeschwindigkeit habe ich nicht verstanden. Wenn die SRT einfach kompletter Blödsinn wäre, dann ist die Lichtgeschwindigkeit genauso wenig fundamental wie die Schallgeschwindigkeit. Da ich Überschallgeschwindigkeiten messen kann, könnte ich dann auch Überlichtgeschwindigkeiten messen.
    Ein Versuchsaufbau könnte per Lichtsignal synchronisierte Uhren verwenden, oder ich synchronisiere zwei Uhren am selben Ort und bewege dann A und B mit exakt gleicher Geschwindigkeit an ihre Endpunkte, so dass die Uhren synchronisiert bleiben, dann lasse ich sie zueinander ruhen und schicke das überlichtschnelle Signal.

    Ein naheliegendes Beispiel ist sicher der Vergleich der Lebensdauern von (instabilen) Teilchen; oder z.B. auch von (sterblichen) Individuen.

    Verstehe nicht, wie das eine Antwort auf meine Frage sein soll.

    Das Einstein-Zitat “Zeit ist das, was man an der Uhr abliest.” hat übrigens etwa 100000 google-Treffer – muss deswegen nicht stimmen, da ich es aber auch anderswo häufig gelesen habe, halte ich es erstmal für korrekt. (@threepoints: Na klar, wenn unterschiedliche Uhren in ihrer Anzeige differieren, dann wird’s damit schwierig, aber der Satz soll meiner Ansicht nach zunächst mal nur zeigen, dass man keine philosophische Definition von Zeit braucht, um mit dem Begriff arbeiten zu können.)

    Entweder ich verstehe dich völlig falsch, oder ich bin absolut entgegengesetzter Ansicht.

  30. #30 threepoints...
    29. Dezember 2010

    “Entweder ich verstehe dich völlig falsch, oder ich bin absolut entgegengesetzter Ansicht.”

    -> meinst du mich und meine Aussage damit?

    jedenfalls bleib ich dabei. Die Uhr misst nicht die Zeit, sondern simmuliert den Verlauf – in dem vom Menschen durch Konstruktion vorgesehenen Rahmen. Ausserdem ist auch der Zeitbegriff zwar im Alltag einleuchtend … aber trotzdem auch (nur) ein standartisiertes System zur Organisation. Wenn wir der SRT glauben wollen, dann haben wir schlichtweg nicht die Zeit, sondern viele Zeiten – je nach Position im Raum.
    Für den Menschen auf der Erde gilt natürlich die in dem Rahmen entwickelte Zeit und sein System als funktional und logisch.
    Es herrscht eben auf der Erdoberfläche fast immer die fast gleiche Gravitation, Luftdichte (Druck), Temperatur, … und auch die Geschwindigkeit der Erde selbst scheint insofern konstant zu sein, das man den “Uhrzeitbegriff” als Massstab anwenden kann.

  31. #31 MartinB
    29. Dezember 2010

    @threepoints
    Nein, das ging an FranzWappler, Tschuldigung für die Unklarheit.

    Ich denke, (Uhr)zeit ist so ein Begriff, der sich ein bisschen wie Münchhausen selbst aus dem Sumpf zieht – mit unserem normalen (unpräzisen) zeitgefühl beobachten wir, dass manche Dinge (anfangs besonders in der Astronomie) regelmäßig sind, unser Physikverständnis hilft uns, zu erkennen, dass einige Prozesse regelmäßiger sein müssen als andere (z.B. Schwingungsfrequenzen vom pendel regelmäßiger als Herzschläge), die verwenden wir dann als Uhren, um den Zeitbegriff zu präzisieren und dann noch genauer herauszubekommen, welcher Prozess nun wie regelmäßig abläuft usw. – immer im Wechselspiel zwischen Theorie (welcher Prozess sollte welchen Störungen unterliegen) und Experiment (welche Perioden messen wir).

    Aber ich sehe nicht, warum eine Uhr die Zeit nicht genauso misst, wie ein Lineal eine Länge misst – oder “simuliert” in deiner Sprechweise das Lineal auch nur die räumliche Ausdehnung?

  32. #32 Walter Orlov
    29. Dezember 2010

    Sie vereinfachen das Problem. Sie denken wohl, dass Sie mit einem Streich alle Zweifel lösen?

    Das ist aber daneben.

    Genauso gut kann Ihr Artikel so heißen:

    “Können Lorettzsche Äthertheorie und Einsteinische spezielle Relativitätstheorie Relativitätstheorie falsch sein?

    Wir sind doch erwachsene Leute und verstehen, dass die Netwtonische Theorie für die Feinabstimmungen nicht taugt.

    Die Lichtgeschwindigkeit als Grenze ist in der Äthertheorie eine Selbstverständlichkeit. Und E = mc^2 hat Hasenöhrl noch vor Einstein ohne jede Relativität hergeleitet. Ich habe in Ihrem Artikel eigentlich keinen Beleg für SRT gefunden 😉

  33. #33 MartinB
    29. Dezember 2010

    @Orlov
    Die Lichtgeschwindigkeit als Grenze ist ja nur ein Aspekt hier – der relativistische Massenzuwachs ist z.B. für die chemischen Effekte wesentlich wichtiger. Und eine Newtonsche/Ätherbasierte Quantenfeldtheorie hat bisher noch niemand in Übereinstimmung mit dem Experiment vorgerechnet.

  34. #34 threepoints...
    29. Dezember 2010

    Hier ist was interessantes zu lesen:

    https://www.faz.net/s/Rub163D8A6908014952B0FB3DB178F372D4/Doc~E3B925B89FF1746BBB495E2F96C0F5087~ATpl~Ecommon~Scontent.html

    Hier wird auch über die Umgebungszustände von Anfang und Ende des Zyklusses vom Universum “gesponnen”. Urknall und endlicher Zustand des Universums sollen als quasi gleiche Zustände interpretiert werden können – also auch physikalisch (offensichtlich).

    Ist es etwas, das man ernstnehmen kann?

  35. #35 Walter Orlov
    29. Dezember 2010

    Wenn es nicht so benannt wird, heißt es nicht, dass es anders ist…

    Auch bei QFT geht’s es vor allem den Loretz-Faktor zu berücksichtiegn. Wie entsteht eigentlich Klein-Gordon-Gleichung? – Man nimmt Energie-Impuls-Beziehung E^2 = m^2 c^4 + p^2c^2, die mit Hilfe vom Lorentz-Faktor hergeleitet wird, und wendet Ersetzungsregeln an… Man benötigt eigentlich kein Relativitätsprinzip, um alle notwendigen Korrekturen vorzunehmen 😉

  36. #36 Name auf Verlangen entfernt
    30. Dezember 2010

    @MartinB: Sie schreiben: “Aber ich sehe nicht, warum eine Uhr die Zeit nicht genauso misst, wie ein Lineal eine Länge misst – oder “simuliert” in deiner Sprechweise das Lineal auch nur die räumliche Ausdehnung?”

    @threepoints hat oben die Antwort schon gegeben: “Wenn wir der SRT glauben wollen, dann haben wir schlichtweg nicht die Zeit, sondern viele Zeiten – je nach Position im Raum.”

  37. #37 TheBug
    30. Dezember 2010

    Ein Relativitätskreationist und ein Astrologe, spannend…

    @MT: Messungen sind immer indirekt durch den Vergleich einer Größe mit einer anderen, im Falle von Zeit verwendet man die Frequenz eines (wie auch immer aufgebauten) Oszillators als Vergleichsgröße. Und es ist irrelevant für die Uhr als Messmittel, dass es verschiedene Zeiten gibt, sie misst wie vorgesehen die Zeit in dem Bezugssystem in dem sie sich befindet, so wie es irrelevant für das Lineal auf meinem Tisch ist, dass in China jemand Nudelsuppe isst.

    Leider war das jetzt wieder vergeblich, weil Markus es eh nicht versteht.

  38. #38 TheBug
    30. Dezember 2010

    Ganz vergessen:

    @Orlov: Und wie alt ist denn nun das Universum?

  39. #39 threepoints...
    30. Dezember 2010

    @Orlov: Und wie alt ist denn nun das Universum?

    -> Nach der derzeitigen Meinung … ~14 Milliarden Erdjahre…von heute

    …also nicht Jahre von vor 6 Milliarden Jahren auf der Erde oder anderswo…

    Es fehlt die Alpha-Zeit – offensichtlich…! Die Zeit nämlich, die immer gleich abläuft. Aber da wird es niemals ein Gerät geben, die es schaffen wird sie zu messen oder zu simmulieren. nach Einstein wäre sie abhängig von der Umgebung, weil sie aus Materie besteht.

    Also, wir hatten hier und anderswo schon mal die Esoterik-Frage. Ich kann den Begriff nicht besonders leiden. Er ist mitlerweile überstrapaziert von allen Seiten. Wenn etwas mal nicht deutlich berechnet werden kann – dann sei das immer gleich Esoterik…. und so! Dabei liegt das Problem dann allermeist im Kopf des Vorwerfenden.

  40. #40 threepoints...
    30. Dezember 2010

    @ TheBug· 30.12.10 · 00:38 Uhr

    Die gleiche Uhr zeigt (misst) die jeweilige Zeit für den entsprechenden Ort – so die Uhr nur am Ort selbst ist. …

    O.k., … wohl logisch, da sich Materie (woraus die Uhr bestehen muß) sich an dem jeweiligen Ort auch entsprechend verhält.

    Also ist die Uhr doch ein Messgerät?

    Nicht, wenn wir die Zeiger und das Zifferblatt zum Ablesen der zeit brauchen. Täten wir die Zeit an einem Stück Materie ablesen können, dann wiederum doch. Die Uhr selbst ist also derart ein Messgerät, so sie aus Materie besteht.
    Die Zeiger und das Zifferblatt (und alle Bauteile im innern) sind jedoch zur Simmulation der Zeit und für die optische Übersetzung für des Menschen Sinnesorgane….damit der die Information überhaupt erst erhalten kann.

    Ich glaub die Uhr ist damit durchgekaut genug…

  41. #41 MartinB
    30. Dezember 2010

    @threepoints
    Bewzüglich des FAZ-Textes: Die Ideen von Penrose sind wie üblich seeeeehr spekulativ -ernst nehmen muss man ihn aber immer, Penrose ist sicher einer der genialsten Physiker der gegenwart (der in meinen Augen Hawking deutlich in den Schatten stellt). Ich habe das Buch auf meiner Wunschliste – wenn es gut ist, schreibe ich was darüber. ich glaube, bei den englischen Scienceblogs bei “Starts with a bang” gab’s ne Rezension.

    Sind die letzten beiden Kommentare wirklich von dir? Lesen sich etwas anders als deine sonstigen texte – oder fehlte nur der Morgenkaffee 😉 ?

  42. #42 Walter Orlov
    30. Dezember 2010

    Ganz vergessen:
    @Orlov: Und wie alt ist denn nun das Universum?

    Ich erfinde keine gewaltigen Zahlen, ich rechne nur. Im Durchschnitt kann es etwa 1 Mrd. Jahre sein, also ca. um eine Großordnung weniger als laut Urknalltheorie.

  43. #43 TheBug
    30. Dezember 2010

    Danke für die prompte Bestätigung des zu erwartenden Niveaus, mehr hatte ich auch nach der Behauptung, dass Ionisationen einen Einfluss auf die Halbwertszeit hat auch nicht erwartet.

    Wer jetzt verwirrt ist wo von ich rede, der möge doch mal den Link von Orlov anschauen und sich so durchlesen was der schreibt.

  44. #44 threepoints...
    30. Dezember 2010

    MartinB· 30.12.10 · 11:31 Uhr

    Am Morgenkaffee kann es nicht gelegen haben – glaube ich. Der war immer vorhanden.

    Eine schöpferische Pause war dazwischen. Hatte andere Themen zur Denkübung…

  45. #45 erik
    30. Dezember 2010

    @threepoints..
    “Ich glaub die Uhr ist damit durchgekaut genug…”

    Nö …
    Eine Uhr benutzt Kraft (heute auch chemische, elektrische und Licht-Energie), um einen Takt nach dem anderen zu erzeugen. Einen Takt kann man auch als einen Abschnitt zwischen Anfang und Ende verstehen. Interessant das gleichzeitig Ende und Anfang zusammenfallen.
    Das oszillieren von aneinander gereihten Anfängen und deren “Enden” finde ich total geil. Ich finde auf die Kreiszahl pi geil.

    Einen Anfang hat die Uhr als Sanduhr genommen. Die Schwerkraft der Erde (das grosse “Sandkorn” im Universum) und die vielen im Glaskolben gefangenen kleinen Sandkörner. Einfach toll, wie Gross und Klein gemeinsam in einem Raum (oder soll ich sagen: Bezugssystem) miteinander “kuscheln”.
    Dann die Kuckucksuhr: Wo die Gewichte in Richtung des Erdmittelpunktes zeigen und die Zeiger uns darauf hinweisen, das jede volle Stunde Ende und Anfang auf zwei Löchern pfeifen: der erste “Kuck” ist wegsaugen (einatmen) und der andere “Kuck” ist ausstossen (ausatmen).

    Den Moment zwischen den zwei “Kuck” hat M. Bäker auf dem Logobanner seines Blogs festgehalten: Licht (Energie) tritt ein in den Raum, wird einigen Verformungen unterworfen bis es wohl wieder in einem schwarzen Loch verschwinden darf. Das geschieht dann beim zweiten “Kuck”.
    Man kann auch das Universum zwischen zwei “Kuck” setzen. Dann wäre eine Menge los zwischen Ende und Anfang. Jedes Ende trägt einen Anfang in sich.

    Beim Penrose-Artikel habe ich Gänsehaut bekommen.

    Das verblüffenste an der Uhr finde ich, das sie mich immer an die Kreiszahl pi erinnert, das jede Sekunde einem n=1 ähnlich ist und ich mit der Uhr eine vielzahl von n sammeln kann, um mich der Eulerschen Zahl e nahe zu fühlen.

    Verblüffend ist auch, das eine Uhr immer mit einer Suche nach Ordnung einhergeht. Eine Ordnung im Raum, welche sich an der Zeit der Uhr orientiert.
    Gemeinsam mit der Newtonschen Mechanik haben Menschen Ordnung in ihr Leben gebracht. Später mit der Quantenmechanik trat das Problem mit der Selbstauflösung der Uhr hinzu. Die Uhr im zu messendem Bezugssystem konnte die zu messende Information nicht mehr an den Beobachter senden.
    Hier ist das, was Zeit wir bisher genannt haben, wohl anders in seinen Eigenschaften.

    Der Raum trägt hier einen anderen Ordnungsfaktor! Jetzt orientiert sich der Raum an dem Takt der Zeit (v=c, QM, SRT und ART).
    In der Newtonschen Mechanik machen wir das genau anders herum: Der Takt der Zeit orientiert sich an den Bewegungen im Raum.

    Für mich als Menschen ist das spürbar: nach einer gewissen Zeit habe ich Hunger, dann mache ich mir Gedanken, wie ich mein täglich Brot verdienen kann und bewege mich (muss heute noch Geld holen).
    Für Goldatome trifft das nicht zu: die richten sich nach der QM: Raum orientiert sich am Takt der Zeit.
    Für einen Goldklumpen auf der Erde gilt aber: Zeit orientiert sich an der Bewegung im Raum.
    Viele Goldatome in einem Goldklumpen auf der Erde, . . . das erinnert mich an etwas. Eriks grosser Assoziationssprung: Das erinnert mich an die Sanduhr! Ein Bezugssystem, die grosse Erde und die vielen kleinen, sich bewegenden, Sandkörner, der Takt der Zeit, Anfang und Ende . . .. …

    Was ordnet Raum und Zeit so, das QM und NM sich verbinden können?
    Die Antwort: e, pi, „i“, 0 und 1.
    Die Kenner entdecken die Eulersche Formel. Eine Formel die ich in mir trage, geistig und körperlich.
    Ich bin quasi wie ein Goldklumpen, liegen gelassen auf der Erde. Nein, das ist kein schönes Bild. Besser ist: ich schürfe in meinem Körper nach „Goldklumpen“. Habe ich doch nicht nur NM an mir, sondern auch QM in mir. Jetzt muss ich nur meinen Verstand nach innen bewegen und entsprechend Licht in mein Dunkel bringen.

    Haben Licht und Gedanken gemeinsame Eigenschaften?

    Ihr seht was Zeit aus mir gemacht hat. Bedauernswert?
    Aber: ich bin hier der Einzige der sich aufgemacht hat, Neues in die Physik zu bringen!
    Ich habe Kraft für das EINE: JA!
    Und jetzt erwarte ich eure Kraft der VIELEN: NEIN!
    Schenkt mir eure Neins, seit gegen mich! Ich find das geil!

    Das machen wir dann solange bis es bei allen „klick“ macht. (eine andere Art mit Zeit umzugehen!)

  46. #46 threepoints...
    30. Dezember 2010

    apropos Sanduhr ….

    Wenn ich die (mal rein hypotetisch) etwa 1 Milliarde Jahre heftig schüttel…damit sich die Sandkörner da drinnen gegenseitig zermalmen …. “zeigt” diese Sanduhr dann noch die selbe Zeit an????

  47. #47 perk
    30. Dezember 2010

    wenn sie noch im selben bezugssystem im gleichen potential befindlich ist: joar

    aber je nach dem was für ne korngröße dann übrig bleibt und welche kräfte in der größenordnung dominieren muss man sie halt neu eichen weil sich die rieselgeschwindigkeit ändern könnte

  48. #48 threepoints...
    30. Dezember 2010

    “…. der möge doch mal den Link von Orlov anschauen …”

    -> Ich habe leider keinen finden können…

  49. #49 rolak
    30. Dezember 2010

    auch nicht den unter seinem nick, threepoints?

  50. #50 threepoints...
    30. Dezember 2010

    Ach ja,… den hatte ich schon gefunden … aber wieder vergessen. Sitze gerade an einem anderen PC.

  51. #51 threepoints...
    30. Dezember 2010

    Das mit der dunklen Materie hatte ich mir schon mal so oder so ähnlich vorgestellt. Ist denn damit der verbleib der fehlenden Massen der Galaxien aufgeklärt? Oder quakt da nur eine Ente? ie in der Zeitung (den roten vorwiegend)

  52. #52 MartinB
    30. Dezember 2010

    @threepoints
    Florian ist unser DM-Experte, guckst du hier
    https://www.scienceblogs.de/astrodicticum-simplex/tag/dunkle%20Materie

  53. #53 threepoints...
    31. Dezember 2010

    O-Ton Florian Fr. :

    “…ich bin auch kein Experte für dunkle Materie und möchte hier nicht noch mehr Verwirrung stiften als sowieso schon herrscht.”

    … in einem der Artikel.

  54. #54 threepoints...
    31. Dezember 2010

    “”Was wir wissen ist, dass ein Problem existiert: unter der Annahme das die “normale” Materie; die Materie die wir sehen, die gesamte Materie ist, die existiert lässt sich das Universum nicht korrekt beschreiben.”

    … die Materie die wir sehen, die gesamte Materie ist, die existiert …

    Und wenn es eben nicht die gesamte Materie ist? Was ja eigendlich genau so sein muß. Wie aber will man die Materie erahnen, welche man zur berechnung braucht? Hochrechnen?
    Wieviel Prozent Materie fehlen denn noch, die derzeit also dunkle sein soll?

  55. #55 MartinB
    31. Dezember 2010

    @threepoints
    Naja, Florian ist halt “unser” Sb-Experte – aber er forscht nicht selbst auf dem Gebiet.

    Ja, die wird hochgerechnet aus den gravitativen Effekten, die man sieht.
    Siehe auch bei Wikipedia
    https://de.wikipedia.org/wiki/Dunkle_Materie
    Es ist nach aktuellen Rechnungen etwa 5-6mal soviel DM wie normale Materie vorhanden.

  56. #56 noanna
    1. Januar 2011

    Um ein Haar hä#tte ich es übersehen:

    https://unglaublichkeiten.org/unglaublichkeiten/htmlphp2/u2_1138ART_SRT.html

    Kürzer geht es wohl nicht, den SRT + ART Quatsch zu widerlegen. Danke im Voraus für alle depperten Kommentare!

  57. #57 noanna
    1. Januar 2011

    “Naja, Florian ist halt “unser” Sb-Experte – aber er forscht nicht selbst auf dem Gebiet.”

    Eben, er ist ein Nachplapperer und ich habe mir glaubhaft bestätigen lassen, daß er wirklich als Nachplapperer zu keiner selbstständigen Forschung fähig ist. Von Erkenntnisfähigkeit ganz zu schweigen.

    “Es ist nach aktuellen Rechnungen etwa 5-6mal soviel DM wie normale Materie vorhanden.”

    Nur Dummis können die aktuellen “Rechnungen” glauben, weil sie vom Rechnen sowieso nichts verstehen. In aller Regel ist man mathematisch nicht einmal in der Lage, ein allgemeines Dreikörperproblem zu lösen. Natürlich können Mathematiker “Formeln” kreieren. Nur haben sie eben nichts mit der physikalischen Realität zu tun, weil sie sie nicht mehr als Zweikörperprobleme behandeln können und meist hat man es mit mehr als einem Atom zu tun, welches bereits einem Vielteilchensystem entspricht.

    Naturbeschreibung geht eben nicht mit der Mathematik von vor 300 Jahren sondern erst seit der Möglichkeiten, welche Newton und Leibniz geschaffen haben: Differentialgleichuungen.

    Und die richtig zu lösen gelingt eben erst seit dem “Computerzeitalter”, wo 5 oder gar Billionen Rechnungen je Sekunde durchgeführt werden können. Wer diese numerische Rechentechnik nicht beherrscht, ist vom wahren Erkenntnisgewinn sowieso ausgeschlossen und kann nur qualitativ babbeln und dies sowieso nur falsch. Warum? Weil eben nicht einmal erkannt wurde, daß die “schönen” Formeln Makulatur sind und grundsätzlich nicht die Realität beschreiben können. Nicht einmal annähernd.

    Wer von den hiesigen Deppen kann überhaupt noch programmieren? Und wenn einer der hiesigen Deppen das tatsächlich einmal gemacht hat, ging die Programierkunst über die zugegebenermaßen für Deppen schwierige Steuererklärung hinaus?

    Ich will nicht behaupten, daß ich intelligent bin. Wahrscheinlich sind alle Anwesenden tausendmal intelligenter als ich. Damit schließe ich sogar den strohdummen Seitenbetreiber mit ein, welcher es gewagt hat, dieses Thema überhaupt anzugehen!

    Intelligenz bedeutet Erkenntnisfähigkeit. Um diese Eigenschahaft erlangen zu können, muß jeder Glaube über Bord geworfen werden. Das bedeutet, alles, was der Logik widerspricht, darf nicht anerkannt werden.

    Nur ein Idiot hatte kann daher glauben, daß 1+1=1 ist. Nur ein IDIOT! Wer dennoch behauptet, 1 + 1 sei 1, kann also entweder ein IDIOT sein oder er verfolgt Ziele, welche der NWO dienlich sind.
    Deinen antisemitischen Mist kannst du anderswo loswerden, das ist ja widerlich.

  58. #58 TheBug
    1. Januar 2011

    noanna: “Ich will nicht behaupten, daß ich intelligent bin. Wahrscheinlich sind alle Anwesenden tausendmal intelligenter als ich.”

    Das war die einzige bisher zutreffende Aussage von noanna…

    Und in diesem Sinne nimm Dir für das neue Jahr mal vor Dich an den Spruch von Nuhr zu halten.

    Allen anderen ein frohes neues Jahr.

  59. #59 kindermund
    1. Januar 2011

    Auch hier gilt wieder dasselbe: Wer die SRT ablehnt, ist in der Pflicht, auch diese Phänomene zu erklären.

    Das ist ein irrationales Totschlagargument. Es genügt vollkommen SRT oder irgendeine andere Theorie zu widerlegen. Für die Frage ob eine Theorie zutreffend ist oder nicht, ist es belanglos, ob man eine Vorstellung hat, womit die fragliche Theorie ersetzen werden könnte.

  60. #60 Schmidts Katze
    1. Januar 2011

    Sperrt doch endlich diesen unsäglichen noanna.

    Die Protokolle der Weisen von Zion braucht auf den ScienceBlogs wohl keiner.

  61. #61 Schmidts Katze
    1. Januar 2011

    Seit dem Ende des Zweiten Weltkrieges sind die Protokolle in West- und Mitteleuropa aus dem öffentlichen Blickfeld verschwunden. Nur noch wenige deutsche Antisemiten berufen sich auf die Protokolle, wie etwa Horst Mahler. In Deutschland wird ihre Verbreitung als Volksverhetzung strafrechtlich verfolgt.

    https://de.wikipedia.org/wiki/Protokolle_der_Weisen_von_Zion#Gegenwart

  62. #62 threepoints...
    1. Januar 2011

    “Die Protokolle der Weisen von Zion braucht auf den ScienceBlogs wohl keiner.”

    -> Ich höre davon zum ersten mal – also wahrscheinlich braucht sie hier keiner.

    Oder doch…?

    Wäre Materie Einbildung … wären auch die Protokolle Einbildung….

  63. #63 threepoints...
    1. Januar 2011

    … und die SRT eben auch …

  64. #64 MartinB
    1. Januar 2011

    @alle
    Ich habe noanna versuchsweise gesperrt – unsere Sperrfunktion ist aber nicht besonders zuverlässig, also nicht wundern, falls noanna wieder auftaucht.
    Bitte noanna einfach ignorieren.

    @kindermund
    Nein, das ist kein Totschlagargument. Es zeigt, wie groß die Beweislast ist, um die SRT zu widerlegen. Gäbe es für die nur schwache Belege, dann würden schwache gegenbelege ausreichen, je stärker die Belege *für* die SRT sind, desto stärker müssen auch die Gegenbelege sein (wenn es denn welche gäbe). Das ist das berühmte “Extraordinary claims need extraordinary evidence”. Mein Post soll deutlich machen, wie außergewöhnlich die Behauptung, die SRT sei falsch, ist.

  65. #65 kindermund
    1. Januar 2011

    @ MartinB
    Mir fallen beim besten Willen keine Belge für A/SRT ein.

    Raum kann man nicht krümmen, weil er keine physischen Eigenschaften besitzt und Zeit kann man nicht dehnen, weil sie ein abstraktes Konzept ist, daß ebenfalls über keine physische Realität verfügt. Diese beiden wahren Aussagen reichen rationalen Menschen, um A/SRT zu begraben.

  66. #66 Ireneusz Cwirko
    1. Januar 2011

    Deinen antisemitischen Mist in einer Zusammenhang mit der Kritik an RT zu bringen ist doch ein durchsichtiger Manöver jegliche Kritik unglaubwürdig erscheinen zu lassen. Dass der Autor des Blogs sich solchen Mittel bedient und durch solche Provokationen von wesentlichen abzulenken zeigt nur wie verzweifelt die Lage schon für die Relativlisten ist.

  67. #67 threepoints...
    1. Januar 2011

    @ kindermund· 01.01.11 · 11:58 Uhr

    Ich wüsste vielleicht ein Szenario, in dem sich Raum Krümmen kann….

    Der nichtkrumme Raum hat keine Verbindung mit dessen Inhalt. Sondern ist ein rein theoretisch angenommenes Konstrukt. a x b x c … und aller Ablauf innerhalb der koordinaten bildet seinen eigenen Raum – der unter bestimmten Umständen dann auch krumm sein kann. Man muß wie bei der zeit derart vorgehen, dass für jede Instanz von Bedingungen ein Raum integriert werden muß Der gedachte Raum … der bedingte Raum … und am Ende könnte ein noch ganz andere Raum dastehen – etwa der Lebensraum des Menschen … der Tiere…. und endlich die Einzeller.

    Welcher Raum endlich der realste sei, dass kann wohl nicht einmal die Wissenschaft plausibel erklären und untermauern. Dazu fehlen noch einige “Räume” und deren innere Abläufe…

    Das mag sich jetzt wieder nach Esoterik anhören, aber daran ist die Wissenschaft solange selbstschuld, bis sie das Problem gelöst hat – unzweifelhaft gelöst. Solange noch dunkle Materie und Energie “erdacht” und erhofft wird, solange wird es noch sowas wie Esoterik geben….

  68. #68 kindermund
    1. Januar 2011

    @ I.C.
    MartinB hat immerhin das Format, den inhaltlichen Teil von “noannas” Beitrag stehen zu lassen und nur den seiner Meinung nach antisemitischen Part zu zensieren. Dafür gebührt ihm Respekt. Auch wenn man Wissenschaft und Politik sicher nicht von einander trennen kann, muss man falschen, eindimensionalen Feindbildern aus vergangenen Zeiten keine Öffentlichkeit gewähren.

  69. #69 MartinB
    1. Januar 2011

    @kindermund
    “Diese beiden wahren Aussagen reichen rationalen Menschen, um A/SRT zu begraben.”
    Das Argument “kann ich mir nicht vorstellen, deswegen muss es falsch sein” ist nun wirklich ziemlich schwach und hat mit Physik nichts zu tun. Die Eigenschaften der Realität (einschließlich Raum und Zeit) können nicht durch bloßes Nachdenken erfasst werden, dazu braucht es Experimente, und die gibt es zuhauf (siehe oben).

    @IC
    Das ist nun wirklich lächerlich: noanna postet hier antisemitischen Mist, und das soll meine Schuld sein – aber wahrscheinlich ist das wieder so eine IC-Verschwörungstheorie, nach der der Blogautor selbst solche Kommentare verfasst. Im übrigen, wolltest du auf meinem Blog nicht nie wieder auftauchen?

    @alle
    Bitte nicht weiter über noanna diskutieren, das ist doch nur, was er erreichen will. Damit müssen wir unsere Zeit nicht verschwenden.

  70. #70 MartinB
    1. Januar 2011

    @threepoints, kindermund
    Falls ihr Probleme mit der Vorstellung eines gekrümmten Raumes habt, empfehle ich die Feynman Lectures Band II, Kap. 42, dort gibt es eine sehr schöne Analogie mit einer heißen Herdplatte. Aber schon prinzipiell ist mir nicht klar, warum die Raumzeitstruktur nun mathematisch ausgerechnet durch eine ungekrümmte Mannigfaltigkeit beschrieben werden soll.

  71. #71 Ireneusz Cwirko
    1. Januar 2011

    @ MartinB “wolltest du auf meinem Blog nicht nie wieder auftauchen?”

    tue ich auch ungern, aber ich kann nicht zusehen mit welchen Methoden hier die Wahrheit vergewaltigt wird.

    @ MartinB “Die Eigenschaften der Realität (einschließlich Raum und Zeit) können nicht durch bloßes Nachdenken erfasst werden”

    Natürlich können sie es wenn man bloß nicht so stur diese primitive Sprache der Mathematik verwenden würde.
    In der Tat ist Raum und Zeit der Schlüssel, der uns der Zugang zu Realität öffnet.
    Das Problem der Physik besteht daran, dass sie die Beständigkeit der Ereignissen voraussetzt. In Wirklichkeit ist das Universum nicht so ein System, sondern es erschafft sich immer wieder aufs Neue. Mit jeder Oszillation des Gravitativen Hintergrunds, entsteht für unglaublich kurze Zeit ein Universum als Ganzes und dann gleich bei der nächsten ein neues Gestalt zu nehmen. Die Änderungen sind klein aber in der Summe ergibt sich in unseren Bewusstsein ein Gefühl von Zeit und Bewegung.

    Und die Physiker versuchen mit ihren Gleichungen eine Verbindung zwischen einzelnen Erscheinungsformen der Wirklichkeit zu stellen wo eigentlich keine gibt’s.

  72. #72 threepoints...
    1. Januar 2011

    q MartinB· 01.01.11 · 12:35 Uhr

    “Die Eigenschaften der Realität (einschließlich Raum und Zeit) können nicht durch bloßes Nachdenken erfasst werden, dazu braucht es Experimente,…”

    Na, wenn er das so genau weiss, dann haben wir wohl das leistungsfähigste Gehirn dieser Erde ausgerechnet hier auf diesem Portal … und sogar deutsprachig….!

    Kleine Parodie. Aber ebenso, wie die Relativitätstheorie ist auch die These der Unmöglichkeit vom bloßen Nachdenken zu Ergebnissen zu kommen über einen komplexen Zusammenhang nur die derzeit vorstellbare These einer Problemstellung. Es kann einfach nicht derart erklärt werden, dass nicht jemand doch noch die Abläufe im Raum durch reines Denken vorhersagen kann.

    Dieses Nachdenken gibt aber wohl nur eine Tendenz aus – keine präzise Vorhersage, wozu dann doch die Experimente und Berechnungen nötig wären.

    Aber vor dem Experiment hat der Geist erstmal wildes Brainstorming vorangestellt – aufgrund jede Berechnung seine Struktur bekommt.

    Ich bröuchte übrigens einen Astrophysiker, einen Banalphysiker, einen Mathematiker …. um meine These vom Ablauf im Raum zu präzisieren…

    (habe ich mich gerade selbst widersprochen?)

  73. #73 MartinB
    1. Januar 2011

    @threepoints
    Ich habe ja nicht gesagt, dass es unmöglich ist, durch bloßes nachdenken zu Ergebnissen zu kommen. Aber ohne Experimente wird man nie wissen, ob diese Ergebnisse etwas mit der Realität zu tun haben oder nicht – es ist ja nicht selbstverständlich, dass die Welt so funktioniert, wie unser gehirn es uns suggeriert.
    Dass vor dem Experiment meist auch eine theoretische Überlegung stattfindet (die mir z.B. sagt, dass es für mein Gravitationsexperiment egal ist, ob die Kugel rot oder blau ist) ist natürlich richtig – wenn diese theoretische Überlegung aber mit der Realität nicht zusammenpasst, dann wird man das im Experiment an seltsamen Widersprüchlichkeiten merken. So geschehen im Michelson-Morley-Experiment oder in der Quantenmechanik. Dann geht man zurück und stellt ne neue Theorie auf. (das war jetzt der cartoonhafte Abriss von Poincares Wissenschaftstheorie.)

    Aber “rein durch Nachdenken” ergeht es einem wie Kant, der aus a priori-Überlegungen ableiten zu können glaubte, dass Menschen den Raum und die Zeit in Newtonscher Form denken müssen. (Wobei schon der absolute Newtonsche Raum in Verbindung mit der Galilei-Invarianz ein Problem ist, wie Penrose sehr schön in “road to reality” darstellt.)

    Ansonsten scheint mir, dass du in deinem ARgument oben verschiedene Raumbegriffe synonym verwendest, die nicht viel (außer einer unterliegenden mathematischen Struktur) miteinander zu tun haben.

  74. #74 TheBug
    1. Januar 2011

    Interessant was so alles zum neuen Jahr aus seinen Löchern krabbelt.

    Widerlegen einer Theorie besteht nicht darin festzustellen, dass man sie nicht begreift, oder entgegen aller Realität zu behaupten es gäbe keinerlei Belege für sie. Martin hat in diesem Artikel eine schöne Auflistung von einigen Belegen für die SRT aufgestellt, hat von Euch Komikern das keiner gelesen? Und das waren nur ein paar Beispiele, mit den Belegen für die SRT kann man mehrere Bücher füllen.

    Auch unglaublich finde ich die Vorstellung, dass hier einige wissenschaftlich unterbelichtete Forenposter glauben schlauer zu sein als die ganze Wissenschaftskommunity. Wie bitte sollte eine Verschwörung in der Lage sein über fast ein Jahrhundert eine falsche Theorie als richtig auszugeben? Hat einer von Euch Flachweltlern eigentlich eine Idee davon was es für ein enormes Prestige für einen Forscher oder eine Forschergruppe wäre etwas zu finden, das eine so gut abgesicherte Theorie wie die Relativitätstheorie in Frage stellt?

    Das wäre die Entdeckung von etwas wirklich Neuem, also genau das was einen Forscher antreibt.

    In diesem SInne mein Wunsch für das neue Jahr: Lest mal weniger Verschwörungstheorien und dafür versucht mal echte Theorien zu verstehen, bevor Ih über diese Mist schreibt.

  75. #75 kindermund
    1. Januar 2011

    @ MartinB

    Das Argument “kann ich mir nicht vorstellen, deswegen muss es falsch sein” ist nun wirklich ziemlich schwach und hat mit Physik nichts zu tun.

    Völlig falsch. Vorstellen kann man sich natürlich alles mögliche, daß macht Spaß und wenn man gut darin ist, verfasst man vielleicht eines Tages Märchenbücher.

    So könnte ich zum Beispiel die Geschichte vom bösen Zauberer erfinden, dem die Freiheit aller Dinge und Lebewesen einfach dorthin zuschweben, wo es ihnen gefällt, nicht ertrug und eines Tages einen Zauber auf die Welt anwandte, der allen Dingen und Lebewesen eine Schwere gab, Kraft derer sie fürderhin zu einander hin und angezogen wurden. Alle Ihre Experimente würden die selben Ergebnisse zeitigen, die Sie kennen und doch wäre die Erklärung ein ganz andere.

    Wenn Sie Wissenschaft betreiben wollen, reicht es nicht Experimente zu machen und dann eine Behauptung aufzustellen, die die Ergebnisse erklären soll. Bezogen auf den gekrümmten Raum, müssen Sie diesen ZEIGEN und ERKLÄREN was ein gekrümmter Raum ist. Wenn Sie das nicht können, dann betreiben Sie keine Wissenschaft.

    Tatsache ist aber, daß die Idee eines “gekrümmten Raumes”, entgegen Ihrer Behauptung, nichts mit Physik zu tun hat. Raum hat keine Eigenschaften, er ist das Gegenteil von Objekt, etwas, das Eigenschaften hat. Sie können nichts “krümmen”, das eigenschaftslos ist!

    Wollte man die Idee des gekrümmten Raumes in ein wissenschaftlich-rationales Bild übersetzen, daß dazu noch etwas mit Physik zu tun hätte, dann könnten Sie nur den Begriff “Raum” durch “Volumen” und “Krümmung” durch “Dichte” ersetzen. Alles andere wäre Esoterik.

  76. #76 MartinB
    1. Januar 2011

    @kindermund
    Die Geschichte mit dem Zauberer erinnert an meinen Schamanen…
    “Raum hat keine Eigenschaften.”
    Aha. Er hat also zum Beispiel keine Dimension? Und kann nicht gekrümmt sein? Warum nicht? Woher wissen Sie das? Nur weil Sie sich das so vorstellen (oder nicht anders vorstellen können) heißt das nicht, dass der physikalische Raum nicht so funktioniert. Sie behaupten das einfach beleglos. Und mathematisch ist ein “Raum” eine Mannigfaltigkeit und hat ne Menge Eigenschaften… Im übrigen empfehle ich nochmal die Lektüre von “road to reality” Kap. 17, wo deutlich wird, dass auch der Newtonsche Raum eigentlich schon “ziemlich” relativ ist – mit Galilei-Invarianz gibt es keinen “absoluten Raum”.

    Die physikalische Weltbeschreibung funktioniert erwiesenermaßen am besten mit der Vorstellung eines gekrümmten Raumes. Wer das anzweifelt, muss eine andere Weltbeschreibung vorlegen, die die Beobachtungen ebensogut oder besser beschreibt.

    Im übrigen sehe ich nach wie vor nicht, warum eine nichtgekrümmte Mannigfaltigkeit so unglaublich viel besser sein soll als eine gekrümmte.

  77. #77 threepoints...
    1. Januar 2011

    Poincares Wissenschaftstheorie N=NP ? Versuch und Irrtum oder geniale Übersprunhandlung?

    Und Kant konnte solche Ableitung wohl noch machen. Zu seiner Zeit gab es scheinbar noch keinen Bedarf an Genauigkeit, wie sie heute erforderlich ist.

    Kants erhobener Zeigefinger entstand also im Prinzip des Bewusstseins durch funktion. Newton gab eine Theorie vor, die sich damals gut genug in alle Anforderungen fügen liess und wenig Mangel blieb zurück. Die Funktion schien zu reichen und das Bewusstsein (hier von Kant) gab es wieder in Form von seiner Einschätzung und Würdigung. Sie konnten eben nicht auf den Mond fliegen…! Hatten also keinen Bedarf an Methoden – nur einen Traum vielleicht.

    Und meine “Räume” sind von Funktion abhängig. Der sterilste Raum ist der ideale Raum (unbedingt – ohne Abhängigkeit)- den ich (und andere) als generelle Struktur über alle weiteren Räume legen, damit der Faden nicht verloren wird.
    Im Grunde endet meine Vorstellung vom Inhalt des Raumes in Form einer Matruska (Puppe in Puppe) … also Raum in Raum (und vielleicht noch mehr Räume in jeweils einem Raum).
    Wobei eben alle anderen Räume schon nicht mehr dem Idealraum entsprechen und ist bedingt durch die Funktion innerhalb der Räume. “Raum” ist allerdings vielleicht nicht der geeignetste Begriff… hat jemand einen anderen?

    @ TheBug· 01.01.11 · 13:16 Uhr

    Die Dummheit der Dummen ist nicht des Dummen Schuld. Schuldig sind sie höchstens, wenn sie diese Dummheit akzeptieren (darauf muß der aber erstmal kommen …). Und der Bauer frisst halt nur, was er kennt…. Wobei man ursache und Wirkung nicht voreingenommen berechnen sollte.

    Und da hilft Schwarmintelligenz (in anlehnung an die SRT-Gläubigen Problematik) nur bedingt. Weil doch auch im Schwarm allerhand Fehlleistung enthalten sein kann, deren Wirkung noch nicht aufgefallen ist. Mich würde es eigendlich auch wundern, wenn 100 Jahre Teilchen- und sonstige hightech-Physik noch nicht darauf gekommen sind…. Aber Zeit für Wunder ist jederzeit….

  78. #78 threepoints...
    1. Januar 2011

    “Raum” durch “Volumen” und “Krümmung” durch “Dichte”

    -> genau so….im reduziertestem Sinne. Erklärt aber schon viel.

  79. #79 threepoints...
    1. Januar 2011

    physikalische Raum = Raum durch und mit Funktion

    absoluter Raum = Raum ohne Eigenschaften (ausser vielleicht absolute Maße) = das Spielfeld der Theorie und eben der physikalischen Räumen.

    O.k., … so ist der Raum gut aufgeteilt.

  80. #80 MartinB
    1. Januar 2011

    @threepoints
    Habe ehrlich gesagt nichts von dem Kommentar verstanden. Raum im Raum? “Steriler” Raum? Idealraum? Was soll das alles sein? Wie ist das definiert? Wie misst man das? Welche Experimente sprechen dafür? Welche Folgerungen kann man daraus ableiten?
    Und dann wieder das beliebte Wort “SRT-Gläubiger”, au weia.

  81. #81 threepoints...
    1. Januar 2011

    3D Manigfaltigkeit = Raum im Raum prinzip…

    Aber die generelle Herleitung aus der Manigfaltigkeit führt eigendlich nur zu Idealergebnissen. So symmetrisch ist der Raum nicht.

  82. #82 threepoints...
    1. Januar 2011

    So symmetrisch ist der Raum nicht.

    -> der physikalische Raum ist nicht so symmetrisch – er besitz aufgrund von Bedingungen nur die Tendenz dahin.

  83. #83 threepoints...
    1. Januar 2011

    Gläubigen Problematik

    Schwarmverhalten … was andere machen ist gut und billig. ich mache das auch.

    Und so kann man schon mal ein Leben lang damit verbringen, sein Weltbild (oder das des Schwarmes) nicht vervollständigen zu können.

    Was mich immer schon störte, war die Einsicht, dass der Fischschwarm nur gejagt wird, weil er im schwarm schwimmt…! Kein Delphin würde sich die Mühe machen, den einzelnen Fisch zu jagen….

  84. #84 MartinB
    1. Januar 2011

    @threepoints
    “Was mich immer schon störte, war die Einsicht, dass der Fischschwarm nur gejagt wird, weil er im schwarm schwimmt…! Kein Delphin würde sich die Mühe machen, den einzelnen Fisch zu jagen….”
    Wenn das so wäre, hätte sich dieses Verhalten evolutionär nicht durchgesetzt.

  85. #85 threepoints...
    1. Januar 2011

    genau … worauf ich mir aber dann doch meine Schlüsse draus ziehe… und beim Anti-Zyklus lande….je nach Begebenheit….!

  86. #86 kindermund
    1. Januar 2011

    @ MartinB

    Er hat also zum Beispiel keine Dimension? Und kann nicht gekrümmt sein? Warum nicht? Woher wissen Sie das? Nur weil Sie sich das so vorstellen (oder nicht anders vorstellen können) heißt das nicht, dass der physikalische Raum nicht so funktioniert. Sie behaupten das einfach beleglos.

    Sie wissen nicht was Raum bedeutet, damit stehen Sie nicht allein. Raum “ist” keine Mannigfaltigkeit, Mathematiker projizieren das Konzept der Mannigfaltigkeit auf das Gegenteil von Objekt – was nicht weiter erstaunt, denn Mathematiker wissen bekanntlich auch nicht was ein Objekt, im wissenschaftlichen Sinne, ist. Das darf man den Mathematikern nicht vorwerfen, schließlich sind sie keine Wissenschaftler.

    “Krümmung” ist eine Eigenschaft, Raum hat keine Eigenschaften. Wenn etwas Eigenschaften hat, dann ist es, per Definition ein Objekt. Wenn sich Mathematiker entschließen Raum als Objekt zu behandeln, dann ist das deren Problem, für die Wissenschaft ist das belanglos.

  87. #87 MartinB
    1. Januar 2011

    @kindermund
    Was ist denn dann Raum? Ein “irgendwas” ohne jede Eigenschaft?
    Und nach wie vor, wo sind die Belege für die Behauptungen?

  88. #88 threepoints...
    1. Januar 2011

    Einen aktiven physikalishcen Raum könnte man in jeweils einer Galaxie erkennen wollen oder auch gut … einen Nebel/Gaswolken ohne Sterne als inaktiven physikalischen Raum. Das Universum als Raum mit den Räumen der Masseansammlungen (Galaxien, Nebel, Gaswolken, Stäuben…). Je nach Konzentration und Anteilen in der Zusammensetzung entsteht ein Raum mit Eigenschaften. Diese Räume können sich gegenseitig beeinflussen. Etwa, wenn Nebel auf Galaxien stoßen – ihnen so nahe kommen, dass eine kollision … oder eher vereinigung unausweichlich wird. Diese Eigenschaften werden sich je nach Fortschritt unterscheiden – aber als Zyklus nur aufgrund der Zusammensetzung und äußeren Bedingungen. D.h., dass eine Galaxie als Idealablauf immer den selben Zyklus durchläuft. Aber durch die äußeren Einflüsse und die Zusammensetzung davon abweicht und die Zyklen nicht “Regelgerecht” (Idealablauf ) durchlaufen muß.

    Das Universum als Raum mit den Räumen der Masseansammlungen/konzentration…
    Dabei ist das “Universum” nicht der absolute Raum, sondern wieder ein physikalischer Raum. Der absolute Raum befindet sich eh nur als Grundlage in der Theorie. … als Matrix zur Beobachtung und geometrischen Auswertung. Schlichtweg also zur Orientierung für den Menschlichen Sinn/Geist – der halt diese koordinaten benötigt. Was aber vielleicht verwirrend wriken wird, wenn sich durch Gravitation und Krümmung eh keine gerade durch den Raum ziehen lässt. Aber diese koordinaten sollen auch von bedingungen unabhängig sein, damit eine Matrix vorhanden ist.

    Aufgrund der Idee der supermassiven schwarzen Löcher und deren angenommenen Raumkrümmung muß angenommen werden, dass es sich hier um unterschiedliche Räume handelt … mit unterschiedlichen Eigenschaften. Der eine Raum basiert auf (oder entstand aus) Materie und der andere basiert auf die Hintergrundstrahlung – also permanente hochfrequente Schwingungen). Beide sind zwar in Kontakt, aber die Materie entwickelt ihre eigenen Abgrenzungen durch Magnetfelder.
    Es bestehen also eindeutige Medium-Unterschiede… also müssen unterschiedliche physikalische Räume vorhanden sein.

    Belege finden sich reichlich, wenn man nur hinschaut. Auf zum Beispiel die verfügbaren visuellen Abbildungen der Objekte im Raum. die darauf erkennbare Energieverteilung und umwandlungsrichtung. Wenn ein Zyklus erkennbar ist, dann
    ist die Stuktur bekannt.

  89. #89 kindermund
    1. Januar 2011

    @ MartinB
    Welchen Teil von “Wenn etwas Eigenschaften hat, dann ist es, per Definition, ein Objekt.” verstehen Sie nicht? “Raum” ist das, wovon sich Objekte unterscheiden. Was wollen Sie da belegen?

  90. #90 Niels
    1. Januar 2011

    @kindermund
    Was ist der Unterschied zwischen “etwas ohne Eigenschaften” und “Nichts”?
    Ist nach deiner Definition eine Eigenschaft ein Raum oder ein Objekt?

  91. #91 MartinB
    1. Januar 2011

    @kindermund
    Sind Sie irgendwie mit Ockham verwandt? Der hat hier vor einiger zeit dasselbe behauptet.
    Sie können das ja von mir aus so definieren. Sie können auch definieren: “Ein Tisch ist ein Fahrzeug mit einem bett drin”. Sie dürfen sich nur nicht wundern, wenn andere Leute Sie nicht verstehen, weil Sie der einzige sind, der diese Definition verwendet. Hat der Raum bei Ihnen drei Dimensionen? Wenn ja, dann hat er damit eine Eigenschaft, wenn nein, dann ist es nicht der Raum, über den wir normalerweise sprechen, wenn wir “Raum” sagen.
    Brauchen wir aber nicht weiter zu diskutieren, da der Streit um Definitionen wenig inhaltlichen Nährwert hat.

    @threepoints
    Ich verstehe kein Wort. Warum soll ein Raum “auf Hintergrundstrahlung basieren”? Warum und wie sollen Räume wechselwirken? Was soll das heißen? Das ist alles so unkonkret, dass es beliebig ist, mit Physik hat das nur sehr wenig zu tun.

  92. #92 H.M.Voynich
    1. Januar 2011

    @kindermund:
    “Was wollen Sie da belegen?”

    Zum Beispiel daß irgendjemand außer Ihnen diese obskure Definition teilt?

  93. #93 threepoints...
    1. Januar 2011

    @ kindermund· 01.01.11 · 18:55 Uhr

    Der versteht das schon noch. Nur will er es immer genau wissen und nicht seine eigene Vorstellung davon entwickeln müssen.

    Aber du verstehst mich wahrscheinlich auch etwas falsch. Das Objekt/die Struktur nimmt Raum ein und bildet ihn damit.Oder das Objekt ist der Raum, so man eine zusammenhängende Struktur erkennen kann. Etwa eine Galaxie. Das Sternensystem nimmt wiederum einen Raum im Raum ein…nämlich innerhalb einer Galaxie. Und der Raum Galaxie befindet sich im Raum “Universum” und belegt darin einen Teil des Raumes. Das Objekt “Galaxie” ist also für mich ein physikalischer Raum in einem physikalischen Raum (das Universum – der Raum zwischen den Galaxien).

    Die Hintergrundstrahlung bildet den Raum. Ohne diese Strahlung wäre er nicht vorhanden. Auch der Rest wohl nicht.
    Ach ja, … keine Physik…. und ich dachte, es wäre welche. Na dann kann ich ja enttäuscht nen Schal stricken gehen. …
    Ich habe auch keine Lust jetzt noch Physik demokratisch zu deffinieren.

  94. #94 MartinB
    1. Januar 2011

    @threepoints
    Nein, tut mir leid, so ist es keine Physik. Das Erstellen von Hypothesen allein reicht nicht. Notwendig ist, aus den Hypothesen Vorhersagen (quantitativ) abzuleiten und diese zu überprüfen.
    Eine einfache Frage z.B.: Wenn ich einen Raum von der Hintergrundstrahlung isoliere (ich expandiere einen Ballon mit für die HG-Strahlung undurchsichtigen Wänden), was passiert dann? Ist drinnen dann kein Raum? oder wie? Was soll es konkret (d.h. beobachtbar) heißen, dass die HG-Strahlung den Raum bildet?

  95. #95 threepoints...
    1. Januar 2011

    Die HG-Strahlung … meint man derart hineinzudenken, dass es eher nach “füllen” scheint, als den Raum bilden. Im Zusammenhang mit dem Universum aber kann man die HG-Strahlung nur entnehmen, wenn man sie ersetzt. Also derart relevant ist, dass sie einen Raum bildet und nicht nur füllt.

    Mit dem Ballon ist es ebenso. … nur dass der die Hülle braucht und eine “Füllung” hat. (aufgrund der Atmosphäre wird die Hülle benötigt). Die Füllung schafft sich hier den Raum. Ohne Füllung fiele der Ballon zusammen und vom Himmel.

    Die Galaxie als Raum täte sich eine Hülle etwa durch Magnetfeld aufbauen. … insofern vergleichbar. Aber es ginge reinst theoretisch auch ohne dem Feld. Die feste Materie (hier Planet) ist sich Hülle genug. Da sie sich aber die Materie in Galaxien ansammelt, ist also diese Ansammlung einer Galaxie mit ihrem Feld ein Raum – als zusammenhängende Struktur mit Funktion. Je größer die Masse der Galaxie, desto größer der Raum den sie einnimmt – und nicht nur mit Materie, sondern auch durch ihr Feld – weiter als Materie in Form von Gasen, Stäuben und andere Materie wie Planeten Sterne usw…im Raum sich befinden. Ohne dem kommt der Raum nicht aus… im Universum.

    In der Atmosphäre der Erde ist die Zusammensetzung durch Gase (Medium) und einfallende Strahlung also ein Grenzfall mit sich überlagernden Räumen. Eigendlich aber bis zu der Grenze der Atmosphäre aber zur Struktur Erde gehörend – als Teil des Sonnensystems und Teil der Galaxie und Teil des Universums.

    So unklar ist das doch gar nicht.

  96. #96 threepoints...
    1. Januar 2011

    “Notwendig ist, aus den Hypothesen Vorhersagen (quantitativ) abzuleiten und diese zu überprüfen.”
    -> Dann werd ich mal eben in den Keller gehen und die abgeleiteten Vorhersagen im Experiement bestätigen.

    Bin gleich wieder da….

  97. #97 Ireneusz Cwirko
    1. Januar 2011

    Sich eine Frage nach dem Anfang des Universums zu stellen, ist für die Physiker eine Herausforderung, den sich fast jeder stellen musste, obwohl jeder Vorschlag in diese Richtung nicht mehr als eine Hypothese bleiben wird, schon aus dem Grund, dass es kaum Möglichkeiten gibt dies experimentell zu falsifizieren.

    Auch der normale Mensch stellt sich diese Frage manchmal. Ich konnte mich auch nicht zurückhalten über diese Frage nachzudenken, wollte aber nicht auf die Modelle der Wissenschaft zurückgreifen. Die Gründe waren zuerst ästhetischer Natur, weil ich in der Physik eine einheitliche Linie bei der Erklärung der physikalischen Phänomene vermisse.

    Wie bei jeder Theorie beginnt dass mir der Annahmen die einen Gerüst bilden auf dem weitere Schlussfolgerungen aufbauen.

    Zu meiner Grundthesen gehörte eine Feststellung, dass das Universum dreidimensional ist und schon immer war.
    Wenn wir diese These konsequent weiter verfolgen, dann müssen auch alle weiteren physikalischen Eigenschaften nur die Folge der Dreidimensioalität des Raumes sein.

    Entsprechend müssen wir Schlussfolgern dass:

    Die Materie nur eine andere Form des Raumes darstellt.

    Die Zeit keine physikalische Größe darstelle sondern nur eine Folge der unwiderruflichen Veränderungen der Oszillationen des Raumes ist.

    Die Naturkräfte keine elementare Elemente darstellen, sondern nur die folge der Modulation des Raumes sind.

    Diese Verengung der Freiheitsgrade der Theorie und weitgehender Verzicht auf die Nutzung der freien Parameter scheint nur auf ersten Blick die Gestaltung einer Theorie zu erschweren, in Wirklichkeit hat den Prozess sogar erleichtert.

    Die vollständige Beschreibung der Theorie würde die Grenzen des Blogs sprengen deswegen nur eine kurze Skizze. Den Weg den ich gegangen bin kann man auf meiner Internetseite nachvollziehen, wo ich meine Theorie seit zwei Jahren weiterentwickle.

    Die Geschichte des Universums hat kein Anfang und kein Ende. Es ist eine unendliche Schleife zwischen zwei Extremen, Einem Universum, dass durch die Materie beherrscht ist und praktisch ein einziges Materiekörper darstellte, den Universumskristall, und einem Universum, dass fast nur aus dem Raum besteht und Materie eine Randerscheinung darstellte

    In der ersten Form des Universums die Grundelemente des Raumes, die Vakuolen, fast ausschließlich in der gebundenen Form einzutreffen sind also in Form der Grundelemente der Materie. Die Materie ist aber nicht gleichmäßig verteilt sondern konzentriert sich zu den Materiekörpern die man auch Schwarze Löcher nennen kann, obwohl die der Definition nicht entsprechen.

    In der zweiten Form besteht das Universum fast ausschlie0lich aus den Vakuolen, ist also praktisch nur ein leeres Raum.
    Ein Übergang von einem Raumuniversum zu einem Universumskristall verläuft langsam und ist gekennzeichnet durch die Schöpfung der Materie in dem die Vakuolen sich miteinander verbinden und die Grundelemente der Materie bilden.

    Ein Übergang zwischen einem Universumskristall zu einem Raumuniversum hat aber ein explosiver Charakter in Folge dessen es zu dem Zerfall der Materie auf die Grundelemente des Raumes, die Vakuolen kommt.

    Die Ursache für diesen Prozess, den ich als Gravitative Entkoppelung bezeichnet habe, ist mit der Dreidimensionalität der Oszillationen der Vakuolen verbunden
    Auch wen sie gebunden sind (und die Materie bilden) bleiben sie nicht statisch, sondern oszillieren sie weiter, es Vergrößert sich nur die Frequenz der Oszillationen, die aber auch weiter von der Frequenz der nicht gebundenen Vakuolen abhängig bleibt.
    Diese alles durchdringende Oszillation des Raumes habe ich die Bezeichnung Gravitative Hintergrund gegeben.

    In der Form von einem materiellen Universum ist die Mehrzahl der Vakuolen in der Form von Materie gebunden. Die Materie ist dann als Materiekörper recht chaotisch in dem restlichen Raum verteilt.

    Die Materiekörper selbst modulieren die Raumoszillationen und verstärken sie Lokal. Je mehr Materie entsteht um so kleiner ist der Raum so dass nach und nach die lokalen Modulationen der Materiekörper beginnen aufeinander zu wirken und die Materie beginnt sich neu in dem Raum zu ordnen.

    Die Meteriekörper, versuchen dann solche Stellung in Raum zu nehmen wo die Gravitative Wirkung der benachbarten Körper möglichst symmetrisch ist.

    Mit der Zeit nehmen also die Materiekörper eine regelmäßige Struktur die einem Kristall nicht unähnlich ist.
    In dieser Form verliert die Gravitative Wirkung die Bedeutung und wird von Elektromagnetismus ersetzt.

    Unser Kristalluniversum verändert sich aber weiter und nimmt nach und nach immer perfekteren Gestalt in dem die Materialkörper einer absolut regelmäßigen Verteilung in dem Raum sich nähern.

    In dem Moment wo eine absolute Perfektion fast erreicht wurde, endet auch materielle Universum. Auf Grund von gegenseitigen konstruktiven Interferenz der Oszillationen des Raumes in dem Perfekten Kristall kommt es zu einer solchen Verstärkung, dass eine so genannte Interferenzkatastrophe entsteht und die Materie auf die Bestandsteile. die freien Vakuolen zerlegt wird. In einem einzigen Augenblick und einer gewaltigen Explosion endet materielle Universum und beginnt das Raumuniversum

    Der Zyklus beginnt von Vorne.

  98. #98 MartinB
    1. Januar 2011

    @threepoints
    “So unklar ist das doch gar nicht.”
    Doch. Völlig.
    Wenn zum Beispiel die HG-Strahlung “den Raum erzeugt”, gilt das für jede elektromagnetische Strahlung? Wenn nein, warum nicht? Wenn ja, was bestimmt, ein wie großes Raumvolumen durch wie viel Strahlung erzeugt wird? Wie entsteht der Raum? Was passiert, wenn ich HG-Strahlung mit einem Spiegel oder einer Linse irgendwohin bündele? Entsteht dann dort “mehr Raum”? Wenn nein, warum nicht? Wenn ja, wie genau?

    Es ist vollkommen legitim, sich solche Ideen zu überlegen – habe ich schon massenhaft gemacht, mit verrücktesten Theorien, wie die Raumzeit funktioniert oder mit Wahrscheinlichkeit als 5. Dimension oder oder oder. Solche Ideen zu erzeugen ist leicht. Das siehst du schon daran, wieviele Leute hier in den Kommentaren auftauchen, beim einen ist alles Raum, beim anderen sind es Raumoszillationen, beim dritten ist alles irgendwie Bewusstsein usw.

    Dann aber beginnt die Physik erst: Fasse deine Idee begrifflich so scharf, wie es geht. Und jetzt leite aus ihr Folgerungen ab, zunächst auch gern qualitativ. Also müsstest du dich fragen, was die ganzen Räume, von denen du sprichst, an messbaren Auswirkungen haben müssten. Was passiert, wenn sich zwei Räume “treffen”? Wird der Raum dann an dieser Stelle “größer”? Was pasiert, wenn man einen Lichtstrahl durchschickt – muss der dann eine längere Strecke zurücklegen? Und so weiter und so weiter.

    Und wenn das alles qualitativ sinnvoll erscheint, dann musst du das gleiche Spiel nochmal quantitativ spielen. Und erst wenn auch quantitative Vorhersagen mit dem beobachteten Universum zusammenpassen, dann musst du noch Vorhersagen machen, die mit der bisherigen Physik nicht übereinstimmen und dann hast du eine physikalische Hypothese.

    @IC
    Das kann man doch alles auf deinem Blog nachlesen, wenn man will. Warum postest du das hier? Ist schon ziemlich seltsam – einerseits erklärst du alle hier zu Deppen, andererseits willst du anscheinend gerade diese Deppen von deinen Hirngespinsten überzeugen. Nächstes mal setze doch einfach einen Link auf deine Homepage.

  99. #99 threepoints...
    1. Januar 2011

    “Dann aber beginnt die Physik erst:…”

    O.k., … soweit kann ich folgen. Allerdings folge ich eh schon der derzeit erklärten Physik – und versuche die noch unklaren Beobachtungen oder Hochrechnungen mit Brücken zu versehen. Etwa die Expansion des Universums. …

    Mit der HG-Strahlung ist es so eine Sache.Es könnte tatsächlich für alle elektromagnetischen Wellen gelten. Dort wo sie vorhanden sind, wird die Umgebung Beeinflusst – also ein zusätzlicher Raum geschaffen.
    um hier ein paar beweise zu erhalten, müssten Experimente im Raum gemachtwerden. Die SSimmulation auf der Erde macht nur sehr bedingt sinn – denn wir haben hier ja Gravitation. Aber mir reicht schon die Tatsache, das Plasmen in Feldern eingeschlossen werden können. Also grenzen Felder einen Raum ab (könnten). Das Plasma wird durch ein Feld von der Umgebung getrennt. Die Galaxie trennt sich durch ein riesiges Feld von den Nachbargalaxien ab und dazwischen bleibt noch etwas Raum in Überlagerung mit den Galaxien für die HG-Strahlung. Dabei sammelt sich die HG-Str. wie in einer Senke in den dazu geeigneten Bereichen zwischen den Galaxien – so, dass an der Stelle der weitesten Entfernung zu allen Galaxien wahrscheinlich die höchsten HG-strahlungswerte (oder niedrigste Strahlungstemperatur…?)

    Wo und wie komme ich an Unterstützung ran? Eine Berechnung unter Anwendung der entsprechenden Formeln werde ich im Leben nicht hinbekommen. Also scheint es aussichtslos, …. oder weiss jemand einen Professor mit Einrichtung, der gerade Forschungskapazitäten frei hat?

    Jetzt fängt er wirklich an zu spinnen….

  100. #100 TheBug
    2. Januar 2011

    WTF? Alles voller Crank-Physik.

    Die oberste Anforderung an eine These um diese überhaupt als solche zu bezeichnen, ist, dass sie nicht der Realität widersprechen darf. Schnapsideen, die schon bei der rein theoretischen Überprüfung nicht mit bekannten und belegten Experimenten übereinstimmen kann man direkt in der Kneipe lassen.

    @threepoints: Die Dummheit ist sehr wohl Schuld des Dummen, wenn der sich weigert etwas dagegen zu tun.

    Ein Plasmaeinschluss in einem Magnetfeld ist nicht ideal, es gibt immer einzelne Teilchen, die diesem entkommen. Was sagt uns das dann schon mal zu Deinen multiplen Räumen?

    @kwirko: Doch nicht eher “Quirko”?

  101. #101 kindermund
    2. Januar 2011

    Sie können das ja von mir aus so definieren. Sie können auch definieren: “Ein Tisch ist ein Fahrzeug mit einem bett drin”. Sie dürfen sich nur nicht wundern, wenn andere Leute Sie nicht verstehen, weil Sie der einzige sind, der diese Definition verwendet.

    Ganz so einfach ist es nicht. Sie können sich natürlich entschließen weiter im gut bevölkerten Wolkenkuckucksheim zu verweilen und mathematische Definitionen für Physik verwenden, nur betreiben Sie dann eben Mathematik, nicht Physik. Dann dürfen Sie sich aber auch nicht wundern, wenn Ihnen Physik ein Rätsel bleibt.
    Die gegebene Definiton ist nicht allein “meine”, so sind Objekt und in der Wissenschaft (nicht in der Nichtwissenschaft Mathematik) definiert. Sollten Sie daran Zweifel haben, tragen Sie doch mal Ihre Definitionen von Raum und Objekt vor, aber achten Sie darauf nicht wieder auf die Ebene der mathematischen Konzepte abzugleiten, sondern geben Sie eine physikalische, wissenschaftliche Definition.

    Hat der Raum bei Ihnen drei Dimensionen? Wenn ja, dann hat er damit eine Eigenschaft, wenn nein, dann ist es nicht der Raum, über den wir normalerweise sprechen, wenn wir “Raum” sagen.

    Natürlich hat Raum keine Dimensionen. Objekte haben Dimensionen. Ersetzen Sie “wir” durch “Mathematiker” und der Satz wird wahr.

    Brauchen wir aber nicht weiter zu diskutieren, da der Streit um Definitionen wenig inhaltlichen Nährwert hat.

    Das ist ein bemerkenswertes Statement von jemandem, der der Mathematik so zugetan ist wie Sie. Allerdings auch nicht völlig überraschend, denn entgegen dem immer wieder gebetsmühlenartig vorgetragenen Mantra vom “strengsten Rigor”, basiert Mathematik bekanntlich doch auf undefinierten Grundelementen und verwendet ständig beliebige Begriffe.
    Ein Kulturpessimist könnte diesen Satz auch so verstehen, daß Sie an Wissensgewinn und Wissenschaft grundsätzlich nicht interessiert sind, denn die eindeutige Definition der Begriffe ist nun einmal das unverzichtbare Fundament der Wissenschaft.

  102. #102 Dr. Webbaer
    2. Januar 2011

    Howdy!

    Was ist hier los? Der Raum sei kein Objekt? Habe keine Eigenschaften?

    Aber hallo, die Herren! – Es wäre ja zumindest theoretisch denkbar, dass der hiesige Inhaltemeister metaphyisch nicht gerade der Weltmeister aller Klassen ist, aber rein philosophisch gehaltenes Genage mit dem Ziel die SRT anzugreifen, ist hier deplatziert.

    Selbstverständlich sind Mathematik und Physik anthropogen, und wenn man so will “letztlich” undefiniert, allerdings kann es im Theoriebetrieb nur darum gehen nützliche und gut beschreibende Theorienmengen zu halten, die empirisch adäquat sind.

    Welche sich durchsetzen (im Sinne des Konstruktiven Empirismus) wird dann in der Debatte entschieden – aber sicherlich nicht in dieser!

    @kindermund: Unbedingt zukünftig substanzieller vortragen!

    MFG
    Dr. Wb

  103. #103 kindermund
    2. Januar 2011

    @ WB

    … rein philosophisch gehaltenes Genage mit dem Ziel die SRT anzugreifen, ist hier deplatziert.

    Danke für den normierenden Eingriff in die Diskussion. Darf ich noch kurz fragen, wer den WB zum Gott ernannt hat? Die Frage was Raum ist hat rein gar nichts mit Philosphie, aber alles damit zu tun, was von A/SRT zu halten ist.

    Selbstverständlich sind Mathematik und Physik anthropogen, und wenn man so will “letztlich” undefiniert, allerdings kann es im Theoriebetrieb nur darum gehen nützliche und gut beschreibende Theorienmengen zu halten, die empirisch adäquat sind.

    Es ist für die in der Überschrift gestellte Frage belanglos worum es im “Theoriebetrieb” geht – der Begriff gibt den Gegenstand ja bereits preis und war bisher nicht Gegenstand der Diskussion. Trotzdem vielen Dank für die Anregung.

    Welche sich durchsetzen (im Sinne des Konstruktiven Empirismus) wird dann in der Debatte entschieden – aber sicherlich nicht in dieser!

    Aber ja doch, Herr Dr. Gott. Vielleicht wird es aber doch in dieser Debatte entschieden, wenn es nicht schon längst entschieden ist und hier nur eine Nachbereitung stattfindet?!

  104. #104 TheBug
    2. Januar 2011

    @kindermund: WB hat lediglich selbstverständliche Anforderungen an eine Argumentation klargestellt.

    Wie schon mehrfach angesprochen: Aufhören mit dem oberflächlichen Geseiere und Butter bei die Fische. Wer versucht eine so gut abgesicherte Theorie wie die SRT anzugreifen muss sich ganz erheblich mehr anstrengen als nur darüber rumzunölen wie schrecklich unlogisch ihm das alles erscheint.

    Mathematik als nicht zu den Wissenschaften gehörig zu bezeichnen ist aber ein Anzeichen dafür wie sinnlos es ist Zeit auf Dich zu verschwenden. Mathematik gehört nicht zu den Naturwissenschaften, das wars dann aber schon. Ohne die Mathematik als Ordnungswissenschaft wären die Naturwissenschaften nicht möglich. Mag sein, dass Dir das nicht passt, aber das ist nur ein Indiz dafür wie unqualifiziert Du bist eine Diskussion über Naturwissenschaften zu führen.

  105. #105 kindermund
    2. Januar 2011

    Wie schon mehrfach angesprochen: Aufhören mit dem oberflächlichen Geseiere und Butter bei die Fische. Wer versucht eine so gut abgesicherte Theorie wie die SRT anzugreifen muss sich ganz erheblich mehr anstrengen als nur darüber rumzunölen wie schrecklich unlogisch ihm das alles erscheint.

    Mehr Butter, als die Feststellung, daß man etwas Eigenschaftsloses nicht mit einer Eigenschaft versehen kann ohne es zu etwas anderem zu transformieren, kann man wohl kaum bei die Fische tun.

    Wenn Sie Probleme haben das zu verstehen, könnte es daran liegen, daß auch Sie mit Begriffen der Mathematik denken. Wenn A/SRT jedoch Realität erklären will, dann muß sie sich schon bequemen Begriffe aus der Realität zu verwenden. Mathematik ist ausschließlich mit Konzepten/Symbolen befaßt, Realität ist kein Konzept/Symbol. Daher ist die Sprache der Mathematik untauglich Realität zu erklären.

    Wenn RT etwas über Realität sagen wollte, dann müßte Sie Begriffe wie “Dichte” verwenden. Aber der zentrale Gegenstand und die gesamte Absicht von RT ist ein abstraktes Konzept, nämlich Zeit mit Objekten der Realität, in der sogenannten “Raum/Zeit”, zu verschmelzen. Das für sich alleine ist bereits irrational und hat mit Wissenschaft nichts zu tun, aber es kommt noch schlimmer, denn
    “Raum” ist nicht nur kein Objekt der physischen Welt, sondern das gestaltgebende Gegenstück dazu.
    Mit anderen Worten bereits um der ursprünglichen Absicht gerecht zu werden, muß sich RT von Wissenschaft verabschieden, nur um dann zu “vergessen” das Universum als physisches Objekt zu deklarieren (was dann natürlich die Frage aufwürfe, was dem Universum Gestalt gibt) und somit die Synthese von 2 Begriffen die nichts Reales bezeichnen als Theorie zur Beschreibung von Realität darzustellen. Das ist Religion, nicht Wissenschaft…

  106. #106 Niels
    2. Januar 2011

    @kindermund

    “Wenn etwas Eigenschaften hat, dann ist es, per Definition, ein Objekt.”
    “Raum” ist das, wovon sich Objekte unterscheiden.

    Ich hab dazu folgende Fragen oben schon einmal gestellt:
    Was ist der Unterschied zwischen “etwas ohne Eigenschaften” und “Nichts”?
    Ist nach deiner Definition eine Eigenschaft ein Raum oder ein Objekt?
    Schließlich gibt es bei dir nur die Kategorien “Objekte” und “etwas, das sich von Objekten unterscheidet” (von dir “Raum” genannt).

    Wenn RT etwas über Realität sagen wollte, dann müßte Sie Begriffe wie “Dichte” verwenden.

    In den Energie-Impuls-Tensor der RT gehen unter anderem die Energiedichte/Massendichte, die Energiestromdichte/Massenstromdichte sowie die Impulsdichte und die Impulsstromdichte ein.
    Welche “Dichte” fehlt denn noch?

    die gesamte Absicht von RT ist ein abstraktes Konzept, nämlich Zeit mit Objekten der Realität, in der sogenannten “Raum/Zeit”, zu verschmelzen.

    Versteh ich nicht. Du sagst selbst: “”Raum” ist das, wovon sich Objekte unterscheiden.”
    Warum ist die Raumzeit also eine Verschmelzung von Zeit mit Objekten?

  107. #107 kindermund
    2. Januar 2011

    @ Niels

    Was ist der Unterschied zwischen “etwas ohne Eigenschaften” und “Nichts”?

    Kein Unterschied.

    Ist nach deiner Definition eine Eigenschaft ein Raum oder ein Objekt?

    Objekte haben Eigenschaften.

    Schließlich gibt es bei dir nur die Kategorien “Objekte” und “etwas, das sich von Objekten unterscheidet” (von dir “Raum” genannt).

    Nein, lies noch mal in Ruhe. Zur Beantwortung der Frage (siehe Überschrift) sind nur zwei Begriffe erforderlich: Raum und Objekt. Daß heißt selbstverständlich nicht, daß ich nicht mehr als diese beiden Begriffe kenne.

    In den Energie-Impuls-Tensor der RT gehen unter anderem die Energiedichte/Massendichte, die Energiestromdichte/Massenstromdichte sowie die Impulsdichte und die Impulsstromdichte ein. Welche “Dichte” fehlt denn noch?

    Das ist alles irrelevant. Nochmal: RT behauptet der Raum sei “gekrümmt”. Da der Raum keine Eigenschaften hat, ist er auch nicht gekrümmt. Wenn etwas Eigenschaften hat, dann ist es ein Objekt, kein Raum.
    WENN man eine Analogie suchen würde, mit der man das Hirngespinnst “Krümmung” der RT in wissenschaftliche Begriffe ÜBERSETZEN wollte, dann müßte man das Universum als Objekt definieren, in dessen Volumen das Medium der Schwerkraft über variierende Dichte verfügt. Der Äther lässt grüssen….

    Warum ist die Raumzeit also eine Verschmelzung von Zeit mit Objekten?

    Ist es nicht, hab ich auch nicht behauptet – ganz im Gegenteil. Lies nochmal in Ruhe.

  108. #108 Niels
    2. Januar 2011

    Zur Beantwortung der Frage (siehe Überschrift) sind nur zwei Begriffe erforderlich: Raum und Objekt. Daß heißt selbstverständlich nicht, daß ich nicht mehr als diese beiden Begriffe kenne.

    Versteh ich nicht. Es gibt doch bei deinen Definitionen nur die beiden Möglichkeiten:

    A) etwas hat Eigenschaften = Objekt

    B) etwas hat keine Eigenschaften = “Raum” = Nichts

    Was bleibt für C übrig?

    “Raum” = Nichts ist wobei genau nützlich?
    Nichts hat einen Einfluss auf… ?

    Zur “gesamte Absicht der RT”:
    Na ja, in Ruhe lesen macht es auch nicht besser. Es stimmt einfach nicht, dass in der ART Zeit und Objekte “verschmolzen” werden. Wozu werden sie denn “verschmolzen”?
    (Wobei ich mit dem Begriff “verschmelzen” in einer physikalischen Theorie nicht besonders viel anfangen kann. Geht es auch präziser?)

  109. #109 kindermund
    2. Januar 2011

    @ Niels
    Daß Denken kann ich Dir nicht abnehmen.

  110. #110 threepoints...
    2. Januar 2011

    @ TheBug· 02.01.11 · 00:19 Uhr

    “Die Dummheit ist sehr wohl Schuld des Dummen, wenn der sich weigert etwas dagegen zu tun.”

    -> Eine kategorie “dumme” hat weder daran Schuld, noch kann er etwas an seiner “minderbemittelten” Lage selbstständig verändern. Sie lebten und leben unter oeden Geistern. Und so geht das Elend ewig weiter…

    “Ein Plasmaeinschluss in einem Magnetfeld ist nicht ideal, es gibt immer einzelne Teilchen, die diesem entkommen. Was sagt uns das dann schon mal zu Deinen multiplen Räumen?”

    -> Das sie genau so im All auch sind. Grenzen, die unter bestimmten Bedingungen ihren Raum abgrenzen und nicht undurchlässid sind – also ein Austausch möglich ist.

  111. #111 threepoints...
    2. Januar 2011

    “… der hiesige Inhaltemeister …”

    -> Es ist Vormittag … und ich habe jetzt lust auf die demokratische Auflösung der diplomatischen Fangfrage vom Webbaeren…

    Wer ist der hiesige Inhaltemeister…?

  112. #112 rolak
    2. Januar 2011

    Dann stimme ich für einen Blick hinter die Kontaktanzeige fast ganz oben auf dieser Seite. Vielleicht findest Du ja iwo in den Weiten des www ein Lexikon WB<=>(Rest der Welt) 😉

  113. #113 MartinB
    2. Januar 2011

    @kindermund
    Es nützt nichts. Du kannst solange behaupten, dass der Raum nicht gekrümmt sein kann, wie du willst, Physik wird daraus nur, wenn du konkrete Vorhersagen machst. Die ART macht mit der Annahme eines gekrümmten Raumes exzellent bestätigte Vorhersagen. Wenn du eine bessere Theorie hast, in der der Raum “nichts” ist, dann bitte sehr, es gilt das gleiche wie oben bei threepoints: Mach bessere oder zumindest gleich gute Vorhersagen (quantitativ, bitte), dann können wir reden.
    Momentan stellst du dich immer nur hin und sagst “Ich habe recht”, ohne Beleg oder Argument. Das meinte ich damit, dass der Streit um Definitionen keinen Sinn hat. Wenn allerdings in deiner Begrifflichkeit “Raum” nicht mal Dimensionen hat, dann wird’s schwierig.

    @threepoints
    “Es könnte tatsächlich für alle elektromagnetischen Wellen gelten. Dort wo sie vorhanden sind, wird die Umgebung Beeinflusst – also ein zusätzlicher Raum geschaffen. ”
    Deine Ideen sind extrem unkonkret – du hast dir nicht einmal diese simple Frage gestellt. Aber was würde das konkret bedeuten, wenn ich z.B. einen hochenergetischen gepulsten Laserstrahl erzeuge – entstehen dann Raum”blasen”, die wieder verschwinden, wenn der Puls aufhört? Was genau soll es bedeuten, wenn irgendwo Raum “entsteht”? Was passiert mit einem Teilchen, das einen Lichtstrahl kreuzt? Müsste das dann nicht gebremst werden, weil der Lichtstrahl Raum erzeugt und das Teilchen mehr Strecke zurücklegen muss?
    Solange das alles so unkonkret ist, wird sich niemand dafür interessieren – wie gesagt, solche Ideen kann jeder zwischen Frühstück und Morgenzeitung erfinden, ohne sich anzustrengen…
    PS: “Inhaltemeister” ist webbärisch für “Blogautor”.

  114. #114 threepoints...
    2. Januar 2011

    O.k., … Der Artikelersteller. Wollte nur mal eben meine Orientierungswaage eichen. …

    Das mit dem WB ist selbst mir bekannt. Ein solches Gefechtsvergnügen hatte ich schon…. und häufiger schon als Schaulustiger anwesend.

    Aussage aus seinem Blog:

    “Machen Sie es Dr. Webbaer aber bitte auch nicht schwer, seien Sie nett und bearbeiten möglichst die Sache, wenn Sie sich schon die Mühe machen hier einen Kommentar zu hinterlassen.”

    -> Also, …. nehmen wir uns ein Beispiel an des Webbaers Methode und werden entspanter mit den eigenen Ansprüchen, um diese Welt daran teilhaben zu lassen….. Auf deutsch: Auf in den Kampf – ziehet zu Felde gegen…. wogegen auch immer.

  115. #115 threepoints...
    2. Januar 2011

    “Momentan stellst du dich immer nur hin und sagst “Ich habe recht”,…”

    -> Ich habe auch ein paar Synapsen frei für Psychologie…. und wenn ich die benutze, wenn ich solche Sätze lese, dann habe ich auch immer den Eindruck, das solche Sätze sagen kann wer will und doch bleibt die Aussage unparteiisch als Mahnmal für beide Seiten …
    Weshalb es mich bohrend belastet, warum man eigendlich zu solchen Aussagen verleitet wird? Ist doch in der Sache schon eine solche Aussage enthalten – wenn man die benötigten Beweise einer Theorie nicht beibringen kann … die Teilnehmer aber darauf hinweisen….

    Nein, solche Aussagen braucht es nicht. Und niemand hier will sich die Phantasien einiger Kommentateure ernsthaft antun, weshalb diese Aussage dann deren leichteste Ausrede ist, sich dem Inhalt anzunähern.

    Der einzigste Grund, warum ich hier überhaupt teilnehme ist, dass ich dadurch erhoffe etwas Einblick ausserhalb der populären Beschreibung von Themen wie diesen zu bekommen.
    Unter solchen Bedingungen aber bleibt diese Hoffnung allermeisst unbefriedigt.

    Und zuletzt … erklärte einer (war doch wieder der Webbaer), dass alles nicht auf diesen Blog geklärt wird.

    Und das entspricht auch meiner Einstellung.

    Gibt es eigendlich ein maximales Zeichenlimmit pro Kommentar? Denn es müssten gar millionen Zeichen auftauchen, um auch nur annähernd auf den Sachverhalt hinweisen zu können.

  116. #116 threepoints...
    2. Januar 2011

    MartinB· 01.01.11 · 21:46 Uhr

    “Was passiert, wenn ich HG-Strahlung mit einem Spiegel oder einer Linse irgendwohin bündele? Entsteht dann dort “mehr Raum”? Wenn nein, warum nicht? Wenn ja, wie genau?”

    -> Ja, so funktioniert es in der Microwelle, wenn ich etwas reinstelle (etwa einen Stein, Kaffee, Katze, Schnitzel und Kartoffeln) Die “Räume” (das Medium im Raum die den Raum einnehmen) nehmen Energie auf und wandeln diese um. Hier wird sich eine Erwärmung ergeben und Ausdehnung folgen, bis diese nicht mehr möglich ist. Daraufhin wird die eingebrachte Energie anders umgewandelt (Temperatur, Ausdehnung, Druck, Impuls und wieder Strahlung) … immer nach den jeweiligen Bedingungen der Materie, die die Strahlungsenergie aufnimmt.

    Es Entsteht dann wohl tatsächlich “mehr Raum” durch Ausdehnung.Und selbst Luft würde sich erwärmen und aus den Raum austreten oder den Druck erhöhen, der erwärmt wird, so er Begrenzt ist.

  117. #117 threepoints...
    2. Januar 2011

    kindermund· 02.01.11 · 07:46 Uhr

    “Das ist alles irrelevant. Nochmal: RT behauptet der Raum sei “gekrümmt”. Da der Raum keine Eigenschaften hat, ist er auch nicht gekrümmt. Wenn etwas Eigenschaften hat, dann ist es ein Objekt, kein Raum. WENN man eine Analogie suchen würde, mit der man das Hirngespinnst “Krümmung” der RT in wissenschaftliche Begriffe ÜBERSETZEN wollte, dann müßte man das Universum als Objekt definieren, in dessen Volumen das Medium der Schwerkraft über variierende Dichte verfügt. Der Äther lässt grüssen…. ”

    Wenn der Raum gekrümmt ist, … dann ist Raum als ein Objekt zu bezeichnen. Aber mit dieser Erklärung kann ich nichts anfangen.
    Etwas anderes ist es, wenn im Raum sich die Bedingungen krümmen, so Masse ausreichend vorhanden ist. Und Masse wie Dichte ist abhängig von Materie. … Also kann der Raum nur hypotetisch gedacht werden und muß immer mit einer Füllung (was auch immer) versehen sein.
    Das lässt den Schluß zu, dass der Raum nicht gekrümmt ist, sonderen dessen Füllung (unter dessen Bedingungen). Und wie es die Theorie vorhersagt, wird alles, was sich in diesem gefüllten Raum bewegen kann … auch auf dessen Bedingungen begründet sein und also sich nur in der Krümmung bewegen oder die Krümmung selbst herstellen – egal, was es ist.
    Tatsächlich schreit es dann nach einem Äther. Aber der ist eben auch so ein Begriff, wie dunkle Materie, dunkle Energie … und so – viel gestammel und wenig fassbar. Und was sei die Hintergrundstrahlung anderes, als eine Art Äther? Oder noch dringlicher: Wozu bräuchten wir die Quantenphysik, wenn kein Phänomen, wie ein “Äther” vorhanden ist? Und auf diese Fragestellung keine Antworten, deren Grundlagen eben auch wieder von oder mit Strahlung in Verbindung stehen oder abhängig sind. Sie würden eben nur betätigen, dass sich ein “Medium” im Raum befindet – etwa ein “Äther”.

  118. #118 kindermund
    2. Januar 2011

    @ MartinB

    Es nützt nichts. Du kannst solange behaupten, dass der Raum nicht gekrümmt sein kann, wie du willst, Physik wird daraus nur, wenn du konkrete Vorhersagen machst.

    Ich behaupte nichts, ich stelle fest. Das würden Sie bemerken wenn Sie mir Ihre rationale Definition von “Objekt” im Sinne der Physik geben würden – wie angefragt.

    Die ART macht mit der Annahme eines gekrümmten Raumes exzellent bestätigte Vorhersagen.

    Wir wissen doch beide, daß das Blödsinn ist. Der Teil an ART der funktioniert stammt von Newton. Einstein hat lediglich Newton umverpackt, eine bedeutungslose Konstante eingeführt und die Fiktion eines gekrümmten Raumes hinzugefügt. Kein Experiment hat die Einsteinsche Science Fiction bisher bestätigt.

  119. #119 kindermund
    2. Januar 2011

    @ threepoints

    Wenn der Raum gekrümmt ist, … dann ist Raum als ein Objekt zu bezeichnen. Aber mit dieser Erklärung kann ich nichts anfangen.

    Nein, wenn etwas Eigenschaften hat, ist es nicht “der” Raum. Daß ist die entscheidende Einsicht, zu der man Mathematiker nur in Ausnahmefällen führen kann.

    Also kann der Raum nur hypotetisch gedacht werden und muß immer mit einer Füllung (was auch immer) versehen sein.

    Nein. “Nichts” ist unabhängig von der An- oder Abwesenheit von “etwas”, aber “etwas” ist nur erkennbar, wenn es sich von seinem Gegenteil abhebt. Eine “Füllung” bezeichnet man in der Wissenschaft als Volumen und das ist eine Eigenschaft von Objekten.

    Das lässt den Schluß zu, dass der Raum nicht gekrümmt ist, sonderen dessen Füllung (unter dessen Bedingungen)

    Nein. Ein Volumen ist nicht “gekrümmt”, jedenfalls nicht im einsteinschen Sinne, die korrekte Entsprechung wäre ein Dichtegradient.

    Tatsächlich schreit es dann nach einem Äther. Aber der ist eben auch so ein Begriff, wie dunkle Materie, dunkle Energie … und so – viel gestammel und wenig fassbar.

    Ich propagiere keinen Äther, ich habe ihn nur erwähnt, weil er in Einsteins Geschichte eine besondere Rolle spielt. Erst “Äther ja”, dann “Äther nein” – der Mann tendierte dazu sein Fähnchen in den Wind zu hängen.
    Auch “dunkle Materie/Energie” ist Science Fiction, daß Produkt eines langen, falschen Weges, der es mit jeder neuen Entdeckung erforderlich macht die angeblich so erfolgreichen Theorie mit neuen Variablen und Phantasieprodukten zu versehen. Denken Sie “dunkle Materie/Energie” als Versagen von “Big Bang” Kosmologie und RT. Achten Sie darauf, was alle diese Konzepte gemein haben: man kann sie nicht beobachten, sie gründen auf mathematischen und/oder argumentativen Zirkelschlüssen und sind experimentell vollkommen unbewiesen.

    Oder noch dringlicher: Wozu bräuchten wir die Quantenphysik, wenn kein Phänomen, wie ein “Äther” vorhanden ist?

    Außer den Mathematikern braucht kein Mensch Quantenphysik. QT wird mißbraucht, um weiterhin an die Existenz von Partikeln glauben zu können und immer dann, wenn man mit Partikeln nicht mehr hinkommt, Wellenfunktionen anführen zu können. Das ist alles PHANTASIE und zwar mathematische Phantasie. Bis heute hat kein Mensch je ein Elektron oder ein Proton gesehen – man muß halt dran glauben, wenn man in dieser Kirche beten will…

  120. #120 threepoints...
    2. Januar 2011

    Also, … mir keimt ein Verdacht auf, dass mir ein Blog mit Kommentarmöglichkeit nicht hilft, wenn ich mir diesen hier ansehe. Entweder es liegt an der Struktur des Blogs und Kommentar – oder an mir (und wenns an mir liegt, dann auch an den anderen Kommentateuren).

    ich versuche es morgen noch mal (einmal drüber schlafen) – und wenn ich dann immernoch nicht durchsehe, von dem was hier an Kommentaren zu widersprüchen führt, dann nutzt es eben auch nichts und muß bleiben gelassen werden.

  121. #121 kindermund
    2. Januar 2011

    @ threepoints
    Gelassenheit ist Trumpf! 🙂

  122. #122 TheBug
    2. Januar 2011

    Jetzt diskutieren schon zwei Cranks darüber welches ihrer Mikey Mouse Universen denn nun richtig ist…

    Jungs, hört sofort auf hier zu posten, Ihr benutzt jede Menge Technologie, die einfach nicht funktionieren kann, weil ja laut Euch die dem zugrunde liegenden Theorien völliger Schwachsinn sind. Also sofort die Computer und andere elektronische Geräte aus Euren Haushalten entfernen, das ist TEUFELSZEUG, ausgedacht von kranken Hirnen die Räume krümmen, die es ja gar nicht gibt, oder gab es davon jetzt jede Menge?

  123. #123 Andreas P.
    2. Januar 2011

    @kindermund
    Alter, dein Ego möchte ich haben … du spazierst hier in ein Blog rein in dem sich reichlich Menschen rumtreiben die mit solchen Fragestellungen ihren Lebensunterhalt verdienen, erklärst alle zu Vollidioten die deine kruden Theorien nicht teilen, und Beweise hast du schon mal gar nicht nötig, denn

    Ich behaupte nichts, ich stelle fest.

    Google mal nach “Dunning Kruger Effect” in ‘ner ruhigen Minute, evtl geht dir ein Licht auf, auch wenn ich’s bezweifel.

  124. #124 MartinB
    2. Januar 2011

    @threepoints
    Das Mikrowellenbeispiel verstehe ich auch nicht: “Es Entsteht dann wohl tatsächlich “mehr Raum” durch Ausdehnung.” Heißt das, das Volumen meiner Mikrowelle wird größer, wenn ich sie anstelle? Oder wie?

    @kindermund
    Es wird langsam lächerlich:
    “Außer den Mathematikern braucht kein Mensch Quantenphysik.”
    Es sei denn, er möchte beispielsweise einen Computer benutzen, den könnte man ohne QM nicht konstruieren…

    “Das würden Sie bemerken wenn Sie mir Ihre rationale Definition von “Objekt” im Sinne der Physik geben würden”
    Objekt ist kein physikalisch sonderlich wichtiger Begriff, ich kenne jedenfalls keine physikalische Theorie, die mit dem Begriff “Objekt” hantiert.

    “Bis heute hat kein Mensch je ein Elektron oder ein Proton gesehen”
    Stimmt. Und das Erdinnere hat auch kein mensch gesehen, das ist auch Fiktion. Auch das Innere der Sonne gibt es nicht, und es gibt überhaupt nur die Dinge, die kindermund sehen kann – der Name passt besser, als man dachte, sehr kleine Kinder neigen ja auch dazu, Dinge für verschwunden zu halten, wenn sie sie nicht sehen können.

    “Kein Experiment hat die Einsteinsche Science Fiction bisher bestätigt. ”
    O.k., gegen derartig hartnäckge Realitätsverweigerung bin ich machtlos, wie schon oben im Post gesagt: “You cannot reason people out of a position that they did not reason themselves into.”

  125. #125 threepoints...
    2. Januar 2011

    @ kindermund· 02.01.11 · 13:17 Uhr

    -> Ja, … ungefähr so streiten sich die schizophrenen Gemüter in meinem Kopf, seitdem ich eben diese Eckpunkte kenne. Mir scheint auch nicht mehr nachvollziehbar, was überhaupt von diesen aussagen als beweisbar und was nur reinste Theorie ist.

    und … nicht dass ich hier im Kommentarbereich tatsächlich irgendwas beweisen könnte … aber verlangen kann man das ja mal – immerzu. Wage ich mich zuweit hinaus, wenn ich meine These/Vorstellung für plausibel halte? Ganz nach dem Motto “Ich habe recht”… Was ich ja nicht mal selbst glaube. Aber die Zukunft könnte es tatsächlich bald aufzeigen – was sich da draussen wirklich tut. Und ob die Grenzen des humanen Geistes auf ewig an dem Urknall scheitern werden….

    Licht ist eine merkwürdige Sache. Ich erinnere mich dabei an goethes Farbenlehre. Und an aus Sicht der Wissenschaft überholte/widerlegte Ansicht seiner Erkenntnisse.
    Er erklärte sich nur, was er sah. Nicht, was er per Werkzeug ermessen konnte.

    Nach deiner Meinung müsste meine Frage nciht lauten:

    “Oder noch dringlicher: Wozu bräuchten wir die Quantenphysik, wenn kein Phänomen, wie ein “Äther” vorhanden ist?”

    … sondern:

    “Wozu brauchen wir den Äther, wenn wir die Quantenphysik haben?”

    … schenkelklopfer… ich werde aber später noch mal lachen.

  126. #126 MartinB
    2. Januar 2011

    @threepoints
    “Wage ich mich zuweit hinaus, wenn ich meine These/Vorstellung für plausibel halte? ”
    Solange du über die Konsequenzen deiner Ideen nicht nachdenkst, ja.
    “Er erklärte sich nur, was er sah. Nicht, was er per Werkzeug ermessen konnte.”
    Ja. und er irrte sich. Spektakulär und poetisch, aber er irrte sich trotzdem.

  127. #127 threepoints...
    2. Januar 2011

    @ MartinB· 02.01.11 · 13:54 Uhr

    “Heißt das, das Volumen meiner Mikrowelle wird größer, wenn ich sie anstelle?”

    -> Das Volumen der Materie (hier Gas) innerhalb des Gerätes.

  128. #128 MartinB
    2. Januar 2011

    @threepoints
    “Das Volumen der Materie (hier Gas) innerhalb des Gerätes. ”
    Wie das? Wenn die Mikrowelle abgeschlossen ist (nehme ich mal vereinfachend an), wie vergrößert sich dann das Volumen des Gases? Dazu müsste sich dann ja der Raum ausdehnen. Wenn er das tut, müsstest du erklären, wie und um wieviel und warum man das auch bei extrem hohen Energiedichten noch nie bemerkt hat.

  129. #129 threepoints...
    2. Januar 2011

    MartinB· 02.01.11 · 13:54 Uhr

    “Heißt das, das Volumen meiner Mikrowelle wird größer, wenn ich sie anstelle?”

    -> Also wer solche Fragen stellt und angibt, er habe Physik studiert und gar tätig im Fach, …..

    ich muß etwas an der Frage falsch verstanden haben…. Unvorstellbar, dass er das so ernsthaft fragen könnte.
    Wenn er mich verarschen will … wieso nimmt er sich die Zeit…? Der kleine threepoints… ist doch dafür kaum der Mühe wert….

    Selbstdisqualifizierung …. irgendwie klassisch.

  130. #130 Niels
    2. Januar 2011

    Wat, die Luftmoleküle werden größer oder wie?

  131. #131 TheBug
    2. Januar 2011

    Nee, der Kopf von threepoints bläht sich gerade auf, weil ihm eine Frage gestellt wurde die nicht sein darf.

  132. #132 threepoints...
    2. Januar 2011

    @ MartinB· 02.01.11 · 14:30 Uhr

    Angenommen dieses Küchengerät hätte eine kleine Modifikation am Garraum erfahren – nämlich luftdicht gemacht. Und das Wasser würde mit ausreichend Energie und Zeit zu Gas… und es sei genug Wasser vorhanden. Das Ergebnis könnte ungefähr so aussehen: Die Microwelle hat seitdem einen leicht gewölbten Garraum…. weil sich das Gas den Raum erschuf…und durch den herrschenden Druck die Wandungen des Garraumes etwas über dessen Belastungsgrenze ….
    Aber das sind Extrembedingungen, die nicht Eintreffen. Aber sie wären nötig, um dieses Prinzip auf der Erde und unter alltäglichen bedingungn überhaupt erkennbar werden sollen.

    Die SRT nimmt dem Raum die klassische Form mit linearen Dimensionen. a x b x c … sind demnach nicht geometrisch berechenbar.
    und aus dieser Tatsache der SRT entnehme ich diese obskure Raumvariante – einen Raum mit vielen Räumen innerhalb derer. Oder eben die Überlagerung aller dieser Räume. und alle haben ihren Zustand (die physikalischen Bedingungen wie temperatur Druck, Dichte, Masse, … usw), der sich unter den bedingungen im Raum und die einwirkenden von ausserhalb des Raumes beeinflussen lässt. Grenzbereiche sind unbedingt erlaubt – in denen sich diese Räume überlagern. Es bedingt immer nur der Energieform und den Bedingungen in der Umgebung – wie sich der Inhalt/das Volumen verhält.

  133. #133 threepoints...
    2. Januar 2011

    Disqualifizert hat sich der “Fachman” in Physik. Nicht ich mit meinem Pseudo-Abschluß….

    Als ich einmal gefordert habe, dass sie sich herablassen sollen, meinte ich das nicht derart weit unten.

  134. #134 threepoints...
    2. Januar 2011

    Disqualifizert hat sich der “Fachman” in Physik. Nicht ich mit meinem Pseudo-Abschluß….

    Als ich einmal gefordert habe, dass sie sich herablassen sollen, meinte ich das nicht derart weit unten.

  135. #135 MartinB
    2. Januar 2011

    @threepoints
    O.k., wenn du auf kritische Fragen so reagierst, hat es wohl wenig Sinn.

    Aber rechne einfach mal die Energiedichte der kosmischen HG-Strahlung oder in einer Mikrowelle aus und überlege, *wieviel* Raum nach deiner Idee geschaffen werden müsste. Und dann schau dir die Strahlungsdichte an der Sonnenoberfläche an. Die ist knapp 2000 mal heißer als die HG-Strahlung, und die Energiedichte geht mit der 4. Potenz der Temperatur (steigt also um das 16E12-fache). Wo ist der ganze Raum, der dadurch entstehen müsste?

    PS: Wo ich mich selbst disqualifiziert habe, sehe ich nicht. Dass sich Materie unter Energiezufuhr ausdehnen kann (nicht muss, siehe Wasser und Eis) hat ja mit deiner Idee, dass sich der Raum selbst ausdehnt, nicht viel zu tun.

  136. #136 TheBug
    2. Januar 2011

    @threepoints: Du hast nur einfach ganz heftig einen an der Waffel. Disqualifiziert haben sich hier nur die Cranks, zu denen Du gehörst. Auf eine klare Frage mit Beleidigungen zu antworten ist nicht gerade ein Indiz für einen verteidigbaren Standpunkt und was Du gerade über die Ausdehnung von Gas/Raum abgelassen hast war nur geistige Diarhö.

    Selbstverständlich gibt es Situationen in denen ein Gas in einem wirklich abgeschlossenen Behälter aufgeheizt wird. Die Mikrowelle gehört nicht dazu und das macht Deinen Crankspace nur noch lächerlicher. Was passiert denn da mit dem Raum der entweichenden Luftmoleküle? Und warum haben die plötzlich mehr Raum? Die sind doch nur wärmer und nicht größer geworden, oder?

  137. #137 threepoints...
    2. Januar 2011

    TheBug· 02.01.11 · 15:12 Uhr

    Ausdehnung von Gas/Raum abgelassen hast war nur geistige Diarhö.

    -> Das sehe ich auch so… nur eben zwingend nach der Frage…

    Und das gleiche Experiment mit einer sich dehnbaren Hülle (Ballon) würde die Ausdehnung deutlicher zeigen.

    @ MartinB· 02.01.11 · 15:06 Uhr

    “Sonnenstrahlung … Wo ist der ganze Raum, der dadurch entstehen müsste?”

    Das Sonnensystem hat Ausmasse die bekannt sind. Warum die nicht größer sind, muß mit dem zusammenhängen, was Gravitation genannt wird. Die Tendenz, dass sich Materie konzentriert. Und warum sie sich in der Folge zu immer höheren Dichten konzentriert, …. kann nur in dem Ablauf sich selbst verstärkenden Prozess liegen. Der einmal anfängt und daraf hin aus sich selbst (Wachstum) fortschreitet und aber von Extern eine Energieform dazu erhält.

    Nun ist ein Stern und sein Prozess sehr verlustreich – was darauf hindeutet, dass die umgesetzte Energie nicht nur in Strahlung endet. Auch, dass ein erheblicher Überschuss an Energie vorhanden ist, der nicht schnell genug abgeführt werden kann, deutet auf weitere Energieformen hin.

    Aber ja, .. Ich habe einen an der Waffel…. das glaub ich selbst schon und höre mir deswegen auch nicht mehr zu….

    Aber ich werde aus anderen Gründen hier keine Energie mehr verschwenden.

    Die Frage war eben schon eine Beleidigung. Wie hätte ich reagieren sollen? Eigendlich habe ich noch nicht mal ernsthaft beleidigt. Kommt wohl noch darauf an, wie ernst einer sich, seinen Fachbereich und sein Output nimmt.

    Ich fand es nur recht merkwürdig… wie einer solche Fragen stellen kann, wenn doch sein Fachbereich genau diese Antworten zum Inhalt hat….

  138. #138 kindermund
    2. Januar 2011

    @ MartinB

    Es sei denn, er möchte beispielsweise einen Computer benutzen, den könnte man ohne QM nicht konstruieren…

    Immer wieder gern genommen, aber trotzdem irreführend, denn worum es geht, kann man u.a. hier nachlesen: https://xxx.lanl.gov/pdf/quant-ph/0609163v2

    Objekt ist kein physikalisch sonderlich wichtiger Begriff, ich kenne jedenfalls keine physikalische Theorie, die mit dem Begriff “Objekt” hantiert.

    Dann betreiben Sie keine Physik, sondern bestenfalls Mathematik. Ich halte zusätzlich fest: Sie drücken sich um die Antwort auf meine Frage.

    Wenn Sie den Begriff “Objekt” für keinen sonderlich wichtigen Begriff halten, dann kann ich eigentlich nur mit einem Zitat antworten: “gegen derartig hartnäckge Realitätsverweigerung bin ich machtlos”. Wobei es sich bei Ihnen tatsächlich um eben das handelt: das Konzept/die Idee/das Symbol der Realität vorziehen.

    Und das Erdinnere hat auch kein mensch gesehen, das ist auch Fiktion. Auch das Innere der Sonne gibt es nicht, und es gibt überhaupt nur die Dinge, die kindermund sehen kann

    Das habe ich nicht behauptet und werte diesen Satz daher als Polemik. Bei der Frage nach den Partikeln gilt es zu beachten, daß seit Einführung derselben mit dem Orbitalmodell, diese erfolglose Theorie von Tag zu Tag komplexer (wegen besagter Erfolglosigkeit) und irrationaler geworden ist. Die Verwechselung von Symbol und realem Gegenstand geht soweit, das es heute erwachsene Männer und Frauen gibt, die glauben ein imaginäres Teilchen namens Gluon könnte ein Vorzeichen einer Formel umkehren um soetwas wie “negatives Moment” (Yukawa Potential g/-g) zu übertragen.

    Sie trauen sich mir “Lächerlich(keit)” an den Kopf zu werfen?

  139. #139 threepoints...
    2. Januar 2011

    “Objekt” ist auch nicht hinreichend befriedigend.

    Ein Objekt ist per (?) Deffinition ein einziges (Ding). Weshalb also ein Inhalt oder Volumen auf ein “Ding” reduziert beschrieben ist – das es nicht ist. Es sind die Luftmoleküle in fast unzählbarer Menge. Und nicht “ein” Molekühl, dass als Objekt bezeichnet werden kann.
    Es sind aber in einem vlumen X wahrscheinlich viele Molekühle vorhanden, die gleiche Eidenschaften besitzen (können – weil es gleiche Elemente sind).

    Das Objekt “Findling” ist physikalisch schlecht gewält, weil der aus mehr besteht als Findling.

    Objekt ist blöd und passt nicht – habe aber selbst keinen Ersatz. Ausser die Materie.

  140. #140 TheBug
    2. Januar 2011

    @threepoints: Die Forderung ein Konzept zu erklären und zu untermauern ist eine Beleidigung? Cool. Ich nehme mal an Dein Raumkonzept ist so was wie eine Religion, da darf man natürlich keine Fragen stellen, sondern muss nur glauben.

    Bisher haben wir also von Dir ein Raumzeitmodell (nicht) vorgestellt bekommen, mit dem die Partyartikelhersteller (Luftballons) leben könnten. Wie wäre es denn nun mal mit ein paar Beispielen wie dieses Modell noch andere beobachtete Effekte erklärt? Die Behauptung es müssten erst entsprechende Experimente durchgeführt werden ist müßig, es gibt jede Menge gut dokumentierter Experimente an denen Du die Anwendbarkeit und Überlegenheit Deines Modells erklären könntest.

  141. #141 MartinB
    2. Januar 2011

    @kindermund
    Ich habe jetzt nur den Abstract gelesen, dort steht jedenfalls nicht “Außer den Mathematikern braucht kein Mensch Quantenphysik.”
    Insofern ist dieses Paper wohl kein beleg für kindermundlichen Blödsinn.
    Und nein, der Satz war keine Polemik, Ihr Argument war “Elektronen kann man nicht sehen, deshalb weiß man nicht, ob es sie gibt.” Meine Beispiele zeigen, dass das ein albernes Argument ist.

    Und die QM/QFT ist also erfolglos. O.k., sie hat uns die Festkörperphysik beschert, macht Tausende korrekter Vorhersagen, ermöglicht Erklärungen von der subatomaren Skala bis hin zur Astrophysik, aber ansonsten war sie na klar ziemlich erfolglos. (Ja, *das* war jetzt Polemik.)

    @threepoints
    “wie einer solche Fragen stellen kann, wenn doch sein Fachbereich genau diese Antworten zum Inhalt hat….”
    Also, meine Physik sagt mir nicht, dass der Raum sich irgendwie ausdehnt, wenn man Energie reinsteckt (obwohl er natürlich gekrümmt wird, aber in einer Mikrowelle oder durch die HG-Strahlung vernachlässigbar wenig). Das mit der Ausdehnung ist für viele (nicht alle) Materialien so, aber nicht für den Raum.

  142. #142 MartinB
    2. Januar 2011

    PS
    Habe das paper jetzt zumindest überflogen – abgesehen von der Schwarzen-Loch-Entropie (da kenne ich mich nicht so aus) steht da nichts drin, was man nicht z.B. auch in guten Büchern wie Morrison etc. findet, soweit ich sehen kann…

  143. #143 threepoints...
    2. Januar 2011

    TheBug· 02.01.11 · 17:48 Uhr

    “Die Forderung ein Konzept zu erklären und zu untermauern ist eine Beleidigung? Cool. Ich nehme mal an Dein Raumkonzept ist so was wie eine Religion, da darf man natürlich keine Fragen stellen, sondern muss nur glauben.”

    -> Na, glauben braucht das keiner. Wer sich nicht was vorstellen kann, der will ja eh nicht glauben.
    Ich hatte mal mit einem Arzt das Thema Glauben – und das wir doch alle glauben müssen. Er darauf nur: “Ich glaube anders…” (na immerhin…)

    Was das Konzept und eure Forderung danach betrifft, habe ich eigendlich nur Forderung nach beweisen vernommen.

    Und … naja, die habe ich nicht. und der Glaubensbeweis gilt ja bei euch nicht…

    “… es gibt jede Menge gut dokumentierter Experimente an denen Du die Anwendbarkeit und Überlegenheit Deines Modells erklären könntest.”

    -> Das könnte sein, muß aber nicht. Denn es werden die Experimente nach bestimmten Anforderungen (eben auch als Beweis für aufgestellte Vermutungen) durchgeführt. Diese werden dann in einer gewissen Erwartungshaltung und entsprechender Planung / Zielführung aufgestellt.

    Aber vielleicht liesse sich tatsächlich etwas finden. Aber vielleicht bedarf es sogar noch der Messtechnik und Strategie – die noch gar nicht erfunden ist oder noch nie angewand worden – weil eben die Zielführung ganz anders war…..

  144. #144 H.M.Voynich
    2. Januar 2011

    … und schon garnicht klärt es darüber auf, wie Computer heute ohne QM aussähen.
    Sollte es das überhaupt, oder wollte kindermund uns nur für eine Stunde beschäftigen?

  145. #145 threepoints...
    2. Januar 2011

    MartinB· 02.01.11 · 18:21 Uhr

    “Das mit der Ausdehnung ist für viele (nicht alle) Materialien so, aber nicht für den Raum.”

    -> Ich setzte auch nicht vorraus, das die Physik das so in ihre Formeln übernommen haben sollte oder noch wird. Ich habe auch nicht ein Element einem Raum zugewiesen oder angedacht, sondern ganze Zusammensetzungen / Systeme von Materie. Als organisations-stütze für besondere Visualisierungen.

    Das muß sich dann wohl für einen Physiker eher seltsam “anhören” – oder gar ganz zu schweigen von dem Bild, was der sich davon macht/machen will/kann. Denn der hat ja seine Methoden und Strukturen zu seinem Überblick parat. Brauch er kein Bild – oder hat eigene.

  146. #146 H.M.Voynich
    2. Januar 2011

    (das bezog sich auf @Martin 18:26)

  147. #147 threepoints...
    2. Januar 2011

    H.M.Voynich· 02.01.11 · 20:15 Uhr

    … und schon garnicht klärt es darüber auf, wie Computer heute ohne QM aussähen.

    -> Und wie sähen die nun aus … ohne QM? Oder genuer gefragt: Was daran ist Quantenmechanik?

  148. #148 Hel
    2. Januar 2011

    Könnte es sein, dass sich Jocelyne Lopez jetzt durch einen Kindermund mitteilt?
    Vgl https://www.mahag.com/neufor/viewtopic.php?f=6&t=324&start=520#p18893
    Unser Spamfilter mag den namen “Lopez” nicht…

  149. #149 Hel
    2. Januar 2011

    Kindermund liest sich irgendwie so altbekannt, vgl auch https://www.mahag.com/neufor/viewtopic.php?f=6&t=324&start=520#p18893

  150. #150 nihil jie
    2. Januar 2011

    @kindermund

    ich verstehe diese debatten um den begriff des objekts nicht. der begriff ist doch absolut universell. ich kann aus allen dingen ein objekt machen… sogar aus rein virtuellen. in der physik kann man sich auch diesen begriffs bedienen… zb. kann ich von einem ganzem komplex (einem atom, molekül usw) als einem objekt sprechen, oder halt über ein einzelnes elementarteilchen… zumindest wenn es zum kontext der darstellung passt. es ist zumindest ziemlich willkürlich wie ich den begriff einsetze. es ist so bisschen wie beim programmieren… ich kann ein xbeliebieges objekt erzeugen und auch wieder zerstören und der kann stellvertretend für so ziemlich alles sein…. ein fenster, eine einzelne combobox uwe… 😉

  151. #151 TheBug
    2. Januar 2011

    @threepoints: Die Halbleitertechnik wäre ohne funktionierende QM und SRT etwa 1990 stehen geblieben. Also hätten wir Computer mit zweistelligen Megahertz Taktraten und zweistelligen Megabytes als Arbeitsspeicher. Optische Speichermedien: Fehlanzeige, weil keine Laser ohne QM und selbst mit Laser funktionieren die nicht ohne SRT.

    Warum das so wäre? Versuch mal mit 193nm Licht Strukturen von 30nm zu erzeugen ohne funktionierende QM.

    Laser sind nur entwickelt worden, weil sich ihre wahrscheinliche Existenz aus der QM ergab und die SRT brauche ich für den Speichermechanismus bereits bei der CD.

  152. #152 threepoints...
    2. Januar 2011

    Ja, was war zuerst da…
    Die vollständige QM, oder die jeweiligen Versuche, die sie dann komplettiert haben? Hat sich die QM ert durch die Versuche mit Licht vervollständigt oder umgekehrt die Theorie die Versuche und Forschungen angeregt und angestossen

    Aber die QM ist ja noch heute nicht fertig? Sie taugt schenbar zur Anwendung, lässt aber wohl noch immer Forschungseinrichtungen Millionen investieren.

    Aus Wiki_Quantenmechanik:

    “Ihre grundlegenden Konzepte wurden im Zeitraum von 1925 bis 1935 von…erarbeitet.

    Und der Laser …irgendwie in den 60`gern oder so.

    Apropos Pentium 90… 32 MB Ram, 2,1 GB HD, …. CD-Rom …. in schönem creme-weiss (RAL 9001?) das Gehäuse… Mein erster PC von etwa 1996…

    Aber ganz erlich?

    Seitdem hat sich prinzipiell nichts geändert. Wir haben immernoch die selbe Hardwaregrundlage … nur alles kleiner, komplizierter im Aufbau (integrierter) …
    CPU, Northbridge, RAM-Anbindung, southbridge, Schnittstellen-Anbindung…..

    prinzipiell immer noch die selben Kondensatoren in CPU und Chipsätzen und Speichern … nur eben kompakter.

    So dringend war die QM wohl nicht für den Computer des endenden 20. Jahrhunderts – also dem AT/ATX-kompatiblen PC

    Aber nicht die Hardware selber hat dadurch direkt profitiert, sondern die Herstellungsmethode ….

  153. #153 TheBug
    2. Januar 2011

    @threepoints: So sehr hast Du es nicht mit dem Lesen…

    Es geht nicht darum, dass die Computerarchitektur die QM bräuchte, es geht darum, dass man Chips auf aktuellen Technologieniveau ohne funktionierende QM nicht herstellen könnte. Bei knapp unter 1µm ist ohne QM einfach mal Schluss mit Licht-Lithographie und die Röntgenlithographie ist bis heute nicht einsatzbereit.

    Wenn Du Dir 96 so eine Möhre als Computer hast andrehen lassen, dann passt das irgendwie ins Bild.

    Lies mal mehr zum Laser und wie es dazu kam.

    Es hat hier niemand behauptet, dass irgend welche Theorien der letzte Schluß der Weisheit sind, es geht nur darum, dass sie wesentlich besser sind als der Quark, den hier einige Cranks verbrechen.

  154. #154 threepoints...
    2. Januar 2011

    MartinB· 30.12.10 · 11:31 Uhr

    @threepoints
    Bewzüglich des FAZ-Textes: Die Ideen von Penrose sind wie üblich seeeeehr spekulativ

    -> Was ich über die Thesen aus dem Buch aufgepickt habe, sieht insgesammt genau nach meinen roten Faden aus. Der muß das Buch und seine vorher veröffentlichten Papers wohl nach meinen Ideen zusammengezerrt haben.

    Allem vorran der “aufdringliche Zyklus” . Und ich finde es nicht so seeeeeehr spekulativ. Wahrscheinlich gibt es auch was über den Zeitraum und den Ablauf der Zeit zwischen Ende und Anfang zu lesen. Es dürfte sich dabei um die Schrumpfphase des Raumes handeln. und den Ablauf, der wahrscheinlich langsam beginnt und immer schneller wird – bis es vielleicht zu einer gewaltigen kollision kommen wird – der Urknall! Zwischen Anfang und Ende sind wir ja gerade irgendwo.

    Aber keine Panik … solange die Observanten und ihre mathematiker erklären, das Universum expandiert noch – und sogar beschleunigt, solange ist alles noch im Lot.

    Jaja, Beweise … und so. Dabei ist der Anfang durch Urknall aus dem Nichts ebenso spekulativ. und davor erst recht. Von nach dem Ende des Universums ganz zu schweigen.

  155. #155 TheBug
    2. Januar 2011

    @Hel: Ja stimmt, klingt exakt genau so infantil und bescheuert. Ist schon bezeichnend, dass die Antirelativisten auch immer schwach in Mathematik sind.

  156. #156 threepoints...
    2. Januar 2011

    TheBug· 02.01.11 · 22:45 Uhr

    @threepoints: So sehr hast Du es nicht mit dem Lesen…

    -> Aber sag ich doch …Herstellungsmethode …. die Technik zur Produktion der Hardware…..im letzten Satz…

  157. #157 walim
    2. Januar 2011

    Um mal diesen Nebenschauplatz mitzubedienen: Nach meinem Verständnis ist die Halbleitertechnik (siehe Tunneleffekt) schon von ihren Begriff an in Quantenmechanik begründet. Aber bitte: es wären auch Relairechner und Apparate aus Röhren möglich gewesen oder schicke Steampunktechnik (https://de.wikipedia.org/wiki/Die_Differenzmaschine )

  158. #158 threepoints...
    2. Januar 2011

    “Ist schon bezeichnend, dass die Antirelativisten auch immer schwach in Mathematik sind. ”

    -> Wenn sie doch so schwach in Mathematik sind, dann müssten sie aber (die Antirelativisten) diese Schwäche relativieren …oder?

    … und so ists wohl nur logisch….

  159. #159 TheBug
    2. Januar 2011

    @kindermund: Typischer Crank-Fehler, geht den Kreationisten und Klimatrollen auch ständig so. Einfach auf ein Paper verweisen, dass einen netten Titel hat, oder sogar noch eine schöne Einleitung geht voll in die Hose. Den eigentlichen Text sollte man auch lesen, der ist nämlich ein Tritt in Deinen Hintern.

  160. #160 TheBug
    2. Januar 2011

    @walim: Streng genommen kann man Halbleitertechnik eine ganze Weile ohne QM betreiben, dafür reicht im schlimmsten Falle das Rutherford-Bohrsche Atommodell. Der Tunneleffekt wird nur in bestimmten Halbleitern verwendet, z.B. EPROM und Flash-ROM. Also ohne QM ist halt nix mit Digitalkamera, zumindest nicht ohne dicke Magnetbandspule.

    Wenn die Strukturen aber kleiner werden, oder die Leistungsdichte höher, bekommt man zunehmend Effekte die sich mit Kügelchen mit Orbitalen drum rum nicht mehr erklären lassen. Dann tritt halt der Tunneleffekt da auf wo man ihn nicht haben will.

    Aber viel gravierender ist, dass wir mittlerweile bei Strukturen auf den Halbleitern angekommen sind, die ganz erheblich unter der Wellenlänge des zur Belichtung verwendeten Lichtes liegen. Bereits wenn man noch beim Vielfachen der Wellenlänge ist bekommt man an den Belichtungsmasken und der Optik erhebliche Probleme mit der Beugung, geht man drunter, dann ist Mathematik angesagt, die eigentlich schon unters Betäubungsmittelgesetz fallen sollte…

    Aber dank QM klappt das alles und so sitzen wir heute alle vor Computern die im GHz Bereich takten, wahrscheinlich mehrere Prozessorkerne haben und über mehrere Gigabytes an RAM verfügen. Verbunden sind wir mit Glasfasernetzen, die wiederum dank QM und teilweise sogar SRT funktionieren. Und das alles klappt obwohl einige Cranks der Meinung sind das sei alles Einbildung.

  161. #161 TheBug
    2. Januar 2011

    @threepoints: Ohne die Herstellungsmethode wären die Steigerungen in der Leistung nicht möglich. Speicherdichte und Transistorzahl sind seit den frühen 1990ern um etwa einen Faktor 1000 gesteigert worden, bei etwa gleichbleibender Chipfläche.

    Also wenn die QM nicht funktionieren würde, dann hätte Dein heutiger Prozessor eine Chipfläche von gut 1000 qcm, also mal locker das Format eines großen Pizzatellers (aktuelle Wafer haben 300mm Durchmesser und sind rund, aus einem Wafer werden mehrere Hunderte bis Tausende Chips, Dein Prozessor wäre bei >31cm Kantenlänge quadratisch), würde schätzungsweise 50kW elektrische Leistung aufnehmen (müsste also mit Flüssigstickstoff gekühlt werden) und die Taktrate dürfte wahrscheinlich 10 MHz nicht übersteigen, da durch die langen Strecken zusammen mit den daraus resultierenden Kapazitäten das Signal stark zeitversetzt in den einzelnen Teilen des Chips(?!?) ankommen würden.

    Ach ja, auf einen Kaufpreis jenseits von 100.000 Euro für den Prozessor alleine würde ich mich auch einstellen, der ganze riesige Wafer müsste ja fehlerfrei sein und das ist selten.

    Und nu Du noch mal: Wie baue ich einen Chip mit 30nm Strukturbreite wenn ich nur eine Lichtquelle mit 193nm habe und die QM Quatsch ist?

  162. #162 Dr. Webbaer
    3. Januar 2011

    @kindermund
    Sie haben ja einen ganz netten Einwurf mit dem Objektcharakter des Raums gehabt, Dr. Webbaer hat das weiter oben denn auch goutiert; nichtsdestotrotz sind Sie der Aufforderung substanziell Ihren Einwurf (wir erinnern uns: Raum kein Objekt, Raum ohne Eigenschaften, Raum zumindest von der SRT falsch verstanden – ein Objekt ist nichts Mystisches, ist ein Ding, eine Sache, ein Gegenstand, ein Konzept(!)) zu begründen nicht nachgekommen.

    Einige Physikerkollegen waren anfänglich ein wenig zögerlich auf das Objekt-Argument einzugehen, Objekte spielen in der Physik (vom Begriff her) keine besondere Rolle, so dass an dieser Stelle noch ein wenig Raum bleibt auf das Objekt (ob-iacere, entgegen-/wegferfen, das Entgegengeworfene – vgl. auch adiacere, subiacere und anderes) an sich einzugehen.

    Also, warum soll der Raum als offensichtliches Containerobjekt mit offensichtlichen (ihm zugesprochenen 🙂 Eigenschaften kein Objekt sein? – Vermutlich stören Sie sich am von der RT angenommenen Abhängigkeit vom Beobachterstandort?! – Hmja, gut, aber die Objekteigenschaft “Krümmung” und die “Verschränkung” von Raum nimmt dem Raum ja nicht die Fähigkeit ein Objekt zu sein!

    Es gibt – wie es bspw. auch theoretische Entitäten gibt – auch theoretische oder virtuelle Objekte.

    Da nichts mehr zu kommen scheint von Ihnen würde der Webbaer – mit Verlaub!, Ihr Einverständnis vorausgesetzend und bei allem Respekt – nun langsam den Sack zumachen wollen…
    BTW, war das ein Zufall, dass “noanna” verschwand und Sie soz. inkarnierten? Kennen Sie den delinquentischen Nutzer mit dem anderen Namen?

    Beste Grüße!
    Dr. Webbaer

  163. #163 Niels
    3. Januar 2011

    @TheBug

    die SRT brauche ich für den Speichermechanismus bereits bei der CD

    Echt? Wozu?

  164. #164 MartinB
    3. Januar 2011

    @TheBug
    Ohne QM gäbe es doch nicht mal ein Bändermodell für Halbleiter, oder? Ich hatte irgendwann mal für ne Diskussion bei den Artefakten die Nobelpreisrede von (ich glaube) Shockley zitiert – der sagte, ohne solides QM-Verständnis und Bändermodell hätte man nicht mal den Transistor bauen können.

  165. #165 kindermund
    3. Januar 2011

    @ MartinB

    Ich habe jetzt nur den Abstract gelesen, dort steht jedenfalls nicht “Außer den Mathematikern braucht kein Mensch Quantenphysik.”
    Insofern ist dieses Paper wohl kein beleg für kindermundlichen Blödsinn.
    Und nein, der Satz war keine Polemik, Ihr Argument war “Elektronen kann man nicht sehen, deshalb weiß man nicht, ob es sie gibt.” Meine Beispiele zeigen, dass das ein albernes Argument ist.

    Es wäre schön, wenn Sie etwas aufmerksamer lesen würden. Ich schrieb: “trotzdem irreführend, denn worum es geht, kann man u.a. hier nachlesen”. Das Papier befasst sich mit der Frage, ob man angesichts pragmatisch funktionierender Elemente von QT davon ausgehen darf, daß die konzeptuellen Elemente zutreffend oder auch nur vorhanden sind (p.3).
    Diese Frage ist von besonderer Bedeutung, weil der Mißstand auf den ich hier aufmerksam mache eben jener der Verdinglichung ist. Es werden Konzepte – etwas das exklusiv in den Gedanken von Menschen existiert – so dargeboten, also ob sie keine Konzepte sondern Realität wären.

    Es ist die Verdinglichung von “Raum”, die es ermöglicht RT ohne auf all die Fehler in der Mathematik, die fehlenden experimentellen Beweise oder gar die Ränkespiele hinter den Kulissen eingehen zu müssen, als irrational zu erkennen. Dort wo angeblich RT funktioniert, funktionieren in Wahrheit Kepler und Newton. Zeitdilatation ist exakt der selbe Fall: Zeit ist ein menschliches Konzept, man kann sie willkürlich einteilen und nicht ohne die Zuhilfenahme von Bewegung oder Prozessen wahrnehmen. Wer Konzepte staucht und dehnt, staucht und dehnt seine Gedanken, sonst nichts…

    Und die QM/QFT ist also erfolglos. O.k., sie hat uns die Festkörperphysik beschert, macht Tausende korrekter Vorhersagen, ermöglicht Erklärungen von der subatomaren Skala bis hin zur Astrophysik, aber ansonsten war sie na klar ziemlich erfolglos. (Ja, *das* war jetzt Polemik.)

    Na, da sind wir uns im letzten Satz ja mal einig. Wie schön. Aber im ernst, was die mathematische Physik hat, ist ein Flickenteppich von Ausdrücken, die in bestimmten und ganz selten allen bekannten Situationen ausreichend präzise Aussagen über Materie machen kann. Das es nach wie vor ein Flickenteppich ist, sieht u.a. man daran, daß QT und RT noch immer unvereint sind und die ganze Mathematik nur so eben funktioniert, wenn ständig neue Teilchen erfunden werden. Das bedeutet nichts anderes, als daß es sich um ein Ratespiel handelt, nicht um Wissenschaft. Es ist irrational zu glauben, daß Teilchen Vorzeichen einer Gleichung ändern können. Das ist Verdinglichung der albernsten Sorte.
    Wenn Sie denn schon an Teilchen glauben wollen, dann müssen Sie z.B. erklären, wie Teilchen MECHANISCH funktionieren. Vor allem aber, müssen Sie in der Lage sein zu zeigen (Foto, Illustration) wie so ein Atom und seine Subteilchen AUSSEHEN. Ich sage voraus, daß Sie schon am Elektron scheitern werden.

    Es gibt übrigens Leute, die das können, doch die finden Sie nicht unter Ihresgleichen, denn das sind natürlich “Cranks” – während Erwachsene die an 0-dimensionale Teilchen glauben, die dann auch noch kollidieren können, Ladung oder Farbe tragen und Vorzeichen in Gleichungen ändern können, Wissenschaftler sind. Brave new world…

    Wenn Sie sich aber mit Wellen rausreden wollen, dann müssen Sie das MEDIUM benennen, in dem diese Wellen propagieren oder Sie bleiben auf der Ebene irrationalem Aberglaubens stecken.

    @ Nihil Jie / Webbear

    ich verstehe diese debatten um den begriff des objekts nicht. der begriff ist doch absolut universell.

    Universell ist der Begriff nur in der Mathematik, wo er gewissermaßen einen beliebigen Behälter darstellt. In der Physik ist ein Objekt etwas das Gestalt hat, ein Gegenstand, ein Ding und dieses können Sie ohne sein Gegenteil, den Raum, weder definieren noch erkennen. Einsteins Raum-Zeit verbindet zwei Konzepte, die nichts miteinander zu tun haben, verdinglicht sie und behandelt sie, je nach Bedarf, mal als real, mal als Abstraktion.

    @ The Bug

    Typischer Crank-Fehler, geht den Kreationisten und Klimatrollen auch ständig so. Einfach auf ein Paper verweisen, dass einen netten Titel hat, oder sogar noch eine schöne Einleitung geht voll in die Hose. Den eigentlichen Text sollte man auch lesen, der ist nämlich ein Tritt in Deinen Hintern.

    Typischer Mathetrottel Fehler: Symbole sehen aber keine inhaltlichen Zusammenhänge herstellen können.

    @ Webbear

    Sie haben ja einen ganz netten Einwurf mit dem Objektcharakter des Raums gehabt, Dr. Webbaer hat das weiter oben denn auch goutiert; nichtsdestotrotz sind Sie der Aufforderung substanziell Ihren Einwurf (wir erinnern uns: Raum kein Objekt, Raum ohne Eigenschaften, Raum zumindest von der SRT falsch verstanden – ein Objekt ist nichts Mystisches, ist ein Ding, eine Sache, ein Gegenstand, ein Konzept(!)) zu begründen nicht nachgekommen.

    Streng genommen bin ich Ihnen zuvorgekommen: “Tatsache ist aber, daß die Idee eines “gekrümmten Raumes”, entgegen Ihrer Behauptung, nichts mit Physik zu tun hat. Raum hat keine Eigenschaften, er ist das Gegenteil von Objekt, etwas, das Eigenschaften hat. Sie können nichts “krümmen”, das eigenschaftslos ist!”
    https://www.scienceblogs.de/hier-wohnen-drachen/2010/12/spezielle-relativitatstheorie.php#comment171378
    Ihr weiterer Vortrag ist etwas wirr, denn er scheint mir meine Position auf den Kopf zu stellen.

    BTW, war das ein Zufall, dass “noanna” verschwand und Sie soz. inkarnierten? Kennen Sie den delinquentischen Nutzer mit dem anderen Namen?

    Das ist Zufall. Ich kenne “noanna” nicht, aber die vorgetragene Kritik ist stichhaltig, ganz gleich ob abgeschrieben oder von “noanna” selbst verstanden. Leider war ‘noannas” Link pauschal judenfeindlich und die berechtigte Kritik an Einstein wird auf den Seiten für rechte Propaganda mißbraucht, Herr Bäkers Eingriff an der Stelle war meines Erachtens gerechtfertigt und maßvoll.

  166. #166 MartinB
    3. Januar 2011

    @kindermund
    “Wenn Sie sich aber mit Wellen rausreden wollen, dann müssen Sie das MEDIUM benennen”
    Nö, muss ich nicht. Wenn die mathematische Beschreibung der Welt mit Wellen (oder etwas moderner ausgedrückt Feldern) ohne tragendes Medium am besten funtkioniert, dann ist das die Beschreibung, die ich wähle – ob sich ein kindermund das anschaulich vorstellen kann, ist mir herzlich egal. Ich muss mir auch keine mechanischen Ersatzbilder von Teilchen bauen, nur weil das der naiv-anschaulichen Physik entspricht.

    Das zitierte Quantenmechanik-Paper enthält wirklich nichts besonders neues, auch wenn es viele Dinge schön zusammenfasst. Sie können ja den Autor mal fragen, ob er der meinung ist, dass Ihre Ideen durch sein paper gestützt werden…

    Und Ihre Aussage, außer den Mathematikern bräuchte niemand die QM ist immer noch so falsch wie am Anfang, auch wenn Sie sich jetzt rauszuwinden versuchen.

    “Raum hat keine Eigenschaften”
    Vielelicht stampfen Sie dabei noch mit dem Fuß auf, dann haben Sie bestimmt recht…
    Ein Argument für diese Behauptung fehlt nach wie vor.

  167. #167 TheBug
    3. Januar 2011

    @nils: Lichtgeschwindigkeit und zwar die im Medium ist ein Faktor bei den optischen Speichern. Im Gegensatz zur populären Meinung funktionieren CD, DVD und Blueray nicht dadurch, dass die Pits nicht reflektieren, sondern dadurch, dass das Pit Lambda/4 tief ist und der Laserstrahl etwa doppelt so breit wie ein Pit, somit also die Hälfte des Lichts um Lambda/2 versetzt reflektiert wird. Also SRT und QM. Einer meiner E-Technik Profs hat sich da mal in einer Vorlesung gewundert warum das Pit nach seiner Meinung zu flach ist, habe ich ihm dann nach der Veranstaltung kurz erklärt.

    @MartinB: Ja, komplett verstehen kann man die Halbleiter nicht ohne QM, aber benutzen schon. Kristalldioden sind schon im 19. Jahrhundert eingesetzt worden. Der Transistoreffekt (Bipolartransistor) wurde durch Messungen an Dioden entdeckt, also nicht zuvor theoretisch hergeleitet.

    @kindermund: Wer auch immer Dir erzählt hat Du hättest das Recht auf ein Universum das einfach zu verstehen ist, ist ein Vollidiot.

    Die Frage nach der Farbe und dem Aussehen von (Subatomaren-)Teilchen ist im allerbesten Fall infantil und es gibt nicht nur mechanische Zusammenhänge, sonst müssten wir Deinen Schrott hier nicht lesen, weil es dann kein Internet gäbe, ganz zu schweigen von kein Leben.

    Nikolic stellt in seinem Paper klar was der tatsächliche Wissensstand ist, nicht mehr und nicht weniger.

    Wenn Du nicht in der Lage bist die Zusammenhänge zu verstehen ist das Dein Fehler und nicht der der Theorien. Ich jedenfalls, und das dürfte für viele Leute hier zutreffen, habe sehr wohl ein klares Verständnis von der SRT und ihren Zusammenhängen. Nur sagt mir meine Erfahrung, dass man die SRT ohne Verständnis der Mathematik nicht erfassen kann und da sind wir dann bei Deinem Defizit. Da Du in religiöser Form alles ablehnst was ausserhalb des Erfahrungshorizontes der Hausfrau stattfindet wird das nichts werden.

  168. #168 MartinB
    3. Januar 2011

    @TheBug
    Ich hab eben nochmal nachgeguckt – es war bardeens lecture, die ich meinte:
    https://nobelprize.org/nobel_prizes/physics/laureates/1956/bardeen-lecture.pdf
    Darin sagt er ziemlich klar, dass Qm und Bändermodell Voraussetzungen für den Bau von Transitioren waren:

    It was dependent both on the sound theoretical foundation largely built up
    during the thirties and on improvement and …

    Vielelicht hätte man einen Transistor auch irgendwie ohne QM zusammentackern können, aber ob man damit vernünftig hätte auslegen und designen können, bezweifle ich.

  169. #169 Niels
    3. Januar 2011

    @TheBug
    Na ja, für die Lichtgeschwindigkeit in Medien brauch ich aber nur die Maxwell-Gleichungen, oder?
    Was genau muss ich denn für CDs wissen, dass nur die SRT mir liefern kann?

  170. #170 kindermund
    3. Januar 2011

    @ MartinB

    Wenn die mathematische Beschreibung der Welt mit Wellen (oder etwas moderner ausgedrückt Feldern) ohne tragendes Medium am besten funtkioniert, dann ist das die Beschreibung, die ich wähle – ob sich ein kindermund das anschaulich vorstellen kann, ist mir herzlich egal. Ich muss mir auch keine mechanischen Ersatzbilder von Teilchen bauen, nur weil das der naiv-anschaulichen Physik entspricht.

    Schön. Dann definieren Sie doch bitte kurz “Welle” in der Physik (sie wissen schon, real), mit besonderem Augenmerk auf den Umstand, daß ein Medium fehlt. Und wenn Sie gerade dabei sind, vergessen Sie nicht auch “Feld” zu erklären.
    “Am besten funktioniert”… interessant. Es funktioniert so “gut”, daß Sie bis heute nicht wissen was Gravitation ist und ständig neue Teilchen erfinden müssen, um die mathematische Scheinwelt am Laufen zu halten. Wenn es zu diesem Voodoo keine Alternative gäbe, könnte man ja noch sagen “warten wirs ab, vielleicht kommt der Durchbruch noch”, aber so wie es läuft, türmen Sie nur Epizirkel über Epizirkel, ohne der Realität auch nur das geringste Bischen näher zu kommen, während Ihre Zunft Leute die Wissenschaft betreiben als “Cranks” diffamiert.

    Und Ihre Aussage, außer den Mathematikern bräuchte niemand die QM ist immer noch so falsch wie am Anfang, auch wenn Sie sich jetzt rauszuwinden versuchen.

    Unsinn. Siehe oben.

    Vielelicht stampfen Sie dabei noch mit dem Fuß auf, dann haben Sie bestimmt recht… Ein Argument für diese Behauptung fehlt nach wie vor.

    Na, jetzt wirds aber immer dünner. Beantworten Sie doch einfach mal die Frage, das würde Ihnen helfen zu verstehen. Oder zeichnen Sie mir Ihre Partikel und Wellen mal auf – als Illustration. Nach 100 Jahren in denen mit Atomen und ihren Teilchen herumgespielt wurde, müssten Sie doch wenigstens wissen, wie die aussehen, oder?!

    @ TheBug

    Wer auch immer Dir erzählt hat Du hättest das Recht auf ein Universum das einfach zu verstehen ist, ist ein Vollidiot.

    Wer immer Ihnen erzählt hat das Universum scherte sich um Mathematik, ist ein Vollidiot.

    Die Frage nach der Farbe und dem Aussehen von (Subatomaren-)Teilchen ist im allerbesten Fall infantil und es gibt nicht nur mechanische Zusammenhänge, sonst müssten wir Deinen Schrott hier nicht lesen, weil es dann kein Internet gäbe, ganz zu schweigen von kein Leben

    Möglich, aber nicht ganz so infantil, wie über Sachen schreiben, von denen man keine Ahnung hat. Google mal “gluon” und schau mal welche Eigenschaften das Selbe haben soll. Vielleicht verstehst Du dann worauf ich anspielte. Ich frage nach der mechanischen Erklärung weil man noch immer munter von subatomaren Teilchen spricht, nicht etwa weil ich viel von Teilchen hielte.

    Nur sagt mir meine Erfahrung, dass man die SRT ohne Verständnis der Mathematik nicht erfassen kann und da sind wir dann bei Deinem Defizit.

    Sieh an, so gut kennen wir uns? Wer Konzepte nicht von Realität unterscheiden kann oder mit undefinierten Begriffen hantiert, um dem Publikum vorzugaukeln man spräche über Realität demonstriert damit nachhaltig seine Defizite, da kann man schon verstehen, das es bequemer ist auf andere zu zeigen, das lenkt ab.
    A/SRT ist ganz offensichtlich irrational, weil Konzepte synonym mit Realität behandelt werden. Was an RT funktioniert sind die Elemente die sich Herr Einstein von Newton “geliehen” hat, sonst nix. So einfach ist das, da hilft auch die ganze Heulerei der Mathematiker nichts….

  171. #171 MartinB
    3. Januar 2011

    @kindermund
    Feld: An jedem Punkt der Raumzeit definierte Größe.
    Tut mir Leid, wenn das deiner Idee
    “Wer immer Ihnen erzählt hat das Universum scherte sich um Mathematik, ist ein Vollidiot. ”
    widerspricht. Solange Physiker Computer bauen können und kindermünder nur schwafeln, weiß ich, wer die realität besser versteht.

    Und Atome sehen nicht aus, weil sie kleiner sind als eine Lichtwellenlänge, deshalb kann man sie auch nicht sinnvoll zeichnen.

    Das wird mir jetzt wirklich zu kindisch.

  172. #172 threepoints...
    3. Januar 2011

    kindermund· 03.01.11 · 08:58 Uhr

    @ The Bug

    Typischer Crank-Fehler. Einfach auf ein Paper verweisen, dass einen netten Titel hat, oder sogar noch eine schöne Einleitung geht voll in die Hose.

    Typischer Mathetrottel Fehler: Symbole sehen aber keine inhaltlichen Zusammenhänge herstellen können.

    -> Selten so amüsantt in den Tag geholt.

  173. #173 walim
    3. Januar 2011

    @MB
    Ohne es genau zu wissen, hatte ich ( 02.01.11 · 22:54 Uhr) sowas wie M.B.10:28 Uhr ungefähr gemeint, war dabei nicht auf das Bändermodell gekommen und hatte dabei auch praktische Anwendung und Theorie verwechselt, was ja nicht zwingend immer in einer definierten Reihenfolge ablaufen muss.
    Da drängt sich mir die Frage auf: Wo gibt es eigentlich Bereiche, in denen man technisch etwas anwendet, das eher unzulänglich verstanden wird (“da müssen wir noch diesen seltsamen Dingsbumskoeffizienten berücksichtigen”)?
    Ich würde da zum Beispiel an die Supraleitfähhigkeit denken, oder ist da das Verständnis schon so weit komplettiert?

  174. #174 threepoints...
    3. Januar 2011

    TheBug· 03.01.11 · 09:44 Uhr

    “@kindermund: Wer auch immer Dir erzählt hat Du hättest das Recht auf ein Universum das einfach zu verstehen ist, ist ein Vollidiot.”

    -> Scheisse noch mal, es ist doch genau umgekehrt. Wenn ich nur 1+1 zusammenzählen muß, dann machts doch keinen Sinn…sich Mühe zu geben. Das kann nun wirklich jeder auf 2 “hochrechnen” …

  175. #175 TheBug
    3. Januar 2011

    @kindermund: “Nach 100 Jahren in denen mit Atomen und ihren Teilchen herumgespielt wurde, müssten Sie doch wenigstens wissen, wie die aussehen, oder?!” Ja wissen wir, sie sehen nicht aus, weil das ein unzutreffender Begriff in diesem Zusammenhang ist. Nur weil in Deinem Kopf nur bunte Murmeln sind muss nicht der Rest der Welt was damit zu tun haben.

  176. #176 TheBug
    3. Januar 2011

    @nils: Ja streng genommen reicht Maxwell. Primär ging es ja um die Frage wie Computer ohne funktionierende QT aussehen würden.

  177. #177 threepoints...
    3. Januar 2011

    kindermund· 03.01.11 · 08:58 Uhr

    “Raum hat keine Eigenschaften, er ist das Gegenteil von Objekt, etwas, das Eigenschaften hat. Sie können nichts “krümmen”, das eigenschaftslos ist!”

    -> So schwirrts mir auch im Kopf herum. Weshalb ich Raum und Inhalt irgendwie verbinden wollte – in etwa auch an der Frage, was Raum ist, wenn sonst nichts ist mit dem Raum – also keine Eigenschaften (ausser absolute Maße). und also mündete diese fragestellung in der Annahme, der raum könne nur sein, wenn ein Inhalt vorhanden. Was dazu führt, das der Inhalt den Raum entstehen lässt. Und dann kann man den Raum auch noch ganz anders interpretieren. Etwa der Stein als Raum, ein See als Raum, ein Magnetfeld als Raum, …jedes sich abgegrenzenste und zusammenhängende Volumen als Raum … also auch jede Materie als Raum. Das führt aber zur Frage, was da ist, wo Raum ist, aber offensichtlich keine Materie…. da nämlich, wo nur HG-Strahlung gemessen werden kann, … aber keine Materie messbar ist.

  178. #178 threepoints...
    3. Januar 2011

    Wir müssen wohl diese Diskussion aus dem Wissenschaftsblog in den PolitikBlog verschieben. Es nimmt hier Aussmasse an, ohne auch nur kleinste Ergebnisse zu liefern. Und dafür ist doch dann die Kunst der Politik zuständig … oder?

  179. #179 TheBug
    3. Januar 2011

    @MartinB: Schockley, Brattain und Berdeen waren nicht die “Erfinder” des Transistors oder Entdecker des Transistoreffektes, sie waren diejenigen, die es geschafft haben die Entdeckung nutzbar zu machen und die grundlegenden wissenschaftlichen Zusammenhänge geklärt haben. Der Halbleitereffekt war vor der Definition der QT entdeckt worden und ein Faktor dafür sich von der klassischen Physik zu verabschieden, weil er wie die elektrische Leitung insgesamt nur unzureichend ohne QT erklärt werden konnte.

  180. #180 TheBug
    3. Januar 2011

    @threepoints: Gibts noch eine Antwort auf die Frage wie Du denn nun moderne Computer ohne Quantentheorie baust?

  181. #181 S.S.T.
    3. Januar 2011

    @kindermund

    Nach 100 Jahren in denen mit Atomen und ihren Teilchen herumgespielt wurde, müssten Sie doch wenigstens wissen, wie die aussehen, oder?!

    So ein Rastertunnelmikroskop gibt schon mal eine Vorstellung davon, inklusive vieler bunter Bilder.

  182. #182 threepoints...
    3. Januar 2011

    TheBug· 03.01.11 · 11:49 Uhr

    @threepoints: Gibts noch eine Antwort auf die Frage wie Du denn nun moderne Computer ohne Quantentheorie baust?

    -> Ich hab keine Ahnung davon und brauch nicht zu antworten … machte es doch keinen Sinn, zu bestätigen, was ich nicht verstehe…oder?

    Gibts noch eine Antwort darauf, warum noch nicht alles gewusst und erklärt werden kann, was wir uns einbilden können? Etwa Gott? … oder den Urknall? … vielleicht auch die Gravitation?

    Ach ja, das ist ja die Bodenhaftung, die uns immer wieder runterzieht … und uns den Dingen auf den Grund gehen lässt. Weil wir so “geerdet” sind und uns quasi “selbstbeschränkt” auf den Boden der Tatsachen bleiben…

  183. #183 Niels
    3. Januar 2011

    @TheBug
    Ist jetzt nicht bös gemeint und stärkt deine Position auch.

    Die Halbleitertechnik wäre ohne funktionierende QM und SRT etwa 1990 stehen geblieben.

    Soweit ich die Sache verstehe wäre man ohne QM bei weitem nicht erst 1990 stehen geblieben, sondern schon etliche Jahrzehnte vorher.
    Wie weit man ohne relativistische Korrekturen des Bändermodelles gekommen wäre ist mir nicht ganz klar, aber bis zum Stand von 1990 höchstwahrscheinlich ebenfalls nicht.

  184. #184 TheBug
    3. Januar 2011

    @nils: Nehme ich auch nicht böse, ich meine nur, dass man gewissermaßen “handwerklich” vielleicht bis 1990 mit den Halbleitern klar gekommen wäre, also durch Anwendung von Erfahrungswerten. Die Ergebnisse wären natürlich nicht so gut gewesen wie sie mit dem notwendigen theoretischen Background sind. Aber etwa beim Unterschreiten von 1µm Strukturbreite wäre es damit endgültig vorbei gewesen. Ich wollte mit meiner Einschätzung den Flachweltlern so weit wie möglich entgegen kommen 🙂

  185. #185 TheBug
    3. Januar 2011

    @threepoints: Du brauchst also nicht auf Dinge zu antworten, die zu Deiner Crank-Physik nicht passen?

    Du behauptest RT und QT hätten keinen Wert und hast dann keine Kenntnisse um das zu belegen, statt dessen beschwerst Du Dich darüber, dass die Naturwissenschaft nicht alle Fragen beantworten kann. Um das dann abzurunden verlangst Du noch, dass Hingespinste (Gott) wissenschaftlich erklärt werden und vermischst das mit physikalischen Tatsachen zu denen noch keine vollständige Beschreibung existiert (Gravitation).

    Kennst Du den Unterschied zwischen “spinnen” und “forschen”? Anscheinend nicht, bzw. scheinst Du erstem einen höheren Wert als zweitem zuzumessen.

    Du lebst in einer Traumwelt und behauptest, dass Dinge die Du wie selbstverständlich benutzt nicht existieren können. Sehr viel schizophrener geht es wirklich nicht mehr.

  186. #186 threepoints...
    3. Januar 2011

    Mögen doch die “Überflieger” den “Flachweltlern so weit wie möglich entgegen kommen” …dass sie (die Überflieger) davon am meisten haben…. wie wir wissen, ist es dann “höchstmöglich”, dass für den Mob noch was übrig bleibt…

    Ich sprehe ja gerne so über manch andere… aber allermeist zu mir selbst … damit die Hoffnung verbunden, es keiner mitbekommt… und ich nicht ganz so arg brüskier…

    … gelingt mir aber nicht besonders erfolgreich (weil Erfolg hier nur 100 % sei)

  187. #187 Ulrich Berger
    3. Januar 2011

    Faszinierend. Bei so einem Crank-Auflauf wie da oben wünscht man sich fast Hartwig Thim zurück.

  188. #188 threepoints...
    3. Januar 2011

    TheBug· 03.01.11 · 12:49 Uhr

    “Du behauptest RT und QT hätten keinen Wert …”

    -> Und ich keine Ahnung davon habe…..

    … und vor allem habe ich niemanden gezwungen, derart zu assozieren, dass mir eine Verachtung der anerkannten Wissenschaft unterstellt wird.

    ich habe nicht behauptet, dass alles (RT, QT, … usw.) scheisse ist …und völlig neben der “Realität” …. woraus sich ergibt, dass, wie ich mit meiner These, dieser Teil meiner Ablehnung der Wissenschaften in Eimbildung durch meinen kontrahenten (hier im Blog) hinzugedichtet wurde.

    Also alles beim alten … und wie gehabt. So unentschieden hats noch nie gestanden.

  189. #189 kindermund
    3. Januar 2011

    @ MartinB

    Solange Physiker Computer bauen können und kindermünder nur schwafeln, weiß ich, wer die realität besser versteht.

    Weder bauen Physiker Computer (das machen Ingenieure) noch schwafele ich, aber Sie haben natürlich ein Recht auf Ihre mathematische Narrenwelt.

    Wie gesagt, weil man Aspekte der Realität näherungsweise berechnen und so teilweise beherrschbar machen kann, heißt das noch lange nicht, daß der benutzte Symbolismus Realität in ihrer Gesamtheit zutreffend abbildet und über die Gestalt sagt es schlicht nichts aus. Klassische Beispiele sind Kepler und Newton, die weder eine Ahnung von subatomaren Teilchen, noch von Q** hatten, und deren Formalismen trotzdem nach wie vor Teilaspekte der Realität ausreichend berechenbar machen. Im Gegensatz zu vielen der heutigen “Mathysikern” war denen aber bewußt, daß es sich nur Formalismen handelt und versuchten nicht sich Phantasiegebilde zur Begründung auszudenken. Und letzten Endes ist es so, daß wenn man sich durch den mathematischen Müll kämpft kommt man in aller Regel bei Gleichungen an, die VOR RT/QT entstanden (QT weniger als RT, die wirklich nur eine -schlechte- Umformung + etwas Unsinn ist). Die ganze “moderne” mathematische Physik lebt von den Kadavern vergangener Zeiten (Kepler, Netwon, Lorentz, Maxwell etc.).

    Ihre strikte Weigerung auch nur in geringster Weise inhaltlich auf meine Aussagen einzugehen, zeigt schon recht deutlich, daß einer von uns beiden tatsächlich ein Schwafler ist.

  190. #190 threepoints...
    3. Januar 2011

    Arthur Conan Doyle schrieb wohl einmal:

    “When you have eliminated the impossible, whatever remains, however impossible, must be the truth.”

    Das passt hier mal ganz gut zur Diskussion. Was nicht heisst, dass die Diskussion in diesen Blog passt…

  191. #192 nihil jie
    3. Januar 2011

    @threepoints & kindermund

    der raum hat sehr wohl eigenschaften… es beinhaltet etwas… zb. energie, vakuum, felder usw…. gebe es das alles nicht gäbe es womöglich auch keinen raum. das schräge bei der geschichte ist auch, dass ohne raum auch die anderen dinge wohl nicht existieren würden. es geht hier jetzt nicht um die frage was früher da war… das huhn oder das ei… es geht in unseren fall eher da drum, dass ohne eier keine hühner existieren würden und ohne hüner keine eier. es bedingt sich halt gegenseitig. ohne raum kein inhalt und ohne inhalt kein raum. so… scheint es… ist unser universum beschaffen.

    @kindermund
    das wort objekt sollte man in der physik echt nicht überstrapazieren. es gibt in der physik wohlbekannte grössen und die sollte man auch benutzen. zumindest wenn damit gerechnet wird. das wort objekt dient lediglich zum umschreiben udn beschreiben. damit rechnet man nicht. ausser dem sagt man nicht einfach nur “das objekt”. wenn man das wort objekt benutzt dann hängt man auch gleich dran um welches es sich handle… man benutzt es nie separat sondern nur in zusammenhang.

  192. #193 MartinB
    3. Januar 2011

    “Weder bauen Physiker Computer (das machen Ingenieure) ”
    Echt? Also ich kenne ne Menge Physiker in der Halbleiterindustrie…

    “kommt man in aller Regel bei Gleichungen an, die VOR RT/QT entstanden”
    Dafür hätte ich gern ein paar beispiele (nein, nicht den beweis, dass die newtonschen Axiome der klassische Grenzfall der Qm sind, den kenne ich.) Also: Welche Gleichungen der QM formt man wie um, dass man bei “klassischen” gleichungen landet und wo ist das praxisrelevant?

    Ich kann ansonsten inhaltlich nicht auf Ihre Aussagen eingehen, weil Sie keine Argumente bringen, sondern nur apodiktische Behauptungen aufstellen (“Raum hat keine Eigenschaften”, “Wer behauptet, dass es Elektronen gibt, muss sagen können, wie sie aussehen” usw.) – gegen Glaubenssätze kann man nicht argumentieren.

    @theBug, Niels
    Ich sehe echt nicht, dass man ohne Bändermodell sinnvoll Halbleiterbauteile auslegen könnte – nein einzelnen transistor vielleicht, aber nen integrierten Schaltkreis? Nen entarteten Halbleiter? Ne Photodiode? Alles nur halb-klassisch mit Beweglichkeiten etc.?

    @walim
    “Wo gibt es eigentlich Bereiche, in denen man technisch etwas anwendet, das eher unzulänglich verstanden wird (“da müssen wir noch diesen seltsamen Dingsbumskoeffizienten berücksichtigen”)? ”
    Ich glaube, da ist die Kernphysik ein Kandidat (bin aber kein Experte), da gibt es ziemlich viele semi-empirische Modelle 8zumindest war das so, als ich studiert habe).

  193. #194 nihil jie
    3. Januar 2011

    @kindermund

    Die ganze “moderne” mathematische Physik lebt von den Kadavern vergangener Zeiten (Kepler, Netwon, Lorentz, Maxwell etc.).

    und ich verstehe schon wieder etwas nicht… denn Kepler, Netwon, Lorentz, Maxwell etc. leben auch von den kadavern vor ihnen lebender philosophen und naturforscher. der erweb von wissen ist ein kontinuierlicher prozess. im grunde kann man sich gleich auch bei unseren urmenschlichen vorfahren bedanken dass sie zb. das feuer anfingen zu beherrschen und aus steinen speerspitzen anfertigten…. geschweige denn, für den bau steinernder himmelsbeobachtungs vorrichtungen 😉 alles absolut grandiose leistungen die sie da vollbrachten, die uns letztendlich auch zur der quantenmechanik und RT geführt haben.

  194. #195 TheBug
    3. Januar 2011

    @nihil jie: Na dann fangen wir vorne an und beten zu unserem Schöpfer: Liebe Evolution…
    Oder bist Du eher dem Urknall zugetan?

    😉

    @MartinB: Die Frage ist wohl schwer zu klären wie weit die “Handwerker” ohne die Physiker gekommen wären, da es ja nun mal so nicht passiert ist, sondern die Theorien entwickelt wurden um zu verstehen was man tut, statt wild zu probieren.

    Und wir sollten jetzt endlich klären welche Farbe ein Rad haben muss, damit es funktioniert.

    Ich komm mir langsam vor als wäre ich bei den Telefondesinfizierern, ach verdammt, da war doch so eine Geschichte mit einem Raumschiff…

  195. #196 kindermund
    3. Januar 2011

    @ MartinB
    Ja klar, Sie weigern sich meine inhaltliche Kritik – das Symbole nichts über die Gestalt von Realität aussagen – auch nur als Argument anzuerkennen (“weil Sie keine Argumente bringen”) und ich setze mich jetzt hin und spiele mit Ihnen im Matheirrenhaus? Eher mal nicht….

    Wer wissen will wie schon von Anfang an Einsteins Scharlatanerie gegen enorme Bedenken namhafter Wissenschaftler durchgesetzt wurde, der wirft einen Blick in die Geschichte (100 Autoren gegen Einstein): https://preview.tinyurl.com/27smhdm

    @ nihil jie
    Das Aufbauen auf Erkenntnissen von Vorgängern ist selbstverständlich nicht verwerflich, darum geht es in diesem Falle aber nicht. Es geht darum, daß ein funktionierender Kern umgeformt, erweitert und pseudo-gestalterisch interpretiert wird, mit dem Ergebnis, daß sich ein höchst fragwürdiges Bild von Realität ergibt.
    Soweit von mir aus soweit auch noch machbar (wenn auch häßlich), wenn es nicht längst wesentlich EINFACHERE und ORIGINÄRE Modelle gäbe, die die Frage nach der Gestalt nicht von Symbolik sondern von Realität herleiten, aber massiv unterdrückt werden.
    In der QT gibt es ja durchaus interessante Insellösungen für praktische Probleme, deren Wert will ich auch nicht in Frage stellen, aber die Vermittlung dieser Symbolik als korrektes Abbild der Natur ist in sich zwingend unlogisch und falsch. Um dies zu verstehen, muß man lediglich den Unterschied zwischen Konzept und Realität zur Kenntnis nehmen. Wer allerdings die beliebigen, undefinierten Begriffe der Mathematik 1:1 in die Realität überträgt, spielt Pippi Langstrumpf…

  196. #197 Niels
    3. Januar 2011

    @MartinB
    Seh ich mit den Transistoren eigentlich auch so.
    Wie weit man mit Elektronenröhren und mechanischen Relais gekommen wäre kann ich schlecht beurteilen.
    Deswegen hab ich von einem Stopp “etliche Jahrzehnte vor 1990” ohne QM gesprochen.

    Der zweite Teil war gemeint als mit QM aber ohne SRT. (Falls so eine Entwicklung der Wissenschaft überhaupt denkbar wäre. Das würde ich eher bezweifeln.)

  197. #198 nihil jie
    3. Januar 2011

    @TheBug

    ich bin grundsätzlich dingen zugetan die einen sinn ergeben… 😉

  198. #199 Hel
    3. Januar 2011

    @Ulrich Berger

    Faszinierend. Bei so einem Crank-Auflauf wie da oben wünscht man sich fast Hartwig Thim zurück.

    Ersetzt denn die hier bereits aufgelaufene Grande Dame aller Einstein-Cranks, deren Phrasengenerator sich diesmal durch einen Kindermund artikuliert, nicht locker ein halbes Dutzend weiterer SRT-Leugner?

  199. #200 MartinB
    3. Januar 2011

    @kindermund
    Du zitierst “100 Autoren gegen Einstein”? Ernsthaft?

    “das Symbole nichts über die Gestalt von Realität aussagen – auch nur als Argument anzuerkennen”
    Natürlich sagen Symbole allein nicht notwendigerweise etwas über die Realität, deshalb muss man die mittels Symbolen abgeleiteten Schlussfolgerungen durch Epxerimente überprüfen. Dann kann man sagen: “Die Welt verhält sich so, dass sie mittels feldern, gekrümmten Räumen etc. beschrieben werden kann”. Entscheidend ist hier das “beschreiben”. Das ist Physik. Mehr geht nicht, muss aber auch nicht.

  200. #201 Niels
    3. Januar 2011

    @kindermund
    Zu dem Buch “100 Autoren gegen Einstein” hat Einstein selbst eigentlich schon alles nötige bemerkt:

    “Wenn ich unrecht hätte, wäre einer genug!”

    Hier kann man dieses Buch übrigens ganz einsehen, nicht nur das Vorwort.
    https://www.literature.at/alo?objid=17088
    Rechts auf PREVIEW klicken.

    Total überzeugend. Auf 68 Seiten geben hundert Autoren (nicht 100 Wissenschaftler oder gar Naturwissenschaftler!) jeweils ein etwa halbseitiges Statements ab, warum die RT falsch sein muss.
    Mathematik brauchen sie dazu offenbar ganz in deinem Sinne nicht.

  201. #202 nihil jie
    3. Januar 2011

    @kindermund

    nur mal ganz kurz weil ich es wirklich eilig habe 😉

    “symbolik” finde ich an sich nicht so tragisch. denn viele dinge sich unseren menschlichen erfahrung einfach entziehen. wir sind, als lebende wesen, an einen punkt vorgedrungen an denen wir zb. atome nicht mit einem blossen auge erfassen können und egal welche modele man einführt und welche rechnungen man anstellt wir werden sie niemals sehen können. nur mit hilfenahme von gerätschaften und im schlimmsten fall sind wir leider nur auf mathematik angewiesen. man rechnet mit grössen die man aber visuell nicht mehr erfassen kann. dann ist 1 femtometer nur mal 1 femtometer… egal ob ich ihr mit meinem verstand erfassen kann oder nicht. unsere einzige chance besteht bei dem ganzen vorhaben, um etwas nachzuweisen, in der suche nach effekten die man erwarten kann wenn unsere modele stimmen. und das tut man auch. es werden effekte vorhergesagt und man sucht nach ihnen in experimenten. sind sie so wie erwartet scheint unser model zu stimmen… wenn nicht… tja… dann haben wir wohl pech gehabt.
    und bis jetzt ist es nun mal so, dass alle bemühungen die effekte, die die RT vorhersagt, recht positive ergebnisse geliefert haben. also so falsch kann sie denn nun echt nicht sein.

  202. #203 MartinB
    3. Januar 2011

    @Niels
    Da sind ja genial gute ARgumente drin, schau mal auf S. 19:
    “Sie geht im übrigen einseitig von der optischen Erfahrung aus und vernachlässigt die Erkenntnisse des tastinns.”
    Na da guckste aber.

    Aber geschockt bin ich ja von S. 20, Autor Emanuel Lasker, der hätte vielleicht lieber mehr Schach spielen sollen (er war aber anscheinend mit Einstein trotzdem nicht ernsthaft verfeindet). Obwohl dessen Argument schon an kindermund erinnert…

  203. #204 TheBug
    3. Januar 2011

    Niedlich, 100 Autoren und ein Forumsposter beschweren sich darüber, dass die Relativitätstheorie zu kompliziert sei.

  204. #205 nihil jie
    3. Januar 2011

    nachtrag:

    und, dass leute von den gegnern der RT und der QM beweise (rechnungen, experimente usw) für ihre thesen verlangen, finde ich an sich nicht falsch. eher im gegenteil, es muss einfach sein sonnst wäre die wissenschaft ein akt der willkür und keine ersthafte forschung. im gunde hätte ich persönlich kein problem damit wenn an den theorien etwas nicht stimmen würde… bloss dass ist mit einer blossen behauptung nicht erledigt… es verlangt nach beweisen. es wäre natürlich schade um die ganze arbeit die man sich gemacht hat um eine theorie auf zu stellen und sie zu unterhalten… da steckt jede menge zeit und engagement drin. aber wenn man was besseres hätte dann würde man zum schluss womöglich auch entlohnt werden. im moment aber sieht es so aus als ob die RT oder QM das beste wäre was wir haben um die “realität” zu beschreiben (ich nehme das wort realität gerne in “” weil es ein recht schwieriger begriff ist)

  205. #206 MartinB
    3. Januar 2011

    @nihil je
    Die arbeit wäre ja auch nicht vertan – wie oben schon gesagt: Jede Theorie, die die SRT ersetzt, wird sie in irgendeiner Form erweitern und als Grenzfall enthalten, dafür ist die SRT einfach zu gut belegt.

  206. #207 TheBug
    3. Januar 2011

    Aber im Cranktinuum muss man nur drei mal laut rufen: “Ich glaub das nicht!” Und dabei fest mit dem Fuß aufstampfen, dann ist jede Theorie widerlegt.

  207. #208 Niels
    3. Januar 2011

    @MartinB
    Der Satz geht aber doch auch noch genial weiter, warum nämlich der Tastsinn wichtiger ist:
    “des Tastsinnes, der ursprünglicher als der Gesichtsinn ist, wie daraus hervorgeht, dass es viele Blinde, aber keine völlig Tastlosen gibt. Das Wort “Begreifen” deutet auffällig auf diese Ursprünglichkeit des Tastsinns hin.”

    Ja, das leuchtet ein. Da dämmert es mir. Da geht mir ein Licht auf.
    Das muss jeder einsehen, der Mann hat es durchblickt und durchschaut.
    Jetzt seh ich endlich klar.

    @TheBug
    “Ich glaub das nicht!” wäre ja ne Stufe harmoser, die sind aber bei “Ich weiß das alles besser! IHR musst mir das einfach glauben!”

  208. #209 nihil jie
    3. Januar 2011

    tja… jetzt sitze ich hier im cafe udn warte auf meine verabredung. es wir oft männern nachgesagt sie seien unpünktlich *tztz glaubt das nicht leute… frauen sind es genau so 😉

    @Martin aka Drachenmann 😉
    ja… das denke ich auch, dass mögliche zukünftige theorien auf der RT aufbauen können. im allgemeinen kann man auch beruhigt sagen, dass für die gewinnung von erkenntnissen auch das irren wichtig ist. denn man möchte nicht nur wissen was funktioniert sondern auch was alles nicht funktioniert 😉

  209. #210 Hel
    3. Januar 2011

    @TheBug

    Aber im Cranktinuum muss man nur drei mal laut rufen: “Ich glaub das nicht!” Und dabei fest mit dem Fuß aufstampfen, dann ist jede Theorie widerlegt.

    Yep, aber das ist ja leider noch nicht alles. Besonders hartnäckige Cranks wenden auch noch die Spiegelgesetze an, ein rhetorisches Mittel aus der Mottenkiste esoterischer Projektionspsychologie. In unserem Fall hier führt das dazu, dass nicht etwa die Cranks, sondern vielmehr die Physiker als “Gläubige”, Voodoo-Anhänger und Esoteriker dastehen sollen, denen es an Skepsis und Kritikvermögen mangelt. Ist doch ganz einfach:

    Nicht IHR seid skeptisch, ICH bin es.

    Nicht ICH bin gläubig, IHR seid es.

    Schließlich die Beweislastumkehr, noch fix ein paar Verschwörungstheorien hinzugefügt – und fertig ist der Crank-Bullshit.

    Falls Kindermund gerade ein paar Buntstifte zur Hand hat, möge sie doch einfach mal nur für sich dieses Bild abmalen, welches das Kernhüllenmodell eines Kohlenstoffatoms zeigt: https://www.tomchemie-forum.de/albums/userpics/10001/6.jpg

    Wäre Malen nicht auch ein viel schöneres Hobby als SRT-Leugnung?

  210. #211 S.S.T.
    3. Januar 2011

    100 Autoren gegen Einstein = 100 Kindermund-Aufsätze zu “Warum übersteigt die RT mein Verständnis”.

    @kindermund, geh buddeln, zwar bist Du eigentlich schon am absoluten Nullpunkt angelangt, aber ich bin fest überzeugt, Du schaffst es noch tiefer. Gib Dir noch ‘ne Chance! Gib Dir noch ‘ne Chance!

  211. #212 Hel
    3. Januar 2011

    @MartinB

    https://www.scienceblogs.de/hier-wohnen-drachen/2010/12/spezielle-relativitatstheorie.php#comment171716

    *ggg* Ich verstehe. Wenn du möchest, kannst du einen meiner beiden Kommentare dazu löschen, aber meinetwegen können sie auch so stehenbleiben.

  212. #213 MartinB
    3. Januar 2011

    @Niels
    Ja, ich war nur zu faul zum Abtippen. Das ganze Argument ist klasse, auch der Quatsch mit “das Relative wird zum absoluten Grundsatz erhoben” – das klassische “alles ist relativ”-Missverständnis in philosophisch klingender Sprache.

    Das lasker-Argument (wenn der leere Raum leer ist, muss die Lichtgeschwindigkeit unendlich sein, weil es ja nichts gibt woran sie sich messen könnte) ist wenigstens noch ein bisschen subtil (letztlich aber auch unsinn – das müsste dann ja auch für teilchen gelten, die den leeren Raum durchfliegen.)

    Für Philosphengeschwurbel empfehle ich noch S. 31f

    Umgekehrt ist auch Existenz ohne absolut gleichzeitige Dinge das absolute Nichts. Da die RTH die die absolute Gleichzeitigkeit bestreitet, so bestreitet sie…die empirische Wirklichkeit. So fabelhaft kurz und einfach lässt sich ein uzwingender Nachweis der empirischen und logischen Unhaltbarkeit dieser “Theorie” gestalten.

    Lothar Mitis ist auch zu empfehlen:

    Da aber c nur ein verschwindend geringer Bruchteil eines Lichtjahres ist…”

    Genau. Und die Schallgeschwindigkeit ist auch nur ein Bruchteil der Entfernung Ohr-Mund.

    In seiner Schlichtheit beeindruckend ist V. Nachreiner:

    Das Raumzeitding der RTH widerspricht der unbezweifelbaren Kant-Schopenhauerschen lehre von Zeit Raum und Materie.

    Auch Prof. Strehl hat wahrhaft fundiertes zu sagen

    Die Theorie von Einstein ist für mich eine funktionale Umformung der Wirklichkeit. Sein Bezugssystem: veränderlich Raum- und zeitmaßstab, unveränderliche Lichtgeschwindigkeit…ist nich mein Geschmack

    Na denn, Physik ist ja Geschmackssache.

    Dank an kindermund und Niels für die referenz, das war wirklich lustig zu lesen, auch wenn einem das Lachen ein bisschen im Hals stecken bleibt, wenn man überlegt, dass dieser Unsinn ernst gemeint war.

  213. #214 MartinB
    3. Januar 2011

    @Hel,
    nö, kein Problem, die lass’ ich so stehen.

  214. #215 rolak
    3. Januar 2011

    Hi MartinB, nee, verfeindet waren die beiden wirklich nicht. Zu diesem Geschehen aus längst vergangenen Zeiten ein indirektes Zitat aus einer Zeit vom letzten Jahr:

    Emanuel Lasker ist einer der stärksten Geister, denen ich auf meinem Lebensweg begegnet bin …
    Ich liebe seine Schriften unabhängig von ihrem Wahrheitsgehalt …

    🙂

  215. #216 MartinB
    3. Januar 2011

    @rolak
    Ja, interessant, wieso Lasker sich dann für so etwas hergab. Was da wohl für eine Geschichte dahintersteckt?

  216. #217 rolak
    3. Januar 2011

    Wäre vielleicht wirklich interessant, da es nach dem Namensregister scheint als habe Lasker diesen Kurztext extra für die Sammlung verfasst. Allerdings halte ich eine höchstwahrscheinlich ungemein aufwendige Recherche für nicht angemessen…

  217. #218 H.M.Voynich
    3. Januar 2011

    @MartinB:
    “Ja, interessant, wieso Lasker sich dann für so etwas hergab.”
    Wer weiß, woher diese 4 Sätze von Lasker stammen und ob er überhaupt um Genehmigung des Abdrucks gefragt wurde (ist bei einem so kurzen Zitat ja nicht nötig).

  218. #219 Niels
    3. Januar 2011

    @MartinB
    Mir gefällt auch

    Die RTH Einsteins mag in der Wissenschaft welche Rolle immer spielen, für die Philosophie ist sie entweder eine Banalität oder ein Unsinn.

    Damit ist dann ja wohl alles gesagt.

    Das ist auch nett:

    Ein schwerer rein logischer Fehler ist es auch, vom gekrümmten Raum zu sprechen, da “Krümmung” ein motorischer Begriff ist

    Dieser Professor hier hat bestimmt die Fields-Medaille bekommen.

    Wenn auch in dem Ausdruck (Wurzel aus 1- v^2/c^2) kleiner als 1 wird, so ist er trotzdem nicht imaginär. In einem Aufsatze in den “Annalen der Physik” habe ich den Nachweis geliefert, dass auch die negativen Zahlen immer reale Werte vertreten, woraus von selbst folgt, dass es keine imaginären Zahlen gibt. Dem Ausdruck unter der Wurzel entspricht daher ein realer Wert, auf keinen Fall bedeutet er “Größe unter Null”. Einsteins “Schluß” ist also überhaupt kein Schluß, zweitens wäre, wenn es ein wirklicher Schluß wäre, die Voraussetzung, auf die er sich gründet, falsch.

    ???
    Frage am Rande: Kann das stimmen, dass er einen Beweis über die Nichtexistenz der imaginären Zahlen in den Annalen der Physik veröffentlichen konnte?
    Ich dachte bisher, dass wäre eine hochseriöse Zeitschrift gewesen.

    Anmerkung am Rande: Dieses Buch lässt mich übrigens sehr stark an der von Jörg Friedrich postulierten Überlegenheit der damaligen humanistischen Bildung zweifeln.
    Eigentlich ist es schon ein Gegenbeleg. Diese Professoren waren doch alle bis zum Abwinken humanistisch gebildet, geholfen hats aber offenbar nix.

    @H.M.Voynich
    Martin hat nur 4 Sätze zitiert, Laskers Beitrag geht aber über 8 Seiten.
    Damit ist es sogar einer der längsten Beiträge im Buch.

  219. #220 H.M.Voynich
    3. Januar 2011

    @Niels:
    Ich sehe da nur einen kurzen Abschnitt von Lasker oben auf Seite 20, genau vier Sätze lang. Direkt darauf folgt ein 8seitiges Traktat von einem Prof.Dr. J. Le Roux.

  220. #221 Niels
    3. Januar 2011

    @ H.M.Voynich
    Arrgh, hast recht
    Ich war zu blöd zum Gucken, hab die Zeile mit dem Professor Roux übersehen und das Traktat zu Laskar dazugerechnet und den tatsächlichen Beitrag von Lasker nur als Einleitung gelesen, warum auch immer.
    Wahrscheinlich hats mir das Gehirn verschwurbelt. 😉

    Ich hätte eine Warnung vor den Link zum Buch setzen sollen:
    Kann zu Konzentrationschwäche oder Verwirrungszuständen führen. Sie sollten deshalb nach der Lektüre kein Fahrzeug lenken, keine Maschinen bedienen und keine Arbeiten ohne sicheren Halt verrichten!”

  221. #222 MartinB
    3. Januar 2011

    Aber bei allem berechtigten Amüsement finde ich doch eins sehr deutlich: Das Buch demonstriert hervorragend, wie untauglich philosophische Begriffsspielereien sind, um physikalische Theorien zu bewerten. (Womit ich nicht sage, dass Philosophie selbst untauglich ist – als Werkzeug innerhalb der Physik hilft sie eben nicht viel, zum Nachdenken über die Grundlagen der Physik schon eher.)

    Die Analyse der Argumente wäre ansonsten ein ziemlich spannendes Thema im Grenzgebiet Wissenschaftsgeschichte/Physik/Philosophie/Politik (denn da spielte ja auch die Kritik an der “jüdischen Physik” eine Rolle, obwohl einer der Hauptautoren mit Namen Hans Israel sicher selbst Jude war.) Anscheinend gibt’s da nicht viel, wenn ich google scholar richtig bedient habe.

  222. #223 TheBug
    3. Januar 2011

    Es wird auch ganz deutlich wie wenig Sinn Philosophie ohne Bezug zur Realität hat, Philosophie die auf vorhandenen Erkenntnissen aufbaut ist andererseits sehr sinnvoll.

  223. #224 walim
    3. Januar 2011

    Ich hatte vor einer Weile mal ein interessantes Buch über diese seltsame Koalition, die sich in der Ablehnung zusammen fand (bzw. sich zusammenfassen ließ) in der Hand, ich glaube, es war das hier:
    https://www.amazon.de/Einsteins-Gegner-%C3%B6ffentliche-Kontroverse-Relativit%C3%A4tstheorie/dp/3593389142/ref=sr_1_1?ie=UTF8&s=books&qid=1294092444&sr=8-1

  224. #225 Dr. Webbaer
    4. Januar 2011

    @kindermund
    Vielleicht lässt sich diese Ihre Aussage –

    Raum hat keine Eigenschaften, er ist das Gegenteil von Objekt, etwas, das Eigenschaften hat.

    – noch positiv deuten. – Vorab aber noch vielen Dank für die Distanzierung von der Doitschen Physik, die bei dieser Debatte keine Rolle spielen sollte. Auch wenn Dr. Webbaer (noch) kein Jude ist, die nehmen wohl auch keine Bären, ist es immer angenehmer Braunvieh nicht in der Debatte zu haben – apropos, wo ist eigentlich der Astrologe weggeblieben?

    Hmja, gut, die Physik ist einerseits die Natur selbst und anderseits als φυσική physikē die Naturforschung. Letztgenannte, und um die bemüht man sich in diesem Kommentarstrang, ist anthropogen und jede Aussage in diesem Zusammenhang ist für den Systematiker immer die Aussage einer Person(engruppe) in diesem Zusammenhang. D.h. jede Aussage über die Natur entspringt einer personengebundenen Theoretisierung oder einer Sicht. Was wiederum bedeutet, dass Dr. Webbaer und kleiner kindermund gerne gemeinsam an der Theoretisierung feilen dürfen, aberr (wir stellen uns hier das rollende R des Willy Brandts der Siebziger vor) – was eben nicht geht ist das haltlose Aufstellen absolut (losgelöster) gemeinter Aussagen zur Sache der Physik.

    Vielleicht seien Ihnen an dieser Stelle diese Texte empfohlen:
    Dr. Russell und der Tisch
    Dr. Bäker – Wissenschaft beschreibt
    Juhnke-Songtext

    Ok, vielleicht lassen Sie den letzten Text mal weg, aber die beiden anderen vielleicht doch kurz lesen…

    Äh, beamen wir uns einmal spasseshalber in die Zeit der Newton-Physik zurück. – Wenn Dr. Webbaer’s Prädezessor seinerzeit zu Ihrem Prädezessor gekommen wäre und sich diesem gegenüber spekulativ theoretisierend und der Newton-Physik agnostisch verhaltend geäußert hätte, wie hätten Sie Ihren Präzedessor seinerzeit antworten lassen?

    MFG + weiterhin Ohren steif halten!
    Dr. Wb

    PS: Wissenschaftler ohne oder mit eng begrenztem metaphysisch-erkenntnistheoretischem KnowHow, die meinen die Wahrheit zu predigen, sind natürlich in der Tat anzutreffen – sogar hier bei den scienceblogs.de!

  225. #226 perk
    4. Januar 2011

    die meinen die Wahrheit zu predigen

    wobei “meinen” hier in der seltener genutzten bedeutung von “ich schreibe ihnen dies nur beleglos zu” auftritt

  226. #227 Dr. Webbaer
    4. Januar 2011

    @perk
    Es gibt halt diejenigen Kollegen, die meinen ganz losgelöst von allem mit wüsten Aussagen zur Natur kommen zu können, frei von der Theoretisierung und frei von jeder Distanz, mindestens ein Beispiel konnte in diesem Kommentarstrang destilliert werden und es gibt diejenigen, die sich der Methoden der modernen Wissenschaft bedienen, aber sozusagen von der anderen Seite her fest überzeugt sind die richtigen Aussagen zur Natur gefunden zu haben.

    Ehrlich gesagt findet Dr. Webbaer Gruppe 2 noch wesentlich uncooler.

    Man kann es immer auch gerne mit dem Theismus und dem Atheismus vergleichen, die im ersten Absatz dieser Nachricht genannte erste Gruppe darf mit den Theisten verglichen werden, die zweite mit den Atheisten – die soo schlau sind, dass sie wissen, dass es keine Götter gibt, zumindest daran glauben, dass es keine gibt…

    Gähn, nicht ganz neu das Thema, fürwahr!, JF fehlt hier ein wenig, gell?, aber man muss schon froh sein, wenn diese Minidebatten der Kommentarbereiche noch halbwegs zivilisiert bleiben!

    GN8 + weiterhin Ohren steif halten!
    Dr. Wb

  227. #228 perk
    4. Januar 2011

    mit provozierenden allgemeinzuschreibungen um sich zu werfen ist bei ihnen in der tat kein neues thema und wohl kaum den anderen kommentatoren hier anzulasten

    ich formuliere es mal als freundliches angebot: schreiben sie konkrete beiträge zum thema um die diskussion voranzubringen

    anderenfalls wird es immer wahrscheinlicher, dass sie mit ihren regelmäßigen angriffen gegen mehr oder weniger genau umrissene gruppen der anderen kommentatoren nur die diskussion stören wollen..

  228. #229 Kampfdenker
    4. Januar 2011

    @kindermund
    Du verbeißt Dich in einen Unterschied zwischen Symbolik und Realität (“Symbole sagen nichts über die Gestalt der Realität”) und übersiehst, dass der Realitätsbegriff an sich höchst schwammig ist. Ist das Rot, dass Du siehst das gleiche, was ich sehe / empfinde, wenn ich etwas “Rotes” sehe? Letztlich gibt es (für mich) keine Realität als die in meinem Kopf. Vielleicht bin ich aber auch nur ein Gehirn, dass in einer Nährlösung schwimmt und alle Sinneseindrücke nur vorgegaukelt bekommt? Eine nette erkenntnistheoretische Spielerei (mir macht sie jedenfalls Spaß… und ein paar Filemmachern auch…) aber sie bringt mich in meinem (empfundenen / eingebildeten?) Leben nicht weiter. In diesm Leben kommen nämlich Zwänge und Bedürfnisse vor (Hunger, Schmerz, Müdigkeit, …), die ich möglichst gut befriedigen sollte, da sich meine “Realität” sonst ziemlich schnell, ganz schön mies anfühlt. Das kann ich aber am besten in Zusammenarbeit mit all den anderen Menschen, die “meine Realität” bevölkern, denn nur mit diesen zusammen kann ich auf all die Errungenschaften zurückgreifen, die mir die Erfüllung meiner Bedürfnisse (ob eingebildet oder “real”) in früher unerreichter Sorglosigkeit und Qualität erlauben. Hierzu muß ich aber mit meinen Mitmenschen Informationen (Erkenntnisse, Erfahrungen) austauschen und hierbei gab es in Vergangenheit und Gegenwart immer wieder Probleme, insbesondere seit die Informationen etwas komplexer wurden, als die Kenntnis über das nächste Wasserloch oder die nächste Höhle. Es kam zu Missverständnissen und gar Missbrauch und somit ergab sich die Notwendigkeit nach mehr Nachvollziehbarkeit und Wiederholbarkeit, um die Zuverlässigkeit der Information zu erhöhen und nach mehr Systematik, um die wachsende Komplexität überhaupt noch zu meistern. Das Ergebnis dieser Bemühungen sind Naturwissenschaften und die in diesem Rahmen immer weiter entwickelte Mathematik, welche geradezu die Quitenssenz an Nachvollziehbarkeit und Systematik darstellt. Es gibt keinen Zugang zur Realität, der sich mit anderen Menschen teilen ließe, der nicht über Symbole funktioniert. Seien es mathematische Symbole, Buchstabensymbole (all diese Posts hier) oder akustische / Wort-Symbole, die unsere gesprochene Sprache ausmachen. Genau genommen machen die Sprachsymbole sogar unser Denken aus, denn welcher Mensch ist in der Lage einen Gedanken zu Denken, der sich in keiner Weise durch Worte ausdrücken ließe? (Siehe auch George Orwells 1984, wo der große Bruder vor hat, die Sprache soweit zu verändern, dass es schwierig wird, einen systemkritischen Gedanken überhaupt noch zu formulieren). Und jetzt wüßte ich gerne, warum Deine Symbolik besser geeignet ist als die mathematische.

  229. #230 kindermund
    4. Januar 2011

    Zum guten Schluß mein Lieblingskommentar:
    “Zu dem Buch “100 Autoren gegen Einstein” hat Einstein selbst eigentlich schon alles nötige bemerkt: “Wenn ich unrecht hätte, wäre einer genug!”” 🙂

    RT ist widerlegt, daran gibt es keinen Zweifel. Dabei ist die Frage der fachlichen Kompetenz einzelner Autoren in “100 Autoren gegen Einstein” gleichfalls nebensächlich, denn ich führte die Veröffentlichung nicht als fachliche Widerlegung sondern als Zeitdokument an, daß einige der Methoden beschreibt, die von den Förderern Einsteins Religion schon vor fast 100 Jahren benutzt wurden. Dies habe ich auch deutlich zum Ausdruck gebracht. Aber vielleicht sollte es nicht wundern, wenn Menschen die an 0-dimensionale Partikel glauben nicht in der Lage oder Willens sind, seriös und inhaltlich auf Kommentare einzugehen. Wer sich auf dieser Plattform umsieht stellt fest: es hat sich in 80 Jahren nicht viel Neues getan an der Propagandefront.

    Raum und Zeit entbehren Realität, sind Konzepte, sie in physikalischem Sinne zu vergegenständlichen ist Blödsinn** (bitte vergessen Sie nicht im LHC auch Zeitteilchen kollidieren zu lassen und achten Sie dabei darauf ob sich eventuell Vorzeichen umkehren). Diese einfache Feststellung wird durch die Forschung bestätigt, denn weder Zeitdilatation noch gekrümmter Raum sind experimentell belegt. Die gerne angeführten Beispiele mit Atomuhren und Lichtbeugung durch “Gravitationslinsen” zeitigten Ergebnisse unterhalb der Messgenauigkeit oder waren schon in der Anlage durch Vernachlässigung lokaler Gegebenheiten, wie das die Sonne und Galaxien umgebende Plasma, fehlerhaft. Trotzdem wird man an den Universitäten nicht müde diesen Hokuspokus zu bewerben und als Wahrheit zu verkaufen. “Verkaufen” ist dabei ein wichtiges Stichwort….

    Und wie, lieber Herr Dr. Bäker, soll man denn überhaupt mit jemandem diskutieren, der sich weigert mit definierten Begriffen zu argumentieren? Dies ist der entscheidende Punkt! Ich habe kein grundsätzliches Problem damit, wenn z.B. Elektronen Ihrer Meinung nach keine Objekte im trivialen Sinne sind, aber dann sollten Sie sie nicht Objekte nennen, denn die “Objekte” der Mathematik sind Konzepte und die “Objekte” der Physik sind Gegenstände, daran erkennbar, daß Sie mindestens 3 Dimensionen haben – keines von beidem beschreibt die Elektronen heutiger Teilchenphysik zutreffend. Das gleiche gilt für Wellen. Eine Welle im mathematischen Verständnis ist ein Konzept, eine Welle im Sinne der Naturwissenschaften ist eine bestimmte Bewegung von ETWAS. Also, entweder Sie benennen ein Medium für Ihre Wellen oder Sie müssen zur Kenntnis nehmen, daß Sie mit Phantomen arbeiten. Und wenn Wissenschaftler auf ihre eigenen Tricks reinfallen und per Verdinglichung auf die Existenz (ja, da kommt sie wieder, die gute alte Realität) von “Schwarzen Löchern” und “schwarzer Materie/Energie” schwören, dann kann einem das Lachen durchaus vergehen. An diesen wenigen Beispielen, die sich lange fortsetzen ließen, wird deutlich, daß etwas faul ist im Staate Dänemark.

    Meine Vermutung: Physiker haben keine Ahnung WOVON sie reden – NEIN, nicht: Phyisker sind dumm und die Gleichungen sind falsch – NEIN, nicht: Physik funktioniert nicht – ABER: mathematische Physik hat nur eine ausschnittartige Ahnung von der Natur ihres Studienobjektes, weil sie sich methodisch auf Teilaspekte beschränkt, was nicht zuletzt daran liegt, daß man sich darin gefällt in ganz trivialer Weise Konzepte zu verdinglichen (reification).

    Sie sind herzlich eingeladen DIES zu widerlegen. Dazu müssen sie INHALTLICH argumentieren, nicht QUANTITATIV. Mein Tip: beginnen Sie damit Ihre Begriffe zu definieren, fangen Sie mit dem Begriff “Punkt” an und arbeiten Sie sich bis “Objekt” vor. Sollten Sie das können, ohne das Ihnen ihre Pseudowissenschaft um die Ohren fliegt, bin auch gerne bereit mit Ihnen zu rechnen oder Einsteins abgekupferten Gleichungen umzuformen*. Nur noch mal ganz klar: DIESER Teil ist unbedeutend, solange nicht mal der Gegenstand der Betrachtung definiert ist.

    @ Webbear
    Ich kritisiere im Wesentlichen nicht die Ergebnisse der mathematischen Physik, sondern ihre Interpretation hinsichtlich der Gestalt der atomaren und subatomaren Welt. Und ich kritisiere ganz entschieden den Auschluß aller anderen Ansätze aus der Forschung. Somit hinkt der Theist/Atheist Vergleich, obwohl er in der Anlage richtig ist. Es gibt eben andere Modelle, die einfacher sind, funktionieren ohne den Kern heutiger Erkenntnisse aufgeben zu müssen und dazu den großen Vorteil haben RATIONAL zu sein (Mathis mechanischen Modell, Gaedes Rope Hypothese u.v.m.) aber komplett ausgeblendet werden.

    Das etwas grundsätzliches an der heutigen Physik falsch ist, kann man schon mit einer einfachen Frage erkennbar werden lassen: Wenn Licht durch Glas geht, reduziert sich die Geschwindigkeit des Lichtes im Glas. Wenn es aus dem Glas in ein Vakuum austritt, nimmt es wieder die bekannte Größe c an. WELCHE Kraft bewirkt die Beschleunigung und WIE kann diese in 0 Zeit stattfinden?

    Schönen Tag noch…

    * weiß allerdings nicht genau warum, denn das ist ja nun wirklich allgemein bekannt. Sie dürfen es ruhig zugeben, deswegen allein schmeißt Sie keiner raus oder gäbe es dann eine Rüge von Scheloske?

    ** wer Raum verdinglicht konvertiert Raum begrifflich zu Objekt/Gegenstand, dessen Inhalt man in der Wissenschaft (Mathematik ist keine Wissenschaft) VOLUMEN nennt

  230. #231 kindermund
    4. Januar 2011

    @ Kampfdenker
    Danke für den ausführlichen und dialogorientierten Kommentar.

    Und jetzt wüßte ich gerne, warum Deine Symbolik besser geeignet ist als die mathematische.

    Warum ist “Mülleimer” ein besseres Symbol für einen Mülleimer als “π r2 h”? Weil die für das menschliche Verständnis wesentlichen Informationen kulturhistorisch mit diesem Symbol assoziiert wurden. Wenn ich etwas ein “Teilchen” oder eine “Welle” nenne, wenn ich tatsächlich nur einen konzeptuellen Teilaspekt vom kulturhistorisch gelernten Symbol meine, dann führe ich mich und andere in die Irre. Wie dies wissenschaftlicher Erkenntnis zuträglich sein kann, entzieht sich meinem Verständnis.

  231. #232 perk
    4. Januar 2011

    Es gibt eben andere Modelle, die einfacher sind, funktionieren ohne den Kern heutiger Erkenntnisse aufgeben zu müssen und dazu den großen Vorteil haben RATIONAL zu sein (Mathis mechanischen Modell, Gaedes Rope Hypothese u.v.m.) aber komplett ausgeblendet werden.

    wissenschaftler sind alles deppen aber leute mit lustigen youtubevideos sind die echten rationalen die alles erklären…

    falls sie das mit den großen vorteilen der ropehypothese ausführen können wäre es hier: https://www.scienceblogs.de/arte-fakten/2010/07/dunkle-energie-oder-dunkles-mysterium.php#comment129756 wohl angebracht

  232. #233 perk
    4. Januar 2011

    achso und für alle die noch nicht kennen was kindermund da für die antwort auf alle probleme der physik (auch die die er künstlich draufprojeziert) hält, hier ist sie:

    https://www.youstupidrelativist.com/

    bill gaedes seite zu seiner rope hypothesis, seinen ansichten zu physikern (wie man sie hängen/einweisen sollte) und seinen eigenen apokalyptischen prophezeiungen

    die gestaltung der seite unterstreicht insbesondere wie rational das hier vorgebrachte ist

  233. #234 MartinB
    4. Januar 2011

    @kindermund
    “mathematische Physik hat nur eine ausschnittartige Ahnung von der Natur ihres Studienobjektes, weil sie sich methodisch auf Teilaspekte beschränkt, was nicht zuletzt daran liegt, daß man sich darin gefällt in ganz trivialer Weise Konzepte zu verdinglichen ”
    Darüber kann man nachdenken, siehe mein text “Physik und Geist”. Wenn Sie dazu etwas konkretes und interessantes zu sagen haben, immer her damit.

    “beginnen Sie damit Ihre Begriffe zu definieren, fangen Sie mit dem Begriff “Punkt” an und arbeiten Sie sich bis “Objekt” vor. ”
    Sie haben anscheinend immer noch nicht verstanden, wie Physik funktioniert: Wir machen uns ein mathematisches Modell, das wir auf die Realität abbilden. Wir prüfen, ob die Vorhersagen des Modells mit der Realität übereinstimmen. Wenn sie das tun, dann sagen wir, dass das Modell die Realität beschreibt. Wie genau dabei die “Objekte” (oder “Entitäten” oder wie immer Sie das nennen wollen) der Realität beschaffen sind, darüber machen wir keine Aussage – wir sagen lediglich, dass sich die Realität so verhält wie unsere mathematische Beschreibung. Sie argumentieren gegen einen naiven Realismus an, den vermutlich kaum jemand so vertritt.

    “ich führte die Veröffentlichung nicht als fachliche Widerlegung sondern als Zeitdokument an, ”
    Dass die Veröffentlichung andersherum als Zeitdokument gedeutet werden kann, das zeigt, dass man schon vor 80 Jahren händeringend nach Argumenten gegen die RT suchte, weil sie einem zu “unanschaulich” war, ist ihnen dabei ironischerweise entgangen. Lustig.

    “WELCHE Kraft bewirkt die Beschleunigung und WIE kann diese in 0 Zeit stattfinden?”
    Gar keine und gar nicht. Lesen Sie mal Feynmans QED-Buch, da erklärt er Ihnen, wie die Verlangsamung durch Wechselwirkungen mit den Atomen zu Stande kommt. Physiker sind gar nicht so doof, wie Sie denken, die stellen sich nämlich auch solche Fragen. Der Unterschied ist nur der, dass Physiker nicht sofort “Alle Theorien sind falsch” schreien, wenn sie die Antwort nicht innerhalb von 10 Sekunden finden, sondern dann lieber ein bisschen länger nachdenken und suchen…

    “…das ist ja nun wirklich allgemein bekannt”
    Mir nicht – hat mir im Studium keiner erklärt. Aber das ist vermutlich wieder eine dieser seltsamen Verschwörungstheorien nach dem Motto “Alle Physiker wissene igentlich, dass die RT falsch ist, aber sagen es nicht.” Welche Motivation man dafür haben sollte, hat sich mir noch nie erschlossen.

    @perk
    Toller Link, jetzt habe ich nen Webdesign-Schock, danke…

  234. #235 Ireneusz Cwirko
    4. Januar 2011

    @TheBug „Es wird auch ganz deutlich wie wenig Sinn Philosophie ohne Bezug zur Realität hat“

    Es reicht schon Blick in Wiki um zu erfahren dass die Philosophie ein Fundament der Erkenntnis bildet.
    Erst durch aussortieren des Unwesentliches aus der Philosophie, ist die Wissenschaft entstanden, oder besser gesagt Technik. Für mich besteht der Unterschied grundsätzlich in dem, dass die Philosophie sich mit Fragestellung beschäftigt und mit dem Weg zu den Antworten.
    Die Technik liefert die Antworten ohne sich um die Erklärung zu scheren. Sie interessiert sich für die Anwendung und Beherrschung der Erkenntnisse. Weil die Techniker diese Beherrschung beschleunigen wollten haben sie es die Wissenschaft gebraucht. Die soll denen die Werkzeuge liefern den Beherrschungsprozes zu beschleunigen.
    An diesen Wurzeln hängt und scheitert die Wissenschaft bis heute weil sie glaubt durch technische Vorgehensweise die Fragen über die Natur antworten zu können. Das wird nicht funktionieren weil die Sprache die sie nutzt, die Mathematik, zu primitiv ist um es artikulieren zu können.

    @ kindermund „Diese einfache Feststellung wird durch die Forschung bestätigt, denn weder Zeitdilatation noch gekrümmter Raum sind experimentell belegt“

    Es ist ein gutes Beispiel für meine oben vorgestellte These. Auf dem Beispiel der Zeitdilatation zeigt sich dass die Fragestellung von den Techniker falsch verstanden wurden. Eine unzulässige Vereinfachung , dass die Veränderung der Frequenz von Strahlungsübergängen der Elektronen freier Atome auf eine Veränderung eines abstrakten Konzepts, wie Zeit, hindeutet muss eigentlich auch für die Techniker widersinnig zu sein.
    Und sie ist es Tatsächlich. Diese Beobachtung bedeutet nur dass sich die Frequenz verändert und nicht dass sich die Zeit verändern kann.
    Also die Frage die man sich als Physiker stellen muss ist warum ist diese Veränderung möglich und welche physikalische Prozesse sind vorstellbar um so eine Veränderung zu verursachen.
    Es ist sogar gegen dem Wesen eines Technikers in dem Moment auf das abstrakten Konzept des Zeit zurück zu greifen. Es muss falsch sein weil er keine physikalisches also reales Objekt darstelle.

    @ kindermund „Das etwas grundsätzliches an der heutigen Physik falsch ist, kann man schon mit einer einfachen Frage erkennbar werden lassen: Wenn Licht durch Glas geht, reduziert sich die Geschwindigkeit des Lichtes im Glas. Wenn es aus dem Glas in ein Vakuum austritt, nimmt es wieder die bekannte Größe c an. WELCHE Kraft bewirkt die Beschleunigung und WIE kann diese in 0 Zeit stattfinden?“

    Sie hängen noch sehr Tief in den Konzepten der Physik wenn sie noch von Kräften in diesem Zusammenhang träumen.
    Die Antwort können Sie bei mir nachlesen und sie ist ganz einfach. Ich habe oben meine Kosmologie kurz skizziert und dort ist sie auch enthalten. Alles aber alles in der Welt ist nur Raum. Dieses Raum hat aber die Eigenschaften. Es besteht aus kleinen Einheiten die dreidimensional Oszillieren. Von der Charakteristik der Oszillationen ist auch die Geschwindigkeit der Photonen abhängig.

  235. #236 Bullet
    4. Januar 2011

    Ich habs jetzt erst durch des Ireneusz’ Zitat gesehen.
    @Kindermund:

    Das etwas grundsätzliches an der heutigen Physik falsch ist, kann man schon mit einer einfachen Frage erkennbar werden lassen: Wenn Licht durch Glas geht, reduziert sich die Geschwindigkeit des Lichtes im Glas. Wenn es aus dem Glas in ein Vakuum austritt, nimmt es wieder die bekannte Größe c an. WELCHE Kraft bewirkt die Beschleunigung und WIE kann diese in 0 Zeit stattfinden?

    Dir kommt natürlich nicht die Idee, daß etwas grundsätzlich an deinem Verständnis von Realität und Physik falsch sein könnte, wenn du so einen Mist verzapfst, oder?
    Die Frage an sich ist in gewisser Weise verständlich. Aber du argumentierst mit “Kraft” und “Beschleunigung”. Bescheuerter gehts kaum.
    – Meinst du tatsächlich, jedes Lichtpaket hätte eine Art hyperdimensionales Raketentriebwerk dabei, das erstmal wieder gezündet werden muß, nachdem das Licht das Glas wieder verläßt?
    – Glaubst du wirklich, BESCHLEUNIGUNG wäre das passende Wort für eine Änderung der Ausbreitungsgeschwindigkeit von Wellen? Kleiner Tip: “Beschleunigung” ist dv/dt. Das heißt: ein beschleunigendes Objekt verändert sein v stetig. Es wandelt dabei übrigens den Gesetzen der Thermodynamik gehorchend Energie aus beliebiger Quelle in Bewegungsenergie um, die nicht verlustfrei rückgewonnen werden kann. Da dies bei Licht nicht auftritt, sollte bereits dein IQ, der ja nachgewiesenermaßen dafür ausreicht, Worte in ein Webformular einzutippen, genügen, zu bemerken, daß etwas an dem Modell, das du hier durch Verwendung der Worte “Kraft” und “Beschleunigung” propagierst, nicht stimmen kann. In diesem Falle wäre es deinerseits angebracht, die höfliche Frage zu stellen, wo denn dein Denkfehler liegt. Daß du stattdessen behauptest, etwas grundsätzliches an der gesamten Physik wäre “falsch” (was auch immer das heißen mag – du tippst Worte in ein Webformular ein, also hast du bereits dadurch bewiesen, daß die Physik funktioniert), zeigt leider, daß du mit dem Tippen der Worte deinen IQ schon verbraucht hast.

    Natürlich gibt es für diese Frage eine ganz einfache Erklärung. Das Problem ist: du wirst sie nicht hören wollen, weil du dann zugeben müßtest, einen Denkfehler gemacht zu haben. Und du müßtest in Zukunft auf dein dümmliches Geseiere von der “falschen” Physik verzichten und hättest keine Ausrede mehr, warum du irgendwelche Sachen nicht verstehst.

  236. #237 Bullet
    4. Januar 2011

    Nachtrag (immer noch an Kindermund):

    Raum und Zeit entbehren Realität[…], sie in physikalischem Sinne zu vergegenständlichen ist Blödsinn[…]. Diese einfache Feststellung wird durch die Forschung bestätigt, denn weder Zeitdilatation noch gekrümmter Raum sind experimentell belegt.

    Lustig. Da fehlt ein Effekt. Relativistische Massenzunahme.
    – daß Raumkrümmung eine geometrische Deutung eines beobachtbaren Effektes ist … nun, darauf lasse ich mich ein. (Ich bin da aber auch nicht an der science frontier.) Immerhin aber läßt diese Deutung Voraussagen zu, die auch beobachtbar sind.
    – Zeitdilatation ist nicht experimentell belegt … ähm, die Verlängerung von Halbwertszeiten instabiler Partikel hoher Geschwindigkeiten zählt nicht? Mutig.
    – Die Massenzunahme ist übrigens bereits dann “experimentell belegt”, wenn du einen beliebigen Teilchenbeschleuniger (ja, auch der Linearbeschleuniger in der Schule kann das – mehr als 200 kV brauchst du nicht) einschaltest und den Stromzähler abliest. Aber nicht nur das: Massenzunahme, Zeitdilatation, Längenkontraktion hängen zusammen und sind entweder alle falsch oder alle korrekt. Beobachte den Stromzähler und erkenne: die drei sind Geschwister und treten nur gebündelt auf. Und das tun sie.
    Gegen die Deutung “Raumkrümmung” darfst du dich natürlich sträuben. Aber obs was nützt? Oder kann hier sonst noch irgendjemand etwas definitives zum Beleg von Raumkrümmung sagen? Ich kanns ad hoc nicht.

  237. #238 Ronny
    4. Januar 2011

    @kindermund

    Halten Sie sich nicht an das was ihr Augen sehen und ihre Finger tasten. Öffnen sie ihren Geist für neue Ideen die vielleicht irreal erscheinen aber trotzdem zu vorhersagbaren Ergebnissen führen.

    Wenn Licht durch Glas geht, reduziert sich die Geschwindigkeit des Lichtes im Glas.
    Falsch. Die Lichtgeschwindigkeit bleibt immer c. Die Bewegung durch Glas ist ziemlich kompliziert. Sie können sich das entweder als Absorption-Reemission vorstellen oder dass das Licht ziemliche Umwege machen muss.

    Wenn es aus dem Glas in ein Vakuum austritt, nimmt es wieder die bekannte Größe c an.
    Falsch, die hat es schon vorher

    WELCHE Kraft bewirkt die Beschleunigung und WIE kann diese in 0 Zeit stattfinden?
    Um die Masse = 0 auf c zu beschleunigen braucht man die Kraft 0. Das ist der Witz, Licht hat IMMER c und nur Licht kann c erreichen weil es masselos ist.
    Jede Glühbirne schafft es übrigens Photonen mit c zu emittieren.

    @Autor
    Gute Zusammenfassung. Mich haben 2 Experimente überzeugt.

    1) Zwei Lichtstrahlen in 180 Richtung entlang der Erdbewegung müssten verschiedene Geschwindigkeiten haben. Haben sie nicht, also muss c konstant sein.

    2) Ein Beobachter auf einem Segelboot sieht einen Lichtstrahl mit c zwischen zwei Spiegeln pendeln, ein ruhender Beobachter sieht c+x. Da c konstant ist muss zwangsläufig die Zeit langsamer werden, was dem in der Physik unangebrachten ‘Hausverstand’ massiv widerspricht und deshalb viele Menschen zu Gegnern macht.

  238. #239 MartinB
    4. Januar 2011

    @Bullet
    Für die raumkrümmung gibt es natürlich auch belege – neben der vielzitierten Ablenkung des Lichts an der Sonne sind vor allem die Gravitationslinseneffekte zu nennen, die spielen in der Astronomie inzwischen ja eine wichtige Rolle. Wenn das jemandem nicht gut genug ist, dann muss er eben eine andere Theorie aufstellen, die alle Beobachtungen genausogut erklärt…

    @Ronny
    Auf die Gefahr hin, dich (und andere) zu verwirren: Auf hinreichend kleiner Längenskala fliegt Licht nicht unbedingt mit Lichtgeschwindigkeit, jedenfalls nicht, wenn man Feynmandiagramme verwendet:
    https://www.scienceblogs.de/hier-wohnen-drachen/2010/10/wie-funktionieren-feymandiagramme.php (Abschnitt Photon-Propagator)

  239. #240 Ronny
    4. Januar 2011

    @MartinB
    Ok. Aber im makroskopischen Bereich von dem Kindermund sprach (lol, passender Name) ist c konstant. Oder versuchst du gerade mich wieder skeptisch der RT gegenüber zu machen ? Ohne diese ‘Seltsamkeit des Lichts’ überall gleich schnell zu sein würde sich die RT ja in Luft auflösen.

    Übrigens, das Thema Licht durch Glas wurde mir schon mindestens 3 mal erklärt und zwar jedesmal anders (Absorption/Reemission, Licht macht Umwege, Überlagerung von Wellengleichungen). Das wär mal ein interessantes Thema (oder gabs da schon was ?)

  240. #241 MartinB
    4. Januar 2011

    @Ronny
    Nein, makroskopisch ist c konstant – von zeilinger-Quantenexperimenten mal abgesehen, die stehen aber nicht im Widerspruch zur SRT.

    Die Erklärung der Lichtbrechung hängt davon ab, auf welchem level du dich bewegen willst – geometrische Optik mit Fermatschem Prinzip, Elektrodynamik mit Wellenausbreitung, Quantenmechanisch mit Photonen, wobei die später genannten die vorherigen jeweils als Grenzfall enthalten.
    Aber du hast recht, das wäre ein interessantes Thema, um daran mal zu erklären, wie Modelle in der Physik funktionieren.

  241. #242 TheBug
    4. Januar 2011

    Geht es noch hypokritischer als kindermund? Sitzt vor einem Gerät und benutzt eine Infrastruktur, die nicht existieren würden wenn die Theorien die er angreift nicht stimmen würden. Vollständige Leugnung der Realität.

    Ich glaube ich hab den Kerl schon mal im Fernsehen gesehen, damals wer er Informationsminister im Irak und berichtete mitten in Bagdad davon, dass es keine Invasion durch Koalitionstruppen gäbe, wobei ihm aus dem Hintergrund einige US Soldaten zusahen.

  242. #243 Ronny
    4. Januar 2011

    @MartinB
    Also als Hobbyphysiker mit Hang zur Quantenmechanik wäre natürlich die quantenmechanische Erklärung die Beste. Ich habe da mal einen englischen Artikel gelesen der sehr interessant war (war sogar auf SC verlinkt). Aber ganz schlau wurde ich daraus nicht.

    Interessant war auch eine Diskussion die schon mal hier auf SC stattgefunden hat wo es darum ging, warum die Elektronen in bestimmten Bahnen kreisen. Die Erklärung: weil dort ganzzahlige Ergebnisse der Schrödingergleichung sind war nicht so ganz erhellend 🙂

    @TheBug
    Um die RT zu begreifen muss man den Schrit vom Realismus zu Mathematik, Modell und Fakten machen. Das schaffen manche nicht.
    Mich erinnert das eher an Gewerkschaftsfunktionäre deren einziges Argument oft ist: So kann das doch nicht sein !

  243. #244 MartinB
    4. Januar 2011

    @Ronny
    Lies Feynmans QED-Buch (“Seltsame Theorie von Licht und Materie”) zum Thema Licht und Glas.

    Was die ganzzahligen Lösungen angeht, schau mal in meine Schrödinger-Serie, teil 2…

  244. #245 TheBug
    4. Januar 2011

    @Ronny: Insbesondere der Schritt dazu zu akzeptieren, dass die Welt nicht wirklich so funktioniert wie wir sie subjektiv wahrnehmen ist für manche Leute zu viel. Macht man die Versuche zur Lichtgeschwindigkeit und Massedilatation, dann wird einem ja deutlich, dass Newton halt nur für kleine Massen und Geschwindigkeiten anwendbar ist.

  245. #246 Basilius
    4. Januar 2011

    @MartinB

    …wobei die später genannten [Modelle] die vorherigen jeweils als Grenzfall enthalten.
    Aber du hast recht, das wäre ein interessantes Thema, um daran mal zu erklären, wie Modelle in der Physik funktionieren.

    Bitte Martin! Bitte machen!

    Das wurde zwar schon immer wieder angesprochen und vorgekaut, aber ich glaube, daß man diese grundlegene Thematik gar nicht oft genug erneut erklären kann. Ich beobachte einfach bei vielen vielen Menschen immer wieder dieses Unverständnis gegenüber den als Spezialfall enthaltenen bisherigen Modellen in der neuen Theorie. Ich denke jeder hier kennt das immer wieder kommende Argument, daß die Wissenschaft ja doch immer wieder alles neu erfinden würde und deshalb diese gefühlte Beliebigkeit auch den noch so absurdesten Quatsch quasi legitimieren würde; weil, morgen könnte ja auf einmal die Erklärung dafür daherkommen, warum es doch (als völlig willkürliches Beispiel) Erdstrahlen geben könnte. Der Florian drüben beim Astro. Simpl. sagt ja auch immer wieder, daß es schon richtig ist, daß wir nicht alles wissen, was da noch sein könnte, aber daß wir sehr wohl schon einiges darüber wissen, was ganz bestimmt nicht sein kann. Auch in Zukunft nicht.
    Diesen Gedankengang zu erfassen und aus dieser Erkenntnis die richtigen Schlüsse zu ziehen würde ich mir für das neue Jahr für einige Mitmenschen wünschen.

  246. #247 togibu
    4. Januar 2011

    @Kindermund
    Deine Argumentation scheint mir schon ein wenig paradox:
    Einerseits beschwerst Du Dich, dass die Begriffe “Raum” und “Objekt” in einer (aus Deiner Sicht) falschen Definition verwendet werden, andererseits beschwerst Du Dich darüber, dass die Quantenphysik sich mit Dingen wie Elektronen beschäftigt, die man nicht sehen kann (und die deshalb nach Deiner Diktion gar nicht existieren). Da frage ich mich doch: Kannst Du denn den von Dir definierten “Raum” sehen (oder sonst wahrnehmen)? Wenn ja, wo und wie? Oder warum beschäftigst Du Dich mit “Raum”, auch wenn Du ihn weder sehen noch sonst wahrnehmen kannst?

  247. #248 TheBug
    4. Januar 2011

    Klar kann er den Raum sehen, sind ja Fenster und eine Tür drin, nur leider lassen die sich alle nur von Aussen öffnen und die Wände sind gepolstert…

  248. #249 Bullet
    4. Januar 2011

    Der war vorhersehbar. 🙂

  249. #250 TheBug
    4. Januar 2011

    Aber notwendig 😉

  250. #251 Gluecypher
    4. Januar 2011

    @Kindermund

    Ich empfehle Dir, selbst ein paar bahnbrechende Werke wie “100 Authoren gegen Einstein” zu verfassen.
    Wie wäre es mit:

    “100 Schreiner gegen Maxwell”
    “50 Fleischereifachverkäuferinnen widerlegen den II. Hauptsatz der Thermodynamik” oder “50 Talkshow-Hosts zerfetzen die QED!”

    Hat DIch schon mal jemand gefragt, was Du an Stelle der QM, RT und sonstigen schwarzmagischen Freveltaten stellen würdes. Ich meine so als Beschreibung, was Licht- und Radio-Wellen so machen, warum Magnete funktionieren, warum LEDs leuchten, wieso Dein DVD-Spieler funktioniert oderoderoder.

    Ach ja, wurdest Du schon, hast es aber vorgezogen, dich wie ein Aal um die Antwort zu drücken, Du darfst es gerne nochmal versuchen.

    @perk
    AAAARRRRRRRGGGGGGG!!!! ICH BIN BLIND! Aber hey, mal ganz ehrlich, mit so ‘ner Hackfresse würde ich mein Photo nicht auf ‘ne Webseite stellen.

    Wo.ist.mein.Bullrichsalz?

  251. #252 kindermund
    4. Januar 2011

    Glaubst du wirklich, BESCHLEUNIGUNG wäre das passende Wort für eine Änderung der Ausbreitungsgeschwindigkeit von Wellen?

    Übrigens, das Thema Licht durch Glas wurde mir schon mindestens 3 mal erklärt und zwar jedesmal anders (Absorption/Reemission, Licht macht Umwege, Überlagerung von Wellengleichungen). Das wär mal ein interessantes Thema (oder gabs da schon was ?)

    Da stehen wir mit beiden Beinen im Problem: Wenn es beliebt ist Licht natürlich eine Welle, wenn es anders besser passt, besteht Licht prompt wieder aus Partikeln. Die Standardantwort ist: Licht ist beides. Wunderbar, dann muß aber das Verhalten von Licht IN JEDEM FALL beiden Beschaffenheiten entsprechen und selbst wenn man sich alles so hinbiegt, daß es unter Zuhilfenahme einiger Fantasiekonstanten und ähnlichem gerade so hinkommt, bleibt das Problem, daß mathysische Partikel und Wellen inhaltslose Worthülsen sind, die nichts mit den Begriffen Partikel oder Welle der Wissenschaft gemein haben.

    Selbstverständlich muß ich von Beschleunigung sprechen, wenn Licht aus Partikeln besteht. Wovon denn sonst?

    v = s / t enthält “s” = Weg, eine Messung von Wellenlängen und Frequenzen ist streng genommen keine Geschwindigkeitsmessung, sondern eine weitere Misrepräsentation von Symbolen. Ich kann mich also, wenn es um Geschwindigkeit geht, nur an die Partikelnatur von Licht halten.

    Wer sich für eine rationale, mechanische Erklärung mit Partikeln interessiert, sollte Miles Mathis lesen. “Licht besteht aus Partikeln MIT einer Welle” – immerhin weiß der Mann, daß Wellen ein Medium brauchen und das obwohl er Mathematiker ist…

    Aber wir kommen deutlich vom Weg ab: Es ist nur bedingt sinnvoll über Licht (oder Raum, Partikel, Atome etc.) zu diskutieren, wenn niemand eine wissenschaftliche Definition von Licht zu geben bereit ist. Begründe ich irgendetwas mit mathematischen Formalismen, baue ich stets auf undefinierten Elementen auf, was kann da anderes herauskommen als ein undefiniertes Ergebnis?

    Zu der Person “Bill Gaede” (Rope Hypothesis, RH): wer seinen Wikieintrag liest, stellt schnell fest, daß er ein ungewöhnlicher Mensch ist. Das äußert sich u.a. darin, daß er, was seine öffentliche Kommunikation betrifft, so eine Art George Carlin der Wissenschaft ist – reine Nervensache sich mit ihm auszutauschen. Trotz Gaedes unvorteilhaftem Stils ist die Rope Hypothese meiner Meinung nach ein hoch interessantes Modell, daß um Längen wissenschaftlicher und einfacher ist, als der mathematische Flickenteppich undefinierter Symbole. Dabei ist es wichtig zu bemerken, daß die allermeisten quantitativen Erkenntnisse bisheriger Forschung nicht im Widerspruch zur RH stehen, die RH aber rational erklärt, wie diese einzelnen Daten zusammenpassen. Ohne RT, ohne QT, ohne Phantombegriffe.
    Es gibt natürlich noch wesentlich mehr bedenkenswerte Vorschläge, z.B. Prime Matter etc.

    Ich erwähne die Alternativen nicht unbedingt, weil ich eine bestimmte davon für den Weißheit letzter Schluß halte, sondern um daran zu erinnern, daß es ernstzunehmende Alternativen gibt.

    Um die RT zu begreifen muss man den Schrit vom Realismus zu Mathematik, Modell und Fakten machen. Das schaffen manche nicht.

    Stimmt. Manche sehen aber auch einfach daß Mathematik mit beliebigen Begriffen arbeitet und konzentrieren sich daher lieber auf Wissenschaft, weil ihnen die Zeit zu schade ist, die man mit bedeutungslosen Formalismen vertrödelt ohne wissenschaftlichen Erklärungen näher zu kommen. Es ist schon eine besondere Ironie, wenn Mathematiker Wissenschaftlern mit dem “Argument” der Realitätsverleugnung (The Bug) kommen.

    @ The bug
    Klar kann er den Raum sehen, sind ja Fenster und eine Tür drin, nur leider lassen die sich alle nur von Aussen öffnen und die Wände sind gepolstert…Nimm heute Abend Deinen Notizblock mit an den Stammtisch, vielleicht fällt Dir nach ein paar Bieren noch was dämlicheres ein, mit dem Du Dich hier als Hecht im Karpfenteich gerieren kannst.

  252. #253 kindermund
    4. Januar 2011
  253. #254 TheBug
    4. Januar 2011

    @kindermund: “Nimm heute Abend Deinen Notizblock mit an den Stammtisch, vielleicht fällt Dir nach ein paar Bieren noch was dämlicheres ein, mit dem Du Dich hier als Hecht im Karpfenteich gerieren kannst.”

    Nein, den Versuch werde ich nicht wagen, es wird mir mit Sicherheit nicht gelingen dämlicheres als Du von mir zu geben. Aber ich bin Dir gerne mit gelegentlichen Bemerkungen behilflich dabei Dich noch lächerlicher zu machen. Das ist meine übliche Vorgehensweise wenn ich es mit Cranks zu tun habe, da von Dir keine Erkenntnisse zu erwarten sind und Dir auch keine zu vermitteln sind, machen wir uns einfach lustig über Dich.

    Schon spannend, laut kindermund haben Wellen also keine messbare Ausbreitungsgeschwindigkeit. Kennt sich jemand damit aus ob das in Takatukaland wirklich so ist?

  254. #255 togibu
    4. Januar 2011

    @Kindermund:
    Falsch: Licht ist keins von Beiden, es verhält sich nur manchmal wie Teilchen, und manchmal wie Wellen. Licht ist Licht, es hat in unserer Wahrnehmungswelt nun mal kein immer passendes Analogon wie Glasmurmel oder Schallwelle (Und das gilt auch für die anderen, von Dir negierten Quantenobjekte).

    Und: Kriege ich noch eine Antwort auf meine Frage?

  255. #256 Bullet
    4. Januar 2011

    Angeregt durch Gluzis Kommentar hab ich mir die Seite jetzt auch angesehen.
    Aber….
    so schlimm fand ich die nicht.
    Okay, sie ist das Ergebnis einer durchrauchten Nacht, in der so einige Kilogramm Substanzen, die auf Listen des BKA/Abteilung BTM stehen, Eingang in einige Gehirne fanden.
    Okay, selbst ohne diese Drogen wären Wortwahl, Stil und Technik der Argumentation in Deutschland ein Versetzungshindernis.
    Okay, die Liste der Falschbehauptungen und unnachvollziehbaren Eigenkreationen an Aussagen ist wahrscheinlich länger als die ganze Seite selbst (so wie 100g Salami ja angeblich 115 Gramm Fleisch enthalten, glaubt man den diversen Aufklebern auf abgepackter Wurst, die ums Jahr 2004 mal eine Weile in Mode waren).

    Aber sonst? Dumme Leute tun dumme Dinge. Und richtig dumme Leute sorgen noch dafür, daß es jeder mitbekommt. Und die allerdümmsten Leute bekommen eben eine Darwin-Award-Nominierung.

  256. #257 Bullet
    4. Januar 2011

    @Kindermund:

    Die Standardantwort ist: Licht ist beides.

    Aha. Das ist sie nicht. Nicht einmal das bekommst du hin. Wie wärs mit: frag mal jemanden, der sich damit auskennt – und hör mal genau hin.

    Selbstverständlich muß ich von Beschleunigung sprechen, wenn Licht aus Partikeln besteht. Wovon denn sonst?

    Tja … dann wäre es ja supereinfach, Licht abzubremsen und in Tüten zu packen. Und das wäre mit Sicherheit auch schon vor vielen Jahrhunderten gelungen.
    Hmmmmmmmmmm … isses aber nicht. Licht besteht eben nicht aus Teilchen, denn das wär aufgefallen. Vom angesprochenen thermodynamischen Problem mal ganz abgesehen.

    Ich kann mich also, wenn es um Geschwindigkeit geht, nur an die Partikelnatur von Licht halten.

    Wenn du meinst … aber erwarte dann nicht, daß du dann in der Realität landest. Denn ich würde gern sehen, wie du einen Ball an die Wand wirfst, der beim Abprall seine Geschwindigkeit nicht verringert, dafür aber kälter wird. Das passiert nun einmal nicht. Und Photonen als Partikel zu betrachten funktioniert auch nicht.
    Aber nur zu, du Pippi-Langstrumpf-Sklave.

  257. #258 TheBug
    4. Januar 2011

    Eine Tüte Licht bitte, das grüne mit den roten Streifen 🙂

  258. #259 Hel
    4. Januar 2011

    AAAARRRRGGG *mühsam Brechreiz unterdrück*

    Ähm, Leute, wozu “Kindermund” durch rhetorische oder fachbezogene Fragen zu noch mehr Gebrabbel ermuntern?

    Dabei kommt eh nichts raus und erst recht nichts an. Fast könnte man Mitleid haben, wäre diese Person nicht so unsäglich aufgeblasen und geltungssüchtig.

    Vgl https://tinyurl.com/yh7s5b4 und https://tinyurl.com/37zzlvd

  259. #260 Georg Hoffmann
    4. Januar 2011

    Fantastischer Artikel. Bald haben wir ja eine Best-of Science Blogs Liste und der Artikel gehoert da sofort rein.

  260. #261 TheBug
    4. Januar 2011

    Autsch!

    Das piekt ja im Auge beim Lesen.

    So lange sich so was nur in Foren rumtreibt ist es ja harmlos, aber solche Leute gibt es leider auch in der Politik und die machen dann z.B. Umweltpolitik ohne die Spur eine Ahnung von Chemie zu haben, Stoffe mit gefährlich klingenden Namen oder Bestandteilen werden dann verboten. Mal sehen wann es das Natriumchlorid erwischt, das ja immerhin aus dem extrem feuergefährlichen Natrium und dem Giftgas Chlor besteht.

  261. #262 Thierbach
    4. Januar 2011

    (so wie 100g Salami ja angeblich 115 Gramm Fleisch enthalten…

    womit wir wieder bei den Fleischereifachangestellten wären 😉
    im übrigen darf Salami gerne auch 130 g Fleisch auf 100g Fertigprodukt enthalten, um so besser schmeckt sie nämlich nach der (wasserzehrenden) Reifung

    Doch besser zurück zur RT:

  262. #263 MartinB
    4. Januar 2011

    @kindermund
    “Die Standardantwort ist: Licht ist beides.”
    Nein, das ist nicht die Standardantwort. Die Standardantowtr ist “Licht hat Aspekte von beidem”, was etwas anderes ist. beides sind anschauliche Modelle, die dem realen Verhalten von Licht (das im Rahmen der QED eindeutig beschrieben ist) je nach Situation mehr oder weniger gut entsprechen. Das einzige, was eas da zu diskutieren gäbe, wäre das quantenmechanische Messproblem.

    “ernstzunehmende Alternativen”
    müssten quantitativ irgendetwas beschreiben können. Tut mir Leid.

    Was genau soll ich an Gluecyphers Kommentar moderieren? Jemand, der andere z.B. als “Mathetrottel” bezeichnet, sollte doch nicht so ton-empfindlich sein, oder? Gluecypher hat das Niveau der “100 gegen Einstein” sarkastisch auf die Schippe genommen, und die Frage, welche Alternativen Sie anzubieten hätten, ist ja wohl nicht unberechtigt.

    @Basilius
    ich setze es mal auf meine Liste möglicher Blogthemen (die wird immer länger, könnte ich bald hauptberuflich machen, wenn’s dafür ordentlich Kohle gäbe…)

  263. #264 Kindermund
    4. Januar 2011

    @ MartinB
    “Gluecypher” hat sich auf SB schon ausreichend als stilsicherer Kommentator profiliert und sein dümmliches “100 Gluecyphers gegen Vernunft” oder was auch immer da steht, berührt mich nicht, weil es ohnehin für sich selbst spricht. Abgesehen davon wußte ich bevor ich den ersten Kommentar schrieb, daß da jede Menge Geschrei der unsachlichen Art auf mich zukommen würde, das ficht mich nicht an.
    Was ich meine ist das hier:

    AAAARRRRRRRGGGGGGG!!!! ICH BIN BLIND! Aber hey, mal ganz ehrlich, mit so ‘ner Hackfresse würde ich mein Photo nicht auf ‘ne Webseite stellen.
    Wo ist mein Bullrichsalz?

    Damit meint er Bill Gaede, der an dieser Diskussion nicht teilnimmt und sich folglich auch nicht wehren kann.

    müssten quantitativ irgendetwas beschreiben können. Tut mir Leid.

    Nein, MUSS es grundsätzlich nicht, tut es im Falle Mathis trotzdem und bei der Rope Hypothese geht es (in diesem Zusammenhang) um die NATUR des Lichts. Wie wiederholt gesagt, erfinden weder Mathis noch Gaede die Welt neu, wo immer die heutige Physik zutreffende quantitative Aussagen produziert (hat), gelten diese weiter. ZUSÄTZLICH eröffnet der Verzicht auf Irrationalität aber neue Einsichten, z.B. im Falle Mathis WAS ein Feld ist und WIE es funktioniert (u.v.m.), z.B. im Falle Gaede was es mit EPR auf sich hat (u.v.m.). Bei Mathis dürfen Sie SRT – mit ein paar Korrekturen – behalten, bei Gaede müssen Sie sich von A/SRT verabschieden. Beiden gemein ist, daß sie mit dramatisch weniger ad hoc Annahmen auskommen, als die Standardmodelle – freilich auch nicht ohne. Ich habe ja bereits überdeutlich gemacht, warum A/SRT meiner Meinung nach Humbug ist.

    Mein ganzes Anliegen an dieser Stelle war über die historisch überholte Frage im Titel auf ALTERNATIVE Modelle hinzuweisen, die ungerechtfertigter Weise vom Mainstream ignoriert werden. Da wir aber nun offensichtlich an dem Punkt angekommen sind, wo sich die Stammtischbrüder darüber einig sind, daß nur noch mit Behauptungen und Spott reagiert werden soll, um die inhaltlichen Fragen nicht diskutieren zu müssen, soll es zu diesem Artikel vorerst genug sein. Ich danke für die Diskussion.

  264. #265 Kindermund
    4. Januar 2011

    @ MartinB
    https://www.scienceblogs.de/hier-wohnen-drachen/2010/12/spezielle-relativitatstheorie.php#comment172268

    Bleibt auch unmoderiert? Ich bin nicht diese Person!

    Zu der Hetzseite “Esowatch” zu verlinken, ist auch nicht besser als zu irgendwelchen braunen Seiten zu verlinken. Was die Haltung zu geistiger und Meinungsfreiheit angeht, nehmen die sich nichts.

  265. #266 Kindermund
    4. Januar 2011

    @ The Bug

    Schon spannend, laut kindermund haben Wellen also keine messbare Ausbreitungsgeschwindigkeit. Kennt sich jemand damit aus ob das in Takatukaland wirklich so ist?

    Schon blöd wenn man zwar ne große Klappe hat, aber unfähig ist Texte inhaltlich zu erfassen und somit nicht nur unverschämt, sondern auch auch am Thema vorbei stänkert. Schau ein bischen Fernsehen, da sind die Worte nicht so wichtig.

  266. #267 Basiljus
    4. Januar 2011

    @Kindermund
    Ich habe bislang noch selten einen so ausgeprägten Fall von Dellörjewubbel gesehen wie bei Ihnen. Sehr beeindruckend. Da frage ich mich schon, ob Sie schlck sollten oder nicht schlck sollten. Aber das wäre ja auch zu einfach und letztlich auch weder wünschenswert, noch wirklich sinnvoll.

  267. #268 Ireneusz Cwirko
    4. Januar 2011

    @ Kindermund “da wir aber nun offensichtlich an dem Punkt angekommen sind, wo sich die Stammtischbrüder darüber einig sind, daß nur noch mit Behauptungen und Spott reagiert werden soll, um die inhaltlichen Fragen nicht diskutieren zu müssen, soll es zu diesem Artikel vorerst genug sein. Ich danke für die Diskussion.”

    Gute Abgang.
    Machen Sie sich keine falsche Vorstellungen, bei dem Stammtisch ist MartinB gerne der Hauptdemagoge.

  268. #269 TheBug
    4. Januar 2011

    @kindermund: “Haltung zu geistiger und Meinungsfreiheit”, na mit Freiheit von Geist kennst Du Dich ja bestens aus. Die Behauptungen kamen von Dir, Antworten auf Fragen keine, wie auch.

    Und immer schön dran denken: Dein Computer ist nur eingebildet, der funktioniert ja gar nicht, weil ja die Physiker alle doof sind und falsche Theorien aufstellen und die Leute die Computer entwickeln sind dann auch noch so doof den Physikern auf den Leim zu kriechen und deren Theorien zu verwenden, kann also alles gar nicht funktionieren.

    @Quirko: Geh am besten gleich mit kindermund mit zum Crank-Stammtisch, da könnt Ihr Euch dann so richtig darüber ausheulen wie dumm die ganzen Physiker sind.

  269. #270 H.M.Voynich
    5. Januar 2011

    @kindermund:
    “Nein, MUSS es grundsätzlich nicht …”
    Dann ist es aber keine Physik.
    Den Physiker interessiert nur: was kommt hinten raus, wenn ich vorne was reinstecke. Die Zusammenhänge, die er dabei empirisch ermittelt, versucht er dann, in einer möglichst einfachen Formel vollständig zu beschreiben.
    Dabei passiert es häufig, daß völlig unterschiedliche Rechenwege, die auf ganz verschiedenen Konzepten basieren, zu identischen Resultaten führen.
    Es ist nicht die Aufgabe des Physikers, zu entscheiden, welchen dieser Konzepte mehr und welchen weniger “Realität” zukommt – darüber dürfen sich die Philosophen streiten.

  270. #271 H.M.Voynich
    5. Januar 2011

    p.s.: ich empfehle die Feynman-Messenger-Lectures, speziell Teil 2: “The Relation of Mathematics and Physics”.
    (Leider ist Spezialsoftware von MS zum Betrachten erforderlich – aber es lohnt sich.)
    https://research.microsoft.com/apps/tools/tuva/index.html

  271. #272 TheBug
    5. Januar 2011

    Diskussionsversuche mit Vollpfosten sind leider nutzlos. Kindermund hat ja schon erhebliche Problem damit zu verstehen, dass ein Erklärungsversuch nichts nutzt, wenn er weder logisch, noch schlüssig ist und dann auch noch der Realität (z.B. Messwerten zur Lichtgeschwindigkeit) widerspricht. Oder kurz: Dass es nicht ausreicht keine Ahnung zu haben um eine Theorie zu widerlegen.

  272. #273 Kindermund
    5. Januar 2011

    @ die Stammtischbrüder
    Mir war von Anfang an klar, daß man in Kreisen mathematischer Physik nur wenig bis kein Interesse an Wissenschaft antrifft, dafür aber umso mehr Begeisterung für “elegante” Formalismen und Ausweichmanöver im Angesicht so schlichter Tatsachen wie “ein Symbol ist nicht identisch mit der Realität auf das es weist”, “Verdinglichung von Konzepten ist Religion” und “reale Objekte können nicht ohne das Konzept Raum definiert werden”. Ich weiche keinen Fragen aus, daß überlasse ich den Mathematkern. Alles was ich verlange, und das sollte in der Wissenschaft eine Selbstverständlichkeit sein, ist die Begriffe zu definieren, weil sonst jede Diskussion von Details Zeitverschwendung ist. Aber bis dahin kommen wir hier ja nicht einmal, weil die versammelte Intelligenz bereits an “Verdinglichung von Konzepten ist Religion” scheitert – obwohl das nun wirklich nicht schwer zu verstehen ist.

    Wie gesagt, ich werde das in Abwesenheit von Ernsthaftigkeit an dieser Stelle nicht weiter diskutieren, aber ich will trotzdem auf ein Papier hinweisen, daß zeigt wie stümperhaft und selbstgefällig entscheidende Teile SRT von Einstein mathematisch präsentiert wurden: https://www.wbabin.net/mathis/spec.htm

    Da dieser Link sich ausschließlich mit Mathematik befasst und SRT generell anerkennt, hoffe ich im Anschluß hier lesen zu können, mit Gleichungen demonstriert, wo Mathis irrt und wieso alles schön ist SRT-Land. Viel Spaß…

  273. #274 TheBug
    5. Januar 2011

    Symbolik verdammen aber Schrift verwenden, kein bisschen hypokritisch nicht wahr?

    Noch mal: Wenn Du zu dumm bist um eine Theorie zu verstehen, hat das nichts damit zu tun ob diese Theorie korrekt ist. Du bist nicht alleine damit die Relativitätstheorie nicht zu verstehen, das ändert aber auch nicht an deren Korrektheit.

    Da ist es auch egal wie viele weitere Links auf Nichtraffer Du hier postest, es bleibt dabei, dass wir Deinen Mist hier nicht lesen könnten (müssten) wenn die Theorien die Du nicht verstehst falsch wären, weil es “hier” mangels moderner Datentechnik nicht geben würde. Dazu fehlt von Euch Cranks auch jede verwertbare Antwort, “glaub ich nicht” ist übrigens nicht ausreichend, DAS ist nämlich Religion.

    Was Deinen Miles Mathis Link angeht, ich habe da auch einen, sogar von seiner eigenen Homepage: https://milesmathis.com/pi2.html

    Für alle denken Menschen auf diesem Blog: Bitte vor dem Öffnen der Seite die notwendigen Sicherheitsmaßnahmen ergreifen, ich möchte nicht für durch Lachkrämpfe verschmutzte Unterwäsche verantwortlich gemacht werden.

  274. #275 Kindermund
    5. Januar 2011

    @ H.M.Voynich
    Vielen Dank für Ihre Kommentare und den Link (kannte ich allerdings schon).

    Dann ist es aber keine Physik.
    Den Physiker interessiert nur: was kommt hinten raus, wenn ich vorne was reinstecke

    Exakt das ist Physik. Meiner Meinung nach unterliegen Sie einem Mißverständnis. Was Sie beschreiben ist die Interessenlage von Mathematikern. Mathematik befasst sich ausschließlich mit Konzepten und weiß nichts über Realität. Das Mathematik nutzbare Formalismen entwickeln kann, was ich weder bestreite noch ablehne (im Gegenteil), sagt nichts darüber aus, wie Realität in ihrer Gesamtheit beschaffen ist. Mathematik kann sich nur mit einem kleinen Teilaspekt von Realität beschäftigen.

    Nehmen Sie das Beispiel mit dem “Mülleimer” Symbol ( https://www.scienceblogs.de/hier-wohnen-drachen/2010/12/spezielle-relativitatstheorie.php#comment172032 ). Hier bereichern mathematische Formalismen das realistische Symbol, weil das Objekt in Frage zufällig einen der menschlichen Erfahrung direkt zugänglichen Maßstab aufweist und der Formalismus so von einem realistischen Symbol abgeleitet werden konnte. Dies führt dazu, daß die mathematische Symbolik nicht zu einer Verzerrung der Wahrnehmung führt (der Mülleimer bleibt Mülleimer).
    In der atomaren und subatomaren Welt treffen wir auf ganz andere Voraussetzungen. Hier wird das mathematische Konzept (aufbauend auf Teilbeobachtungen), zum Ausgangspunkt des Verständnisses, der Entwicklung eines realistischen Symboles (analog des “Mülleimers”), dieser Weg führt vermittels der Tatsache das Mathematik ausschließlich dynamische Konzepte beschreiben kann zwangsläufig zu einer Misrepräsentation, wenn nicht vermieden wird durch Verdinglichung aus einem abstrakten Konzept ein realistisches Symbol zu machen. Exemplarische Konsequenzen des leichtfertigen Umgangs mit systembedingt schlaglichtartigen Erkenntnissen sind die allerorten als Realität verkauften Konzepte “schwarzes Loch”, “Raumzeit” und “Zeitdilatation”.

  275. #276 Kindermund
    5. Januar 2011

    @ TheBug
    Weil Du einfachste Tatsachen nicht verstehst, bin ich dumm? Ich weiß ja nicht was Du rauchst, aber Du solltest damit aufhören, vielleicht kommt dann auch die Denkfähigkeit zurück. Aber wieso bin ich nicht überrascht, daß Du den Unterschied zwischen “Strecke” und “zurückgelegter Strecke” nicht kennst?

  276. #277 Niels
    5. Januar 2011

    @Kindermund
    Na ja, der Mann hat offenbar nicht einmal die Galilei-Transformation verstanden.
    Ich hab das jetzt nur 5 Minuten überflogen und mir sind gleich zwei Brüller aufgefallen.

    Let us go back for a moment to the first equation, x’ = x – vt .
    Again, this is supposed to be the Galilean transformation equation for x.
    But this is not the case. There is no such thing as a Galilei transformation equation. For Galileo and Newton, no transformation was necessary for a linear problem like this. x in S’ would equal x in S. The whole universe was a single co-ordinate system, and the train would not have been given a system of its own. It is clear that the equation for x in a Galilean system would have looked just like the other equations (for y, z, and t). Meaning, x’ = x. Or x = vt. But not, x’ = x – vt. This is one of the biggest mathematical blunders in history, sitting in the open for a century. And until now unquestioned.

    Aha. Wenn ich auf das Dach eines Zuges ein x’ male und darunter auf die Schienen ein x, dann bleibt der Abstand der beiden gemalten Kreuze zueinander immer gleich, auch wenn der Zug von Stuttgart nach Moskau fährt.
    Leuchtet ein.
    Ganz bestimmt hat Newton das auch so gesehen.

    Hier sieht man deutlich, dass es nicht einmal zu simpelster Mathematik reicht:

    But even as a Lorentz equation, the equation makes no sense. Let us look at Einstein’s transformation equation for x.

    x = [x’ + vt]/[(1 – v^2/c^2)^1/2]

    At low values for v, x is greater than 2x’.
    You may say, But if v is zero, then x = x’.
    Yes, but for any velocity greater than zero, no matter how slow, x is greater than 2x’. I do not even understand how one might go about plugging numbers into such an equation. What if our train is going ten miles per hour? And what if we watch it for a hour? Does Relativity want to tell us that the train is going to appear to have gone more than twenty miles

    x ist also immer mindestens größer als 2x’?
    Das beweist mathematisch, dass die SRT falsch sein muss?
    Wie wärs, wenn man einfach mal eine Zahl einsetzt und es ausprobiert?
    Zum Beispiel einfach mal t=1s und v= 1 m/s? Das geht doch sogar im Kopf?

    Außerdem hat er offenbar nicht im geringsten kapiert, was man mit diesem x’ überhaupt berechnet.

    Wenn du das für dich überzeugend ist, kindermund, fehlen dir eigentlich grundlegende Fähigkeiten, um über die Schwächen der Physik oder der Mathematik urteilen zu können.

  277. #278 TheBug
    5. Januar 2011

    Schwarze Löcher sind beobachtbare Objekte, Zeitdilatation ist messbar und kindermund demonstriert hier fortlaufend seine umfassende Inkompetenz. Finger in den Ohren und lautes “Lalala”-Schreien ist keine Wissenschaft.

  278. #279 kindermund
    5. Januar 2011

    @The Bug

    Schwarze Löcher sind beobachtbare Objekte

    Kuckuck, Kuckuck… Klar, wenn es sie denn gäbe und man unendlich viel Zeit mitbrächte, dann könnte man SLs beobachten. Aber was wann ist denn “unendlich viel Zeit” vorbei? Genau: niemals. Ehrlich, Du bist noch dümmer als ich dachte.

    @ Niels
    Schau Dir die erste Illustration nochmal an, dann verstehst Du es auch.

  279. #280 Bullet
    5. Januar 2011

    Da ist das Problem ja endlich mal in Worte gefaßt:

    Meiner Meinung nach unterliegen Sie einem Mißverständnis. Was Sie beschreiben ist die Interessenlage von Mathematikern. Mathematik befasst sich ausschließlich mit Konzepten und weiß nichts über Realität.

    In deiner Welt wäre ein Physiker dazu verdammt, bei – sagen wir: Fallversuchen – die Wertetabelle mit vergangener Zeit gegen zurückgelegten Weg auszufüllen, aber dann nicht mehr zu beachten. Aus den gewonnenen Meßwerten auch nur ein Diagramm zu erstellen, wäre bereits Anwendung von Mathematik. Daraus sogar noch einen grundsätzlichen Mechanismus abzuleiten (im doppelten Wortsinn: “ableiten” hier auch in der Bedeutung s/t-Diagramm -> v/t-Diagramm ->a/t-Diagramm) und dann frevelhaft VORAUSSAGEN zu konstruieren und zu zeigen, daß in realen Situationen nur wenig Abweichungen von mathematischen Idealfällen auftreten, ist dann wahrscheinlich Ketzerei.
    Übrigens ist deine Vorstellung von Physik unzutreffend:
    Physik ist nämlich, wenn man aus Beobachtungen der Welt die Regelmäßigkeiten der Abläufe in ihr erfassen und in generalisierbare Beschreibungen gießen kann.
    Wäre dem nicht so, gäbe es keine Bremsweg-Faustformeln. Du bräuchtest eine Tabelle, auf der jeder Kfz-Typ gegen jede Geschwindigkeit aufgetragen ist. Das hilft dir dann sehr viel.
    Und bitte hör auf mit diesem Schwachsinn “Exemplarische Konsequenzen des leichtfertigen Umgangs mit systembedingt schlaglichtartigen Erkenntnissen sind die allerorten als Realität verkauften Konzepte “schwarzes Loch”, “Raumzeit” und “Zeitdilatation”.” Hier kann niemand etwas dafür, daß dein religiöses Weltbild dir bestimmte Gedanken verbietet. Solange sich “zu weit gereiste” instabile Teilchen und Laufzeitunterschiede genau gehender Uhren quantitativ messen lassen und du außer “das geht nisch” kein Argument hast, wär deinerseits Stille angebracht.

  280. #281 MartinB
    5. Januar 2011

    @kindermund
    Gluecyphers Ausdruck “Hackfresse” ist sicher nichts, was ich selbst scheriben würde – wer aber solche Cartoons auf seine homepage setzt, wie z.B. das “Relativity Asylum” (das ja z.B. Hawking verunglimpft), der muss auch mit solchem Echo rechnen.
    Siehe auch seine eigene Seite
    https://www.youstupidrelativist.com/Ridicule/01Rid.html

    Inhaltlich wird es übrigens nicht besser, wenn Sie immer absurdere Internetseiten von RT-Leugnern finden…

  281. #282 Niels
    5. Januar 2011

    @kindermund
    Kennst du dir wirklich keine Koordinatentransformation zu einem mit v bewegten Koordinatensystem vorstellen?
    Vielleicht, wenn du Koordinatenachsen zum einen auf ein Blatt Papier malst und zum anderen auf eine durchsichtige Folie.
    Dann legst du die beiden Koordinatenachsen genau aufeinander und ziehst langsam die Folie in die x-Richtung weg.
    Dann sollte dir eigentlich einleuchten, warum x’ = x nicht sein kann.

    Außerdem könntest du einfach mal Zahlen in die Formel einsetzen und uns vorrechnen, wie x immer größer als 2x’ wird.
    Du kannst ja mit den der Geschwindigkeit von 10 Meilen pro Stunde und der Zeit von einer Stunde aus dem Text anfangen.
    Das müsstest du uns doch vorführen können, oder?

  282. #283 togibu
    5. Januar 2011

    @Basilius
    Natürlich sollten sie schlck, denn dann sind sie reif für die Undulation…:-)

    Und das hat wesentlich mehr Sinn als alles, was der Babymund (Kindermund ist eine Beleidigung für alle Kinder) von sich gegeben hat

  283. #284 Basilius
    5. Januar 2011

    Kindermund wa dekkai Fugu desu.

  284. #285 MartinB
    5. Januar 2011

    @Basilius
    Ich kann kein Japanisch wie vermutlich die meisten hier. Falls der Kommentar inhaltlich sinnvoll ist, dann bitte übersetzen, falls es sich nur um noch eine humorige “kindermund-ist-doof”-Variante handelt, dann bitte in Zukunft so etwas unterlassen.

    @alle
    Ich weiß, manchmal ist es schwer, wenn Leute vollkommen argument- und realitätsresistent sind. Könnt ihr bitte trotzdem persönliche Anfeindungen und Pauschalurteile wie “Vollpfosten” etc. unterlassen. (Und, ja, das gilt für alle, auch für kindermunds “du bist noch dümmer als ich dachte”.)
    Greift Argumente an, nicht Personen.

  285. #286 TheBug
    5. Januar 2011

    Welche Argumente?

  286. #287 MartinB
    5. Januar 2011

    @TheBug
    Z.B. die, die Nils oben so schön zerlegt, oder HM Voynich.

  287. #288 Basilius
    5. Januar 2011

    @MartinB
    Ich kann auch nicht wirklich japanisch (nur ein kleines bisschen, aber es wird immerhin langsam mehr). Deine Vermutung lag richtig. Übersetzt lautet es soviel wie: “dadada ist ein Riesenkugelfisch.” Das war ein Versuch des Maskierens, eben weil ich den persönlichen Angriff (zumindest direkt beleidigende Wörter wie ‘Vollpfosten’, ‘Hackfresse’, … auch nicht mag.
    Aber letztlich hast Du natürlich Recht, es bleibt eine weitere Variante. Argumente sollte man angreifen und keine Personen. Ich will das in Zukunft wieder mehr beherzigen. Zur Diskussion sinnvolles kann ich derzeit halt nicht beitragen, da Niels, H.M.Voynich, Bullet, Du selber, etc. eigentlich schon alles gesagt haben und Kindermund ja nicht wirklich auf die Argumente eingeht.

  288. #289 erik
    5. Januar 2011

    @ kindermund
    @LLE anderen

    „Mathematik befasst sich ausschließlich mit Konzepten und weiß nichts über Realität. Das Mathematik nutzbare Formalismen entwickeln kann, was ich weder bestreite noch ablehne (im Gegenteil), sagt nichts darüber aus, wie Realität in ihrer Gesamtheit beschaffen ist. Mathematik kann sich nur mit einem kleinen Teilaspekt von Realität beschäftigen.“

    Das @lles ist doch nur ein grosses Missverständnis !
    Natürlich kann Mathematik nur „Formalismen“ entwickeln, welche ihre Quelle in der „Realität“ besitzen. Grundsätzlich sind materielle „Formalismen“, wie Tisch, Stuhl und die Aufforderung: „Hinsetzen und Kindermund halten!“ alles Dinge die der „Realität“ entspringen. Die Realität ist etwas, wohin wir mit unseren Dingen nie hingehen können. Das sind getrennte Welten!!! Nie werde ich in der Realität Experimente ausführen können, nie werde ich dabei sein, wenn der Quelle in der Realität irgendetwas entspringt, → Das dann in der Verdinglichten Welt aus dem Kindermund heraus tritt.
    Mir geht es genau so wie dir: Irgendwas aus der Realität hat eine Verbindung mit mir (Mensch in der Dinglichen Welt) aufgenommen und jetzt denkt es sich mir: „Du, lieber Erik, wirst die Weltformel erklären! Fang mal mit der Eulerschen Formel an und wende dich an die Kapazitäten des Blogs „Hier wohnen Drachen“.

    Meine Güte, habe ich gedacht, was für ein Schmarren! NEIN, PENG, KRACH: Da meldet sich das für mich Unerreichbare und ich spüre die Resonanz von Schwingungen in meinem Kopf: es formen sich aus der Substanz der Realität heraus Gedanken in meinem SELBST! „ Kein Schmarren, meint das Unergründliche aus der Realität, hör dir an, was die Autoren im Blog zu sagen haben und fasse @lles zusammen was da geredet wird. Niemand soll ausgegrenzt werden, du stehst über dem @llem und wirst das Gefassel verwenden, um die WF zu formulieren. @lles aus dem Kindermund ist wahr, eine WF muss es schaffen den Texten aus dem Kindermund einen Sinn zu geben, die angesprochenen Themen mit allen anderen Themen der Mathematiker, den Biologen, den Chemikern und natürlich den Physikern zu vereinheitlichen.“

    Ich wollte gerade einen Stopp einlegen, eine kleine Pause: Da wischt mir irgendwas im Kopf meinen Pausengedanken weg. Ich erhalte die Aufforderung: !Weiterdenken!
    Scheisse, denke ich, muss das sein? „JA!“ kommt als Antwort. Na ja, denke ich, die unergründliche Substanz aus der Realität weiss schon was sie will! Da steckt wohl ein Sinn dahinter, welcher mir übertragen wird aus der Realität.
    Scheisse, die Substanz aus der Realität hört mit! Die überwacht meine Gedanken, schreibt mir vor was ich zu denken habe, redet mir in meinen Text hinein und was auch immer die noch mit mir machen kann.
    ABER: ich sollte sie testen: Hat sie recht, dann geht das sicher gut für mich aus: Die WF zu formulieren. Hat die unergründliche Substanz nicht recht, dann taugt die Realität nicht viel und die Substanz kann sich von dannen machen. Ich muss nur garantieren mich ihr mit Leib und Seele, vollständig, hinzugeben. Kein Zipfel in mir darf noch etwas anderem gehören als der Führung aus der Realität.
    Dann bekomme ich das schon hin mit der WF und den Dingen von Allem (ausgenommen der Realität, denn die gehört nicht zu @llen Dingen, denn die ist der Herr, oder ?Mutter? @ller Dinge!?).

    Aber wieso muss ich Realität ausschliessen, wenn ich die Ordnung aller Dinge beschreiben will? PENG: ein Gedanke ist gerade geliefert worden: „Schau dir die imaginäre Zahl „i“ einmal an und lass mich (sagt die Substanz aus der Realität) mal machen!“
    Hu, hu … da will etwas, von mir nie Erreichbares, etwas mit mir thun? Mich erschaudert das ein wenig, aber die Vorstellung macht auch ein wenig geil, nein toll wollte ich sagen.
    Ich geh dann mal zu „i“ und sende einen Erlebnisbericht etwas später.
    → Wünscht mir viel Glück!
    → Muttermund, ähh Kindermund, komm doch mal mit, vielleicht gelingt uns was gemeinsam?

  289. #290 Bullet
    5. Januar 2011

    Ist jetzt nur mir allein etwas unwohl oder geht es anderen Lesern auch so?

  290. #291 TheBug
    5. Januar 2011

    Ich hatte gerade so ein kaltes Gruseln als wenn ein Bullshit durch den Raum gegangen wäre…

    Es könnte aber auch ein Hoax gewesen sein, die sind manchmal schwer zu unterscheiden.

  291. #292 erik
    5. Januar 2011

    Warum wir Kindermund nicht verstehen können: Schuld trägt die SRT!!!
    Begründung:
    SRT: Licht übt Einfluss auf die Form des Raumes aus
    Eigenschaften von Zeit in der SRT sind andere als in der NM (Kindermundweg)

    SRT: Zeit übt einen Zwang auf die Form des Raumes aus
    NM: Kraft und Masse üben Zwang auf den Raum aus

    ich: Kraft und Masse sind Eigenschaften von Zeit im Raum von v=c
    Zwang wird im Raum allgemein ausgeübt (Universum: St-Pf-Tier: Mensch/Masch)
    Zwang wird im Raum im speziellen ausgeübt (Denk-Raum / Vorstellungsraum)

    Zwang wird durch Ordnungstrukturen in zwei Räumen umgesetzt: Materie / Verstehen

    Die QM beschreibt die Eigenschaften von Dinglichkeit, welche die Quelle der Dinglichkeit der NM darstellt.

    Im Raum der QM wirkt der Zwang auf ein Raumvolumen.
    Im Raum der NM wirkt der Zwang in Richtung eines Punktes.
    Zwang in QM und NM streben aufeinander zu, haben aber antagonistische Eigenschaften.
    QM und NM haben eine gemeinsame Ordnungstruktur und entwickeln durch die Eigenschaften der Zeit zwei voneinander getrennte Bewegungsräume.
    Das ist somit die Geburt von dialektischem Denken.

    Oder auch: In dem einen Raum sitzt Bullet und TheBug und im anderen sitzt Kindermund. Sie fühlen sich voneinander angezogen und werfen sich voneinander abstossende Worte zu. Das alles trägt einen Sinn.

    Die Begrenztheit von eigenem Denken /Verstehen des anderen, liegt in dem physikalischen Prozess begründet, welchen die SRT /ART und die NM beschreibt.

    Mit anderen Worten: Wir können Kindermund nicht verstehen, weil die SRT/ART gemeinsam mit der NM dies nicht zulässt.

    Fazit: Weitermachen! Irgendein Resultat wird schon dabei rauskommen.

    (Vom Besuch bei „i“ bin ich noch nicht zurück.)

  292. #293 nihil jie
    5. Januar 2011

    @kindermund

    Mathematik befasst sich ausschließlich mit Konzepten und weiß nichts über Realität.

    sehe ich nicht so… denn die grundlage der mathematik ist das hantieren mit mengen die uns alle immer umgeben. und wenn sie mit sichbaren mengen umgehen kann, kann man sie auch getrost für gedachte und hypothetische mengen nutzen. das mathematische werkzeug besteht aus lauter algorithmen die für gruppen von aufgaben oder eine aufgabe zugeschnitten sind. was natürlich nicht heissen soll, dass man sich dabei irren kann. man kann auch fehler machen oder sogar noch keinen geeigneten algorithums für ein problem gefunden haben… dennoch hat die mathematik schon einen bezug zu realität die uns umgibt. egal ob sie für uns sichtbar ist oder nicht. deswegen werden nicht zuletzt auch experimente durchgeführt um annahmen zu überprüfen die die theoretiker vorhergesagt haben.

    man kann die mathematik sogar als eine universelle sprache beschreiben. ein ausserirdischer, wenn er denn genügend intelligenz besitzt um seine umwelt quantifizieren zu können wird früher oder später auch mit mengen hantieren und davon wahrschenlich die selbe oder zumindest ähnliche mathematik ableiten wie wir.
    ein atom wird auch dort ein atom sein, genau so wie eine pflanze wird auch bei ihm eine pflanze und nicht 20 sein. das einzige was man dann in erfahrung bringen müsste ist nur welches “zeichen” für bestimmte mengen steht und welche für die gewünschte opperation um einen algorithmus zu bilden (subtraktion, addition, potenzieren usw) um den ausserirdischen zu verstehen.

  293. #294 nihil jie
    5. Januar 2011

    nachtrag @kindermund:

    selbst tiere benutzen mathematik. zb eine krähe (habe selbst eine grossgezogen) kann sofort abschätzen ob auf dem einem haufen 4 und dem anderen 5 körner liegen um sich dann auch für den grösseren haufen zu entscheiden *gg ja ja… das einfache hantieren mit mengen ist sogar vielen tierarten gegeben die da drauf angewisen sind oder daraus einen vorteil haben 😉 das ist eine art einfacher mathematik… und das egal ob sie die mengen mit namen versehen können und irgend welchen schreibzeichen zuordnen können. unsere gehirne sind einfach da drauf ausgelegt damit umgehen zu können. es ist wichtig für unser überleben. darüber hinaus haben wir menschen aber auch die fähigkeit unsere umwelt auch hypothetisch mit zahlen zu beschreiben. wenn das nicht so wäre wären wir nicht im standen gebäude zu bauen deren statik wir zuerst theoretisch berechnen können. und das ist nur ein kleiner beispiel… so zu sagen die spitze des eisberges 😉

  294. #295 erik
    5. Januar 2011

    Was könnte Schrödingers Katze durch den Kopf gehen?

    https://theory.gsi.de/~vanhees/faq/physik/node5.html:
    “Im mikroskopischen Bereich von Atomen dagegen gibt es tatsächlich solche ,,Zwischenstadien“ von zwei Zuständen, die sich ,,eigentlich“ gegenseitig ausschließen. So können Atome mehrere verschiedene Anregungszustände quasi gleichzeitig besetzen, ja sie können sich sogar an zwei Orten gleichzeitig aufhalten – so lange niemand genau hinschaut. Leider sind auch dies wieder nur bildliche Darstellungen. Was da tatsächlich ,,passiert“, kann man sich meiner Meinung nach nicht veranschaulichen.”

    Katzenkopf: “Da gibt es Dinge, die doch wirklich beides, Realität und Materie besuchen können.”
    oder auch:
    “Dunkel wars, der Mond schien helle,
    als ein Wagen blitzeschnelle langsam um die Ecke fuhr,
    drinnen sassen stehend Leute, schweigend ins Gespräch vertieft,
    als ein totgeschossner Hase auf der Sandbahn Schlittschuh lief.”

  295. #296 TheBug
    5. Januar 2011

    @erik: Nein, das ist komplett falsch so. kindermund kann uns nicht verstehen, weil er sich weigert über den Tellerrand zu gucken, bzw. zu denken. Newton steht einem nicht im Weg wenn man versteht, dass er nur einen Grenzfall der SRT erfasst.

    Genau so wie ich in der Elektrotechnik mit technischem Strom hantiere, der von Plus nach Minus fließt, statt mich mit den tatsächlichen Leitungseffekten auseinander zu setzen, bei denen Elektronen von Minus nach Plus fließen und durch Valenzbänder der verwendeten Materialien beeinflusst werden, muss ich beim Ausrechnen der Fahrstrecke eines Zuges bei einer gegebenen Geschwindigkeit und Zeit nicht Einstein verwenden, schlicht weil es einfacher ist und hinreichend genau.

    Fange ich aber an Halbleiterbauteile zu entwickeln, dann muss ich mich mit der QM auseinander setzen, so wie ich die SRT und ART brauche, wenn ich präzise Bestimmungen von Satellitenbahnen benötige (z.B. GPS), oder einen Teilchenbeschleuniger bauen möchte.

    Was Deine etwas inkongruente Darstellung zur QM betrifft: Der wichtigste Satz ist, dass auch diese Analogien falsch sind. Beobachten bedeutet bei der QM nicht “da guckt einer”, sondern “da wird eine Messung durchgeführt”. So eine Messung basiert immer auf einer Interaktion mit einem Teilchen, also einer Beeinflussung. Im Gegensatz zu makroskopischen Objekten, die sich um ein Bisschen Licht relativ wenig scheren, reagiert ein Atom wenn es z.B. von einem getroffen wird sehr wohl und so etwas ist dann das “Beobachten” im Sinne der QT.

  296. #297 MartinB
    5. Januar 2011

    @TheBug
    Das Messproblem ist aber schon mehr als “da gibt es eine beeinflussung” – deswegen wird das ja hier auch gern und oft diskutiert.

    @erik
    “Was da tatsächlich ,,passiert“, kann man sich meiner Meinung nach nicht veranschaulichen”
    Naja, ein bisschen schon – siehe meine Schrödinger-Serie. Und mit der richtigen Mathematik lassen sich Experimente (statistisch) korrekt vorhersagen.

  297. #298 nihil jie
    5. Januar 2011

    @TheBug (ergänzung)

    und um so kleiner die teilchen um so kleinere wellenlängen des lichts werden benütigt um das teilchen “zu beobachten”… und um so kleiner die wellenlänge um so grösser die energie. da hat so manch ein kleines teilchen schwer zu kämpfen wenn er damit gestört wird 😉

  298. #299 TheBug
    5. Januar 2011

    Wer misst misst Mist und wer sich darüber nicht im Klaren ist, der misst ganz großen Mist, besonders wenn er mit Mist Mist misst.

    @MartinB: Ist mir klar, ich arbeite u.a. an Sensoren und bin (nicht nur) dadurch mit der Messproblematik vertraut. Man kann halt nicht Messen ohne gleichzeitig zu beeinflussen und hat dann auch noch das Problem sich bestens mit seinen Messfehlern vertraut machen zu müssen. Das ist mikroskopisch wie makroskopisch gleich, nur dass sich dann natürlich haufenweise spezifische Probleme je nach Anwendung ergeben.

    Leider ist das vielen Leuten nicht klar. Einem Kunden habe ich mal auf die Frage wie genau denn unsere Beschleunigungssensoren messen, zur Abtastungsauflösung die ganzen Fehler, also Offset, Drift, Alterung, Unlinearität, Übersprechen usw. genannt um dann die Frage zu hören bekommen: “Und wie genau sind die nun?”.

  299. #300 Hel
    5. Januar 2011

    @MartinB

    Auch und gerade ich als notorischer Schulversager in Physik – der darauf keineswegs stolz ist -, möchte dir übrigens nochmal meinen Dank und ein fettes Kompliment für diesen lehrreichen und gut verständlichen Artikel aussprechen.

  300. #301 MartinB
    5. Januar 2011

    @TheBug
    “Man kann halt nicht Messen ohne gleichzeitig zu beeinflussen ”
    Das ist erstens (zumindest im theoretischen Grenzfall) falsch (na klar kann ich den Impuls eines Elektrons im Impulseigenzustand theoretisch beliebig genau messen, da gibt es keine prinzipielle Schranke) und zweitens ja nicht wirklich das “Messproblem” – das ist ja eher der Kollaps der WF (obwohl das natürlich zusammenhängt).

    @Hel
    Danke.

  301. #302 Basilius
    5. Januar 2011

    @TheBug

    “Man kann halt nicht Messen ohne gleichzeitig zu beeinflussen “

    Da muss ich dem MartinB zustimmen. Es gibt durchaus Möglichkeiten eine Messung ohne Beinflussung vorzunehmen. Oder habe ich da bei einer Messbrücke nach z.B. Wheatstone etwas falsch verstanden? Diese Art der Messung hat ja zudem noch den Vorteil, daß man hier schon verdammt präzise messen kann.

  302. #303 TheBug
    5. Januar 2011

    @Basilius: Präzision ist eine Sache, die kann ich mit der notwendigen Sorgfalt natürlich hoch drücken. Messfehler habe ich aber immer, egal wie klein sie sind und da auch eine Messbrücke keine unendliche Auflösung hat, habe ich auch da immer einen Fehlerstrom der übrig bleibt und damit letztlich eine Beeinflussung.

  303. #304 Frank Wappler
    6. Januar 2011

    MartinB schrieb (29.12.10 · 10:50 Uhr):

    > Wenn die gemessenen Lebensdauern von teilchen, die mit 99% c fliegen, sich nicht gegenüber ruhenden Teilchen verändern, dann wäre das eine Verletzung der SRT.

    Heißt das (unter Gebrauch der oben vorgeschlagenen, eher konkreten und drastischen Formlierung) also:

    Die SRT ist falsch, falls sich zwei (instabile) Teilchen finden ließen, die sich zueinander bewegten (ob nun exakt mit einem Tempo von 99/100 c, oder nicht, sei dahingestellt), und deren Leben (exakt?) gleich lang dauerten
    ??

    Mal ganz abgesehen vom sicherlich unstrittigen Umstand, dass es verschiedene bekannte Teilchensorten (mit verschiedenen mittleren Lebensdauern) gibt und es sogar weitere noch unbekannte geben könnte,
    und ganz abgesehen dass z.B. die PDG für bestimmte Teilchensorten nur “mittlere Lebensdauer” (aber nicht “Lebensdauer jedes einzelnen Teilchens” angibt, und auch das wohl nur für bestimmte “Zerfallsbedingungen”; etwa Zerfall eines “freien Teilchens” in “freie Zerfallsprodukte”) — weswegen ich oben anzuregen versuchte, stattdessen die Lebensdauern von “ein-eiigen Zwilligen” zu diskutieren:

    Konkret wie sollen denn die Lebensdauern zweier sich gegenüber einander bewegender Teilchen (oder Zwillinge) überhaupt miteinander verglichen werden?
    Das ist doch das experimentelle Problem, mit dessen Lösung sich die SRT ganz wesentlich beschäftigt. Welche eventuellen anderen nachvollziehbaren Lösungsmöglichkeiten gäbe es denn dafür?

    Bzw.: konkret welche Messgröße soll beim entsprechenden Experiment unter “Dauer” verstanden werden?
    Etwa die Messgröße (der SRT), die von der PDG (bzw. den Detektor-Kollaborationen, die Messwerte beitragen) unter “Dauer” verstanden wird (und deren Werte “gemittelt” werden)? …

    > Wenn Die Teilchen im CERN mit Überlichgeschwindigkeit durch den Beschleuniger fliegen, wäre das eine Widerlegung der SRT
    > […] Da ich Überschallgeschwindigkeiten messen kann, könnte ich dann auch Überlichtgeschwindigkeiten messen.

    Vermutlich wäre das mit der Forderung verbunden, dabei auch den Brechungsindex der Region zu messen und zu berücksichtigen, in der ein solches hypotetisches Teilchen ausgetauscht würde. (Vgl. die obige Voraussetzung, dass “der Brechungsindex [d]er gegebene[n] Region gleich 1” gemessen wurde.)

    Es bleibt noch festzulegen, wie denn “(Durchschnitts-)Geschwindigkeit” (z.B. eines gegebenen Teilchens gegenüber einem Startbeobachter und einem Zielbeobachter, die messen, dass sie dabei zueinander ruhten) ermittelt werden soll;
    nämlich (deine Zustimmung vorausgesetzt) als

    “Distanz zwischen Startbeobachter und Zielbeobachter /
    Dauer des Zielbeobachters von seiner Anzeige gleichzeitig zur Startanzeige bis zu seiner Anzeige der Passage des Teilchens”.

    > Ein Versuchsaufbau könnte per Lichtsignal synchronisierte Uhren verwenden

    Wozu Synchronisation? Reicht nicht die Feststellung von Gleichzeitigkeit, für gerade einmal ein einziges Paar von Anzeigen?
    (Entsprechend des obigen Ansatzes: die Anzeige des Starts, dass das Teilchen aufgebrochen ist, und der dazu gleichzeitigen Anzeige des Ziels.)

    > oder ich synchronisiere zwei Uhren am selben Ort und bewege dann A und B mit exakt gleicher Geschwindigkeit an ihre Endpunkte […]

    Das würde doch offenbar die Feststellung und den Vergleich von “Geschwindigkeits”-Werten schon erfordern und voraussetzen. Aber wir sind doch hier noch mitten in der Diskussion und Festsetzung der Messmethode, wie solche Messwerte überhaupt ermittelt werden sollten.
    Wir fallen also zwangsläufig auf die erstgenannte Methode (“per Lichtsignal“) zurück, da diese offenbar keine “Geschwindigkeits”-Messwerte erfordert.

    dann lasse ich sie zueinander ruhen und schicke das überlichtschnelle Signal.

    Na schön.
    Und??
    Wie soll denn die Distanz (zwischen den relevanten Beteiligten) während des Versuches ermittelt werden?
    Insbesondere, falls diese Messung entsprechend der SRT-Methode (und d.h. mittlerweile wohl auch: entsprechend der SI-Methode) erfolgen soll:
    Sind dabei alle Ping-Signale während des Versuches zu berücksichtigen? (Oder welche nicht?)

    > Die Behauptung, Energien seien nur im Rahmen der SRT sinnvoll definierbar (so verstehe ich dich im Moment) wird auch dadurch widerlegt, dass der Energiebegriff älter ist als die SRT und dass die Newtonsche Mechanik und die QM nicht-relativistisch sinnvoll mit dem Energiebegriff operieren können.

    Und zwar wie??
    Wie sollte “nicht-relativistische Physik” denn nachvollvollziehbar sein, wenn nicht (bestenfalls) als mehr oder weniger gute Näherung an (relativistische) Physik für mehr oder weniger hinreichend kleine Werte “beta” und “1/Krümmungradius”?

    > Photoeffekt in Verbindung mit einer bias-Spannung – bias-Spannung für Licht unterschiedlicher Wellenlängen (an unterschiedlichen Orten im Spektrum) messen, dann werde ich eine schöne lineare Beziehung zwischen Ort im Spektrum und bias-Spannung (sprich Energie) bekommen, ganz ohne Frequenz.

    Wie wären “Spannungs“-Werte zu ermitteln? (abgesehen davon, dass diese Werte wohl affin zu den Werten von “Orten im Spektrum” sein können; für geeignet ausgewählte gültige Versuche).

    Und nochmal: was soll das mit dem Begriff
    > Energiezunahme […]
    zu tun haben, den du oben benutzt hast?
    (Oder soll “bias-Spannungs”-Zunahme schlicht ein Synonym für Energiezunahme sein?)

    > Das Einstein-Zitat […]

    (Ich kenn jedenfalls die aus Ann. Phys. 17, 1905, zitierte Formulierung; und von anderen ist mir Einstein nicht als Quelle bekannt.)

    > […] soll meiner Ansicht nach zunächst mal nur zeigen, dass man keine philosophische Definition von Zeit braucht, um mit dem Begriff arbeiten zu können.

    Es scheint mir recht nebensächlich, ob man die Einsteinsche Definition “philisophisch” nennt, oder nicht.
    Und ja: entscheidend ist die Nachvollziehbarkeit des Begriffes. Das ermöglicht und schließt sicher auch die Unterscheidungen ein, von

    – “Zeit” im Sinne einer einzelnen Anzeige einer bestimmten Uhr,

    – “Zeit” im Sinne der Reihenfolge aller Anzeigen einer bestimmten Uhr,

    – “Dauer” zwischen zwei bestimmten Anzeigen einer bestimmten Uhr, als Maß der Zeit dieser Uhr von der einen zur der anderen ihrer Anzeige,

    – “gute Uhr” im Sinne einer Uhr, deren Anzeigen reelle Zahlen (“t”) so zugeordnet wurden, dass sie affin zur Dauer zwischen entsprechenden Anzeigenpaaren sind, oder

    – “ideale Uhr” im Sinne einer Uhr, zwischen deren aufeinanderfolgenden Anzeigen gleiche Dauer festgestellt wurde.

  304. #305 TheBug
    6. Januar 2011

    @Frank Wappler: Leider ist Dein Beitrag nicht gerade das was man übersichtlich oder verständlich nennen kann.

    Die Zeitdilatation wird ständig vom Polarlicht demonstriert, kurz zusammengefasst im zweiten Kapitel über Myonen bei Wikipedia nachzulesen:
    https://de.wikipedia.org/wiki/Myon

    Bei den restlichen Punkten(?)/Fragen(?) ist mir nicht klar worauf Du hinaus willst.

  305. #306 Frank Wappler
    6. Januar 2011

    TheBug schrieb (06.01.11 · 08:57 Uhr):
    > Die Zeitdilatation wird ständig vom Polarlicht demonstriert, kurz zusammengefasst im zweiten Kapitel über Myonen bei Wikipedia nachzulesen:
    https://de.wikipedia.org/wiki/Myon

    Das Wort “Polarlicht” kann ich im genannten Wikipedia-Artikel nicht finden.

    Seinem zweiten Abschnitt (“Myon#Myonzerfall”) kann man entnehmen, dass für (alle bisher gemessenen Stichproben von) Myonen konsistent eine bestimmte mittlere Lebensdauer gefunden wurde (bezogen auf “freie Myonen” und “freie Zerfälle”, also insbesondere nach Berücksichtigung/Abzug des im Artikel beschriebenen zusätzlichen Zerfallskanals von “negativen Myonen in Materie”).

    SWIV, wurden dabei zur Feststellung des Wertes der mittleren Lebensdauer von Myonen (als Teilchenart insgesamt) die Lebensdauern einzelner Myonen entsprechend den Messdefinitionen der (S)RT festgestellt und gemittelt
    (also insbesondere unter jeweiliger Ermittlung und Berücksichtigung des im Rahmen der SRT herzuleitenden “Lorentz-Faktors”;
    und ggf. sicher unter Berücksichtigung/Gewichtung von systematischen Unsicherheiten, die ebenfalls entsprechend den Messdefinitionen der (S)RT festgestellt wurden).

    Der erste Abschnitt (“Myon#Kosmische Strahlung”) beschreibt dagegen, dass für bestimmte verschiedene Stichproben von Myonen drastisch verschiedene Halbwertszeiten gefunden worden wären.

    Zusammen mit dem verlinkten https://de.wikipedia.org/wiki/Halbwertszeit und dem wiederum dort weiterführenden https://de.wikipedia.org/wiki/Lebensdauer_(Physik) ist zu schließen, dass demnach
    für bestimmte verschiedene Stichproben von Myonen drastisch verschiedene Lebensdauern gefunden worden wären.

    Die beiden genannten Abschnitte des Artikels über Myonen bei (der deutschen) Wikipedia widersprechen sich also offenbar in ihren Aussagen zur mittleren Lebensdauer von Myonen.

    (Leider ist aber dieser Artikel wohl viel zu spärlich verlinkt, als dass dieser Widerspruch auch ohne besondere Recherche auffallen würde.
    Die im einen oder anderen Abschnitt gebrauchten Worte “Halbwertszeit” bzw. “(mittlere) Lebensdauer” sollten wohl anders verlinkt bzw. disambiguiert werden, sofern sie überhaupt enzyklopädisch wären …)

    p.s.
    Worauf ich (unmittelbar) hinauswill ist, dass diejenigen, die behaupten, die SRT sei experimentell zu testen, den hypothetischen Fall einer experimentellen Falsifikation mal möglichst konkret formulieren und durchdenken:
    Die SRT ist falsch (experimentell falsifiziert), falls folgender Befund erhalten wurde: …”.

    Allein deswegen, weil gewisse Wikipedia-Artikel widersprüchlich geschrieben bzw. verlinkt sind, wäre die SRT ja sicherlich nicht “falsifiziert” zu nennen.

  306. #307 TheBug
    6. Januar 2011

    [] Du verstehst den Unterschied zwischen einer Größenordnung und 1 Promille.
    [] Du verstehst, das eine noch kürzere Lebensdauer das Gegenteil eines Gegenbeweises für dieses Beispiel ist.

  307. #308 MartinB
    6. Januar 2011

    @FrankWappler
    Mir geht es ähnlich wie TheBug – ich verstehe auch nicht recht, was du sagen willst.

    “Die SRT ist falsch (experimentell falsifiziert), falls folgender Befund erhalten wurde: ”
    a. In einem teilchenbeschleuniger auf 0.9 c beschleunigte instabile Teilchen der Sorte X haben dieselbe Lebensdauer (Halbwertszeit) wie ruhende teilchen derselben Sorte. Könnte man in einem Ringbeschleuniger beim jeweiligen Durchqueren der umlaufenden teilchen an einem Ort messen.

    b. In einem elektrischen Feld entsprechender Stärke können Elektronen mit Überlichtgeschwindigkeit beschleunigt werden. Versuchsaufbau: Elektronenkanone A beschleunigt Elektronen, die dann zum Ort B fliegen. Bei Eintreffen der elektronen am Ort B wird ein entsprechender Rückimpuls ausgesandt. Der zeitliche Abstand bis zum Wiedereintreffen der elektronen am Ort A wird mit einer herkömmlichen Quarzuhr bekannter Präzision gemessen. Der Abstand zwischen A und B wird mit z.B. mit einem geeichten metermaß gemessen (würde bei hinreichend großen Abständen natürlich lange dauern, ist aber prinzipiell möglich). Natürlich muss die Messgenauigkeit der Apparaturen berücksichtigt werden.

  308. #309 Bullet
    6. Januar 2011

    Zusatz zu Martin: es ist leicht errechenbar, daß Elektronen bereits bei einer Beschleunigungsspannung von 255 kV (also nicht wirklich so eine waaahnsinnige Geschichte) laut klassischer Berechnung c überschreiten. Absolut JEDES Verhalten von Elektronen, die >1MeV aufm Tacho haben, aber langsamer als Licht unterwegs sind, sollte schon gut begründet werden, wenns kein relativistischer Faktor sein soll. Der zeigt nämlich ziemlich gute Vorhersagen.

  309. #310 TheBug
    6. Januar 2011

    Man könnte auch einfach sagen: Relativity sucks less 😉

    Weil andere Ansätze bisher nur schlechtere Erklärungen liefern.

  310. #312 erik
    6. Januar 2011

    TheBug: Relativity sucks less 😉
    Bullet: Ach ja: der Rechenweg

    Der Rechenweg zeigt mir deutlich, wo das Problem sich darstellt: rechnen kann ich mit der „NM-Mathematik“ sehr wohl Geschwindigkeiten V>c, nur das sich bei v=c die Eigenschaften des NM-Raumes ändern wird im Rechenbeispiel nicht berücksichtigt. Der QM-Raum saugt auch nicht, sondern gestaltet den Raum nach Bedingungen, die ich bisher nicht beschrieben habe.

    Gleiches gilt beim Messen: Dauer, Strecke … sind Grössen des NM-Raumes und alle Messmethoden tasten sich aus dem Verstehen des NM-Raumes an die Grenzzone zum QM-Raum heran. Die Beispiele von den vielen Arten von Ungenauigkeiten müssen nicht mal Ungenauigkeiten beim messen sein, sondern sind vielleicht schon Eigenschaften des „sich gestaltenden“ QM-Raumes. Nur kann ich sie aus der Sicht und der Erfahrungen mit dem NM-Raum nicht anders deuten.

    Wie weiter aus meiner Sicht?
    Ich muss Ordnungssysteme finden, welche einen Raum immer im GANZEN gestalten, beeinflussen.
    Vorher muss ich eine Vorstellung aufbauen, was denn ein GANZER, ein Vollständiger Raum ist. Da hilft mir WIN-TIME:RZ (ich würde einen Link setzen, aber habe keine Ahnung wie der dann zum entsprechenden Kommentar scrollen würde).
    Wenn dieser Vollständige Raum eine Struktur besitzt in der QM und NM ihren Platz eingenommen haben, dann beschreibe ich die Ordnungssysteme in ihrem Wirkungfeld /-raum (z.B. Entropie und Eigenschaften von Gasen …).
    Dann bemerke ich, das Wissenslücken in einem Ordnungsraum in einem benachbarten Ordnungsraum beschrieben sind, dort die Wissenslücken an anderen Stellen auftreten.
    Dann geben mir zum Beispiel die „Biologie“ im Universum Inspiration Dinge im Raum der „Chemie“ im Universum zu verstehen.

    Dann brauche ich einen „ordentlichen“ Rechner der mir entsprechende Daten auswertet. Den habe ich auch!
    → und da kommt meine Friky-Natur (oder wie schreibt sich das Wort, welches Missgeburt heissen kann oder einen Menschen mit aussergewöhnlichen Fähigkeiten beschreibt?)
    → Ich assoziiere so gerne und kann das auch wirklich gut: Ich mache das in der sagenumwobenen Stadt, die es eigentlich nicht geben soll. Dann gehe ich nach down town Bielefeld und schmeisse meinen Bio-A-Körper-Rechner an, lege mich in das NICHT von Bielefeld und komme nach und nach zu Resultaten, die kein Messgerät der physikalischen Welt je erzielen wird.

    Warum ich meinen Körper mit zugehörigen Geist so „benutzen“ kann hat mir Dr.E.Berndt in einem früheren Kommentar zwar nicht erläutert, aber treffend beschrieben: „Zwecke und Naturgesetze entstammen unserem Selbstbewusstsein. Selbstbewusstsein und Welt sind identisch.“
    Ich füge hinzu: Der Verstand, gepaart mit SELBST und dessen Bewusstsein, können zumindest die Welt vollständig verstehen. Schon alleine dadurch muss (wird, ausgeübter ZWANG des VERSTEHENS) die Welt sich verändern. (Fast so wie beim Messen in der QM.)

    Gerne würde ich eine Reise mit Herrn Dr. E. Berndt zu den Komplexen Zahlen und zur imaginären Einheit „i“ machen. Ich würde mich anstrengen Mathematik und Assoziation in vernünftiger Art und Weise zu verbinden. Dann kann aus meiner Sicht das Ausgebrannte und Ausgelaugte mit NEUEM aufgefüllt werden.

  311. #313 TheBug
    6. Januar 2011

    Schon wieder dieses kalte Schauern, ich glaube hier spukt es…

  312. #314 MartinB
    6. Januar 2011

    @erik
    Ich weiß nicht, ob hier irgendjemand deine Texte liest (ich tue es nicht mehr). Kannst du dir nicht vielleicht einen eigenen Blog besorgen und dort weiterschreiben, so richtig passt das hier nicht hin.

  313. #315 erik
    6. Januar 2011

    Warum passen Lösungsansätze zu oben beschriebenen Problemen der SRT nicht in diesen Blog?

    Was bedeutet: “so richtig pass das hier nicht hin?” Ist das statistische Genauigkeit?

  314. #316 TheBug
    6. Januar 2011

    Na der Gegenstand dieses Blogs ist halt nicht: Wissenschaft aus dadaistischer Sicht.

  315. #317 nihil jie
    6. Januar 2011

    noch mal ich mit ein paar belegen für die gültigkeit von der QM 🙂 all die medizinischen gerätschaften wie zb. die MRT oder auch als Kernspintomografie bekannt 😉 wer dabei die richtigkeit der QM modelle in frage stellt agiert so wie jemand der sagt alle grundlagen des baus eines verbrennungsmotors seien falsch und sich dann ein paar minuten später in sein auto setzt um damit herum zu fahren 😉

  316. #318 Bullet
    6. Januar 2011

    Oder im Klartext:
    Erik, ich versteh dich nicht. Von hier aus sieht das alles wie zusammenhangloses Gerede aus. Und ich konnte deinen auf mich wirr wirkenden Worten nicht folgen, also hab ich einfach aufgehört zu lesen.
    Mir geht es offenbar nicht allein so.
    Vielleicht versuchst du es mal mit nur einem Gedanken in wenigen kurzen Sätzen?
    Das wäre möglicherweise hilfreich.

  317. #319 nihil jie
    6. Januar 2011

    aber einen habe ich noch 😉 einer der quantenmechanischen phänomene die wir tag täglich erleben ist wenn man durch eine scheibe schaut. ein teil der photonen gehen dabei durch die scheibe durch und ein teil nicht… welches von den ankommenden photonen die scheibe passieren wird und welches nicht ist dabei nicht bekannt.

  318. #320 Michael.S
    6. Januar 2011

    Zitat:
    ###############################################
    Kommentar-Direktlink noanna· 01.01.11 · 02:48 Uhr

    Um ein Haar hä#tte ich es übersehen:

    https://unglaublichkeiten.org/unglaublichkeiten/htmlphp2/u2_1138ART_SRT.html

    Kürzer geht es wohl nicht, den SRT + ART Quatsch zu widerlegen. Danke im Voraus für alle depperten Kommentare!
    Ende Zitat ###############################################

    Ich habe mir diesen Link einmal näher angeschaut. War recht interessant.
    Danach sah ich mir diese Webseite genauer an und lese diesen ganzen
    Quatsch von der Antarktis, den Flugmaschinen usw. Dadurch wird der oben angegebene Artikel natürlich etwas unglaubwürdiger.

    Allerdings reichen meine mathematischen Kenntnisse nicht aus, um zu verifizieren ob das absoluter Blödsinn ist, was der da behauptet.

    Frage: Wird auf diesen Artikel deswegen nicht eingegangen, weil das schon einmal besprochen wurde (wenn ja bitte einen Hinweis wo das abgehandelt wurde). oder weil das so unnsinnig ist, dass man sich die Mühe sparen kann.

    Ich möchte ausdrücklich darauf hinweisen, dass meine Frage an die “wissenschaftlich Denkenden” hier im Forum gerichtet ist. Die “alternative” Betrachtungsweise wird ja bereits im Artikel dargelegt (ich benötige keinerlei Einführung in Esoterik,Ufo’s,Astrologie, oder wo die Reichsdeutschen ihre angeblichen Kolonien haben – das interessiert mich nicht)..

    Entschuldigung – im Spamfilter hängengeblieben, hab ich erst heute gemerkt.

  319. #321 TheBug
    6. Januar 2011

    Was? Das Glas wird nicht im Laufe der Zeit dicker weil es die Lichtteilchen absorbiert?

    😉

  320. #322 nihil jie
    6. Januar 2011

    klarr… es wächst von selbst… so macht man auch glass *gg 😉

  321. #323 MartinB
    6. Januar 2011

    @erik
    Du produzierst keine Lösungsansätze, sondern reihst freie Assoziationen aneinander. Das mag zwar ein Mittel sein, um zu neuen Ideen zu kommen, ist aber selbst noch nicht soo interessant – ich hab’s ja schon mehrfach geschrieben: Verrückte Ideen, wie die Welt sein könnte, kann ich zwischen Tagesschau und Wetterkarte im Dutzend produzieren. Solche Ideen sind für sich allein nicht interessant, erst wenn sie weitergedacht werden, mit bekannten Beobachtungen im Einklang stehen und zumindest grob quantitativ erfasst werden, wird daraus Physik.

    @nihilje
    Guter Punkt mit dem MRT, muss ich mir merken.

  322. #324 roel
    6. Januar 2011

    @nihil jie Ah, das Phänomen habe ich bis heute morgen an meinem Haustürvordach bewundert. Da hing ein Eiszapfen der hat auch fleißig immer Photonen absorbiert und ist ziemlich groß geworden. Leider ist es jetzt zu warm geworden. Obwohl – funktioniert das auch mit Wasser?

  323. #325 roel
    6. Januar 2011

    @Erik warum gibt es Bielefeld nicht. Das ist doch die Stadt in Deutschland mit dem höchsten Berg und demnächst auch mit dem tiefsten Stollen.

  324. #326 nihil jie
    6. Januar 2011

    aber mit den scheiben ist es wirklich so… ein teil geht durch und ein teil wird reflektiert… je nach glassorte natürlich. und man kann nicht verhersagen welches photon die scheibensubstand passieren wird und welches reflektiert wird. vielleicht finde ich das paper da zu… ich hatte mir das schon vor langer langer zet auf einer meiner alten festplatten abgespeichert 😉 aber bei meiner unordnung… naja… mal sehen 😉

  325. #327 nihil jie
    6. Januar 2011

    @MartinB

    ja… ich frage mich warum mir das nicht früher eingefallen ist. die medizin (also zumindest die radiologie) hat in den letzten beiden jahrzehnten sehr oft von den erkenntnissen der teilchenphysik, im speziellen von der QM, profitiert. gerade was die echtzeit abbildungen betrifft. aber nicht nur die medizin… auch die archeologie oder biologie usw. manche verfahren werden heute auch bei der tumorbekämpfung eingesetzt…

    Teilchenbeschleuniger sind längst zu einem unverzichtbaren Werkzeug für die moderne Wissenschaft geworden. Dabei finden sie nicht nur in der Teilchenphysik Anwendung, sondern kommen auch in der Medizin – etwa zur Therapie von Tumoren – zum Einsatz.

    Quelle: Welt der Physik

  326. #328 MartinB
    6. Januar 2011

    @nihilje
    Natürlich ist das so: Glas ist nicht perfekt durchsichtig, einige Photonen werden reflektiert. Wie immer in der Quantenmechanik (oder QED) kann man nicht vorhersagen, welches Photon das ist. Wie üblich empfehle ich Feynmans QED-Buch zum Thema – da ist genau das das Aufhängerproblem.

  327. #329 erik
    6. Januar 2011

    @MB
    Genau das ist der Weg!
    Mal sehen ob ich diesen Weg alleine gehen muss.
    Durch unsere Diskussion seid ihr ja schon indirekt dabei.

  328. #330 TheBug
    6. Januar 2011

    Betr.: Bielefeld

  329. #331 Frank Wappler
    6. Januar 2011

    MartinB schrieb (06.01.11 · 12:23 Uhr):

    > “Die SRT ist falsch (experimentell falsifiziert), falls folgender Befund erhalten wurde: ”
    > a. In einem teilchenbeschleuniger auf 0.9 c beschleunigte instabile Teilchen der Sorte X haben dieselbe Lebensdauer (Halbwertszeit) wie ruhende teilchen derselben Sorte. Könnte man in einem Ringbeschleuniger beim jeweiligen Durchqueren der umlaufenden teilchen an einem Ort messen.

    In den Webseiten der Particle Data Group (der internationalen Kollaboration, deren Aufgabe es ist, Teilchenphysik betreffende Befunde zu erfassen und öffentlich zu machen) findet man Messwerte der mittleren Lebensdauer von “geladenen Kaonen”: https://pdg.lbl.gov/2010/listings/rpp2010-list-K-plus-minus.pdf

    Angegeben sind Messwerte sowohl für Stichproben “at rest” als auch “in flight” (wobei sich diese Kaonen z.B. bei der in PR185, 1676 (1969) beschriebenen Messung gegenüber dem Detektor mit ca. 0.95 c bewegten; vgl. https://prola.aps.org/abstract/PR/v185/i5/p1676_1 ).

    Die Messwerte beider Stichproben unterscheiden sich um weniger als 1 % voneinander.

    Ist die SRT demnach falsch (experimentell falsifiziert)?
    Oder welche Schlussfolgerung ist zu ziehen?

    > b. In einem elektrischen Feld entsprechender Stärke können Elektronen mit Überlichtgeschwindigkeit beschleunigt werden. Versuchsaufbau: Elektronenkanone A beschleunigt Elektronen, die dann zum Ort B fliegen. Bei Eintreffen der elektronen am Ort B wird ein entsprechender Rückimpuls ausgesandt. Der zeitliche Abstand bis zum Wiedereintreffen der elektronen am Ort A

    … die Dauer As vom Senden der Signal-Elektronen zu B bis zum Empfang der Antwort-Elektronen zurück von B …

    > wird mit einer herkömmlichen Quarzuhr bekannter Präzision gemessen. Der Abstand zwischen A und B wird mit z.B. mit einem geeichten metermaß gemessen

    Entsprechend den SI-Einheiten-Definition, s. z.B.
    https://www.bipm.org/en/si/si_brochure/
    besteht das “Eichen eines Längenmaßes” (oder schlicht: die Feststellung der Distanz zwischen zwei gegebenen Enden, sofern diese dabei zueinander ruhten) doch gerade darin,
    dass (jeweils) eines der beiden Enden eine Signalanzeige darstellte (z.B. das Aussenden einiger Elektronen), das andere Ende dieses Signal wahrnahm (spätestens “in Tateinheit” mit der Beobachtung des Eintreffens der Signal-Elektronen), das erste Ende wiederum das Echo wahrnahm (spätestens “in Tateinheit” mit der Beobachtung des Eintreffens von Antwort-Elektronen)
    und die Distanz dieser beiden Enden zueinander einvernehmlich als
    “c/2 Pingdauer”
    bewertet wird.

    Die Pingdauer jedes der beiden Enden, von der Signal-Anzeige bis zur Wahrnehmungs-Anzeige des Echos, ist demnach höchstens so groß wie (und i.A. kleiner als) die schon diskutierte Dauer As vom Senden der Signal-Elektronen zu B bis zum Empfang der Antwort-Elektronen zurück von B;

    die Distanz von A und B zueinander ist demnach höchstens so groß wie (und i.A. kleiner als)
    “c/2 Dauer As vom Senden der Signal-Elektronen zu B bis zum Empfang der Antwort-Elektronen zurück von B”;

    die (Durchschnitts-)Geschwindigkeit des Elektronenaustausches ist demnach höchstens so groß wie (und i.A. kleiner als)
    “c”;

    der hypothetische Befund eines Wertes der (Durchschnitts-)Geschwindigkeit größer als “c” ist durch die Mess- bzw. Eich-Methodik von vornherein ausgeschlossen;
    es handelt sich also nicht um eine Hypothese, die noch einer ausdrücklichen experimentellen Prüfung (durch Anwendung der Messoperation auf konkrete Beobachtungsdaten) bedürfte;
    die SRT wäre durch derartige Versuchsanordnungen keinesfalls zu falsifizieren.

    Bullet schrieb (06.01.11 · 12:51 Uhr):

    > es ist leicht errechenbar, daß Elektronen bereits bei einer Beschleunigungsspannung von 255 kV (also nicht wirklich so eine waaahnsinnige Geschichte) laut klassischer Berechnung c überschreiten.

    Sicher ist es leicht genug, etwas wie

    “Sqrt[ 2 * 255 / 511 ] c”

    hinzurechnen;
    oder meinentwegen auch

    “Sqrt[ 2 * 256 / 511 ] c”.

    Das heißt keineswegs, das eine derartige “klassische Berechnung” irgendeine nachvollziehbare Bedeutung hätte; außer als eine mehr oder weniger miese Näherung für den Ausdruck

    “Sqrt[ 1 – (1 / (256 / 511) + 1)^2 ] c”,

    der bei der Berechnung der Endgeschwindigkeit eines Elektrons (von ca. 511 keV / c^2 Masse) auftritt, das über eine Potentialdifferenz von 256 kV aus der Ruhelage beschleunigt wurde.

    > relativistischer Faktor […] Der zeigt nämlich ziemlich gute Vorhersagen.

    Was denn für Vorhersagen??
    Etwa die 256 kV ?
    Oder die ca. 511 keV / c^2 ?? …

  330. #332 TheBug
    6. Januar 2011

    1. Ist der Lorentzfaktor exponentiell.
    2. Könntest Du mal bitte genauer angeben wo Du diese Daten gefunden haben willst? Ich habe wenig Bock mich durch diesen Datenhaufen durchzubuddeln und anderen hier wird das auch so gehen. Die zweite Referenz spricht übrigens nicht von Ruhe und 0,95c, sondern von 1,6 und 2,0 MeV und das ist alles andere als Ruhelage.

  331. #333 Bullet
    6. Januar 2011

    @ Frank W.:
    Und ich hatte einen Link zum Thema gesetzt, weils bei Ulrich Berger bereits Thema war. Für dich extra hier nochmal:
    https://www.scienceblogs.de/kritisch-gedacht/2010/09/der-einsteinwiderleger-im-kepler-salon.php#comment155716

    Lies dir die Rechnung durch. Die ist echt simpel.

    Das heißt keineswegs, das eine derartige “klassische Berechnung” irgendeine nachvollziehbare Bedeutung hätte

    Wenn du es nicht nachvollziehen kannst, kann ich dir nicht helfen. Die Formelsammlung sagt etwas darüber aus, wie Elektronen in einem Ladungsfeld beschleunigt werden. Wenn ich für die Endgeschwindigkeit c eingebe und nach U auflöse, komm ich auf 255 kV. Sprich: ohne RT ist ein Elektron mit nur höchstens 255 kV auf c zu bringen. Da das aber mit der beobachtbaren Realität nicht übereinstimmt (du weißt bestimmt, mit welchen Tricks Elektronen heutzutage auf GeV-Orgien gebracht werden), ist die klassische Näherung unzutreffend – und der relativistische Ansatz passend. Was für ein Zufall.

    Im Übrigen bin ich mit deinem komischen Eichkrempel nicht einverstanden. Die Lauflänge einer Strecke bestimmst du bestimmt nicht mit einem Elektronenstrahl – sondern mit einem Laserpuls.
    Sobald ein Elektronenstrahl dieselbe Distanz schneller überbrückt als ein Laserstrahl (und das ist ein reiner Vergleichswert, also nix mit “eichen”), könnten wir nochmal über “superluminal” sprechen.
    Du kennst dich doch sonst so gut aus. Warum jetzt plötzlich so ein wirres Zeug?

  332. #334 perk
    7. Januar 2011

    In den Webseiten der Particle Data Group (der internationalen Kollaboration, deren Aufgabe es ist, Teilchenphysik betreffende Befunde zu erfassen und öffentlich zu machen) findet man Messwerte der mittleren Lebensdauer von “geladenen Kaonen”: https://pdg.lbl.gov/2010/listings/rpp2010-list-K-plus-minus.pdf

    Angegeben sind Messwerte sowohl für Stichproben “at rest” als auch “in flight” (wobei sich diese Kaonen z.B. bei der in PR185, 1676 (1969) beschriebenen Messung gegenüber dem Detektor mit ca. 0.95 c bewegten; vgl. https://prola.aps.org/abstract/PR/v185/i5/p1676_1 ).

    Die Messwerte beider Stichproben unterscheiden sich um weniger als 1 % voneinander.

    Ist die SRT demnach falsch (experimentell falsifiziert)?
    Oder welche Schlussfolgerung ist zu ziehen?

    da du immer passende quellen und gute informationen zur verfügung hast, kann ich bei deiner (von mir gefetteten) frage nur noch glauben, dass du uns verarschen willst

    der dritte absatz des 2. kapitels des von dir angeführten papers lautet:

    Since the measurement has to be made in flight, the observed lifetime in the laboratory must be transformed to the rest frame of the meson. Thus, an exact knowledge of the momentum is essential; this, coupled with the knowledge of the decay length, is equivalent to the precise timing required in an experiment at rest. However, the relativistic time dilation, depending on the momentum chosen, imposes a necessity for long decay paths.

    2. Könntest Du mal bitte genauer angeben wo Du diese Daten gefunden haben willst? Ich habe wenig Bock mich durch diesen Datenhaufen durchzubuddeln und anderen hier wird das auch so gehen. Die zweite Referenz spricht übrigens nicht von Ruhe und 0,95c, sondern von 1,6 und 2,0 MeV und das ist alles andere als Ruhelage.

    es waren sogar GeV statt MeV also deutlich größer als die ruheenergie der kaonen und damit voll relativistisch

  333. #335 MartinB
    7. Januar 2011

    @FrankWappler
    Langsam begreife ich gar nichts mehr – das mit den Kaonen war ja wohl ein schlechter Witz, oder?

    Den Teil mit dem Ping-Experiment verstehe ich gar nicht. ich kann den Abstand zwischen den Punkten A und B auch über de Austausch eines Ping-Signals mit Licht messen – habe ich hier nicht so vorgeschlagen, weil das ja die Konstanz von c impliziert. Aber selbst wenn ich das so mache, dann könnte ich trotzdem sehen, dass ein Elektronen-Ping schneller ist als das. Ich kann deinem text beim besten Willen nicht entnehmen, warum das nicht gehen soll.

    Dieser Satz hier

    Die Pingdauer jedes der beiden Enden, von der Signal-Anzeige bis zur Wahrnehmungs-Anzeige des Echos, ist demnach höchstens so groß wie (und i.A. kleiner als) die schon diskutierte Dauer As vom Senden der Signal-Elektronen zu B bis zum Empfang der Antwort-Elektronen zurück von B;

    gilt eben nur, wenn c die Grenzgeschwindigkeit ist, und das kann ich wie beschrieben experimentell prüfen.

    Nimm als Analogie die Schallgeschwindigkeit: Ich kann zum Bestimmen der Entfernung auch einen Schall-Ping verwenden. Trotzdem heißt das nicht, dass ich nur aufgrund dessen niemals Überschallsignale sehen kann – jedes Gewitter beweist das.

  334. #336 Frank Wappler
    7. Januar 2011

    perk schrieb (07.01.11 · 07:17 Uhr):

    > kann ich bei deiner (von mir gefetteten) frage nur noch glauben, dass du uns verarschen willst

    Mein Satz bezieht sich unmittelbar auf die (oben im Zusammenhang zitierte, und zweckmäßig zugespitzt formulierte) Aussage von MartinB. Sicher hat er diese Aussage auf meine Anregung hin geschrieben, für die wiederum die vorausgegangene Diskussion und sein Artikel an sich Anlass gaben.
    Ich würde mich nicht zur Unterstellung versteigen, dass er uns damit verarschen will; aber der mit Hilfe der zugespitzten Formulierung gerade gezeigte Widerspruch bedarf sicher einer Klärung.

    > der dritte absatz des 2. kapitels des von dir angeführten papers lautet:

    Danke für das Zitat; Zugang zu PR-Artkel-Inhalten habe ich ansonsten nämlich nur in der Bibliothek. (Wohl dem, der hat. &)
    Aus dem oben gezeigten Abstract hatte ich lediglich den Impulsbereich der Stichprobe (1.6 – 2.0 GeV / c) entnommen.
    >

    […] the observed lifetime in the laboratory must be transformed to the rest frame of the meson

    Falls Lobkowicz et al. mit “observed lifetime in the laboratory” nicht etwa die (gemessene) individuelle Lebensdauer eines (jeden) einzelnen Myons meinen, sondern stattdessen die (gemessene) Dauer von Laborbestandteilen (Beschleuniger, Detektor) während sich ein (jedes) einzelne Myon darin aufhielt bzw. bewegte,
    dann ist wohl genau dieser doppelsinnige Gebrauch des Wortes “lifetime” dazu geeignet, jemanden zu verarschen.

    (Wie oben bereits bemerkt, wäre in einer ordentlichen Enzyklopädie solcher Doppelsinn zumindest durch ausdrücklich disambiguiertes Verlinken aufzulösen, oder besser ganz zu vermeiden.)

  335. #337 MartinB
    7. Januar 2011

    @FrankWappler
    Das Zitat von perk ist doch eindeutig: Die im labor gemessene lebensdauer wurde mit Hilfe der SRT auf das Eigensystem des kaons umgerechnet. Im Laborsystem lebte das Kaon als entsprechend dem gamma-Faktor länger. Die Lebensdauer bei unterschiedlichen geschwindigkeiten stimmen also nur nach Umrechnung mit den Formeln der SRT überein. Täten sie das nicht, dann wäre das ein Problem für die SRT.
    Das kann man doch eigentlich nicht missverstehen.

  336. #338 perk
    7. Januar 2011

    Ich würde mich nicht zur Unterstellung versteigen, dass er uns damit verarschen will; aber der mit Hilfe der zugespitzten Formulierung gerade gezeigte Widerspruch bedarf sicher einer Klärung.

    nö, denn der widerspruch entsteht nur wenn man absichtlich den zusammenhang zwischen messvorgang+messergebniss und datenverarbeitung/interpretation+endergebnis ignoriert und nach belieben aussagen missversteht

  337. #339 perk
    7. Januar 2011

    hoppla ein s zuviel.. lesbar ist es hoffentlich trotzdem

  338. #340 Bullet
    7. Januar 2011

    jup

  339. #341 Frank Wappler
    7. Januar 2011

    Bullet schrieb (06.01.11 · 23:01 Uhr):

    > https://www.scienceblogs.de/kritisch-gedacht/2010/09/der-einsteinwiderleger-im-kepler-salon.php#comment155716

    und dort (30.10.10 · 12:05 Uhr):

    > [… von] Leuten, die jeden Satz für blödsinnig halten, in dem das Wort “relativistisch” vorkommt.

    Wo gibt’s denn sowas?? Ich jedenfalls rechne mich zu den Leuten, die “klassische Rechnungen” nur insofern für sinnvoll halten, als sie gewisse Näherungen von (relativistisch) nachvollziehbaren Rechnungen darstellen; und das auch nur insofern als darin auftretende Symbole auch in der Näherung ihre (relativistisch) nachvollziehbare Bedeutung als Messgrößen bzw. als Messwerte haben und behalten.

    > Von daher werd ich mich also an die klassischen Formeln halten. Mein Formelbuch sagt: Nach dem Durchlaufen der Beschleunigungsspannung U erhöht sich die Energie eines Teilchen mit der Ladung q um ΔE = q*U.

    Das sagt “mein Formelbuch” auch. (Das “Buch mit den Herleitungen” beschäftigt sich im Wesentlichen mit Variationsrechnung und wie zu entscheiden wäre, ob und in wie fern ein gegebenes System “geschlossen” blieb.)

    > Zusätzlich ist v = √(2U * q/m). Klar soweit?

    Nein, “mein Formelbuch” gibt stattdessen zu aller erst

    v = c √[ 1 – (1 / ((U * q)/(m c^2)) + 1)^2 ] an

    (und die Herleitung interessiert sicherlich auch …).
    Die Formel

    v = √[ (2 * U * q/m) ]

    steht da lediglich als Fußnote, nämlich als (etwas simplere) Näherung für Fälle von
    U * q viel kleiner als m c^2.

    Und was den Rest der gezeigten Rechnung angeht:
    16 / 9 = 1,7777777… .

    > Im Übrigen bin ich mit deinem komischen Eichkrempel nicht einverstanden. Die Lauflänge einer Strecke bestimmst du bestimmt nicht mit einem Elektronenstrahl – sondern mit einem Laserpuls.

    Man benutzt: Signalanzeigen der beiden Enden.
    (Da die Enden nach MartinBs Vorgabe u.a. in der Lage gewesen sein sollen, zur Signalanzeige Elektronen loszuschicken bzw. wahrzunehmen, handelt es sich sogar ausdrücklich um elektro-magnetische Signalanzeigen.)

    Wesentlich ist: die Distanz zwischen den Enden muss ermittelt werden,
    wenn man die Geschwindigkeit dessen feststellen will, was auch immer zwischen den Enden ausgetauscht wurde.

    > Sobald ein Elektronenstrahl dieselbe Distanz schneller überbrückt als ein Laserstrahl (und das ist ein reiner Vergleichswert, also nix mit “eichen”), könnten wir nochmal über “superluminal” sprechen.

    Erstens trägt _spätestens_ der Elektronenburst das (elektro-magnetische) Signal, das zur Distanzfeststellung (gemäß) RT zu berücksichtigen wäre.

    Zweitens: natürlich gibt es (“in der Praxis”) auch Versuche, in denen sich z.B. mit (hinreichend “Hochenergie”-)Elektronen kürzere Pingdauern zwischen gegebenen Enden finden ließen, als mit einem (hinreichend niederfrequenten, quasi-monochromatischen) Laserpuls.
    Man sagt (bzw. Mensch schließt) dann, dass diese Versuche in “Medien”, mit “Brechungsindex n > 1”, durchgeführt wurden;
    und gerade solch ein Befund wurde in der vorangegangenen Diskussion (zunächst sinnvoller Weise, aber vielleicht nur vorläufig) ausgeschlossen.

  340. #342 perk
    7. Januar 2011

    Zweitens: natürlich gibt es (“in der Praxis”) auch Versuche, in denen sich z.B. mit (hinreichend “Hochenergie”-)Elektronen kürzere Pingdauern zwischen gegebenen Enden finden ließen, als mit einem (hinreichend niederfrequenten, quasi-monochromatischen) Laserpuls.

    in welchem medium?
    verlink mal bitte das paper

  341. #343 perk
    7. Januar 2011

    hm.. ich hatte deinen nachfolgenden absatz überlesen.. ich dachte dir war klar dass wir über das hochvakuum innerhalb von teilchenbeschleunigern/speicherringen sprechen und da würde mich ein solches ergebnis deutlich überraschen..

    aber du wolltest einfach mal etwas das nicht zum thema gehört hinschreiben (eventuell um klugzuscheißen.. glückwunsch, es ist dir gelungen)

    Nein, “mein Formelbuch” gibt stattdessen zu aller erst

    wenn es bei klassischen formeln diese version ausgibt solltest du es wegschmeißen

    oder du hast bereitwillig den satz von bullet ignoriert den du trotzdem zitiert hast:

    Von daher werd ich mich also an die klassischen Formeln halten.

    wie auch immer.. dein post scheint erneut kein ziel zu haben.. du versuchst zwanghaft fehler zu korrigieren die erst auftreten indem du missverständnisse einbaust, sätze ignorierst oder deine eigenen quellen nicht liest.. denke bitte noch einmal darüber nach was du damit für diese diskussion erreichen willst

  342. #344 TheBug
    7. Januar 2011

    Komisch, ich habe schon wieder so ein Deja’vu, mir kommt es gerade so vor als wenn ich Jerusalem auf dem Markplatz stehe und ein Bärtiger einen Vortrag über Verwirrung hält…

    @Frank Wappler: Sortier doch bitte mal was Du schreibst, das ist ja unlesbar.

  343. #345 JS
    7. Januar 2011

    Wie weiter oben eh schon festgestellt wurde: Dunning Kruger lässt grüßen.

    Liebe Leugner, wo sind eure Daten, die die SRT widerlegen? Habt ihr noch was anderes auf Lager als “Die SRT widerspricht meinem gesunden Menschenverstand” oder falsch verstandene Definitionen (wie z.B. die der Zeit)? Und wieso steigert ihr euch da überhaupt so rein? Macht es euch echt so fertig, dass etwas was ihr nicht versteht, die Welt gemeinsam mit vielen anderen großartigen Erkenntnissen in eine neue Ära geführt hat? Kratzt diese Sache wirklich so sehr an eurem Ego?
    Nicht jeder kann die SRT verstehen und niemand kann alles verstehen. Habt doch ein bisschen mehr Vertrauen in die Fähigkeiten eurer Mitmenschen.

  344. #346 Frank Wappler
    7. Januar 2011

    MartinB schrieb (07.01.11 · 09:53 Uhr, 10:11 Uhr):

    > […] das mit den Kaonen war ja wohl ein schlechter Witz, oder?

    War denn deine Aussage, auf die ich mit dem PDG-Kaonen-Beispiel geantwortet habe, als schlechter Witz gemeint?
    Falls nicht, können wir die Diskussion ja gern so ernsthaft wie bisher fortsetzen.

    Welche Schlussfolgerung ist denn nun zu ziehen?
    Habe ich die Phrase “MEAN LIFE”, der tatsächlich in der PDG-Tabelle benutzt wird, etwa inkorrekt als “mittlere Lebensdauer” übersetzt und verstanden??

    Falls nicht, könnte es sein, dass du mit “Lebensdauer (Halbwertszeit)” in deiner Aussage trotzdem eine andere Größe meinst, als u.a. die PDG und (sicherlich) die Physiker, die sich auf deren Angaben berufen?

    > Die im labor gemessene lebensdauer wurde mit Hilfe der SRT auf das Eigensystem des kaons umgerechnet.

    Man sollte das (nicht zuletzt im Sinne der PDG) anders formulieren, um zu unterstreichen, dass “Dauer” eine nachvollziehbare Messgröße ist und “Lebensdauer eines gegebenen, instabilen Teilchens” einen eindeutigen reellen Wert hat; etwa:

    “Aus der Anzeige einer Laboruhr bei Erzeugung des Kaons und der Anzeige einer (i.A. anderen) Laboruhr beim Zerfall des Kaons wurde mit Hilfe der SRT (Feststellung von Gleichzeitigkeit bestimmter Anzeigenpaare) die Dauer dieser beiden Laboruhren festgestellt, während der sie das Kaon unter einander austauschten;
    und, unter Verwendung von Messwerten zur Distanz dieser Laboruhren zueinander bzw. der (Durchschnitts-)Geschwindigkeit des Kaons bzgl. dieser beiden Uhren wurde mit Hilfe der SRT (gamma-Faktor) die Lebensdauer des Kaons errechnet.”

    Es wurde ja (von perk) zurecht darauf hingewiesen, dass verschiedene Messgrößen sorgfältigst (insbesondere namentlich) zu unterscheiden sind; hier die Unterscheidung zwischen Dauer einer bestimmten Laboruhr (zwischen zwei bestimmten Anzeigen dieser Uhr) und Dauer eines bestimmten Kaons (z.B. seine Lebensdauer von seiner Entstehungs- bis zu seiner Zerfallsanzeige).

    > Die Lebensdauer bei unterschiedlichen geschwindigkeiten stimmen also nur nach Umrechnung mit den Formeln der SRT überein.

    Die MEAN LIFE Werte der Kaonen, die die PDG angibt, stimmen für die verschiedenen Stichproben gut überein, wenn sie unter Einsatz der SRT wie beschrieben ermittelt werden.

    Die jeweiligen Dauern der Laboruhren, während der sie ein Kaon unter einander austauschten, haben innerhalb jeder Stichprobe sehr verschiedene Werte (deswegen führt man ja eine anschließende Mittelung durch).

    Die entsprechenden _mittleren_ Dauern der Laboruhren sind für verschiedene Kaonen-Stichproben (von jeweils einer bestimmten Geschwindigkeit bzgl. der Laboruhren) verschieden, und hängen mit der gut übereinstimmenden mittleren Lebensdauer der Kaonen über den gamma-Faktor zusammen.

    > Täten sie das nicht, dann wäre das ein Problem für die SRT.

    Soll das heißen:
    “Die SRT ist falsch (experimentell falsifiziert), falls
    Kaonen-Stichproben gefunden würden, deren mittlere Lebensdauern (die — wohlgemerkt! — wie beschrieben unter Einsatz der SRT ermittelt wurden) deutlich verschieden voneinander sind.”
    ?

    Wären in einem solchen hypothetischen Fall nicht eher Erwartungen falsifiziert, die sich mit dem Standardmodell (der Teilchenphysik) verbinden, und/oder z.B. gewisse Erwartungen bzgl. der Gleichheit der Zerfallsbedingungen in den verschiedenen Versuchen??

    > Nimm als Analogie die Schallgeschwindigkeit: Ich kann zum Bestimmen der Entfernung auch einen Schall-Ping verwenden.

    Das scheint keine sinnvolle Analogie, weil oder sofern man unter “Schall” prinzipiell Signalaustausch in einem Medium mit Brechungsindex (mehr oder weniger) größer als 1 versteht …

    > ich kann den Abstand zwischen den Punkten A und B auch über den Austausch eines Ping-Signals mit Licht messen

    Der Austausch von geladenen Teilchen (wie dem Elektronen-Burst im Beispiel) ist ja auch (ziemlich) zwangsläufig mit einem elektro-magnetischen Signal verbunden.

  345. #347 nihil jie
    7. Januar 2011

    @Frank Wappler

    zitat von TheBug

    @Frank Wappler: Sortier doch bitte mal was Du schreibst, das ist ja unlesbar.

    ja wirklich… und ich dachte immer ich bin hier der wirrste schreibling *gg Du läufst mir gerade meinen rang ab 😉

  346. #348 MartinB
    7. Januar 2011

    @FrankWappler
    Ich verstehe dich immer noch nicht. Was soll das hier heißen:
    “Aus der Anzeige einer Laboruhr bei Erzeugung des Kaons und der Anzeige einer (i.A. anderen) Laboruhr beim Zerfall des Kaons wurde mit Hilfe der SRT (Feststellung von Gleichzeitigkeit bestimmter Anzeigenpaare) die Dauer dieser beiden Laboruhren festgestellt,”
    Warum soll ich zwei verschiedene Uhren nehmen – meine Kaonen sind doch brav auf ner Kreisbahnund kommen immer wieder zur Uhr zurück? Deshalb brauche ich auch keine SRT um irgendeine Gleichzeitigkeit festzustellen.
    Uhr wird gestartet – Kaonen gehen auf Kreisbahn, wenn sie wieder bei der Uhr sind (gleichzeitig am selben Ort), wird geguckt, ob und wieviele noch da sind. Ich kann auch ne Uhr auf die Kreisbahn schicken, die zur zurückbleibenden Uhr identisch ist, wenn die das besser gefällt. Auch da vergleiche ich dann nur Ereignisse am (mit beliebiger genauigkeit) selben Ort zur selben Zeit, für die SRT und Newtonphysik keine unterschiedlichen Aussagen machen.

    Dafür brauche ich die SRT nicht. Du tust irgendwie so, als hätte man vor Einstein keine Uhren ablesen können – das begreife ich nicht.

    Und wenn dir die Elektronen zum Pingen nicht gefallen weil sie geladen sind, dann nehmen wir halt pionen oder auch wieder kaonen oder Neutrinos.

    Tut mir Leid, ich begreife nach wie vor absolut nicht, warum du davon ausgehst, dass man ohne SRT keine Strecken messen und keine Uhren ablesen kann.

  347. #349 Basilius
    7. Januar 2011

    @nihil jie
    Da mach’ Dir mal gar keine Sorgen. Auch wenn die Orthographie manchmal noch etwas Verbesserungsfähig wäre, so kann ich doch das wirklich Allermeiste von Dir ganz gut verstehen. Ich kann hier ja auch den Willen erkennen, verstanden zu werden. Dieser erscheint mir allerdings nicht immer bei allen Kommentatoren so leicht erkennbar.

    @Frank Wappler
    Sagen Sie mal, hat das einen tieferen Grund, warum Sie zunehmend skurilere URL Einträge in Ihren Kommentaren posten? Da ich den Inhalt der Kommentare schon länger nicht mehr verstehe habe ich halt angefangen drum herum zu lesen und da kam diese Frage bei mir auf.

  348. #350 Frank Wappler
    8. Januar 2011

    MartinB schrieb (07.01.11 · 18:03 Uhr):

    > Warum soll ich zwei verschiedene Uhren nehmen – meine Kaonen sind doch brav auf ner Kreisbahnund kommen immer wieder zur Uhr zurück? Deshalb brauche ich auch keine SRT um irgendeine Gleichzeitigkeit festzustellen.

    Wie willst du denn (im Falle dieser Versuchsanordnung) sonst nachweisen, _dass_ die betrachteten Kaonen sich von Treffen zu Treffen “auf ner Kreisbahn” bewegt hätten? (Und noch dazu “brav“, was bei der Berechnung ihrer mittleren Lebensdauer durchaus relevant sein mag.)

    Woher sollte man die erforderlichen Werte von “beta^2” bzw. von “gamma” nehmen, um die (mittlere) Lebensdauer der Kaonen mit der Dauer der Uhr von Treffen zu Treffen in Beziehung zu setzen?
    (Mal vorausgesetzt, dass dabei jede Dauer der Uhr von Treffen zu Treffen gleich war, was unter Einsatz der SRT nachzuweisen wäre.)

    > Ich kann auch ne Uhr auf die Kreisbahn schicken […]

    Eben.
    Das ist sicherlich ein Anlass, (noch) einmal darüber nachzudenken, wie denn nachzuweisen wäre, ob und in wie fern die einzelne Uhr, auf die du dich beschränken willst, während des Experiments nicht ebenfalls “auf die Kreisbahn” geschickt gewesen wäre.

    > […] die zur zurückbleibenden Uhr identisch ist

    Identisch“??
    Bzgl. welcher Messgröße(n) sollen diese beiden unterscheidbaren Uhren gleich gewesen sein,
    bzw. wie sollte nachgewiesen werden, _dass_ diese beiden dermaßen gleich waren und blieben, insbesondere während sie voneinander getrennt waren?

    > Du tust irgendwie so, als hätte man vor Einstein keine Uhren ablesen können

    Wohl kaum.
    Viele tun aber irgendwie so, als könne man entscheiden, ob eine gegebene Uhr gut war, oder nicht, ohne dazu eine bestimmte nachvollziehbare Messmethode festzusetzen;
    als habe das Aussehen von Anzeigen einer gegebenen Uhr (und manche Anzeigen lassen sich ja durchaus als Zahl lesen) von vornherein irgendetwas mit der Reihenfolge dieser Anzeigen zu tun, oder gar mit der Dauer zwischen Anzeigenpaaren.

    > warum du davon ausgehst, dass man ohne SRT keine Strecken messen […] kann

    Ich gehe lediglich davon aus, _dass_ man mit der SRT-Methode (Synge nannte sie die “chrono-geometrische”) Distanzen messen kann; bzw. Distanzverhältnisse als rationale Zahlen auswerten kann;
    und ich kenne (noch) keine andere nachvollziehbare Methode zur Feststellung von geometrischen Beziehungen zwischen zueinander ruhenden Beteiligten.

    Entscheidend ist die Konsequenz für die angebliche Falsifizierbarkeit der SRT:
    Messwerte, die auf Grundlage der SRT-Messoperation(en) gewonnen wurden, eigenen sich keinesfalls für die Schlussfolgerung, dass die SRT (einschl. genau der eingesetzten Messoperationen) falsch sei.
    Und zu Messwerten, die nicht auf Grundlage der SRT-Messoperation(en) gewonnen wurden, macht die SRT keine Aussage, und kann folglich durch diese ebensowenig flasifiziert werden.

    Du hast im letzten Kommentar schon gar keinen weiteren Versuch gemacht, die hypothetische Vorgabe “Die SRT ist falsch, falls …” konkret zu formulieren.
    Vielleicht begreifst du ja mittlerweile, dass eine Theorie eben nicht experimentell zu testen ist; sondern lediglich Modelle (wie das Standardmodell), die auf ihrer Grundlage formuliert werden.

  349. #351 Niels
    8. Januar 2011

    @Frank Wappler
    Nur mal am Rande: Unter Wissenschaftlern ist es üblich, sich möglichst klar und verständlich auszudrücken. Vielleicht könntest du dich darum in Zukunft etwas verstärkt bemühen?

    Zu deinen Problemen, “welcher Messgröße” bei Uhren betrachtet wird, ob jede “Dauer der Uhr” gleich ist , ob ein Zusammenhang zur “Reihenfolge der Anzeigen” und dem “Aussehen von Anzeigen” besteht und in welcher Messgröße identische Uhren wohl übereinstimmen sollten:
    Schau dir mal die Definiton der Einheit “Sekunde” an und lies mal etwas über die Internationale Atomzeit.
    Man könnte dein Eindruck gewinnen, du wolltest absichtlich falsch verstehen.

    Was sind eigentlich SRT-Messoperation?
    In der Definition der Sekunde lese ich nichts von SRT.
    Wenn dich bei der Einheit des Meters die Verwendung der Lichtgeschwindigkeit so stark stört, kannst du auch gerne wieder den Urmeter auspacken. Oder es mit der Definition von 1960, also mit dem vielfachen der Wellenlänge einer bestimmten Strahlung probieren.
    Es ändert sich absolut nichts an den Ergebnissen der Messungen.

    Außerdem könntest du definieren, wie du die Begriffe Modell und Theorie verwendest.
    Wenn nach deinem Verständnis physikalische Theorien prinzipiell nicht falsifizierbar sind, entspricht deine Verwendung eindeutig nicht der Norm.
    Im Übrigen würde ich das Standardmodell als eine physikalische Theorie bezeichnen.

  350. #352 Name auf Verlangen entfernt
    8. Januar 2011

    @ Niels: Sie schreiben: “Schau dir mal die Definiton der Einheit “Sekunde” an … ”

    Die Sekunde ist und wird immer durch den Umlauf der Erde um die Sonne definiert sein, alles andere ist und bleibt logischerweise daran geeicht.

    “Wenn dich bei der Einheit des Meters die Verwendung der Lichtgeschwindigkeit so stark stört, kannst du auch gerne wieder den Urmeter auspacken.”

    Eben nicht. Dadurch, daß die Lichtgeschwindigkeit den Meter bestimmt, wird die Definition der Strecke (Meter) vorausgesetzt, die doch eben hinterher erst durch die Lichtgeschwindigkeit definiert wird. Nicht vergessen, daß durch die Längendilletation nirgendwo ein Ur-Meter zu finden sein dürfte. Von welchem Standpunkt aus?

  351. #353 rolak
    8. Januar 2011

    Zum Thema Einheiten bisher wohl immer von SI weggehört, was Markus? Mußt nur zählen können – allerdings weiter als bis 12.

  352. #354 TheBug
    8. Januar 2011

    Name auf Verlangen entfernt, mit traumwandlerischer Treffsicherheit voll daneben, wie immer…

    https://de.wikipedia.org/wiki/Sekunde
    https://de.wikipedia.org/wiki/Meter

    @Frank Wappler: Meinst Du die Kaonen machen mal einen kleinen Abstecher um Blumen zu pflücken?

  353. #355 Name auf Verlangen entfernt
    9. Januar 2011

    @ TheBug: das ist jetzt echt gut mit der Atomsekunde. Ich warte darauf, bis Dir ein Licht …

  354. #356 JS
    9. Januar 2011

    @Name auf Verlangen entfernt

    sag mal, kommt es dir manchmal in den Sinn – bevor du solchen Unsinn postest – mal irgendwo nachzusehen, ob deine kognitiven Ergüsse in irgend einer Weise mit der Realität korreliert sind?
    Also, nimm dir mal eine Sekunde Zeit … und ab damit aufs Eichamt.

  355. #357 JS
    9. Januar 2011

    @Name auf Verlangen entfernt

    hast du den Artikel auch weiter gelesen als bis zum Wort “Atomsekunde”?

  356. #358 TheBug
    9. Januar 2011

    @JS: Vergebliche Liebesmüh, Markus wird uns jetzt gleich erklären warum die Sekunde eine Naturkonstante ist, die sich aus der extrem präzisen Umlaufzeit der Erde um die Sonne ableitet, oder so was ähnliches.

    Und das Licht was wahrscheinlich nicht nur mir, sondern allen, die sich um tatsächliche Zusammenhänge scheren, aufgehen soll, ist wahrscheinlich, dass das Zeitnormal nicht mehr stimmt, wenn man sich als Beobachter z.B. relativ dazu bewegt, oder einer anderen Gravitation ausgesetzt ist.

    Wie er von da aus dann noch versuchen will zu erklären, dass die relativistischen Korrekturen z.B. beim GPS nur ein Hoax sind, ist mir momentan noch nicht klar, aber das schafft er schon.

    @MartinB: Um mal wieder etwas Sinn in diesen Thread zu bringen: Welches Messprinzip verwendet man eigentlich um die Kaonen im Ringbeschleuniger zu detektieren, ohne sie dabei zu stören? Einfach auf ein Target klatschen lassen wäre da ja kontraproduktiv.

  357. #359 MartinB
    9. Januar 2011

    @FrankWappler
    Habe eben nochmal überlegt, was du uns vielleicht zu sagen versuchst: Dass man eine Theorie streng genommen nie verifizieren kann, weil man immer beliebig viele Hilfsannahmen machen kann, um sie zu retten? Das war ja eine Kritik an popper. Mag in der Theorie richtig sein, in der Praxis wird eine solche Theorie inter dem Gewicht der Zusatzannahmen irgendwann zusammenbrechen.

    “und ich kenne (noch) keine andere nachvollziehbare Methode zur Feststellung von geometrischen Beziehungen zwischen zueinander ruhenden Beteiligten.”
    Also, bei mir im keller liegt so ein Zollstock, und den kann ich so langsam bewegen, wie ich will, da sind relativistische Effekte im Rahmen jeder Messgenauigkeit egal.

    “_dass_ die betrachteten Kaonen sich von Treffen zu Treffen “auf ner Kreisbahn” bewegt hätten? ”
    Indem ich auf der Kreisbahn in hinreichend dichtem Abstand ne Menge Detektoren setze zum Beispiel. Ich injiziiere einen bunch von instabilen teilchen, lasse die umlaufen und sehe hinterher, wie die Zahl der detektierten teilchen an jedem Detektor abnimmt. Damit du keine Krise mit unterschiedlichen Uhren bekommst, werte ich nur eine Uhr an einem Ort aus, die anderen Detektoren nehme ich nur, um sicherzustellen, dass keine nennenswerte Menge an Kaonen entkommt.

    “bzw. wie sollte nachgewiesen werden, _dass_ diese beiden dermaßen gleich waren und blieben, insbesondere während sie voneinander getrennt waren? ”
    Ich baue zwei Uhren so identisch wie möglich. Ich lasse sie laufen und bestimme die zeitliche Abweichung zwischen beiden. Dann mache ich das beschleuinigerexperiment. Um sicherzustellen, dass beim beschleunigen nichts kaputt geht, teste ich, ob sie hinterher immer noch gleich laufen. Das mache ich mit unterschiedlichen Konstruktionstypen (Quarzuhr, Atomuhr, mechanische Uhr) – dann kann ich davon ausgehen, dass ein konsistenter Gangunterschied nicht auf die Konstruktion der Uhr zurückzuführen ist.

    Ich kann die SRT übrigens auch durch ein positives Michelson-Morley-Experiment in die Bredouille bringen – wäre das damals anders verlaufen, dann hätte es die SRT ja niemals gegeben. Das ist überhaupt ein interessantes Gedankenexperiment: nehmen wir mal an, das MM-Experiment hätte einen Laufzeitunterschied für’s Licht gemessen. Und dann kommst du und erfindest die SRT und sagst, “die kann man nicht widerlegen”. Hältst du das für plausibel?

    Oder wir machen folgendes Experiment zur Widerlegung der SRT: ich erzeuge einen 1000stelligen Zahlencode in zwei Kopien (oder nehme ein beliebiges anderes Objekt, das nahezu unmöglich zu kopieren ist – wenn ich der QM vertraue, könnte ich auch eine gigantische Menge verschränkter Teilchenzustände nehmen).
    Ich bringe mit einer Rakete ein Exemplar nach Alpha Centauri, lasse es dort bewachen und fliege wieder zurück zur Erde (o.k., das dauert ein bisschen). Wieder auf der Erde sage ich dir erst jetzt, wohin ich geflogen bin. Wenn du innerhalb von 10 Sekunden das Objekt von Alpha Zentauri geholt hast (und das machen wir ein paarmal und holen James Randi hinzu, damit du keine Tricks anwendest, um an den Code zu kommen), dann betrachte ich die SRT als widerlegt.

    Keine dieser Methoden ist absolut unanfechtbar – vielleicht ist ja das, worauf du hinauswillst; man kann immer passende Szenarien innerhalb der SRT konstruieren (genau im entscheidenden Moment hatte der Computer durch ein Teilchen der kosmischen Strahlugn einen Bitfehler oder was weiß ich). Das mag für Philosphen ein Problem sein, fr mich als Physiker ist es das nicht – ich sehe das pragmatisch: Wenn ich unglaublich viele Zusatzannahmen brauche, um die SRT zu retten, dann betrachte ich sie irgendwann als widerlegt. Man könnte übrigens genau solche Argumente für die Newtonsche Physik anführen – vielleicht hatten ja alle Experimentatoren immer eine Halluzination beim Auswerten der Ergebnisse, die die SRT zu bestätigen scheinen. Das ist nicht unmöglich, aber so unwahrscheinlich, dass man es nicht in Betracht ziehen muss.

    @TheBug
    S.o.: Man würde einen teilchenbunch injizieren und die zerfälle pro Runde messen – es müssen ja eh viele teilchen sein, wenn wir ne Halbwertszeit messen wollen.

  358. #360 Bullet
    9. Januar 2011

    Die Sekunde ist und wird immer durch den Umlauf der Erde um die Sonne definiert sein, alles andere ist und bleibt logischerweise daran geeicht.

    Es wurde zwar bereits erwähnt, ich meld mich aber trotzdem gern dazu zu Wort:
    Hallo Markus, du Schwachsinnsschleuder. Wie gehts denn so? Neues Jahr, neues Bullshitkontingent? Das soeben zitierte Bröckchen Bullshit ist mal wieder ein schönes Beispiel für dein Versagen auf ganzer Linie. Wie rolak schon erkannte: wenns um Zahlen >12 geht, geht bei dir eine Sicherung kaputt, wa?
    Wenn du doch wenigstens die Nachbeben des Haiti-Events auf den MONAT genau vorhergesagt hättest. Du bist ein echtes Glanzlicht deiner Zunft.

  359. #361 Frank Wappler
    9. Januar 2011

    Niels schrieb (08.01.11 · 21:47 Uhr):

    > Was sind eigentlich SRT-Messoperation[en]?

    Besonders bekannt sind natürlich die Methode zur Feststellung von Gleichzeitigkeit von Anzeigen zueinander ruhender Beteiligter (jemand als “Mitte zwischen” den beiden entscheidet, ob die entsprechenden Signale koinzident oder “eines nach dem anderen” wahrgenommen wurden),
    sowie die chrono-geometrische Methode zur Feststellung von Distanzverhältnissen zueinander ruhender Beteiligter (durch Feststellung der Verhältnisse zwischen den entsprechenden Pingdauern; bzw. die formale Setzung von Distanz als “c/2 Pingdauer).

    Darauf aufbauend die Operation zur Messung von Geschwindigkeit (bzw. des Wertes “beta”) dessen, was zwischen zwei zueinander ruhenden Beteiligten ausgetauscht wird; und daraus wiederum herzuleiten die Operation zum Vergleich von Dauern von Beteiligten, die nicht zueinander ruhen (nämlich unter Berücksichtigung des “gamma”-Faktors).

    > Schau dir mal die Definiton der Einheit “Sekunde” an

    Fairer Vorschlag. Den Link zur definitive Referenz, um sich das anzuschauen (nämlich die Webseite des BIPM: https://www.bipm.org/en/si/si_brochure/ ), hatte ich ja bereits oben erwähnt …

    > In der Definition der Sekunde lese ich nichts von SRT.

    Ich finde darin ausdrücklich das Wort “duration” und die Phrase “at rest”;
    und die Messungen, ob und in wie fern ein gegebenes System bestehend aus 55 Protonen, 55 Elektronen und 78 Neutronen einen “ground state of the caesium 133 atom” darstellte, und wie seine Periodendauern ggf. wie gefordert “corrected for the shift due to ambient radiation” werden müssten, sind sicherlich auch relevant.

    > Wenn dich bei der Einheit des Meters die Verwendung der Lichtgeschwindigkeit so stark stört, kannst du auch gerne wieder den Urmeter auspacken.

    Warum sollte die chrono-geometrische Distanzdefinition irgendjemanden stören?
    Wie sonst sollte beurteilt werden, ob z.B. die beiden Enden des Urmeters gleiche Distanz zueinander behielten, falls sie mal nicht ständig zueinander ruhten sondern (einzeln oder gemeinsam) “transportiert” würden?
    Wie sonst sollte man “Ausdehnungskoeffizienten” eines gegebenen Endenpaares (z.B. in Korrelation zu Umgebungs-(Temperatur-)Strahlung oder angelegter (Piezo-)elektrischer Spannung usw. usf.) bewerten (und z.B. feststellen, dass diese Koeffizienten für die beiden Enden des Urmeters bisher recht klein waren)?

    > Außerdem könntest du definieren, wie du die Begriffe Modell und Theorie verwendest.

    Kurz gefasst:
    Eine Theorie ist ein System aus Axiomen und (unter Einsatz der axiomatischen Begriffe formulierten) Definition bzw. Festsetzungen; sowie deren logische Konsequenzen/Theoreme. (Bei der SRT handelt es sich insbesondere um eine Theorie, die sich mit Messoperationen zur Feststellung geometrischer Beziehungen befasst.)
    Ein Modell ist eine Menge von Mess- oder Erwartungswerten bestimmter Messoperationen (z.B. das Standardmodell der Teilchenphysik, u.a. mit bestimmten Mess- und Erwartungswerten für die Verhältnisse der mittleren Lebensdauern bestimmter Teilchenarten).

    > Wenn nach deinem Verständnis physikalische Theorien prinzipiell nicht falsifizierbar sind, entspricht deine Verwendung eindeutig nicht der Norm.

    Welche Bezeichnung für ein System aus Axiomen, Definitionen und (… ähmm …) Theoremen betrachtest du denn stattdessen als normativ?
    Oder folgst du etwa der Norm, Axiomen und Definitionen “experimentell falsifizierbar” zu nennen? …

    > Im Übrigen würde ich das Standardmodell als eine physikalische Theorie bezeichnen.

    Im Übrigen ist “ca. 137.036” ein _Messwert_ (des entsprechenden Kopplungsparameters der elektro-schwachen a.k.a. GWS-Theorie, bezogen auf 0 K).

  360. #362 Name auf Verlangen entfernt
    9. Januar 2011

    @ Frank Wappler: “Warum sollte die chrono-geometrische Distanzdefinition irgendjemanden stören?
    Wie sonst sollte beurteilt werden, ob z.B. die beiden Enden des Urmeters gleiche Distanz zueinander behielten, falls sie mal nicht ständig zueinander ruhten sondern (einzeln oder gemeinsam) “transportiert” würden?
    Wie sonst sollte man “Ausdehnungskoeffizienten” eines gegebenen Endenpaares (z.B. in Korrelation zu Umgebungs-(Temperatur-)Strahlung oder angelegter (Piezo-)elektrischer Spannung usw. usf.) bewerten (und z.B. feststellen, dass diese Koeffizienten für die beiden Enden des Urmeters bisher recht klein waren)?”

    Daher stört es gewaltig:

    Entscheidend ist die Konsequenz für die angebliche Falsifizierbarkeit der SRT:
    Messwerte, die auf Grundlage der SRT-Messoperation(en) gewonnen wurden, eigenen sich keinesfalls für die Schlussfolgerung, dass die SRT (einschl. genau der eingesetzten Messoperationen) falsch sei.
    Und zu Messwerten, die nicht auf Grundlage der SRT-Messoperation(en) gewonnen wurden, macht die SRT keine Aussage, und kann folglich durch diese ebensowenig flasifiziert werden.

    All das wäre weiter kein Problem, wenn wir es mit einer bescheidenen Position zu tun hätten, die nicht ihre eigene Verfasstheit als mathematische Fiktion ins Weltall projeziert, als könnte sie sich zutrauen, in diesem Bereich, sowie subatomar, irgenwelche vernünftigen Aussagen zu machen, außer eben bestimmten Zahlen bestimmte Korrelationen zuzuordnen.

    @ Bullet: Auch heute noch kommt das “Normalzeitsignal” aus der Sternwarte Hamburg-Bergedorf.

    @ Martin B.: “Wenn du innerhalb von 10 Sekunden das Objekt von Alpha Zentauri geholt hast (und das machen wir ein paarmal und holen James Randi hinzu, damit du keine Tricks anwendest, um an den Code zu kommen), dann betrachte ich die SRT als widerlegt.”

    Da sie ausdrücklich “verschränkter Teilchenzustände” zulassen, ist das ja eben laut Zeilingerscher Quantenteleportation nachgewiesen.

  361. #363 MartinB
    9. Januar 2011

    @MT
    Müssen Sie eigentlich ihr vollkommenes Unverständnis der Physik ständig auf meinem Blog kundtun? Ein Objekt, das sich in einem verschränkten Quantenzustand befindet, mit Überlichtgeschwindigkeit zu transportieren ist nicht dasselbe wie die Quantenverschränkung auszunutzen, um den Zustand des zweiten verschränkten Teilchens zu ändern.

    @FrankWappler
    Dein Theoriebegriff entspricht nicht dem der normalen Naturwissenschaft, in dem für eine gültige Theorie auch übereinstimmung mit der Realität gefordert wird. Das scheint mir hier ein Knackpunkt zu sein.

  362. #364 Name auf Verlangen entfernt
    9. Januar 2011

    @MartinB: “… die Quantenverschränkung auszunutzen, um den Zustand des zweiten verschränkten Teilchens zu ändern.”

    Darum geht es eben. Genau die Veränderung kann zwischen hier und Alpha Zentauri ohne zeitliche Verzögerung zuverlässig vorgenommen werden.

    Im übrigen äußere ich mich hier nur dann, wenn Sie sich mal wieder die Freiheit nehmen, zwischen Philosophie und Physik einen willkürlichen Unterschied zu machen.

  363. #365 MartinB
    9. Januar 2011

    @MT
    “Darum geht es eben.”
    Nein, darum geht es eben nicht. Die Verschränkung ist hier nur Mittel zum Zweck, um ein “fälschungssicheres” Objekt zu haben – Quantenverschränkung kann man nicht kopieren. Trotzdem soll in meinem Experiment das zweite teilchen natürlich tatsächlich transportiert werden.
    Hier nochmal gaaaaanz langsam:
    Zwei verschränkte Teilchen A und B, A auf der Erde, B bei Alpha Zentauri.
    Messung bei B machen und dabei den Zustand von A ändern ist *nicht* das was ich meine. Ich meine:
    Teilchen B von Alpha zentauri zur Erde zurückholen und dann hier durch entsprechende Experimente nachweisen, dass es tatsächlich das verschränkte und nach Alpha Zentauri geschickte Teilchen ist. (der Nachweis funktioniert natürlich nur statistisch, also mit vielen Teilchen.)

  364. #366 Name auf Verlangen entfernt
    9. Januar 2011

    @ MartinB: auch noch mal ganz langsam: ” … es tatsächlich das verschränkte und nach Alpha Zentauri geschickte Teilchen ist.” Die nachgewiesene Teleportation bezieht sich nicht auf das Teilchen, sondern auf die Verschränkung. Das reicht aber völlig für Ihren geforderten Gegenbeweis.

  365. #367 MartinB
    9. Januar 2011

    @MT
    Ich geb’s auf – Sie wollen mich nicht verstehen. Frag mich nur, warum Sie dann hier auf meinem Blog sind, wenn Sie mir eh nicht zuhören…

  366. #368 volki
    9. Januar 2011

    @MartinB: Also hier ein bescheidener Versuch zu erklären, was Frank Wappler evt. meinen könnte. In der mathematischen Logik gibt es den Unterschied zwischen Theorie und Modell und ich glaube Frank versucht die mathematischen Begriffe auf die Physik zu übertragen, was dort aber nicht passt. Siehe hier Wikipedia:

    https://de.wikipedia.org/wiki/Axiomensystem (Axiomensystem= Theorie)
    https://de.wikipedia.org/wiki/Modelltheorie

    So gesehen hat Frank Wappler vielleicht sogar recht, dass die SRT nicht falsizierbar ist (mathematisch gesehen als mathematische Theorie) das ist jedoch in der physikalischen Welt volkommener Blödsinn. Eventuell denkt er auch an den Gödelschen Satz der mathematisch gesehen in dieser Situtation zwar auch nicht greift (glaube ich bin mir nicht sicher, da Gödel nur etwas über Verifizierbarkeit sagt) aber physikalisch noch immer Blödsinn ist.

    lg
    Volki

  367. #369 Basilius
    9. Januar 2011

    @Frank Wappler
    Sagen Sie mal, hat das einen tieferen Grund, warum Sie meine Frage bezüglich der zunehmend skurileren URL Einträge in Ihren Kommentaren nicht beantworten? Liegt das vielleicht daran, daß Sie gar nicht mitlesen, sondern nur Ihre Kommentare posten? Falls ja, so finde ich es schade, da mich eine Antwort wirklich interessieren würde.

  368. #370 TheBug
    9. Januar 2011

    @MT: Wieder mal bei dem Versuch Schrödingers Katze zu reiten? Habe ich Dich doch schon mehrfach vor gewarnt, das wird nichts, die mag nicht in die geistige Ödnis von Esoteria.

    Bei der Quantenteleportation wird keine Information übertragen, die muss schon auf dem einsteinschen Weg ans Ziel gelangen.

  369. #371 axel
    9. Januar 2011

    Bin neu hier im blog, habe aber einen prinzipiellen Einwand:

    Es mag ja schön sein, so viele Beispiele zu finden, die das Funktionieren der SRT eindrucksvoll beschreiben, aber für die Fragestellung, ob die SRT richtig oder falsch ist, ist dies eher belanglos.

    Mich hätte interessiert, ob es Stellen gibt, wo es mit der SRT hakt, was aber leider nur gestreift wird.

    Was feststeht ist jedenfalls, dass die SRT nicht vollständig wird, wie im Artikel richtig dargelegt wird, ich füge als Beispiele mal das Unvermögen der SRT an, den Urknall oder Schwarze Löcher zu beschreiben. Wie sieht’s aber mit den “dunklen Fragen” der Astronomie aus? Dunkle Energie und dunkle Materie? Spielt die SRT da eine Rolle und uns eventuell einen bösen Streich?

  370. #372 Name auf Verlangen entfernt
    9. Januar 2011

    @ The Bug: doch: die Verschränkung ist die Information.

  371. #373 TheBug
    9. Januar 2011

    @axel: Schon mal was von der Allgemeinen Relativitätstheorie gehört?

  372. #374 Frank Wappler
    10. Januar 2011

    MartinB schrieb (09.01.11 · 12:00 Uhr):

    > Ich kann die SRT übrigens auch durch ein positives Michelson-Morley-Experiment in die Bredouille bringen

    Nicht die SRT — sondern (bestenfalls) die Erwartung (verbunden mit den Standardmodellen des Sonnensystems und der irdischen Atmosphäre), dass der Brechungsindex in M-Ms Apparaturgehäuse praktisch genau den Wert 1 hatte und behielt;
    und/oder die Erwartung (verbunden u.a. mit dem Standardmodell der Mineralogie), dass die Bohrlöcher in M-Ms Granitplatte praktisch ihre Distanzverhältnisse untereinander während der Versuche beibehielten.

    (Oder hätten M-M etwa ausdrücklich gemessen, dass das während der Versuche der Fall war und blieb?)

    > […] Indem ich auf der Kreisbahn in hinreichend dichtem Abstand ne Menge Detektoren setze zum Beispiel.

    Du pflasterst also die Kaonenbahn mit Detektoren, aber “gönnst mir” nicht die zwei Uhren in meiner oben skizzierten “linearen” Version einer Lebensdauer-Messung? …

    > Ich injiziiere einen bunch von instabilen teilchen, lasse die umlaufen

    Da es ja insbesondere darum geht, in wie fern diese Versuchsanordnung auf der SRT beruht, hier nochmal ausdrücklich die Frage:
    Wie soll unterschieden werden (sofern das Versuch zu Versuch relevant wäre), ob die besagten Teilchen (von einem Detektor zum nächsten) “umliefen“, oder die Detektoren “umliefen” (wobei jeder ab und zu mal Teilchen trafi) ?

    > Ich baue zwei Uhren so identisch wie möglich. Ich lasse sie laufen und bestimme die zeitliche Abweichung zwischen beiden. Dann mache ich das beschleuinigerexperiment.
    Um sicherzustellen, dass beim beschleunigen nichts kaputt geht, teste ich, ob sie hinterher immer noch gleich laufen. Das mache ich mit unterschiedlichen Konstruktionstypen (Quarzuhr, Atomuhr, mechanische Uhr) – dann kann ich davon ausgehen, dass ein konsistenter Gangunterschied nicht auf die Konstruktion der Uhr zurückzuführen ist.

    Ohne eine Festsetzung, wie zu messen wäre, ob denn “das beschleuinigerexperiment” zumindest hinsichtlich seiner Geometrie und im Rahmen der geforderten Genauigkeit Versuch für Versuch nachvollziehbar gleich blieb,
    dürfte nicht viel Konsistenz zu finden sein.

    > Oder wir machen folgendes Experiment zur Widerlegung der SRT: […]
    Ich bringe mit einer Rakete ein Exemplar nach Alpha Centauri, lasse es dort bewachen und fliege wieder zurück zur Erde (o.k., das dauert ein bisschen). Wieder auf der Erde sage ich dir erst jetzt, wohin ich geflogen bin. Wenn du innerhalb von 10 Sekunden das Objekt von Alpha Zentauri geholt hast (und das machen wir ein paarmal und holen James Randi hinzu, damit du keine Tricks anwendest, um an den Code zu kommen), dann betrachte ich die SRT als widerlegt.

    Das könnte doch allenfalls unsere modellhaften Erwartungen in Bezug darauf widerlegen, dass
    – das fragliche Exemplar ggf. samt Bewachung und Beleuchtung dir nicht in geeigneter Weise hinterhergeflogen wäre, und/oder
    – der Brechungsindex in der betrachteten Region nicht sehr drastisch verschieden von 1 wäre, und/oder
    – die Distanz zu Alpha Tsentauri während des Versuches nicht schlicht und einfach geringer war als c/2 10 Sekunden (und wer weiß über welche Blümchenwiesen du geflogen bist, wenn du Geometrie eben partout nicht gemessen haben willst).

    Wie sollte irgendeiner dieser Befunde widerlegen, dass man sich zuerst festlegen musste, wie gemessen wurde?

    Wenn du einen solchen Befund als “Widerlegung der SRT” vorschlägst, also ggf. insbesondere die Methode zurückweist, geometrische Beziehungen aus Pingdauern zu erschließen,
    (und man Methoden, die man ggf. zurückweisen müsste, am besten sowieso von vornherein verwirft) —
    wie kommst du ausgerechnet auf 10 Sekunden, d.h. woher käme speziell deine Erwartung bzgl. irgendeiner bestimmten Untergrenze der entsprechenden Pingdauer??

    > Wenn ich unglaublich viele Zusatzannahmen brauche, um die SRT zu retten, dann betrachte ich sie irgendwann als widerlegt.

    Occam’s razor beschäftigt sich damit, verschiedene vorhandene konkrete Erwartungen zu unterscheiden; unter verschiedenen Modellen auszuwählen, die gegebene Messwerte und verschiedene weitere Erwartungswerte zusammenfassen.

    Hier aber geht es darum, wie Messwerte überhaupt erst zu ermitteln und Erwartungen überhaupt konkret auszudrücken wären.
    Mein mittlerweile recht systematisch demonstriertes “Austricksen” deiner (woher auch immer stammenden) Erwartungen bzw. Vorurteile stellt keine zusätzlichen (und trotzdem mit deinen Erwartungen verträglichen) Annahmen dar;
    sondern soll illustrieren, dass Erwartungswerte auf bestimmten Messdefinitionen begründet sein und bleiben müssen.

    > Man könnte übrigens genau solche Argumente für die Newtonsche Physik anführen

    Selbstverständlich ist auch die Newtonsche Physik an sich nicht experimentell falsifizierbar. Das Aus- bzw. Abwahlkriterium für Theorien ist (vor allem) deren Nachvollziehbarkeit. Was Newton aber mit Worten wie z.B. “geradlinig” und “gleichförmig” gemeint haben könnte (falls irgendetwas Bestimmtes), oder mit Werten der Variablen “r” oder “t”, lässt sich höchstens (und oft nur näherungsweise) durch Bezug auf die SRT nachvollziehen.

  373. #375 TheBug
    10. Januar 2011

    @Frank Wappler: Und nun stell Dir mal vor die ganze Welt besteht aus bunten Blümchen die “LaLa” singen.

    Was Du hier praktizierst ist Beliebigkeit, abseits jeglicher Realität und frei der normalen Bedeutung der verwendeten Worte.

  374. #376 Frank Wappler
    10. Januar 2011

    Basilius schrieb (09.01.11 · 22:48 Uhr):
    > Sagen Sie mal, hat das einen tieferen Grund, warum Sie meine Frage bezüglich der zunehmend skurileren URL Einträge in Ihren Kommentaren nicht beantworten?

    Also … ich hatte eine ganze Weile überlegt, in wie fern die von mir angegebenen URLs (zu Webseiten der PDG, BIPM oder APS) skuril wirken könnten.
    Mittlerweile vermute ich, dass eventuell mein (mehr oder weniger thematisches) Präambel-Geschreibsel gemeint sein könnte, durch das mein Name so hübsch blau geschrieben wird. Das ist bestenfalls als “Mouse-over” gemeint; nicht zum anklicken.

    p.s.
    Seit wann wird denn in Blogs gesietzt??

  375. #377 JS
    10. Januar 2011

    @Name auf Verlangen entfernt

    “doch: die Verschränkung ist die Information.”

    mag sein, dass es eine Information ist, dass Teilchen verschränkt sind, aber das heißt noch lange nicht, dass hier irgendwo Information übertragen wurde. Außerdem bin ich mir gar nicht sicher, was du unter Information verstehst… Der Begriff mag trivial erscheinen, dass er es nicht ist sieht man aber schnell:

    https://de.wikipedia.org/wiki/Information

  376. #378 MartinB
    10. Januar 2011

    @FrankWappler
    “Selbstverständlich ist auch die Newtonsche Physik an sich nicht experimentell falsifizierbar. ”
    Das und die Antworten oben scheint mir deutlich zu machen, dass meine vermutung stimmt: Du sagst, dass man eine Theorie nicht widerlegen kann, weil man immer beliebig viele andere Erklärungen konstruieren kann, warum etwas wie gemessen wurde (“Standardmodell der Mineralogie”??).

    “die Distanz zu Alpha Tsentauri während des Versuches nicht schlicht und einfach geringer war als c/2 10 Sekunden”
    Ja, und vielleicht hat Gott auch das Universum gerade vor einer Sekunde geschaffen mit allen Erinnerungen usw. Oder vielleicht wird die Gravitation doch von unsichtbaren rosa Einhörnern verursacht, die die Planeten schieben. Auch das sind Aussagen, die nach deiner Logik nicht widerlegt werden können. Das mag philosophisch richtig sein, ist aber von jeder realen physikalischen Praxis so weit entfernt, dass es in meinen Augen irrelevant ist.

    Ich sage es hier nochmal ganz deutlich:
    1. Die Physik ist keine axiomatische Wissenschaft.
    2. Die Physik beinhaltet (zumindest in der aktuellen Praxis) immer nicht sauber definierte Konzepte, die nur durch “Hinzeigen” erklärt werden können.
    3. Die Physik beinhaltet immer Schlussfolgerungen, die nicht logisch wasserdicht gemacht werden können. (Nur weil bisher das Gravitationsgesetz galt, könnte ja theoretisch die Schwerkraft in fünf Minuten ausfallen. Diese Annahme ist nicht widerlegbar, aber unphysikalisch.)

    Ich empfehle noch einmal nachdrücklich die Lektüre von Feynmans “Character of Physical Law”, wo der Unterschied zwischen Physik und Mathematik sehr deutlich zur Sprache kommt.

    @axel
    Die SRT stößt an ihre Grenzen da, wo die Allg. RT relevant wird (von der sie ein Spezialfall ist). Sie mag Probleme bei der Planckskala bekommen, eine mögliche Lösung diskutiere ich hier:
    https://www.scienceblogs.de/hier-wohnen-drachen/2010/08/die-doppeltspezielle-relativitatstheorie.php

  377. #379 Name auf Verlangen entfernt
    10. Januar 2011

    @ JS: “aber das heißt noch lange nicht, dass hier irgendwo Information übertragen wurde.”

    Was denn sonst?

  378. #380 Bullet
    10. Januar 2011

    @MT: genau das ist der Punkt. Vielleicht wurde ja überhaupt nichts übertragen. Du erzählst seit vier Wochen, das Quantenteleportationsexperiment Zeilingers würde genau das (also den Informationstransport) belegen. Eine Welle im Ozean sieht auch danach aus, als würde irgendwas eine Strecke zurücklegen. Tut es aber nicht. Das Wasser, das als Tsunami die Strände SO-Asiens verwüstete, war nicht das, das über dem Bebenzentrum aufgewühlt wurde. Und ob ein Quantenzustand (du hast übrigens immer noch nicht zeigen können, daß du weißt, was das ist – und “Ort” ist kein Quantenzustand) “Information” ist, war deine Interpretation.

  379. #381 Name auf Verlangen entfernt
    10. Januar 2011

    @ Bullet: Du glaubst doch nicht, daß ich mich von Dir examinieren lasse?

    “Vielleicht wurde ja überhaupt nichts übertragen.” Na klar, den Tsunami hat´s wohl gar nicht gegeben.

    “Eine Welle im Ozean sieht auch danach aus, als würde irgendwas eine Strecke zurücklegen. Tut es aber nicht”

    Interessanter Gedanke. Nimm Dir die Freiheit, das auf´s Licht zu übertragen. Die Lücke, die sich auftut, ist in etwa der Erkenntnisbereich, den die SRT verschüttet hat.

  380. #382 Bullet
    10. Januar 2011

    @MT:

    “Vielleicht wurde ja überhaupt nichts übertragen.” Na klar, den Tsunami hat´s wohl gar nicht gegeben.

    Kleine Verständnisfrage für dich:
    Bei einem Tsunami, ausgelöst durch ein Erdbeben an der argentinischen Küste, wird auf dem Meer eine Ausbreitungsgeschwindigkeit von 800 km/h erreicht.
    Mit welcher Geschwindigkeit kommen die Wasserwellen am japanischen Strand an? wie wird diese Änderung erreicht?
    Wieviel Wasser von der argentinischen Küste landet in Japan?

    Zusatzfrage: was wird bei der Ausbreitung von Licht transportiert? Welches Medium ist beteiligt?

  381. #383 Frank Wappler
    10. Januar 2011

    MartinB schrieb (10.01.11 · 08:40 Uhr):

    > dass meine vermutung stimmt: Du sagst, dass man eine Theorie nicht widerlegen kann, weil

    Also nochmal kurz und bündig:
    Weil die Festsetzung einer bestimmten Operation, wie Messwerte zu ermitteln sind, nicht dadurch falsch und hinfällig wird, dass man einen bestimmten so erhaltenen Messwert nicht erwartet hätte.
    Ohne an der zugrundeliegenden Messoperation festzuhalten, kann man weder am Erwartungswert noch am Messwert festhalten.

    > Ich sage es hier nochmal ganz deutlich:
    1. Die Physik ist keine axiomatische Wissenschaft.
    2. Die Physik beinhaltet (zumindest in der aktuellen Praxis) immer nicht sauber definierte Konzepte, die nur durch “Hinzeigen” erklärt werden können.
    3. Die Physik beinhaltet immer Schlussfolgerungen, die nicht logisch wasserdicht gemacht werden können.

    Dem möchte ich (nochmals ganz deutlich) widersprechen:

    1. Die Experimentalphysik ist eine axiomatische Wissenschaft.
    (Dafür, was manche Theoretiker den lieben langen Tag treiben, kann ich mich nicht verbürgen.) Insbesondere ist dabei das Konzept der Beobachtbarkeit und Unterscheidbarkeit (“Herzeigbarkeit”) von Beteiligten ein wesentliches Axiom.

    2. Konzepte, die nicht axiomatisch sind (z.B. Messoperationen zur Feststellung der Geometrie von Versuchsanordnungen) sind als weitere Festsetzungen unter Einsatz der Axiome zu formulieren, und diesbezügliche Messwerte sind entsprechend zu ermitteln.

    3. Tatsächliche Messwerte gleichen nicht unbedigt erwarteten Werten (sogar dann, falls die Erwartungswerte gemäß dem Prinzip der stationären Wirkung mathematisch optimal ermittelt würden). Tatsächliche Messwerte und Erwartungswerte sind überhaupt nur insofern kommensurabel, als sie sich auf ein-und-die-selbe Messoperation beziehen.
    (Im Übrigen wird das meiste, was nicht wasserdicht geschlussfolgert wurde, im Reviewprozess als “nicht mal falsch” verworfen.)

    > (“Standardmodell der Mineralogie”??).

    Ich empfehle die Lektüre der Messwertsammlung von Landolt/Börnstein (u. Nachfolgern) zu Erwartungswerten bestimmter physikalischer Größen für Mineralien, wie (verschiedene) Granite.

  382. #384 MartinB
    10. Januar 2011

    @FrankWappler
    “Die Experimentalphysik ist eine axiomatische Wissenschaft.”
    Vielleicht ein Missverständnis: Ich meinte: Ausschließlich axiomatisch, d.h. nich alles in der Physik ist eine Folgerung aus bekannten Axiomen. Das scheinst du ja in Punkt 2 auch so zu sehen. Man kann natürlich Axiome formulierren, die sind aber (bisher) nie vollständig.

    Vielleicht wird es ja klarer, wenn wir es tatsächlich mal an der Newtonschen Physik aufhängen, die ja deiner Ansicht nach auch nicht widerlegbar ist. Also: wie erklärst du das Verhalten von teilchen bei hohen Geschwindigkeiten z.B. in teilchenbeschleunigern im Einklang mit der Newtonschen Physik?

  383. #385 Name auf Verlangen entfernt
    10. Januar 2011

    @ Bullet: typisch, diese Verdrehungen: “was wird bei der Ausbreitung von Licht transportiert? Welches Medium ist beteiligt?

    Da solltest Du doch wissen, daß Licht gemäß eurer Theorie in gar nichts transportieren kann. Vom Licht aus betrachtet, ist die Welt zwei-dimensional: in so eine Schachtel passt kein “Medium” rein.

  384. #386 Bullet
    10. Januar 2011

    Markus … DU wirfst MIR Verdrehungen vor?
    Starkes Stück.
    Lies das.

  385. #387 Aragorn
    10. Januar 2011

    Quantenverschränkung und Informationsübertragung:

    Soweit ich das (vielleicht auch falsch) verstanden habe, kann mit Superpositionszuständen keine Information übertragen werden, weil das Ergebnis der Messung rein zufälliger Natur ist, und vom Sender nicht in eine bestimmte Richtung beinflußt werden kann.

    Der Sender kann das Ergebnis seiner Messung nicht erzwingen (es ist immer fifty / fifty), und dementsprechend keine Information zum Empfänger senden.

    Der Empfänger kann ebensowenig erkennen, ob sein Systemteil noch in einem Überlagerungszustand ist, oder nicht.

    Das was beim Empfänger ankommt ist daher, in jedem Falle, nur statistisches Rauschen. Der Empfänger kann damit zwar Rückschlüsse auf den Zustand des (nun nicht mehr verschränkten) Sendesystems (bsw. deren Polarisationsrichtungen) ziehen.
    Da diese aber Zufallsergebnisse darstellen (Rauschen), haben diese für ihn keinerlei informellen Wert.

    Der Vorteil der Kombination eines Quantenkanals mit einem klassischen Informationskanals, liegt darin, daß so erkannt werden kann, wenn der Verschränkte Kanal abgehört wird. Das Rauschsignal des Quantenkanals ist dabei der Sicherheits-Schlüssel, der zur Auswertung des Informationskanals benötigt wird.

  386. #388 MartinB
    10. Januar 2011

    @Aragorn
    Du hast, soweit ich sehe, in allen Punkten Recht.
    Das wird aber MT nicht davon abhalten, in deine Äußerungen hineinzuinterpretieren, was immer im gerade passt. Ist ja letztlich sein Beruf – in Sternkonstellationen interpretiert er ja auch hinein, was er will.

  387. #389 Name auf Verlangen entfernt
    10. Januar 2011

    @ Aragorn: “weil das Ergebnis der Messung rein zufälliger Natur ist, und vom Sender nicht in eine bestimmte Richtung beinflußt werden kann.”

    Das muss es auch gar nicht. Wichtig ist allein, daß wir wissen, daß am anderen Ende der verschränkten “Teilchen” die “gegenteilige” Informtion unserer eigenen angezeigt werden wird, bzw. angezeigt worden ist. Diese Information ist Information genug.

  388. #390 JS
    10. Januar 2011

    @Name auf Verlangen entfernt

    “typisch, diese Verdrehungen”

    Projektionen sind ganz normal, wenn man argumentativ in die Enge getrieben wird… mach dir nix draus und glaub was du willst… tschüss

    https://de.wikipedia.org/wiki/Projektion_(Psychoanalyse)

  389. #391 JS
    10. Januar 2011

    @MT

    “Diese Information ist Information genug.”

    Nur blöd, dass “diese Information” gar keine ist …

  390. #392 Aragorn
    10. Januar 2011

    @MT
    Dann erkläre mir mal, anhand eines konkreten Beispiels, wie du bsw. die Bitfolge 0110010001110101, per Quantenverschränkung übertragen willst.

  391. #393 JS
    10. Januar 2011

    … zumindest keine Information, die irgendwohin übertragen wurde.

  392. #394 Bullet
    10. Januar 2011

    Mit dieser Information kannst du aber nichts anfangen, weil vom Erzeugungsort der Quanten die Pärchenhälften immer zufällig in die eine oder andere Richtung geschickt werden. Vereinfacht gesagt: du bekommst an der einen Abhöranlage Rauschen, und an der anderen Abhöranlage das mit dem Rauschen *vielleicht* noch verschränkte Invers-Rauschen. du kannst keine Information senden, weil die statistische Hälfte der Information zum falschen Empfänger geht.

  393. #395 Bullet
    10. Januar 2011

    ups. Das sollte an MT, 19:48 anknüpfen. 🙂

  394. #396 Name auf Verlangen entfernt
    10. Januar 2011

    @ JS, Bullet, Aargorn: wir sprechen hier über etwas ganz Einfaches: den sogenannten “nichtlokalen Charakter” der QM – “daß die Messung an einem Photon tatsächlich den Zustand des anderen Photons beeinflusst.” (Quantetheorie, Ingold, S. 93) – und zwar unabhängig von der Entfernung.

    Statt nun mit irgendwelchen Taschenspielertricks diese Tatsache aushöhlen zu wollen, indem man semantische Spielchen betreibt, erklärt ihr doch bitte in Übereinstimmung mit eurer lächerlich gespaltenen Relativitätheorie diese Tatsache.

    Im Klartext: Wie findet die Beeinflussung des “anderen Photons” statt?

    Nennt es Information oder nennt es Pfannenkuchen. Aber erklärt das mal. Und der Einsatz des lustigen Wörtchens “nichtlokal” allein reicht wohl nicht.

  395. #397 Dete
    10. Januar 2011

    @Name auf Verlangen entfernt

    Im Klartext: Wie findet die Beeinflussung des “anderen Photons” statt?

    Nicht durch Information, es gibt nur die 50:50 Chance. Das andere Teilchen ist zwangsläufig so, völlig ohne Informationsübertragung. Ich denke mal der Nachweis das das nicht so wäre, wäre sowie die Falsifikation der QM als auch der RT. Dafür müsste man nämlich Information übertragen.

    Gruß Dete

  396. #398 Aragorn
    10. Januar 2011

    @MT
    a) Enthält statistisches Rauschen Information?
    b) Wäre es möglich einem Alien im Andromedanebel, dein negiertes Rauschsignal instantan zuzusenden?
    c) Wird die Kausalität (bsw. die eindeutige Zuordnung von Ereignisreihenfolgen) durch die Quantenverschränkung verletzt?

    d) Von einer “lächerlich gespaltenen Relativitätstheorie” habe ich noch nie gehört. Kannst du einen Link dazu angeben?

    e) Was ist mit deiner noch ausstehenden Erklärung, wie du Bitfolgen per Quantenverschränkung versenden willst?

    Weil dein Physikverständnis so absolut unterirdisch ist, will ich dir ausnahmsweise beim letzten Punkt nochmals unter die Arme greifen.

    Also:
    Damit das alltagstauglicher wird, übertragen wir das Quanten-Verhalten dazu mal auf makroskopische Boxenpaare.

    Du hast zehn Boxen, von denen jeweils immer zwei, auf geheimnisvolle Weise miteinander gekoppelt sind. Wenn du die eine öffnest, dann ist da entweder ein Apfel oder eine Birne darin. Du hast keinerlei Einfluß darauf, was drin sein wird, wenn du sie öffnest.

    Es besteht aber immer eine Korrelation zwischen den Boxenpaaren. Öffnest du deine und findest einen Apfel, dann liegt in der gekoppelten Box immer eine Birne, und umgekehrt.

    Mit den fünf Boxenpaaren willst du nun die Bitfolge 11010 verschicken, und legst bsw. fest: 1=Apfel und 0=Birne.
    Dein Gefolgsmann schnappt sich nun die fünf Boxen, welche mit deinen 5 gekoppelt sind, und fliegt mit denen nach Athen.

    Und nun erkläre uns mal, wie du ihm diese Bitfolge zukommen lassen willst.

    Auf gehts!

  397. #399 Frank Wappler
    10. Januar 2011

    MartinB schrieb (10.01.11 · 14:06 Uhr):

    > Ich meinte: Ausschließlich axiomatisch, d.h. nich alles in der Physik ist eine Folgerung aus bekannten Axiomen.

    Gut, dem kann ich zustimmen;
    und das sollte in meiner Antwortliste (1. – 3.) auch deutlich sein.
    Welcher Zusammenhang besteht nun zu experimenteller Falsifizierbarkeit? …

    > Vielleicht wird es ja klarer, wenn wir es tatsächlich mal an der Newtonschen Physik aufhängen,

    Es hätte wohl keiner damit gerechnet, dass die Diskussion ausgerechnet diesen Umweg nimmt — aber schaun ‘mer mal:

    > die ja deiner Ansicht nach auch nicht widerlegbar ist.

    Ich hatte oben — einigermaßen sorgfältig — geschrieben:
    >> Selbstverständlich ist auch die Newtonsche Physik an sich nicht experimentell falsifizierbar.

    Soll heißen, dass “Newtonsche Physik an sich” als Theorie sorgfältig z.B. von “SRT, eingeschränkt auf Versuche mit Messwerten beta viel kleiner als 1” unterschieden werden sollte. (Zumindest begrifflich, im Ansatz. Auf die Frage, in wie fern gewisse Gleichheiten bestünden, kommen wir vielleicht zurück.)
    Und natürlich: dass wir experimentelle Falsifizierung diskutieren …

    > Also: wie erklärst du das Verhalten von teilchen bei hohen Geschwindigkeiten z.B. in teilchenbeschleunigern im Einklang mit der Newtonschen Physik?

    (Diese Frage vermittelt wirklich den Eindruck, dass meine Argumentation bisher weitgehend an dir vorbeigerauscht ist. Also: …)

    Was soll denn “Geschwindigkeit” im Rahmen der Newtonschen Physik an sich bedeuten?
    Wie sollen Werte dieser Größe der Newtonschen Physik zumindest im Prinzip ermittelt und miteiander verglichen werden, um zumindest z.B. “hohe” von “nicht so hohen” Werte zu unterscheiden?
    Und zwar, sofern es sich eben nicht um eine Version der SRT handeln soll:
    ohne die chrono-geometrische Definition von Distanzverhältnissen durch Verhältnisse von Pingdauern, und
    ohne die SRT-Gleichzeitigkeitsdefinition (zur Entscheidung “wann” die Zieluhr gestartet worden sein sollte).

    Sicher, “Limit{ t –> t0 }[ (x( t ) – x( t0 )) / (t – t0) ]”
    oder auch nur “(x( tZiel ) – x( tStart )) / (tZiel – tStart)”
    lässt sich leicht hinschreiben —
    aber wie sollen konkrete, kommensurable Werte aus experimentellen Beobachtungen durch Newtonsche Physik gewonnen werden??
    Wie soll “das Verhalten” Versuch zu Versuch durch Newtonsche Physik an sich quantifiziert werden?

    Ich meine, dass Newtonschen Physik an sich schon an dieser Klippe scheitert.
    Die Frage nach “Erklärung” eventueller Korrelationen stellt sich in diesem Falle gar nicht erst, und von experimenteller Falsifizierung kann demnach keine Rede sein.

    Von der SRT wird die besagte Klippe dagegen genommen — das, was du oben (grob gesagt) “das Verhalten bei hohen Geschwindigkeiten” nennst, nenne ich (grob gesagt) “die SRT-Messmethode zum Vergleich von Lebensdauern, zwischen Beteiligten, die nicht zueinander ruhten”.

    Die daran anschließende Frage nach experimenteller Falsifizierung haben wir oben schon recht ausführlich diskutiert …
    Zum Perspektivwechsel schlage ich deshalb folgende Analogie vor:

    Nehmen wir die “Fußball-Theorie”, bestehend insbesondere aus dem Fußball-Regelwerk und die dadurch definierte Methode zur Unterscheidung von “regulären Spielzügen” (mit denen Tore geschossen werden könnten, die man ggf. zum Spielresultat zusammenzählt) und “irregulären Spielzügen” (die von den Schiedsrichtern verworfen werden sollen).

    Und jetzt betrachten wir die Aussage:
    “Fußball-Theorie ist falsch (experimentell falsifiziert), falls:
    mein Lieblingsclub sein nächstes Fußball-Spiel/Experiment nicht gewinnt.”

    So eine Aussage ist nach meiner Auffassung absurd;
    und ebenso absurd wäre jede Aussage der Form
    “SRT ist falsch (experimentell falsifiziert), falls: …”.

    Falsch (experimentell falsifiziert) wäre stattdessen meine Erwartung, dass mein Lieblingsclub sein nächstes Fußball-Spiel gewinnt; und damit u.a. das Modell, dass mein Lieblingsclub jedes seiner Spiele gewinnt.

    Und diese Unterscheidung passt auf die o.g. Unterscheidung von axiomatischen und nicht-axiomatischen Aspekten der (Experimental-)Physik.

  398. #400 TheBug
    10. Januar 2011

    @Aragrn: MT wird sich wohl mit dem “gespalten” auf SRT/ART beziehen.

    Machen wir es doch mal einfach.

    @MT: SRT ist die einfache Variante der Relativitätstheorie, die ähnlich wie die Newtonschen Gesetze in vielen Situationen ausreichend ist. Die ist etwa so kompliziert, dass Du nach Anschauung Deiner hier demonstrierten Leistungen frühestens in ca. 100 Jahren in der Lage sein könntest sie zu begreifen.

    Bei der ART steht das “Allgemeine” dafür, dass sie nicht nur wie die SRT in Grenzfällen anwendbar ist, sondern, dass sie auf eigentlich alle oberhalb der Quantenebene stattfindenden Effekte anwendbar ist. Da wird dann leider die Lebensdauer der Sonne nicht mehr ausreichen, bevor Du in die Nähe des Verständnisses kommen könntest.

    Für alle Leute die nicht MT sind ist zu sagen, dass man sich die SRT und ART durchaus mit etwas Arbeit und soliden Grundkenntnissen in Mathematik und Physik erschließen kann und damit dann ein Werkzeug hat, mit dem man sehr viel mehr Dinge auf dieser Welt verstehen kann. Der erste Schritt da hin ist es sich klar zu machen, dass subjektive Wahrnehmung bei Dingen die sich wie die Lichtgeschwindigkeit um Größenordnungen unseren Sinnesorganen entziehen, nichts wert sind und man objektive Messwerte benötigt.

    Dann kommt der Punkt, dass objektive Messwerte sich halt in einigen Fällen reproduzierbar von dem unterscheiden was der “gesunde Menschenverstand”, oder in diesem Falle eher “das Bauchgefühl des Laien” sagt. Spätestens an der Stelle scheiden sich dann die Leute, die die Realität akzeptieren von den MTs dieser Welt. Markus hat halt die blaue Pille geschluckt…

  399. #401 Basilius
    11. Januar 2011

    @Aragorn
    Verschlossene Schachteln mit verwschränktem Obst drin. Sehr anschauliches Beispiel. Und sehr treffend, soweit ich im Thema noch drin bin.
    ^_^

  400. #402 H.M.Voynich
    11. Januar 2011

    @Name auf Verlangen entfernt:
    “Wichtig ist allein, daß wir wissen, daß am anderen Ende der verschränkten “Teilchen” die “gegenteilige” Informtion unserer eigenen angezeigt werden wird, bzw. angezeigt worden ist. Diese Information ist Information genug.”

    Ich sehe auf der Erde, wie ein 10 Lichtjahre entfernter Stern explodiert. Gleichzeitig weiß ich, daß diese Information auf der anderen Seite der Supernova, 20 Lichtjahre von hier entfernt, ebenfalls genau jetzt wahrgenommen wird.
    Ich “weiß” also, was 20 Lichtjahre von hier genau jetzt passiert. Dafür brauche ich kein Quantensystem. Ist diese Information wirklich Information genug?

  401. #403 H.M.Voynich
    11. Januar 2011

    @Aragorn:
    Ich verabrede vorher mit dem Athener, wenn er in der ersten Kiste einen Apfel sieht, soll er die zweite öffnen und das darin enthaltene Obst essen.
    Die zweite Kiste habe ich aber vorher vergiftet.
    Wenn ich nun in der ersten Kiste eine Birne sehe, weiß ich instantan, daß der Athener tot ist – noch vor seinen unmittelbaren Nachbarn.

  402. #404 H.M.Voynich
    11. Januar 2011

    “Im Klartext: Wie findet die Beeinflussung des “anderen Photons” statt?”

    Im Klartext: die Beeinflussung fand statt, als die beiden miteinander verschränkt wurden.
    Schwer vorstellbar wird das ja erst dadurch, daß der Zustand eines einzelnen Photons vor der Messung nicht festgelegt ist (wohl aber der Zustand beider zusammen).
    Nimm doch einfach mal das Everettsche Viele-Welten-Modell als Interpretation der QM, dann wird gleich deutlich, daß keine neue Information entsteht: im einen Zweig sieht A die Birne und B den Apfel, im anderen umgekehrt. Damit weiß A1 nur, daß er sich im selben Zweig wie B1 befindet, und A2 “erfährt”, im selben Zweig wie B2 zu sein; doch das war von vornherein klar.

  403. #405 Bullet
    11. Januar 2011

    Leute … ich will euch ja nicht desillusionieren, aber MT hat bereits gezeigt, daß er den Unterschied zwischen “Ortsveränderung” und “Bewegung” nicht versteht. Und ihr kommt hier mit Beispielen, bei denen man denken muß?

    Das heißt übrigens nicht, daß ich das Obstkistenbeispiel schlecht finde. Aber MT denkt, wenn er “Photonen” sagt, an kleine Billardkugeln, die durchs Weltall flitzen. Und er denkt, die hätten eine Art Funkgerät dabei, mittels dessen sie sich absprechen.
    Voynich: dein Viele-Welten-Ansatz in allen Ehren – aber er wird es nicht begreifen. Aber natürlich wird er weiterhin versuchen, hier mit gezieltem Name- oder Factdropping den Anschein zu erwecken, er sei der Checker. Obs klappt … naja.

  404. #406 MartinB
    11. Januar 2011

    @FrankWappler
    “Was soll denn “Geschwindigkeit” im Rahmen der Newtonschen Physik an sich bedeuten?”
    Weg pro Zeit.
    Weg gemessen mit einem Metermaß (ich nehme das Urmeter, dann bin ich in der klassischen periode), Zeit gemessen mit einer hinreichend ganggenauen Uhr, so wie man sie im 19. Jahrhundert zur Verfügung hatte.
    Messe mit dem Urmeter eine hinreichend lange Strecke (Vorschlag: Ein paar Millionen Kilometer – ja, das wird ein bisschen dauern, aber das ist hier ja ein Gedankenexperiment) A-B ab. Synchronisiere zwei Uhren mit den Mitteln des 19. Jahrhunderts. Positioniere eine bei A, eine bei B.

    Nimm ein negativ geladenes Objekt, das vor A mit einem Plattenkondensator beschleunigt wird. Berechne die notwendige Spannung im Plattenkondensator, um das Teilchen mit der Gleichung F=m a auf eine Geschwindigkeit von 1000000m/s zu bringen.
    Detektiere das Teilchen bei A und schreibe die Zeit auf. Detektiere das teilchen bei B und schreibe die zeit auf. Sende die zeitinformation von B nach A und berechne daraus die Geschwindigkeit.
    Du wirst experimentell feststellen, dass du bei beliebiger Wahl der Uhrkonstruktion, der Teilchensorte etc. niemals eine Geschwindigkeit größer als (im Rahmen der Messgenauigkeit) 300000km/s aus den Messwerten berechnest.

    Erkläre das im Rahmen der Axiome der Newtonschen Physik.

    PS: Die Fußballanalogie habe ich gar nicht vestanden, weil der Sieg der mannschaft nicht aus den Fußballregeln folgt.

  405. #407 Aragorn
    11. Januar 2011

    @H.M.Voynich
    Mit Zusatzbedingungen, die in der Quantenwelt nicht vorhanden sind, kann man das Apfel-Birne-Modell natürlich kippen.

    In der Quantenwelt ist es halt nicht so, daß man selbst festlegen kann, was in der Kiste liegt. Es ergibt sich, bei vielen Messungen, immer ein Verhältnis der zwei möglichen Messungergebnisse von inetwa 50:50.

  406. #408 Aragorn
    11. Januar 2011

    @H.M.Voynich
    Ich habe nochmal nachgedacht. Das mit dem vergifteten Obst bezog sich auf “Schrödingers Katze”?
    Ok, dann stimmt was du geschrieben hast, und der böse Experimentator weiß nach der Messung ob sein Versuchsobjekt lebt oder tot ist.

  407. #409 Frank Wappler
    11. Januar 2011

    MartinB schrieb 11.01.11 · 08:34 Uhr:
    > PS: Die Fußballanalogie habe ich gar nicht vestanden, weil der Sieg der mannschaft nicht aus den Fußballregeln folgt.

    Aber du verstehst und betrachtest offenbar (oh Wunder! 😉 endlich mal in einem konkreten Fall,
    _dass_ ein bestimmtes Messergebnis aus einem bestimmten Versuch (hier: das Resultat des nächsten Spieles meines Lieblings-Fußballclubs) nicht zwangsläufig aus den von vornherein festgesetzten und fortbestehenden Regeln folgt, nach denen Messergebnisse Versuch für Versuch aus Beobachtungsdaten zu ermitteln sind.
    (D.h.: Messoperatoren haben i.A. einen Wertebereich mit mehr als einem Element.)

    Und du verstehst wohl auch (soviel von Fußball), _dass_ ohne dessen bestimmte, festgesetzte Regeln Spielergebnisse entweder gar nicht ermittelt werden könnten, oder zumindest nicht Spiel zu Spiel kommensurat genug wären, um daran Erwartungen über das Resultat des nächsten Spieles zu knüpfen.
    (D.h.: Sonst wären Modelle wie “mein Club gewinnt immer” ja noch nicht mal “falsch” zu nennen.
    In dieser Hinsicht hatten Pauli bzw. Popper gewiss recht.)

    Stimmst du also zu, dass es absurd wäre, die Fußball-Theorie (google ruhig mal!) falsch (experimentell falsifiziert) zu nennen, nur weil ein regelgerecht ermitteltes Spielergebnis nicht irgendjemandes modellhaften Erwartungen entspräche?
    Oder nicht?
    Oder kannst du begründen, in fern diese Frage(n) nicht nachvollziehbar wäre(n)?

    Die konkreten Werte der (mittleren) Lebendauern verschiedener Teilchensorten folgen doch auch nicht aus der SRT,
    sondern die SRT legt lediglich nachvollziehbare Regeln fest, wie man ggf. diese Werte Versuch für Versuch kommensurat ermitteln kann.

    Aus der SRT folgt doch nicht mal, dass überhaupt bestimmte (Teilchen-)Sorten auffindbar wären, für die sich ein mehr oder weniger konsistenter Wert der (mittleren) Lebendauer ermitteln ließe, sondern die SRT legt lediglich nachvollziehbare Regeln fest, wie man ggf. Werte der Lebensdauer einzelner beteiligter Versuch für Versuch kommensurat ermitteln kann.

    p.s.
    > Weg gemessen mit einem Metermaß (ich nehme das Urmeter, dann bin ich in der klassischen periode),

    In welchem Maße sollte irgend ein bestimmtes Stück Blech, dessen Enden- oder Kerbenpaar du “Urmeter” nennst, Versuch für Versuch nachvollziehbar bleiben?

    > Zeit gemessen mit einer hinreichend ganggenauen Uhr

    Wie stellst du dir vor, ausgerechnet eine “hinreichend ganggenauen Uhr” (von sicherlich vielen gleichwertigen Uhren) aus der denkbar ungeheuer großen Anzahl von Uhren herauszufischen, die _nicht_ “hinreichend ganggenau” wären bzw. (wichtiger) im betreffenden Versuch blieben??

    > Synchronisiere zwei Uhren mit den Mitteln des 19. Jahrhunderts.

    Was bedeutet das; und erfordert das (schon) den Begriff “Geschwindigkeit” als Messgröße, mit dessen Definition im Rahmen der “Newtonschen Theorie an sich” wir uns gerade beschäftigen?

    p.p.s.
    Granit ist (natürlich! &) nicht im strikten Sinne ein Mineral, sondern ein i.A. aus verschiedenen Mineralen bestehendes Gestein; insofern ging es oben eher ums “Standardmodell der Petrologie”.

    p.p.p.s.
    Die (Ungefähr-)Zahl “137.036” ist natürlich der Kehrwert der (häufig ausgewerteten) Kopplungskonstante der GWS-Theorie, bezogen auf 0 K.

  408. #410 MartinB
    11. Januar 2011

    @FrankWappler
    “In welchem Maße sollte irgend ein bestimmtes Stück Blech, dessen Enden- oder Kerbenpaar du “Urmeter” nennst, Versuch für Versuch nachvollziehbar bleiben?”
    In hinreichendem maße – d.h. ich gehe davon aus, dass das urmeter seine Länge nicht um mehr als ein paar prozent ändert, weil das der Erfahrung entspricht. Dasselbe gilt für die Uhren. Man konnte im 19.Jahrhundert Strecken und Zeiten tatsächlich ohne Rückgriff auf die SRT messen, oder musste man mit Entdeckung der SRT alle Weltkarten neu vermessen und z.B. alle gemessenen Planetenumlaufzeiten?

    Entsprechend mache ich das mit den uhren: ich baue meinetwegen 1000 Uhren mit unterschiedlichen pendel- und Federantrieben und unterschiedlichen konstruktionen und vergleiche diese miteinander: Genau so, wie man im 17.-19.Jahrhundert Uhren tatsächlich immer besser geeicht hat, was übrigens *eine Voraussetzung* für die Entdeckung der SRT war.

    Dass man das letztlich nie ohne Rückgriff auf nicht-axiomatische Erkenntnisse (z.B.: Metermaßstäbe ändern ihre Länge unter Alltagsbedingungen nicht signifikant, was sich durch einen Vergleich sehr vieler Maßstäbe mit unterschiedlichen Eigenschaften und anderer Längenmessinstrumente ergibt – die Ergebnisse sind jeweils konsistent) kann, ist klar, aber für die Physik irrelevant.

    Ich versuche es nochmal anders: Ich nehme die SRT als axiomatisches System mit einem kleinen Unterschied: ich ändere den Wert von c in meiner Variante der SRT auf 0.00000001mm/s ab. Ist diese Theorie dann deiner Ansicht nach widerlegbar?

    “Die konkreten Werte der (mittleren) Lebendauern verschiedener Teilchensorten folgen doch auch nicht aus der SRT,”
    Natürlich nicht. Hat auch keiner behauptet – Teilchen sind nur wegen ihrer absoluten Identität ein praktisches beispiel.

    PS: Nein, die Fußballanalogie erschließt sich mir nicht – willkürlich zu verlangen, dass aus bestimmten Axiomen ein Ergebnis folgen soll, hat mit dem, was wir hier diskutieren, nichts zu tun. Übrigens kann man die Verletzung der Fussballregeln in der Praxis durchaus feststellen, soweit ich weiß: Dafür gibt es Schiedsrichter.

  409. #411 perk
    11. Januar 2011

    hm vllt kann ich auch noch was zur verwirrung beitragen:

    zuerst was das fußballspiel angeht, wir haben den regelsatz also unsere theorie, zusätzlich haben wir spieler (mit masse und kopplungskonstanten (zum spielfeld, den anderen spielern und dem ball, so im groben)), und die sind die werte des modells in diesem szenario

    die regeln sagen nicht vorraus, dass spieler masse und kopplungskonstanten haben, aber erst die konkreten werte dieser größen bestimmen das spielergebnis, deswegen können wir aus dem spielergebnis nur auf die modellgrößen schließen, wenn wir die regeln kennen

    interessant an dieser analogie ist: wenn man als fußballunkundiger am spielfeldrand sitzt (also nicht nur das ergebnis kennt, sondern alle interaktionen der modellparameter, dabei kennt er aber deren werte nicht, da er die massen und kopplungskonstanten nicht misst)) kann man nach endlicher zeit mit großer sicherheit die korrekten regeln ableiten

    diese ableitungsfähigkeit ist nicht 100% sicher aber sie ist vorhanden, und genau an der stelle vermute ich den grund für den widerspruch zwischen martin und frank

    frank weiß dass man vom zuschauen nie die regeln wird messen können, da all die spieleraktionen auch mit vollkommen anderen spielregeln genauso funktionieren könnten, martin hingegen hat 1000te spiele gesehen und hat sich ein paar fußballregeln vorgestellt die zu allen spielen passen die er beobachtet hat und hält sie für gut, da ihm egal ist, dass regeln nicht richtig oder falsch sein können, sondern nur beliebig gesetzt werden

    wenn ein foul nicht geahndet wird hat martin ein problem, er glaubt das die regeln die er sich zusammengereimt hat widerlegt sind und sucht einen spezialfall von regeln der die nichtahndung dieses fouls mit einschließt

    für frank wäre die nichtahndung des fouls egal, da die regeln nur sagen was ein faul ist und was eine schiedsrichterentscheidung ist, nicht aber welches verhalten von fouls (wann sie auftreten) und schiedrichterentscheidungen zu erwarten ist..

    und an der stelle versuch ich den schwenk zur srt, die physikalische theorie sagt was uns was ne strecke ist und was zeiten sind, aber sie sagt uns nicht ob das kaon mit 0,95c 5 meter oder 50 meter weit kommt, so wie ein kaonischer schiedsrichter vllt blind ist und deswegen nie schiedsrichterentscheidungen fällt

    anhand der srt können wir lebensdauer und flugstrecken von verschiedenen partikeln untersuchen und über statistik schlussfolgern, dass sie sich unterscheiden

    so wie wir anhand der fußballregeln einen “blinden” von einem “sehenden” schiedsrichter unterscheiden können (kopplungskonstante 0 bis 1 zu den spielern)

    soweit zu meiner beschreibung des bisher bereits gesagten..

    was nun die frage der falsifizierbarkeit der srt angeht bin ich trotz dieser darstellung merkwürdiger weise nicht franks meinung sondern martins.. ganz einfach weil die srt keine perfekte gesetzte theorie ist (wie die fußballregeln) sondern ein durch beobachtungen des spiels zusammengereimtes halbgares empirisches konzept und in diesem sinne nicht wie eine theorie (franks verständnis) sondern wie eine theorie (martins verständnis) gehandhabt wird/werden muss

    ein nicht geahndetes foul würde die von uns angenommenen regeln falsifizieren und wir würden unseren abgeleiteten regelsatz um ausnahmen erweitern, aber wir würden nie beim exakten wortlaut des fifaregelwerkes ankommen

  410. #412 Name auf Verlangen entfernt
    11. Januar 2011

    @ Bullet, H.M. Voynich, Aragorn:

    “Ich “weiß” also, was 20 Lichtjahre von hier genau jetzt passiert”: da siehst Du mal, wie verwirrt Du bist: genau jetzt, oder 20 Lichtjahre von hier: entscheide dich mal.

    “Im Klartext: die Beeinflussung fand statt, als die beiden miteinander verschränkt wurden.”

    Wie kann man nur so frech große Töne spucken, und sich dermaßen konsequent um die entscheidenden Beobachtungen bislang unwiderlegter Experimente drücken?

    Im Klartext: wir können beweisen, “daß die Eigenschaften von Dingen nicht da sind, bevor wir sie beobachten.” (Zeilinger, Einsteins Spuk S. 238) – d.h., die Beeinflussung findet erst in dem Augenblick statt, indem wir die Dinge im weitesten Sinn betrachten: das ist die erste Möglichkeit, und die zweite, wohl zutreffende, ist unsere offensichtlich unzureichende Vorstellung von Zeit und Raum bisher und der Grund dafür ist die ART.

    Die Begriffs- und Sinnverwirrung durch eure Relativitätstheorien fängt tatsächlich bei der manirierten Unterscheidung von SRT und ART an, und es ist nicht besonders schwierig – Mathematik braucht es dazu nicht – sie ist sogar (siehe euer Beispiel) – gehirnwäschemäßig hinderlich; die SRT als Knoten und die ART als Doppelknoten aufzufassen: eine Theorie der Gravitation, die ihren Sinn aus der Schöpfung einer zusätzlichen Pseudodimension gewinnt, wie das Kaninchen aus dem Hut. Wie verwirrend das ist, zeigen ja selbst befissene “Wisser” mit den “genau jetzt” stattfindenden 20 Lichtjahren Entfernung. Kann man eigentlich nicht verdenken.

  411. #413 Kuchlbacher Rudolf
    11. Januar 2011

    Juhuu – da is er ja wieder….

    genau jetzt, oder 20 Lichtjahre von hier: entscheide dich mal.

    Jetzt mal ehrlich – zu solchen Leistungen ist nur einer fähig! The one and only……

    Gaaanz langsam zum mitdenken: Mit Lichtjahren misst man Entfernungen… “jetzt” bezieht sich auf Zeit!

  412. #414 Aragorn
    11. Januar 2011

    @MT
    Dir etwas zu erklären ist wirklich völlig sinnlos. Du scheiterst schon an den wirklich einfachen Grundbegriffen, die selbst bei jedem Hauptschüler besser sitzen.

    Du bist noch nicht einmal in der Lage, eine konkretes Beispiel um den winzigen Rest zu ergänzen, wenn deine Lehrer dir bereits 99% vorgegeben haben.

    Was für ein armseliges, aber große Töne spuckendes Astrologen-Würstchen du doch bist. Schäm dich!

    Setzen 6!

  413. #415 perk
    11. Januar 2011

    witzig wie es immer wieder passiert euch von einem troll so weit aufregen zu lassen, dass ihr persönlich werdet..

    ich dachte das internet und wie foren/newsgroups/kommentarstränge funktionieren ist schon ne weile bekannt..

  414. #416 Basilius
    11. Januar 2011

    @perk
    Danke für den vorigen Beitrag. Ich glaube, daß Du mich damit wieder ins Boot geholt hast. Ich bin aber natürlich gespannt, wie es weitergeht…

  415. #417 Basilius
    11. Januar 2011

    Uargs!
    Synchronisationsproblem. Perk, ich meinte natürlich Deinen Kommentar von 11.01.11 · 13:10 Uhr

  416. #418 Aragorn
    11. Januar 2011

    @perk
    Das ist hier der Drachen-Blog. Hier darf Feuer gespuckt werden!

  417. #419 H.M.Voynich
    11. Januar 2011

    Ich frag jetzt lieber nicht, was jemand unter “Quantenteleportation” eigentlich versteht, wenn für ihn schon in einem klassischen Gedankenexperiment Gleichzeitigkeit und große Entfernungen sich gegenseitig ausschließen.
    Nur fürs Protokoll: die ART enthält keine “zusätzliche Pseudodimension”.

  418. #420 Bullet
    11. Januar 2011

    @Voynich:

    wenn für ihn schon in einem klassischen Gedankenexperiment Gleichzeitigkeit und große Entfernungen sich gegenseitig ausschließen.

    Das gefettete muß aber wirklich betont werden. Vielleicht sehen es dann auch alle Beteiligten. *hint, hint*

  419. #421 Andreas P.
    11. Januar 2011

    knapp am topic vorbai, aber das resume passt IMHO: https://xkcd.com/836/

  420. #422 H.M.Voynich
    11. Januar 2011

    @Andreas P.:
    At least, with p<0.05 confidence.

  421. #423 H.M.Voynich
    11. Januar 2011

    arghs.
    “At least, with p<0.05 confidence.”

  422. #424 MartinB
    11. Januar 2011

    @perk
    Guter Versuch, finde ich, macht mitr die Analogie etwas deutlicher als das, was Frank schreibt..

    Das hier finde ich aber nicht ganz zutreffend:

    frank weiß dass man vom zuschauen nie die regeln wird messen können, da all die spieleraktionen auch mit vollkommen anderen spielregeln genauso funktionieren könnten, martin hingegen hat 1000te spiele gesehen und hat sich ein paar fußballregeln vorgestellt die zu allen spielen passen die er beobachtet hat und hält sie für gut, da ihm egal ist, dass regeln nicht richtig oder falsch sein können, sondern nur beliebig gesetzt werden

    Da Frank gesagt hat, dass er schon die Newtonsche Physik für nicht widerlegbar hält, hättest du meiner Meinung schreiben müssen

    Frank weiß, dass man aus der Beobachtung endlich vieler Spielzüge nie die Regeln ableiten kann. Er hält es deshalb für möglich, dass das Spiel dort auf dem rasen nach den regeln des Basketballs gespielt wird. Martin gibt zu, dass das theoretisch denkbar ist, wenn man hinreichend absurde Zusatzannahmen macht (die Schiedsrichter haben ständig Halluzinationen, der Basketball sieht nur aus wie ein Fussball, weil das Flutlicht ihn seltsam anstrahlt, die Spieler sind alle Mutanten, deren Füße in Wahrheit Hände sind usw), hält diese Zusatzannahmen aber für hinreichend absurd, um sich darüber keine Gedanken zu machen. Für alle praktischen zwecke kanne rgewisse Regeln widerlegen. Er hat einen Regelsatz gefunden, der zu seinen Beobachtungen passt und ist damit bis auf weiteres zufrieden.

    Das mit dem Ableiten der Spielregeln (allerdings beim Schach) aus dem beobachten von Spielen ist übrigens eine Analogie im viel zitierten Feynman “Character of Physical Law”.

  423. #425 TheBug
    11. Januar 2011

    Verdammt, wo ist Michael Buffer wenn man ihn braucht?

    “Ladies and Gentleman, the world champion in bullshit flinging, king of the inept remark, master of ignorance Maaaaaaaarkus Teeeeeeeeeeeeeeeeeeermin!!!”

    Noch mal für die ganz Langsamen:
    Newton – Für relativ kleine Massen, langsame Bewegungen, bzw. geringe Genauigkeit
    SRT – Gleichförmige, nicht beschleunigte Bewegung, also auch keine, bzw. vernachlässigbare Gravitation
    ART – The full monty

    Aber wie schon bemerkt, eher wird die Sonne verlöschen, als das Markus das versteht.

  424. #426 MartinB
    11. Januar 2011

    @MichaelS
    Tut mir Leid, dein Kommentar oben ist im Spamfilter hängengeblieben. Soweit ich auf die Schnelle sehen kann, ist das eine andere Variante des “Beweises”, den wir anderswo schon diskutiert haben:
    https://www.scienceblogs.de/hier-wohnen-drachen/2010/11/ein-kleines-perpetuum-mobile.php
    Mein Kommentar vom 8.12., 20:51 hat einige Referenzen zu ähnlichen “Rechnungen”.

    Generell ist es keine gute Idee, aus der Newtonschen Mechanik Folgerungen ableiten zu wollen, die der ART widersprechen und das dann für eine Widerlegung zu halten.

  425. #427 perk
    11. Januar 2011

    Das mit dem Ableiten der Spielregeln (allerdings beim Schach) aus dem beobachten von Spielen ist übrigens eine Analogie im viel zitierten Feynman “Character of Physical Law”.

    daher hab ichs nicht.. aber die analogie ist ja eine übliche und weit verbreitete, die immer mal wieder auftaucht

  426. #428 Name auf Verlangen entfernt
    11. Januar 2011

    Das Problem scheint ja offenbar zu sein, daß ihr die Konsequenzen eurer eigenen Lehre nicht verstanden habt:

    “Gaaanz langsam zum mitdenken: Mit Lichtjahren misst man Entfernungen… “jetzt” bezieht sich auf Zeit!”

    Es gibt in der “Raumzeit”, der zusätztlichen aus dem Hut gezauberten Dimension, kein “Zeit” oder “Raum”. Das Bild der Gegenwart, daß wir hier vermittelt bekommen, ist die Vergangenheit der Gegenwart dessen, was wir z.B. aus dem großen Bären gegenwärtig sehen.

    Das sollte doch wenigstens klar sein. Es ist meine Meinung nicht. Wenn nun aber jemand inbrünstig die ART vertritt und dazu noch meint, zwischen Zeit und Entfernung einen kategorialen Unterschied machen zu können, dann … ja dann haben wir ein Beispiel von “über-den-eigenen-Wahnsinn-gestolpert”.

  427. #429 TheBug
    11. Januar 2011

    Ja Markus, ganz klar, Du hast bestimmt recht und nu halt mal bitte für 100 km die Klappe und rück 2 Minuten weg.

  428. #430 Name auf Verlangen entfernt
    11. Januar 2011

    @ TheBug: den Gefallen tue ich Dir nicht: wer sollte euch denn sonst auf eure Widersprüche aufmerksam machen?

  429. #431 walim
    11. Januar 2011

    Größenwahn hat einen Namen

  430. #432 Aragorn
    11. Januar 2011

    Name auf Verlangen entfernt ist die männliche Ausgabe des Plapperautomaten “Jocelyne Lo pez”

  431. #433 Bullet
    11. Januar 2011

    Nanu? Ist der Master of incompetence wieder in einer manischen Phase?

    Das Problem scheint ja offenbar zu sein, daß ihr die Konsequenzen eurer eigenen Lehre nicht verstanden habt:

    Das Problem ist zuerst mal, daß du keine Ahnung von nix hast, aber große Reden schwingst. Bislang war jeder Versuch, dir auch nur einen einzigen Krümel Fakten darzubringen, erfolglos. Sieh mal: du scheiterst doch schon daran, den Unterschied zwischen Ortsveränderung und Bewegung zu verinnerlichen. Selbst auf deinem angeblich eigenen Gebiet, der Astrologie, kommst du auf keinen grünen Zweig: du schaffst es nicht, die Nachbeben des katastrophalen Haiti-Bebens auf den Monat genau vorherzusagen.

    Dann, nachdem du es schlauerweise mal geschafft hast, ein wenig den Rand zu halten, schlägst du wieder mit einer richtigen Bombe hier ein. Du behauptest:

    Die Sekunde ist und wird immer durch den Umlauf der Erde um die Sonne definiert sein, alles andere ist und bleibt logischerweise daran geeicht.

    Ach? Was ist denn eine Sekunde deiner Meinung nach? Hast du eine Definition für “Sekunde” die das hergibt, was du behauptest?
    Aber nein, wenn da Widerworte kommen, seilt sich MT ab und geht nach diesem bonmot

    Im übrigen äußere ich mich hier nur dann, wenn Sie sich mal wieder die Freiheit nehmen, zwischen Philosophie und Physik einen willkürlichen Unterschied zu machen.

    – Wer soll dir das abnehmen –
    zum nächsten Punkt über:

    Die nachgewiesene Teleportation bezieht sich nicht auf das Teilchen, sondern auf die Verschränkung. Das reicht aber völlig für Ihren geforderten Gegenbeweis.

    Tut es das? Ein wunderbarer Zeitpunkt, zu meinem Versuch mit dem abgelenkten Laser zurückzukommen.
    Markus, du kleiner Dummkopf: beim Auslesen eines verschränkten Quants wird keine Energie auf die andere Seite übertragen und keine Information. Daher kann diese verschränkte was-auch-immer beliebig schnell was-auch-immer machen. Wie der Laserstrahl beliebig schnell von einem Auftreffpunkt auf den anderen wechseln kann. Wenn du das schon nicht raffst, warum wechselst du sogleich wieder den Fokus und fabulierst munter weiter:

    doch: die Verschränkung ist die Information.

    Man darf hierbei nicht vergessen: Markus GröLaZ Termin bezeichnet sich selbst offiziell als Laie, scheut sich aber nicht, immer wieder falsche Faktenbehauptungen über ein Thema, von dem er keine Ahnung hat (Physik), in die Runde zu werfen. Auch hier nochmal: wenn man keine Ahnung hat und den Satz von Nuhr nicht anwenden will, sollte man um Informationen erfragen. Nicht – so wie du – einfach “alles Quatsch” nölen. Aber es ist noch nicht zuende:

    “Vielleicht wurde ja überhaupt nichts übertragen.” Na klar, den Tsunami hat´s wohl gar nicht gegeben.

    Tja … da war es ihm wieder zu schwer, darüber nachzudenken, was bei einer Wasserwelle denn übertragen wird, wenn der Großteil des Wassers nur kleine Schwingungen um einen Ruhepunkt herum ausführt. Schade. Da wäre er fast selbst in der Lage gewesen, zu erkennen, was eine Welle eigentlich ist … aber wieder verkackt. wie immer. Naja. Wie gehts weiter? Ach ja, so:

    Da solltest Du doch wissen, daß Licht gemäß eurer Theorie in gar nichts transportieren kann. Vom Licht aus betrachtet, ist die Welt zwei-dimensional: in so eine Schachtel passt kein “Medium” rein.

    Also … wenn es ihm zu schwer wird, dann behauptet er plötzlich, “unsere Theorie” würde so und so aussehen, und dann: *schnipp* kommt wieder eine Interpretation, die sich gewaschen hat. Vom Licht aus betrachtet, ist die Welt zwei-dimensional. Ja, Markus – du warst ja selbst mal eine Leuch.. äh, ein Licht und weißt das deswegen. Aha. Warum Licht lichtschnell ist, ist dir auch ein Rätsel, nicht wahr? Und warum keine Masse jemals lichtschnell werden kann, kannst du auch immer noch nicht fassen. Hmja. Weiter:

    Wichtig ist allein, daß wir wissen, daß am anderen Ende der verschränkten “Teilchen” die “gegenteilige” Informtion unserer eigenen angezeigt werden wird, bzw. angezeigt worden ist. Diese Information ist Information genug.

    Hört, hört: der Laie Name auf Verlangen entfernt holt sein verborgenes Fachwissen raus. Nur leider, Markus, solltest du doch inzwischen begriffen haben, daß eine Information nur dann entstehen kann, wenn ich einen Zustand identifizieren, auswählen und verändern kann (und meine Gegenstelle weiß, was welcher Zustand bedeutet). Bei der Verschränkung ist leider das “Auswählen” nicht dabei. Zu doof, das. Immerhin scheinst du so langsam mal zu bemerken, daß es sinnvoll ist, beim Thema zu bleiben:

    wir sprechen hier über etwas ganz Einfaches: den sogenannten “nichtlokalen Charakter” der QM

    Wertes Publikum: etwas ganz Einfaches, nämlich das noch viel einfachere Thema “Bewegung vs. Ortsveränderung” war dem Herrn Termin zu viel. Aber jeeeeeetzt: der nichtlokale Charakter der Quantenmechanik (jaha – eine Wellenfunktion muß nicht immer *einen* scharfen Peak haben) ist etwas wirklich echt und wahrhaftig einfaches.
    Aber Markus T. wäre nicht Markus T., wenn er nicht noch einige Joker im Ärmel hätte:

    Statt nun mit irgendwelchen Taschenspielertricks diese Tatsache aushöhlen zu wollen, indem man semantische Spielchen betreibt, erklärt ihr doch bitte in Übereinstimmung mit eurer lächerlich gespaltenen Relativitätheorie diese Tatsache.

    Can I get the Key section again?
    in Übereinstimmung mit eurer lächerlich gespaltenen Relativitätheorie
    Danke. Hat hier jemals irgendjemand behauptet, die Wellennatur des Lichts wäre ein Thema der Relativitätstheorie? Also … außer dem Laien MT, der von einem Napf in den anderen stolpert. Aber er kommt wieder in Fahrt: jetzt wirds wirklich lustig (und etwas schizophren). Eben noch voller Verachtung für die “lächerlich gespaltene RT”, haut er nun raus:

    “Ich “weiß” also, was 20 Lichtjahre von hier genau jetzt passiert”: da siehst Du mal, wie verwirrt Du bist: genau jetzt, oder 20 Lichtjahre von hier: entscheide dich mal.

    Markus: du kannst doch nicht fordern, man solle sich in einem System bewegen, das du rundheraus ablehnst, wenn man gerade versucht, dir klarzumachen, wie diesess System aussieht? DU bist doch derjenige, für den diese beiden Aussagen keinen Widerspruch darstellen, weil eben nur die RT den Begriff der “Gleichzeitigkeit” abschafft. Sei doch froh, daß wir dir ein wenig entgegenkommen. Aber ich sehe, du bist ferade in Pöbellaune:

    Wie kann man nur so frech große Töne spucken, und sich dermaßen konsequent um die entscheidenden Beobachtungen bislang unwiderlegter Experimente drücken?

    äh … dir ist klar, daß dieser eine Satz gespeichert wurde, um ihn dir ab sofort bei jedem deiner Kommentare ins Gesicht zu kleben? Wenn sich hier einer um irgendwas drückt, dann bist immer du derjenige. Und das mit dem “frech große Töne spucken” ist wohl deine Paradefähigkeit. Deine einzige übrigens, wenn ich das mal anmerken darf. Du weißt ja: Nachbeben und Monat und so. Ach ja: und großspuriges Wiederholen längst widerlegter Interpretationen:

    Im Klartext: wir können beweisen, “daß die Eigenschaften von Dingen nicht da sind, bevor wir sie beobachten.”

    Nein, das stimmt so nicht. Die Eigenschaften sind da – die Werte jener Eigenschaften werden durch die Messung verändert. Aber das wurde schon einmal lang und breit ausgewalzt, und du gefällst dir offenbar in der Rolle des stumpf repetierenden Mühlsteins. (Kleiner Tip am Rande: der IQ eines Mühlsteins ist weniger eine Zier.) Damit ließe sich aber vielleicht die folgende Entgleisung erklären:

    Das Problem scheint ja offenbar zu sein, daß ihr die Konsequenzen eurer eigenen Lehre nicht verstanden habt

    Es gäbe noch eine andere Möglichkeit, Markus: da DU diese Lehre nicht verstanden hast, führst du Konsequenzen ein, die nicht existieren. Die lehnst du dann *eigentlich* nicht überraschend ab – aber der Großteil der anwesenden Leserschaft deiner Wunschträume kämpft dann erst einmal mit einem WTF-Gesicht. Bei mir tauchte dieser Gesichtsausdruck an dieser Stelle auf:

    Es gibt in der “Raumzeit”, der zusätztlichen aus dem Hut gezauberten Dimension

    “Zusätzlich”? Hast du mal ein Minkowski-Diagramm gesehen? “Zusätzlich”???
    WTF??
    Und zum Schluß:

    Wenn nun aber jemand inbrünstig die ART vertritt und dazu noch meint, zwischen Zeit und Entfernung einen kategorialen Unterschied machen zu können, dann … ja dann haben wir ein Beispiel von “über-den-eigenen-Wahnsinn-gestolpert”.

    Oje, oje, oje. Da hab ich doch letztens einen schönen Filmschnipsel zum Thema gesehen… wo war der nur … ach ja, hier.

  432. #434 walim
    11. Januar 2011

    Ja der Vergleich mit dieser speziellen Jelo trifft die Sache.
    Der ganze Kommentarthread drifted unrettbar ins immer offtopischere ab. Ich ärgere mich ein bißchen für meinen überflüssigen Aufschrei oben angesichts des Ausmaßes der Absurdität. Der Hauskonfusius wird natürlich gleich feststellen, alle hier seien absurd und widersprüchlich außer ihm selber. Aber klar: Entfernung ist Zeit und Zeit existiert nicht und tüdelüt. Infolgedessen ist alles egal.
    Es wäre schön, wenn es hier mal wieder was Substanzielles gäbe. Andererseits hat MB ja im Text die Forderungen an die Theorie beschrieben und das wirre Begriffsgewusel das davon angelockt hier abgesondert wurde hat nun die übliche Pfützentiefe.

  433. #435 Frank Wappler
    11. Januar 2011

    MartinB schrieb (11.01.11 · 11:35 Uhr):

    > die Fußballanalogie erschließt sich mir nicht – willkürlich zu verlangen, dass aus bestimmten Axiomen ein Ergebnis folgen soll, hat mit dem, was wir hier diskutieren, nichts zu tun.

    Diese Bemerkung ist sicher wichtig, aber ich bin mir beim besten Willen nicht sicher, dass ich sie richtig verstehe. Würdest du bitte nochmal ausdrücklich unter Bezug auf das Fußball-Beispiel zuordnen:
    Was wird deiner Meinung nach “verlangt“?,
    was ist “das Ergebnis” (ich meinte: der Endstand eines bestimmten Fußballspiels),
    was sind “die Axiome” (ich meinte: die allgemein bekannten Fußballregeln),
    und woraus schließt du (in meiner Darstellung des Beispiels), dass das eine aus dem anderen folgen sollte?

    > “Die konkreten Werte der (mittleren) Lebendauern verschiedener Teilchensorten folgen doch auch nicht aus der SRT,”
    Natürlich nicht. Hat auch keiner behauptet

    Und deine Reaktion, bitte, auf meine Anschlussbemerkung:
    “Aus der SRT folgt doch nicht mal, dass überhaupt bestimmte (Teilchen-)Sorten auffindbar wären, für die sich ein mehr oder weniger konsistenter Wert der (mittleren) Lebendauer ermitteln ließe”
    ?

    > Teilchen sind nur wegen ihrer absoluten Identität ein praktisches beispiel.

    Wie kommst du darauf, dass insbesondere instabile Teilchen “(kollektiv) absolute Identität” besäßen? — auch nur “sortenweise”, überhaupt, und erst recht von vornherein??

    Instabile Teilchen sind selbstverständlich ein praktisches Beispiel — als Anwendungsfall, dass und wie Dauer als Messgröße definiert werden kann und sollte, um Werte von (individueller) Lebensdauer zu ermitteln und ggf. auf Werte von “mittlerer Lebensdauer” zu schließen.

    > Ich versuche es nochmal anders:
    (Auch falls ich deine Argumentation wieder mal verreißen sollte, hoffe ich, es versteht sich dennoch, dass ich die Versuche wertschätze, dadurch schließlich zu Argumenten zu gelangen, die eben nicht verrissen werden können. Wenn es nur nicht zu aufwändig wird …)

    > Ich nehme die SRT als axiomatisches System mit einem kleinen Unterschied: ich ändere den Wert von c in meiner Variante der SRT auf 0.00000001mm/s ab.

    Gebe ich diese Vorgabe (auch) richtig wieder, wenn der Vorgabe-Wert als
    “10^-11 m/s” geschrieben würde?
    (Dann könnte ich mich in der folgenden Beantwortung ganz unmissverständlich auf “das m” in der Darstellung dieses Vorgabe-Wertes beziehen.)

    Falls so:
    Sind wir uns (wie bislang) einig, dass Betrachtungen zu “Brechungsindex” außen vor bleiben?
    (“Vakuum” bzw. “Brechungsindex” als auch “gegenseitige Ruhe” sind zwar keine Begriffe, die ohne Definition selbstverständlich wären; aber ihre Definitionen als Messgrößen werden innerhalb der SRT als axiomatisch vorausgesetzt, und sind erst innerhalb der ART zu erstellen.)

    Der Buchstabe “c” soll ja (sicherlich) wie üblich in der chrono-geometrischen Distanzdefinition (der SRT) dazu dienen, um “Distanz”-Werte von Werten der “halben Pingdauer an sich” symbolisch zu unterscheiden.

    Die SRT, also die Definitionen von Messgrößen wie “Distanz”-Verhältnissen, “Geschwindigkeits”-Beträge (als ein Zahlenbruchteil und der Buchstabe “c”), “beta”-Bewertung, Lebensdauer-Verhältnissen usw. usf. ergibt sich damit wie üblich.

    Insbesondere haben die Messwerte “beta” (die Zahl, versteht sich; nicht der Vektor) wie immer den Wertebereich “von 0 bis 1”, ganz unabhängig davon, ob der Geschwindigkeitsbetrag “c” außerdem in irgendeiner bestimmten Einheit (außer “c”) ausgedrückt würde, oder nicht.

    Wir haben es also wie üblich mit der SRT zu tun (die wir schon ausgiebig diskutiert haben …).

    Es wäre also lediglich zu klären, was denn mit “dem m da” bzw. “dem s da” in deinem Vorgabe-Wert gemeint sein könnte (falls überhaupt etwas Nachvollziehbares).

    Unter Berufung auf die SRT, wie üblich, bliebe jedenfalls die Vorgabe der “Sekunden”-Definition https://www.bipm.org/en/si/si_brochure/chapter2/2-1/second.html nachvollziehbar (auch ohne Repräsentation der erforderlichen Messwerte in irgendwelchen bestimmten Einheiten; insbesondere versteht sich auch “Temperatur Null” ohne konkrete Vorgabe von “K” oder dergleichen.)

    Falls diese Bezeichnung “s” auch in deiner Vorgabe den üblichen Wert dieser (Einheits-)Dauer meint, dann ergibt sich aus deiner Vorgabe, dass mit “dem m da” der Distanzwert
    “c 10^11 s” gemeint ist;

    und nicht der übliche “Meter”-Wert

    “c 1/299792458 s”.

    > Ist diese Theorie dann deiner Ansicht nach widerlegbar?

    Kann man durch den Einsatz einer bestimmten Messoperation einen Messwert erhalten, und diesen durch Einsatz einer bestimmten Einheitendefinition in dieser entsprechenden Einheit repräsentieren,
    und daraus den Schluss ziehen, dass die gerade benutzte Messoperation und/oder die gerade eingesetzte Einheitendefinition damit experimentell falsifiziert (und folglich zu verwerfen) wären?? — Nein, natürlich nicht; man wundert sich, wie irgendwer darauf kommen würde.

    Gegen die vorgeschlagene Einheitendefinition (die Theorie was ja so-wie-so SRT) spricht lediglich die praktische Erwägung, dass sie ohne ausdrückliche Angabe der unüblichen Bedeutung “des m”s sicher missverständlich wäre.

    p.s.
    > ich gehe davon aus, dass das urmeter seine Länge […]

    Seine — was??
    Seine zweifellos nachweisbaren zwei Kerben, oder was sonst?

    > oder musste man mit Entdeckung der SRT alle Weltkarten neu vermessen und z.B. alle gemessenen Planetenumlaufzeiten?

    Natürlich; und geographische bzw. geodätische, astronomische, kosmologische, oder auch mikro-, nano- usw. -skopische Messungen werden nach wie vor gemacht.
    Und man hat damit systematische Fehler der vorausgegangenen Abschätzungsbemühungen überhaupt erst bewerten können.

    Viele waren (schon damals) ganz erstaunlich gut bzw. genau (so wie entsprechend der SRT-Methodik gemessen wurde) — wir leben eben in einer hinreichend kalten und flachen Weltgegend.

    > weil das der Erfahrung entspricht.

    Experimentalphysik jedenfalls ist daran orientiert, auch über jemandes unmittelbaren Erfahrungsbereich hinaus nachvollziehbar zu sein. Je systematischer und gründlicher das beachtet und umgesetzt wird, insbesondere mittlerweile durch den Einsatz der RT, um so mehr dient Experimentalphysik als Mittel der Forschung.

    Sicher: Vieles “funktioniert” ganz prima auch ohne gedankliche Auseinandersetzung mit Physik. Zumindest, bis die Erwartung, dass “es” immer so prima funktionieren würde, möglicherweise enttäuscht wird.

  434. #436 Name auf Verlangen entfernt
    11. Januar 2011

    @Bullet: “Nein, das stimmt so nicht. Die Eigenschaften sind da – die Werte jener Eigenschaften werden durch die Messung verändert.”

    Ich halte mich hier kurz, und bitte Dich, das auch zu tun; mir fehlt einfach die Geduld, Deinen ganze Kram zu lesen.

    Was obigen Satz angeht, bemerkt man halt, daß Du die Sache nicht verstanden hast. Tatsächlich ist beweisbar, daß die Eigenschaften vorher nicht da sind. Und zwar: “indem man an einer Messsation im allerletzen Moment entscheidet, was gemessen wird, und dann kann das Signal nicht mehr rechtzeitig an der anderen Seite ankommen.” (Zeilinger, Spuk, S 237)

  435. #437 Bullet
    12. Januar 2011

    Markus:
    was “Eigenschaft”?
    Beispiel bitte.

  436. #438 Aragorn
    12. Januar 2011

    @MT

    Was obigen Satz angeht, bemerkt man halt, daß Du die Sache nicht verstanden hast.

    Nein, du hast nichts verstanden. Bullet hat deine falsche Behauptung:

    Im Klartext: wir können beweisen, “daß die Eigenschaften von Dingen nicht da sind, bevor wir sie beobachten.”

    korrigiert. Da allerdings jede Begründung dieser Tatsache von dir ignoriert oder nicht verstanden wird, erspare ich mir weitere Worte und setze dich jetzt einfach auf “ignore”.

  437. #439 Bullet
    12. Januar 2011

    Ach ja, aus Rezension “Einsteins Spuk”:
    “Kritisch anzumerken sind jedoch Formulierungen, die instantane physikalische Wirkungen auf verschränkte Partner behaupten, zumal im selben Buch ausdrücklich diskutiert wird, dass das Aufgeben der Lokalität nicht die einzig mögliche Konsequenz aus der Verletzung Bellscher Ungleichungen ist.”

    Und jetzt… der ultimative Maulkorb. Markus: lesen. Und dann nochmal lesen. Folgendes Zitat ist aus einem Interview mit ANTON ZEILINGER. Wenn du schon hier immer mit dem Zeilinger-Buch wedelst, dann laß es dir doch von deinem Idol selbst erklären:

    Sie übertragen diese Information schneller als mit Lichtgeschwindigkeit, aber das widerspricht nicht der Relativitätstheorie. Wie ist das erklärbar?

    Das System im Endzustand nimmt in gewissen Fällen schneller als mit Lichtgeschwindigkeit den richtigen Zustand, die richtigen Eigenschaften an. Das ist kein Widerspruch zur Relativitätstheorie, die ja sagt, dass man nicht schneller als mit Lichtgeschwindigkeit irgendetwas übertragen darf. Der Grund ist der, dass die Nachricht, die ankommt, nicht verstehbar ist ohne zusätzliche Information. Empfänger wissen nicht, wann es geklappt hat und wann es nicht geklappt hat. Und diese zusätzliche Information kann nur mit Lichtgeschwindigkeit kommen.

    Quelle: https://www.asta-uni-mainz.de/index.php?option=com_content&view=article&id=482%3Aspuk-und-wirklichkeit-ein-gespr-mit-dem-quantenphysiker-anton-zeilinger&catid=202%3Aunipress-343&Itemid=145&lang=de

    Solltest du weiterhin darauf beharren, deinen Mist hier anzubringen, dann ist natürlich allen Lesern klar, daß du – eigentlich erwartungsgemäß – dieses Buch zum Türen stoppen verwendet hast. Möglicherweise, weil es dir zu schwierig war.

  438. #440 TheBug
    12. Januar 2011

    Nur mal so eine Vermutung: Markus verwendet wahrscheinlich alle Bücher bis auf das Telefonbuch als Türstopper oder zum Ausbalancieren von wackelnden Möbeln. Jedenfalls wehrt er sich sehr erfolgreich gegen jede Informationsaufnahme.

    @Frank Wappler: PLONK!

  439. #441 Kuchlbacher Rudolf
    12. Januar 2011

    @MT

    Es wird nur noch absurder…

    Es gibt in der “Raumzeit”, der zusätztlichen aus dem Hut gezauberten Dimension, kein “Zeit” oder “Raum”.

    Erstens ist die “Raumzeit” keine zusätzliche Dimension, sondern das 4-dimensionale Gebilde aus 3 Raumkoordinaten und der Zeit, und zweitens: Warum sollte man in einem mehrdimensionalen Koordinstensystem (z.B. 3-dimensional x,y,z) die einzelnen Koordinaten nicht mehr unterscheiden können?

    @Bullet

    Ich hab an anderer Stelle diesbezüglich auch schon mal ein Interview von Prof. Zeilinger verlinkt – bringt aber nix, weil unser großer Meister sich anscheinend nicht bemüßigt sieht auch nur irgendwas mit substantieller Kraft an sich heran zu lassen…

  440. #442 MartinB
    12. Januar 2011

    @FrankWappler
    ich habe deinen text zweimal gelesen, aber verstehe ihn einfach nicht. Irgendwie scheinst du fundamentale Begriffe ganz anders als ich zu verwenden, aber ich blicke nicht so recht durch, welche genau und wie. Dass du z.B. darauf zu bestehen scheinst, dass man Längen nur mit “ping”-Signalen messen kann, leuchtet mit nicht ein.

    Ich meine herauszulesen, dass man deiner Ansicht nach eine SRT mit c=1e-11m/s nicht experimentell widerlegen kann. Das halte ich für eine absurde Aussage.

    Auch dies hier versteh ich nicht:
    “> oder musste man mit Entdeckung der SRT alle Weltkarten neu vermessen und z.B. alle gemessenen Planetenumlaufzeiten?
    Natürlich;”
    Du meinst also, 1905 und in den Folgejahren hat man alle Längenmessungen in allen Wissensgebieten nochmal gemacht? Echt jetzt? Oder wie ist das gemeint?

    “wir leben eben in einer hinreichend kalten und flachen Weltgegend.”
    Was hat jetzt das mit der SRT zu tun? In der SRT ist die Metrik immer flach, und Temperaturen spielen überhaupt keine Rolle, oder?

    “Die SRT, also die Definitionen von Messgrößen wie “Distanz”-Verhältnissen, “Geschwindigkeits”-Beträge (als ein Zahlenbruchteil und der Buchstabe “c”), “beta”-Bewertung, Lebensdauer-Verhältnissen usw. usf. ergibt sich damit wie üblich.”
    Dieser Satz setzt SRT gleich mit der Definition von Messgrößen. Die SRT definiert aber keine Messgrößen, sondern verwendet sie (setzt die existenz der Möglichkeit von z.B. Längenmessungen doch schon voraus).

    Ich habe langsam Zweifel, dass es mir noch gelingen kann, deinen Standpunkt zu verstehen. Er scheint mir auch im klaren Widerspruch zur Physik, wie sie praktiziert wird, zu stehen, denn deiner Ansicht nach kann man ja Newton nicht widerlegen – während die heutige Physik ja Newton als widerlegt (genauer gesagt als Grenzfall einer umfassenderen Theorie) ansieht.

    PS: Die Fußball-Analogie hat mir von vornherein nicht besonders eingeleuchtet – so wie du es geschrieben hast, hast du gesagt (oder so habe ich es verstanden): Wenn es Fußballregeln gibt, können diese dann dadurch widerlegt werden, dass meine Mannschaft ein Spiel verliert? Das ergibt für mich keinen Sinn, weil das keine gültige Folgerung aus den regeln ist.

  441. #443 Bullet
    12. Januar 2011

    @Rudolf:
    vielleicht war ja das Problem, daß du nur *verlinkt* hast. Da hätte man noch draufklicken müssen, was bedeutet, daß man der unbequemen Faktenlage durch Nichtklicken entgehen kann. Deshalb hab ich jenen Text hier reinkopiert.
    Ich bin mal gespannt, wie er sich da rauswindet. Vielleicht mit dem Hinweis, daß Zeilinger die Implikationen seiner eigenen Entdeckungen nicht versteht (im Gegensatz zum Laien Termin natürlich)??

  442. #444 TheBug
    12. Januar 2011

    Die Fußball “Analogie” macht alleine schon deshalb keinen Sinn, weil es sich dabei um einen Vorgang handelt, der nicht einem festen Mechanismus folgt, anders als ein physikalischer Effekt, der unter gleichen Bedingungen auch immer zum gleichen Ergebnis führt, auch wenn dieses möglicherweise statistisch zu erfassen ist, wie im Quantenbereich.

    Aber wir haben da mit FW ein ähnliches Kommunikationsproblem wie mit MT, er scheint zwar prinzipiell zu komplexeren Gedanken als MT fähig, hat sich aber in eine Ecke verrannt, aus der er nicht wieder herauskommen mag und scheint auch zumindest etwas verwirrt zu sein.

  443. #445 perk
    12. Januar 2011

    martin lass dich nicht wieder von franks merkwürdiger ausdrucksweise 3 kapitel zurückwerfen

    Ich meine herauszulesen, dass man deiner Ansicht nach eine SRT mit c=1e-11m/s nicht experimentell widerlegen kann. Das halte ich für eine absurde Aussage.

    mit c = 1e-11m/s würde die srt immernoch genauso gut funktionieren (die theorie braucht nur ein c, wie groß dessen wert ist ist für die theorie wurscht)

    für uns ist der wert von c natürlich wichtig, deswegen bauen wir ein modell innerhalb der srt, das einen konkreten wert c hat und vergleichen experimentalergebnisse gegen dieses modell, wir können also im rahmen der srt nur konkrete modellannahmen widerlegen und wenn wir irgendwann kein modell innerhalb der theorie mehr finden können, das noch zum experiment passt bezeichnen wir physiker eine theorie als widerlegt (und konstruieren zb die art, da wir darin wieder modelle finden die zum beobachtungsbefund passen)

    Dieser Satz setzt SRT gleich mit der Definition von Messgrößen. Die SRT definiert aber keine Messgrößen, sondern verwendet sie (setzt die existenz der Möglichkeit von z.B. Längenmessungen doch schon voraus).

    hmm nein der satz sagt, dass ein wichtiger bestandteil der srt die definition von messgrößen ist, genauer zeit und länge…
    und damit zeitdiletation und längenkontraktion tatsächlich sinnvolle begriffe sind ist es sinnvoll sich etwas von der materie loszulösen, zb muss man die uhren die am anfang und ende der messstrecke sind nach dem synchronisieren hinreichend langsam zu ihrem endpunkt bringen (oder beide in der mitte starten lassen) damit sie bei ankunft noch die gleiche zeit anzeigen

    solche probleme sind für die propere definition der physikalischen größen und ihre bestimmung wichtig, weshalb die srt lösungen (aus der sicht der pragmatischen physiker) bzw axiomatische forderungen (aus franks sicht) für messungen enthält

    Wenn es Fußballregeln gibt, können diese dann dadurch widerlegt werden, dass meine Mannschaft ein Spiel verliert? Das ergibt für mich keinen Sinn, weil das keine gültige Folgerung aus den regeln ist.

    warum gehst du soweit zurück, ich dachte nach meinem vorletzten beitrag war die fußballanalogie soweit durch und verstanden?

    Er scheint mir auch im klaren Widerspruch zur Physik, wie sie praktiziert wird, zu stehen, denn deiner Ansicht nach kann man ja Newton nicht widerlegen – während die heutige Physik ja Newton als widerlegt (genauer gesagt als Grenzfall einer umfassenderen Theorie) ansieht.

    so wie ich frank bisher verstehe wurde newton wurde ja nie widerlegt, sondern nur alle modelle, die uns im rahmen dieser theorie einfielen.. ob man daraus die widerlegung der theorie schlussfolgert, ist dann nur geschmackssache (bei der ich deinen geschmack sie als widerlegt zu bezeichnen teile, wobei sie halt nicht widerlegt ist, sondern nur als nicht ausreichend zutreffend für die von uns experimentell zugängliche realität gekennzeichnet)

    aber auch ich habe noch probleme einiges von frank zu verstehen..
    zb welche bedeutung nimmt der mechanismus, wie geschwindigkeiten zu komponieren sind, im rahmen einer theorie ein (teil des modells ist er ja nicht)

    v_e = v_1+v_2 wäre so wie ich das sehe noch teil der newtonschen theorie der kinetik
    und gilt im rahmen dieser für den fall, dass 3 bewegte bezugssysteme existieren .. haben wir in teilchenbeschleunigern ja bei gegenläufigen strahlen, aus dem bezugssystem des einen strahls fliegt die strahlröhre mit fast c ständig an ihm vorbei, und der entgegenkommende strahl? nicht mit 2c sondern mit 0,99c

    meiner ansicht nach wurde hier kein modellparameter benutzt sondern nur geschwindigkeitsdefinitionen die newtonsch zu beobachten sind und eine bezugssystemstransformation der theorie

    beides zusammen (vollkommen modellunabhängig) scheitert am experimentellen befund
    und das ergibt im canon der physik eine widerlegung der theorie

  444. #446 klauszwingenberger
    12. Januar 2011

    mit c = 1e-11m/s würde die srt immernoch genauso gut funktionieren

    Richtig, und wer das nicht verstanden hat, verwechselt Anfangsbedingungen mit Naturgesetzen.

  445. #447 MartinB
    12. Januar 2011

    @perk
    Meine andere Wahl von c hatte folgende Bewandtnis:
    Es scheint mir offensichtlich, dass eine Theorie, die den Formalismus der SRT beinhaltet *und zusätzlich* den Wert von c auf diesen absurden Wert setzt, durch Experimente widerlegt ist. Ich wollte gern wissen, ob das für Frank konsensfähig ist.
    Falls es das ist, dann möchte ich gern verstehen, warum das festlegen dieses einen Zahlenwertes eine prinzipiell nicht widerlegbare Theorie zu einer widerlegbaren macht.
    Das ist nach wie vor ein versuch zu verstehen, was er eigentlich mit “Widerlegen” meint.

    Und wenn ich dann die SRT mit einem konkreten Zahlenwert für c widerlegen kann, dann könnte ich sie auch für jeden anderen konkreten Wert widerlegen (wenn sie denn falsch wäre). und daraus könnte man zumindest eine konkrete untere Schranke für c bestimmen, und wenn die größer ist als der gemessene Wert von c, dann wäre die Theorie widerlegt. Oder nicht? Ich glaube, ich verstehe es einfach gar nicht.

    Für mich sagt der Satz
    “ie SRT, also die Definitionen von Messgrößen wie “Distanz”-Verhältnissen, ”
    dass die SRT die Definition von Messgrößen beinhaltet. Das ist meiner Ansicht nach falsch – Größen wie Länge und Zeit werden von der SRT vorausgesetzt (deswegen doch einstens Satz “Zeit ist,w as die Uhr misst”.). ich hätte ansonsten gern die Definition von zeit und Länge aus der SRT gewusst.

    “ich dachte nach meinem vorletzten beitrag war die fußballanalogie soweit durch und verstanden?”
    Aber was Frank wieder dazu schrieb, habe ich wieder anders verstanden.

    “so wie ich frank bisher verstehe wurde newton wurde ja nie widerlegt, sondern nur alle modelle, die uns im rahmen dieser theorie einfielen.. ob man daraus die widerlegung der theorie schlussfolgert, ist dann nur geschmackssache”
    Dann ist es aber die Aufgabe des Newton-Anhängers, ein konkretes Modell aus der Theorie abzuleiten, das nicht widerlegt ist. Ansonsten ist das so ein bisschen wie Pseudowissenschaft (o.k., es gibt keine Evidenz für homöopathie, aber sie könnte ja trotzdem richtig sein usw.).

    “beides zusammen (vollkommen modellunabhängig) scheitert am experimentellen befund und das ergibt im canon der physik eine widerlegung der theorie”
    Eben.

  446. #448 JS
    12. Januar 2011

    @ alle, die mit Name auf Verlangen entfernt diskutieren:

    Verschwendet nicht eure Zeit mit ihm, das ist nämlich genau das, was er erreichen will. Er hat jetzt in unzähligen Diskussionen und bei den verschiedensten Themen bewiesen, dass er nicht gewillt oder nicht in der Lage ist selbst die einfachsten und offensichtlichsten Zusammenhänge, die außerhalb seines Wunschdenkens liegen, zu begreifen. Dass hier wissenschaftlich gut gebildete Menschen mit ihm diskutieren gibt ihm auch noch das Gefühl, als würde sein Gerede tatsächlich von Bedeutung sein … eine kapitale Fehlwahrnehmung.
    Wenn die Wissenschaft schon seiner Astrologie keinerlei Aufmerksamkeit schenkt, dann sollen ihm zumindest jene, die Wissenschaft betreiben ein wenig Aufmerksamkeit schenken, auch wenn es keine Aufmerksamkeit im positiven Sinne ist. Es geht also gar nicht um Inhalte, sondern nur darum von jemandem wahrgenommen zu werden, der nicht jeden Unsinn schluckt, wie all seine Kunden. Kann man ihm das verübeln? Wenn man bedenkt, dass er sich im 21. Jahrhundert selbst (vollen Ernstes!!) als Astrologe bezeichnet, natürlich nicht.

  447. #449 Bullet
    12. Januar 2011

    @JS:

    Dass hier wissenschaftlich gut gebildete Menschen mit ihm diskutieren gibt ihm auch noch das Gefühl, als würde sein Gerede tatsächlich von Bedeutung sein …

    Ich weiß nicht. Ein gefühltes Drittel der Kommentare an MT liest dich etwa so:
    “du offenbarst immer wieder eklatante Wissenslücken in elementaren Aspekten der Materie…”
    oder
    “es gibt kaum einen Satz von dir, den man so einfach stehenlassen könnte”

    Jemand, der bei solchen Ansagen immer noch glaubt, er sei auf Augenhöhe, hat mehr als nur das eine kleine kognitive Herausforderung zu meistern.

  448. #450 TheBug
    12. Januar 2011

    @JS: Wir diskutieren nicht mit ihm, wir betreiben Feldstudien zum Dunning Kruger Syndrom.

  449. #451 Name auf Verlangen entfernt
    12. Januar 2011

    @ wen´s angeht: Unendlichkeit des Weltalls oder auf einen Detektor, einen Messapparat begeben. Wenn Du nun mißt, was der eine Strahl für eine Farbe hat, dann kannst Du sicher wissen, daß der andere dieselbe zeigt. Nun veränderst Du mit einem Filter die Farbe des ersten Strahls. Du stellst sicher, daß er keinen Kontakt mit dem zweiten Strahl (Physiker sprechen gern kryptisch von Photonen) hat. Und doch wird sich, ungeachtet des Zwischenfilters (man kann auch sagen: Messung), der erste so verhalten, wie der zweite. Man nennt das “nichtlokalen” Charakater der Quantenphysik – übertragen auf die Dramentheorie von Aristoteles ist damit die Einheit von Ort und Handlung nicht infrage gestellt, aber die Handlung findet tatsächlich synchron und zeitunabhängig an zwei Orten zugleich statt. Und das “spukhaft”, außerhalb der Zeit. Wir wissen das und können das beobachten. Das Problem an diesem Versuch ist, daß die wenigen Menschen, die jemals die Möglichkeit hatten, diese Versuche konkret nachzuvollziehen und ihre wahre Bedeutung nachvollziehen können, zum Teil total kindisch sind, was gemeinhin an ihrem Zeitsubjektivismus liegt, der durch die ART ausgelöst wurde. Zeilinger z.B zieht ernsthaft Zeitreisen in Betracht. Als hätten wir die Gegenwart schon verstanden.

  450. #452 S.S.T.
    12. Januar 2011

    Jetzt ist es wirklich nicht mehr von dieser Welt, noch nicht einmal mehr mystisch, sondern

    zum Teil total kindisch

  451. #453 Name auf Verlangen entfernt
    12. Januar 2011

    S.S.T.: Nun erklär Du mal, was Einstein meinte, als er von “spukhaft” spracht?! Hat der Alte phantasiert? Warum wohl wählte er dieses Wort? Aus Jux und Tollerei? Weil nix dran ist, denn es kann ja keine “Information” von A nach B gelangen? Den EPR-Versuch hat es nicht gegeben? Ich bin mal dran mit Fragen: was bedeutet “nichtlokal” in der QM?! Hat sich jemand was ausgedacht, um euch zu verarschen?!

  452. #454 Frank Wappler
    12. Januar 2011

    MartinB schrieb (12.01.11 · 08:29 Uhr):

    [Falls ich im Folgenden auch Zitate oder Anregungen aus deinen anderen Korrespondenzen entnehme, sehe man mir bitte nach, wenn ich trotzdem vorrangig versuche, unsere Korrespondenz weiterzupflegen. – FW]

    > [… Du] setzt SRT gleich mit der Definition von Messgrößen.

    Ja, mit Definitionen bestimmter Messgrößen;
    sowie den axiomatischen Begriffen, mit denen die Definitionen konstruiert werden, den Theoremen, die sich aus Zusammenhängen ergeben.

    Besonders prominent dabei die Messgröße “Gleichzeitigkeit (Boolscher Messwert: ja oder nein)” für Anzeigenpaare zueinander ruhender Beteiligter, nach Einsteins Definition;
    die Messgröße “chrono-geometrische Distanz: c 1/2 Pingdauer” für zueinander ruhende Beteiligte;
    und die Messgröße “(Durchschnitts-)Geschwindigkeitsbetrag” (Messwert: “c” verbunden mit einer rationale Zahl zwischen 0 und 1) für ein bestimmtes Starttor und eine bestimmtes Zielband, die zueinander ruhen sollen, festzustellen für “das, was” sie austauschen, als
    “Distanz zwischen Start(tor) und Ziel(band)
    geteilt durch
    Dauer der Ziel(band)uhr während des Austausches”
    (wobei letztere Dauer die Messung von Gleichzeitigkeit erfordert, welche Anzeige der Zieluhr entsprechend der Startanzeige war; also die Messung “wann die Zieluhr hätte loslaufen sollen”);

    beruhend auf Einsteins selbstverständlicher (und wohl kaum vernüftig in Frage zu stellender, also axiomatischer) Annahme, dass jeder zumindest im Prinzip beurteilen kann ob man “das Zeigen des kleinen Zeigers meiner Uhr auf 7 und das Ankommen des Zuges” entweder koinzident wahrnahm, oder “das eine nach dem anderen”,
    sowie weiteren Festsetzungen, die darauf aufbauen;

    und mit aus den Definitionen herzuleitenden Theoremen wie z.B.

    “Distanz zwischen Lokomotivenspitze und Zugende ist gleich
    dem 1/Sqrt[ 1 – beta^2 ] fachen der
    Distanz zwischen einer Schwelle, die die Lokspitze traf, und einer anderen Schwelle, die das Zugende traf”,
    sofern die Anzeige des Treffens der einen Schwelle mit der Lokspitze und die Anzeige des Treffens der anderen Schwelle mit dem Zugende zueinander gleichzeitig waren,
    und sowohl der Geschwindigkeitsbetrag, den die Schwellen für Lokspitze und Zugende feststellten, als auch der Geschwindigkeitbetrag, den Lokspitze und Zugende für die Schwellen feststellten, den gleichen Wert
    “c beta” hatte;

    und (unter den selben Bedingungen) weitergehende Theoreme wie z.B.

    “beta_Zug[ Gegenzug ] =
    (beta_Schwellen[ Zug ] + beta_Schwellen[ Gegenzug ]) /
    (1 + beta_Schwellen[ Zug ] * beta_Schwellen[ Gegenzug ])”.

    > Die SRT definiert aber keine Messgrößen, sondern verwendet sie

    Da kommen wir dem Kern der Auseinandersetzung doch schon näher:
    Wie nennst du denn die Theorie bestehend aus dem oben beschriebenen Axiom, den darauf aufbauenden Definitionen, insbesondere zur Messung von Gleichzeitigkeit bzw. von chrono-geometrischer Distanz, und den daraus herzuleitenden bekannten Theoremen
    ?

    Was davon gehört zu dem, was du unter “SRT” verstehst?
    Und was gehört außerdem zu dem, was du unter “SRT” verstehst, dass es dir ermöglicht, ihre experimentelle Falsifikation als Hypothese in Betracht zu ziehen?

    […]

    p.s.
    > Du meinst also, 1905 und in den Folgejahren hat man alle Längenmessungen in allen Wissensgebieten nochmal gemacht?

    Man hat systematische Unsicherheiten von Werten beurteilen können, die (unter Namen wie “Abstandsverhältnisse” oder “Geschwindigkeitsverhältnisse”) durch das von dir oben propagierte “Hinzeigen” schon vorher in Umlauf gebracht wurden. Und natürlich ist es nie verkehrt (solang es ökonomisch vertretbar ist), immer wieder zu messen. Schließlich könnte sich die Modell-Erwartung, immer wieder die gleichen Werte zu finden, zumindest in einigen Fällen als falsch erweisen.

    MartinB schrieb (12.01.11 · 13:01 Uhr):
    > Dann ist es aber die Aufgabe des […] Anhängers [einer bestimmten Theorie], ein konkretes Modell aus der Theorie abzuleiten, das nicht widerlegt ist.

    Eine bestimmte Theorie stellt, in Form definierter Messgrößen bzw. -operatoren, nur Bausteine zum Modellbau zur Verfügung.

    Aus dem gesamten Wertebereich der Messgröße “Distanzverhältnisse” kann man z.B. die unterschiedlichsten Modelle von “Küstenlinien der Erde” aufstellen; sowohl falsche als auch (im Vergleich zur jeweiligen Messung Versuch für Versuch) das Richtige.

    Aus dem gesamten Wertebereich der Messgröße “regulärer oder irregulärer Fußball-Spielzug” kann man z.B. verschiedene Modelle darüber formulieren, ob man Zeuge eines Fußball-Spiels wäre (bzw. einer ganzen Saison usw.) oder ob man sich zum Synchronschwimmen-oder-wer-weiß-was verlaufen hat.

    Aus dem gesamten Wertebereich der Messgröße “Tor durch regulären Spielzug oder kein Tor” kann man die unterschiedlichsten Modelle zu Spielausgängen formulieren.

    Die experimentell nachweisbare Tatsache, dass darunter jede Menge falsche Modelle sind, falsifiziert (vernichtet) aber nicht die Bausteine der Modelle.

    > eine Theorie, die den Formalismus der SRT beinhaltet *und zusätzlich* den Wert von c auf diesen absurden Wert setzt

    Was du (11.01.11 · 11:35 Uhr) angeboten hast, kam bei mir an als
    “eine Theorie, die den Formalismus der SRT beinhaltet *und zusätzlich* ungewöhnlich bzw. ungeschickt benannte Maßeinheiten”;

    bzw. sofern “s” dabei trotzdem wie üblich als der Dauerwert “eine Sekunde” gemeint war
    (was ja durchaus denkbar wäre, weil oder besser sofern die entsprechende SI-Definition auch ohne irgendwelche bestimmten Maßeinheiten nachvollziehbar ist):

    “eine Theorie, die den Formalismus der SRT beinhaltet *und zusätzlich* den Distanzwert “c 10^11 s”, der mit dem Buchstaben “m” benannt ist, obwohl man damit üblicherweise den “Meter” genannten Distanzwert “c 1/299792458 s” meint”.

  453. #455 TheBug
    12. Januar 2011

    Einstein ist wohl der am häufigsten aus dem Zusammenhang gerissen oder vorsätzlich falsch zitierte Wissenschaftler des 20. Jahrhunderts. Dadurch, dass es sich bei ihm um einen überaus gebildeten Menschen handelte, sind die durchschnittlichen Esoteriker mit seinen Aussagen und seinem Humor halt überfordert, selbst wenn er sich nicht in den höheren Spähren der Physik bewegte.

    @Frank Wappler: Echt keine Ahnung aus welchem Universum Du stammst, aber es könnte glatt eins aus einem Douglas Adams Buch sein.

  454. #456 MartinB
    13. Januar 2011

    @FrankWappler
    Ich fang am Ende an, da scheint mir der kernpunkt deutlich zu werden:
    “”eine Theorie, die den Formalismus der SRT beinhaltet *und zusätzlich* den Distanzwert “c 10^11 s”, der mit dem Buchstaben “m” benannt ist, obwohl man damit üblicherweise den “Meter” genannten Distanzwert “c 1/299792458 s” meint”.”
    Heißt das, du bist der Meinung, dass die SRT sozusagen selbst den Wert von c setzt? Dann schließt deiner Meinung nach die SRT also die Maxwell-Gleichungen mit ein, denn nach denen ist c ja ebenfalls bestimmt durch c^2=1/eps0 mu0. Und da man in der realen Welt ja das meter letztlich zumindest grob über die Längenskala definiert hat, die sich aus unserer Biologie ergibt, gleich noch ein bisschen mehr.

    Ich kann übrigens eine Längenskala auch unabhängig von der SRT zum Beispiel über die Quantenmechanik definieren, beispielsweise als Radius des Bohr-Atoms. Das ist eine unabhängige Definition. Auch in der kann ich c festlegen, als soundsoviele Atomradien pro Zeit, wobei ich die Zeit über E=h nu und die Grundzustandsenergie festlegen könnte. Da geht c nicht ein.

    Was die Messgrößen angeht: Du bist der meinung, dass die SRT die Längeneinheit komplett neu definiert hat über “ping”s, richtig? Und was man vorher als Längeneinheit definiert hat (z.B. der Abstand bestimmter Punkte auf dem Urmeter) ist damit hinfällig geworden. Entspricht das deiner Meinung?

    Deine Auflistung der durch die SRT definierten Messgrößen (nebenbei, warum sprichst du eigentlich immer von Operatoren? Das ergibt doch in der klassischen Physik nicht viel Sinn, oder?) verstehe ich nicht so ganz. Wenn du c als die fundamentale Größe siehst, mit der Messgrößen definiert werden, dann brauchst du doch zusätzlich noch entweder eine Definition für Sekunde oder für Meter, sonst fehlt was. Wo ist denn die? Wie folgt die aus der SRT?
    Implizit hast du übrigens auch angenommen, dass du bestimmen kannst, wann zwei Objekte relativ zueinander ruhen, auch dafür brauchst du zusätzlich zu c entweder eine Längen- oder eine Zeitdefinition.

  455. #457 Bullet
    13. Januar 2011

    @MT:

    Ich bin mal dran mit Fragen: was bedeutet “nichtlokal” in der QM?

    Jede Welle ist nichtlokal, da sie nicht auf einen Punkt fixiert werden kann. Da eine Welle Energie transportiert, ist auch die von der Welle transportierte Energie (und, soweit ich das verstanden habe, demzufolge auch ihre Quantenzustände … Martin???) nicht auf einen Punkt beschränkt. (Im Gegensatz zu der veralteten, weil unpassenden Analogie des sog. “Teilchens”, die davon ausgeht, daß das Teilen einen Ort hat und an diesem Ort auch alle Energie, die das Teilen mit sich führt, versammelt ist.)
    Soweit verständlich?

  456. #458 Aragorn
    13. Januar 2011

    “nichtlokal” bedeutet:
    MT soll sich NICHT andauernd im LOKAL (Kneipe) besaufen, und danach hier stockbesoffen über Physik labern.

  457. #459 Bullet
    13. Januar 2011

    Dabei fällt mir ein: ich hab mal “Nichtlokalität” gegoogelt und gleich diesen Link gefunden:
    https://www.quantenwelt.de/quantenmechanik/wellenfunktion/nichtlokalitaet.html

    Und was steht da? Fast genau das, was ich mir da oben aus den Fingern gesogen hab. Krass. Jetzt siehts so aus, als hätt ich nur abgeschrieben.

  458. #460 Bullet
    13. Januar 2011

    Andererseits hätte MT da auch selber drauf kommen können…

  459. #461 TheBug
    13. Januar 2011

    @Bullet: Nee, hätte er nicht, sonst wärs nicht MT.

  460. #462 Name auf Verlangen entfernt
    13. Januar 2011

    @ MartinB: “Wenn du c als die fundamentale Größe siehst, mit der Messgrößen definiert werden, dann brauchst du doch zusätzlich noch entweder eine Definition für Sekunde oder für Meter, sonst fehlt was. Wo ist denn die? Wie folgt die aus der SRT?”

    – tut es das nicht aus der Definition von Masse als Proportionalitätskonstante aus Kraft und Beschleunigung?- : m/s² – so ist der Meter in der SRT vorausgesetzt, während er gleichzeitig von ihr definiert wird.

  461. #463 Name auf Verlangen entfernt
    13. Januar 2011

    @ Bullet, Aragorn:

    “Unter jener Quantenteleportation versteht man dabei im physikalischen Sinn das instante Übersenden der in einem unbekannten Quantenzustand enthaltenen Information an einen beliebig weit entfernten Empfänger unter Ausnutzung von Verschränkung.” (aus D.Bruß, Quanteninformation, Fischer 2003)

    “Ein Skeptiker mag einwenden, daß hier nur der Polarisationszustand des Photons übertragen wurde, oder allgemeiner, sein Quantenzustand, aber nicht das Photon ‘selbst’, doch da ein Photon vollständig durch seinen Quantenzustand charakterisiert wird, ist die Teleportation seines Zustands völlig äquivalent zur Teleportation des Teilchens.” Anton Zeilinger (Nature, 2000)

    Nun tut also bitte mal nicht so, als hättet ihr diese Entwicklung der Physik verschlafen. Das ist so neu nicht. Das anloge Beispiel mit den Wasser-Wellen ist hier völlig absurd, und zeigt eben, daß du den springenden Punkt nicht geblickt hast: es gibt keine Verbindung vom einen zum anderen Photon, außer der Verschränkung.

    Gerade zur Zeit sind ein paar wunderbar allgemeinverständlich lesbare Werke von seriösen WissenschatlerInnnen im Umlauf. Ihr lest das mal, dann sprechen wir uns wieder, o.k.?

  462. #464 MartinB
    13. Januar 2011

    @MT
    Leider wird keine Information in dem Sinne übertragen, dass der Zustand des Photons vorher am “Aussendeort” hätte bekannt sein können. Dazu müsste man ihn messen. Das würde die Verschränkung zerstören. Deswegen kann der “Sender” so auch keine Information übertragen.

    Egal wieviele populärwissenschaftliche Kommentare du hier anführst – du hast die QM leider nicht verstanden, und allgemeinverständlich lesbare Werke sind in der Wissenschaft aus gutem Grund nicht so sehr hoch angesehene Quellen.

  463. #465 TheBug
    13. Januar 2011

    Ich denke mal MT meint man könnte den Zeitpunkt zu dem an Ort A die Messung durchgeführt wird an Ort B durch das Verhalten des Zwillingsteilchens erfahren.

  464. #466 Name auf Verlangen entfernt
    13. Januar 2011

    @ Martin B: “Leider wird keine Information in dem Sinne übertragen, dass der Zustand des Photons vorher am “Aussendeort” hätte bekannt sein können.

    – das ist doch überhaupt nicht die Frage und wird doch nicht bezweifelt. Trotzdem besteht eine symmetrische und entfernungsunabhängige Korrelation zwischen den verschränkten Systemteilnehmern. Eben jene von Einstein aus gutem Grund so genannte “spukhafte Fernwirkung”.

    Wozu soll das auf der Ebene von Informationsübertragung diskutiert werden? So nach dem Motto: ein Ding, daß wir nicht nutzen können, ist nichts wert? Wir wollen wissen, was das physikalisch bedeutet, und keine komplett ignoranten Bemerkungen, wie die Ihre eben.

  465. #467 Name auf Verlangen entfernt
    13. Januar 2011

    @ The Bug: nein, M.T. denkt, daß Teilchen A und Teilchen B entfernungsunabhängig symmetrisch sind, auch wenn komplett ausgeschlossen werden kann, daß Teilchen B die Messung des Teilchen As mitbekommen hat.

  466. #468 Bullet
    13. Januar 2011

    @MT:

    Gerade zur Zeit sind ein paar wunderbar allgemeinverständlich lesbare Werke von seriösen WissenschatlerInnnen im Umlauf. Ihr lest das mal, dann sprechen wir uns wieder, o.k.?

    Moment mal: DU forderst von MIR, daß ich gefälligst die BILD-Zeitung zu lesen hätte, damit ich in der Quantenphysik mal richtig was lerne und dann auf deinem Niveau mit dir reden kann???
    Wie wärs, wenn du dir endlich mal eine Tüte Verstand besorgst, damit du die richtigen Bücher, deren Autoren sich mit der BILD (bzw. deinen lustigen Kinderbüchern) den Hintern abwischen, überhaupt ansatzweise verstehst?

    Jetzt mal im Ernst: du bekommst regelmäßig einen manischen Schub und beginnst mit völlig neuen Themen, die du ausgiebig mit Kommentaren à la “ich Wisser, ihr nix” würzt. Nichts könnte weiter von den realen Verhältnissen entfernt sein. Ich hab noch nie ein sachliches Argument von dir gelesen. Jedesmal, wenn unter deinem Namen Sätze auftauchen, über die man zumindest nachdenken kann, sind sie deutlich oder wie auch schon vorgefallen: weniger deutlich als Zitate gekennzeichnet. Ich hab mal eine Skizze verlinkt, die du “unbedingt” haben wolltest – nur um dann dich in die Welt hinauszuposaunen zu sehen, daß die paar Striche zu hoch für dich waren. Das enttäuscht.
    Du bist hier nicht in der Position, irgendwas fordern zu können. Und daß ich dich nicht jedesmal mit “MKT”, wobei das “K” für “Kretin” steht, anspreche, ist nicht etwa dem Umstand geschuldet, daß ich so tu, als wärst du keiner, sondern entspringt lediglich dem Zugeständnis an Martin, nicht ausschließlich meine berechtigte Verachtung für dich auszudrücken, wenn ich das Wort an dich richte, sondern ein gewisses Maß an Sachlichkeit zu wahren. Natürlich gibt es auch ein passendes Video zu allen deinen Äußerungen, das ich dir hiermit nicht vorenthalten mag:

    Außerdem erwarte ich von dir, daß du an dich gestellte Fragen pronto beantwortest, speziell dann, wenn du diese Fragen selber haben wolltest.
    Meine letzte Frage an dich war (was du eigentlich wissen solltest, aber wir sind hier unter uns und wissen um deine Defizite):
    Soweit verständlich?

  467. #469 Frank Wappler
    14. Januar 2011

    MartinB schrieb (13.01.11 · 07:52 Uhr):

    [Auch ich versuche, deine Anfragen so zu sortieren, dass meine Antworten einigermaßen aufeinander aufbauen.]

    > nebenbei, warum sprichst du eigentlich immer von Operatoren?

    Um nicht immer noch umständlicher zu schreiben:
    von jeweils einer bestimmten nachvollziehbaren Messmethode, mit der bestimmte Messwerte aus gegebenen Beobachtungsdaten zu ermitteln sind, und die einen bestimmten Wertebereich hat,
    und im Unterschied zu einem einzelnen bestimmten Messwert, der so aus den Beobachtungsdaten eines einzelnen Versuches ermittelt wurde, oder der erwartet wird.

    Ich war außerdem nicht ganz sicher, dass das Wort “Messgröße” allein wirklich nur im ersteren Sinn (als Operator, mit Wertebereich) aufgefasst wird.

    > Das ergibt doch in der klassischen Physik nicht viel Sinn, oder?

    Vermutlich hat man auch schon im Altertum erlebt, dass man Messwerte (z.B. “ich habe diese Runde beim Würfeln gewonnen”) nur insofern einvernehmlich und “einklagbar” erhalten kann, wenn die entsprechenden Spiel- bzw. Bewertungsregeln (“wie soll festgestellt werden, ob …”) von vornherein nachvollziehbar festlegt und dann beibehält.

    Das allgemeine Interesse an solcher Nachvollziehbarkeit, und die (mathematischen) Mittel, sie formal auszudrücken, haben sich natürlich nur allmählich entwickelt.

    > Implizit hast du übrigens auch angenommen, dass du bestimmen kannst, wann zwei Objekte relativ zueinander ruhen

    Ich meine, dazu hatte ich mich schon recht explizit und früh in der Diskussion geäußert — Frank Wappler schrieb (28.12.10 · 23:30 Uhr):
    >> Die Messdefinitionen, wie zu entscheiden wäre, ob gegebene Beteiligte zueinander ruhen (d.h. zueinander starr sind, und zueinander Euklidisch-flach liegen), oder nicht, und ob der Brechungsindex einer gegebene Region gleich 1 ist, oder nicht, gehören wohl zum Aufgabenbereich der ART.

    > auch dafür brauchst du zusätzlich zu c entweder eine Längen- oder eine Zeitdefinition.

    Nein. Die entsprechenden Beweise, im Rahmen der ART, “über Verhältnisse von Pings”, füllen ganze Bücher. Plausibel sollte aber Folgendes sein:
    ein bestimmtes Paar Enden, die zueinander ruhen, haben nicht unbedingt die gleiche Distanz voneinander wie ein anderes bestimmtes Paar Enden, die ebenfalls zueinander ruhen. Das eine Paar wäre ebenso als “Einheits”-Repräsentant zu gebrauchen, wie das andere; und insgesamt wäre die Auszeichnung des einen Paars gegenüber dem anderen nicht erforderlich, weil es (zumindest sofern schon gemessen wurde, welche Paare zueinander ruhten, und welche nicht) sowieso nur noch um Verhältnisse geht.

    > Was die Messgrößen angeht: Du bist der meinung, dass die SRT die Längeneinheit komplett neu definiert hat über “ping”s, richtig?

    Nein, ich meine dass die SRT irgendeine bestimmte Längeneinheit weder definiert, noch benötigt. (Nebenbei: ich nenne das (Aus-)Maß zweier zueinander ruhender Enden lieber “Distanz” als “Länge”, damit nicht die unzutreffende Assoziation mit “Kontraktion” aufkommt; ähnlich wie die oben diskutierte und von der PDG tabulierte mittlere Lebensdauer von Kaonen genau diese Kaonen selbst betrifft, und nicht die Dauern irgendwelcher Detektoruhren.)
    Die chrono-geometrische Definition besagt lediglich, dass der entsprechende Wert “c 1/2 Pingdauer” beträgt.

    > Heißt das, du bist der Meinung, dass die SRT sozusagen selbst den Wert von c setzt?

    Na sozusagen. Ich meine: wenn man unter Einsatz der entsprechenden Messdefinitionen (chrono-geometrische Distanzdefinition, Definition von Gleichzeitigkeit) den Wert der Geschwindigkeit bestimmt, mit der ein Signal zwischen zwei zueinander ruhenden Enden ausgetauscht wurde
    (und zwar gerade solch ein Signal, dass auch bei den “Pings” zur Distanzfeststellung und Gleichzeitigkeitsfeststellung zu berücksichtigen ist),
    dann erhält man (zwangsläufig) den Wert
    “1.0 c”.
    Dabei geht es ausschließlich um den Austausch des Signals, und nicht um den Austausch von irgendjemandem oder irgendetwas, der/das wiederum Signale austauschen müsste oder auch nur könnte.
    Dieser (zwangsläufige) Messwert der Signalgeschwindigkeit, “1.0 c” bzw. kurz “c”, wird deshalb auch (Wert der) “Licht”-Geschwindigkeit genannt.

    > Dann schließt deiner Meinung nach die SRT also die Maxwell-Gleichungen mit ein, denn nach denen ist c ja ebenfalls bestimmt durch c^2=1/eps0 mu0.

    Wodurch wäre(n) denn “eps0 mu0” (zusammen oder einzeln) sonst bestimmt?
    (Spielen dabei nicht u.a. geometrische Beziehungen eine große Rolle? …)

    > Ich kann übrigens eine Längenskala auch unabhängig von der SRT zum Beispiel über die Quantenmechanik definieren, beispielsweise als Radius des Bohr-Atoms. Das ist eine unabhängige Definition.

    Beim Versuch, etwas über “Radius des Bohr-Atoms” herauszufinden, stößt man u.a. auf
    https://de.wikipedia.org/wiki/Bohrsches_Atommodell#Atomgröße
    wo der Begriff “Geschwindigkeit v” (leider unverlinkt) auftritt.
    Welche Messgröße (bzw. welcher Messoperator) würde sich denn dahinter verbergen,
    insbesondere solang man noch mitten im Versuch steckte “eine Längenskala zu definieren”?

    p.s.
    In diesem Blog steht nun schon mehrfach:
    > “Zeit ist,[ w]as die Uhr misst”

    Einstein ist wohl der am häufigsten aus dem Zusammenhang gerissen oder vorsätzlich falsch zitierte Wissenschaftler des 20. Jahrhunderts.
    Wie oben schon einmal betont, findet sich in Ann. Phys. 17 (1905) Einsteins maßgebliche Bemerkung:

    […] daß ich an Stelle der „Zeit“ die „Stellung des kleinen Zeigers meiner Uhr“ setze

    nachzulesen z.B. in https://wikilivres.info/wiki/Zur_Elektrodynamik_bewegter_K%C3%B6rper

  468. #470 Name auf Verlangen entfernt
    14. Januar 2011

    @ Bullet: ” … und beginnst mit völlig neuen Themen.” Das Thema lautet im Augenblick und klar verständlich: “Was haben wir uns in der Physik unter ‘Nichtlokalität’ vorzustellen”. Wie üblich, kommst Du mit riesen Tam-tam, ad personam Blödsinn und null Argumenten.

    Du gehst offensichtlich davon aus, daß Deine bisherigen Irrtümer zukünftige irgendwie legitimieren. Ich muß Dich noch weiter enttäuschen, und Dir sagen, daß ich Deine Kommentare in Zunkunft der Einfachheit halber nicht mehr lese. Gehab Dich wohl!

  469. #471 Bullet
    14. Januar 2011

    @erwartungsgemäßer MT:

    Das Thema lautet im Augenblick und klar verständlich:

    Und schon beim nächsten Kommentar kanns anders sein. Wissen wir alle. Außerdem liest du sie sehr wohl noch, du alter Lügner. Du kannst vielleicht vermeiden, sie zu beantworten… aber lesen wirst du sie. Und wenn ich die anderen Kommentatoren bitten muß, meine Texte in ihre Posts mitaufzunehmen.
    Aber schön, daß du nicht eine einzige einfache Antwort hinbekommst.

  470. #472 erik
    14. Januar 2011

    Warum kochen die Emotionen nur so hoch? Worin liegt der Sinn, sich die Köpfe einzurennen?
    Worin liegt die Vernunft im gegenseitigen martern der Gehirnwindungen?

    Jede Seite gebirt Gedanken, um den anderen den Spiegel seiner Begrenztheit vorzuführen.
    In der Demonstration der Begrenztheit sehe ich einen Sinn.
    Gedanken kommen und gehen, sie selber können wir nicht messen. Was wir messen ist das Wirken von “Denken” auf die Biologie des menschlichen Körpers.
    “Denken” kann man nicht “be-greifen”, wir können Reaktionen auf unseren Körper messen.

    Selbiges Problem hat die QM und NM: den Raum der QM können wir nicht “be-greifen”, können aber an der Grenze von QM und NM Messungen durchführen und unser Denkvermögen trainieren.

    Ich möchte einmal eine “un-begreifbare” Aussage in den Raum stellen:
    Kann das Licht Christi in einem Menschen den Raum um ihn herum so formen (gestalten), wie es Licht mit dem Raum in der QM tut?

    Wenn ja, dann kann man die QM und NM an Hand der Geschichte der Bibel interpretieren und hätte somit auch einen Universellen Zusammenhang.

    Solche Gedanken gehen mir durch den Kopf und ich renne diesen nicht gegen andere Köpfe.

  471. #473 erik
    14. Januar 2011

    Ich befinde mich gerade wieder in einem Gedankenstrom.

    Wenn obige Aussage wahr wäre, dann würden Gegner von Religion auch die Wissenschaft von(Licht, Masse und Kraft und Temperatur), also (SRT / ART,) in Frage stellen.

    Wenn obige Aussage falsch wäre, dann würden alle Spezialisten ihres Faches nur eine Hälfte alles Seins leben können.
    Was ist ein halber Mensch schon wert? Er müsste sich dann eine bessere Hälfte suchen.
    Ein Professor müsste sich Professorinnen suchen, oder?

  472. #474 Name auf Verlangen entfernt
    14. Januar 2011

    @ erik: das sind natürlich die allerwichtigsten Gedanken, aber die wichtigste physikalische Formulierung in dieser hochinteressanten Diskussion zwischen Inhaltemeister und Frank Wappler ist dies:

    Heißt das, du bist der Meinung, dass die SRT sozusagen selbst den Wert von c setzt?
    Na sozusagen. Ich meine: wenn man unter Einsatz der entsprechenden Messdefinitionen (chrono-geometrische Distanzdefinition, Definition von Gleichzeitigkeit) den Wert der Geschwindigkeit bestimmt, mit der ein Signal zwischen zwei zueinander ruhenden Enden ausgetauscht wurde
    (und zwar gerade solch ein Signal, dass auch bei den “Pings” zur Distanzfeststellung und Gleichzeitigkeitsfeststellung zu berücksichtigen ist),
    dann erhält man (zwangsläufig) den Wert
    “1.0 c”.
    Dabei geht es ausschließlich um den Austausch des Signals, und nicht um den Austausch von irgendjemandem oder irgendetwas, der/das wiederum Signale austauschen müsste oder auch nur könnte.
    Dieser (zwangsläufige) Messwert der Signalgeschwindigkeit, “1.0 c” bzw. kurz “c”, wird deshalb auch (Wert der) “Licht”-Geschwindigkeit genannt.

  473. #475 MartinB
    15. Januar 2011

    @FrankWappler
    Sei mir nicht böse, aber ich gebe auf. Egal wie oft ich deine Kommentare lese, ich verstehe eigentlich immer noch nicht, worauf du hinauswillst oder warum die genau die Größen als gesetzt ansiehst, die du so ansiehst.
    Z.B. definierst du Strecken über ein c-Ping und eine Zeiteinheit. Ich halte eine Längendefinition für wesentlich einfacher als eine Zeitdefinition (jedenfalls im Prinzip).
    Ping-Signal mit Licht, schön und gut, kann man als Definition nehmen, aber warum soll diese Lichtgeschwindigkeit zwangsläufig auch eine Signalgrenzgeschwindigkeit sein? Das folgt aus der Theorie nicht und ich verstehe nach wie vor nicht, warum du meinst, man könnte ein überlichtschnelles Signal niemals messen.

    eps0 und mu0 sind natürlich aus der Elektro- und Mganetostatik bekannte Größen, die man gemessen hatte, bevor irgendjemand die Idee hatte, Licht könnte irgendwas mit Elektromagnetismus zu tun haben (von der SRT ganz zu schweigen).
    Dein Weltbild scheint irgendwie zu sein, dass man ohne das Gedankengebäude der SRT keine Physik betreiben könnte (aber irgendwie auch nicht, weil man ja auch alles im Newtonschen Weltbild erklären könnte, wenn ich dich richtig verstehe, was ich auch nicht nachvollziehen kann).
    Mir scheint, dass allein die Geschichte der Physik deine Ansicht widerlegt: Man hat den Wechsel von Newton zur SRT gemacht, und zwar durch Experimente getrieben. Insofern habe ich trotz der endlosen Diskussion hier eigentlich immer noch keine Ahnung, was du eigentlich sagen willst.

  474. #476 erik
    15. Januar 2011

    @MartinB
    Die Ansichten von @Frank Wappler werden von der Zukunft der Physik genauer beschrieben werden. So wird eher ein Schuh draus. Einen Schuh den sich die Füssiger anziehen werden, um nicht am Fleck zu verharren.
    – (kurzer Text – – zum überfliegen!)

  475. #477 Bullet
    15. Januar 2011

    Ach was. Hältst du dich für einen Propheten?

  476. #478 galileo2609
    16. Januar 2011

    @Martin Bäker,

    stell dich schon mal auf ein Personalgespräch morgen ein. 😉

    Grüsse galileo2609

  477. #479 Frank Wappler
    17. Januar 2011

    MartinB schrieb (15.01.11 · 10:42 Uhr):

    > Sei mir nicht böse, aber ich gebe auf. Egal wie oft ich deine Kommentare lese, ich verstehe eigentlich immer noch nicht, worauf du hinauswillst

    Sei mir bitte nicht böse, wenn ich darauf trotzdem antworte.
    Zunächst mit der Feststellung, dass mir die Diskussion leichter fiele, und wohl eher zu Ergebnissen führen würde, wenn du genauer zitieren würdest, welchen Formulierungen oder Argumentationsschritten du nicht folgst.

    > Z.B. definierst du Strecken über ein c-Ping und eine Zeiteinheit.

    Ich habe mich mehrfach auf die chrono-geometrische Distanzdefinition berufen,
    die von J. Synge (nach den 2. Weltkrieg) so benannt, von A. Einstein
    (als “der Erfahrung gemäß”) eingesetzt, und von O. Römer (im 17. Jh.)
    wohl schon in Betracht gezogen wurde; nämlich:
    den Wert der Distanz (zweier zueinander ruhender Beteiligter) als
    “c 1/2 PingDauer”
    darzustellen.

    Eine Einheit von Dauer (dem Maß von Zeit) kommt darin nicht vor.
    Was darin vorkommt, ist ein Buchstabe (sofern man dieser Konvention
    überhaupt folgt), “c”.

    Unter Einsatz der Messgrößen “Distanz” und “Dauer” lässt sich die Messgröße
    “Geschwindigkeit” definieren;
    und man erhält insbesondere für den Wert der Geschwindigkeit des
    Ping-Signal-Austausches des Wert “1.0 c”.

    Dieser spezielle Wert bietet sich folglich als eine Einheit für Geschwindigkeitswerte an.

    > Ping-Signal mit Licht, schön und gut, kann man als Definition nehmen, aber warum soll diese Lichtgeschwindigkeit zwangsläufig auch eine Signalgrenzgeschwindigkeit sein?

    Vor allem, weil unter “Ping” verstanden wird: nachdem man eine bestimmte Signalanzeige dargestellt hat, berücksichtigt man diejenigen Echos von den anderen Beteiligten, die man jeweils zuerst wahrnahm; nicht irgendwelchen “Nachhall”.
    (Eventuelle Dispersion von Echos ist ein Maß für das Vorhandensein von
    “Medien” in der Versuchsregion; also für Versuche, die nicht im Rahmen
    der SRT auszuwerten sind.)

    > Das folgt aus der Theorie nicht

    Das folgt sehr wohl:
    falls die beiden gegebenen Enden dabei zueinander ruhten,
    die Distanz zwischen den beiden gegebenen Enden chrono-geometrisch gemessen wurde,
    und die Dauer des Signalaustausches für beide Enden einvernehmlich festgestellt wurde, durch Anwendung von Einsteins Gleichzeitigkeitsdefinition zur einvernehmlichen Feststellung der Anzeigen zu Beginn des Signalaustausches, und zur einvernehmlichen Feststellung der Anzeigen zum Ende des Signalaustausches.

    Es fragt sich einmal mehr, was du unter der SRT verstehst …

    > eps0 und mu0 sind natürlich aus der Elektro- und Mganetostatik bekannte Größen, die man gemessen hatte, bevor irgendjemand die Idee hatte, Licht könnte irgendwas mit Elektromagnetismus zu tun haben (von der SRT ganz zu schweigen).

    Überlegungen zur Statik allein geben wohl kaum Anlass, “1/Sqrt[ eps0 mu0 ]” auszuwerten.

    Ansonsten kann ich in diesem Zusammenhang die (offenbar von dir vertretene) Idee des “Auswertens durch einfaches Hinzeigen” vielleicht so ins Wanken bringen:
    Stell dir die zwei Kerben vor, die du durch Hinzeigen als “Meter”-Repräsentation identifizierst. Nun stell dir elektrische Ladungen vor (der Einfachheit halber: gleich-“namige”), die auf beide Kerben verteilt und dort statisch gegenüber gehalten werden sollen. Kannst du dir so viel Ladung in den Kerben vorstellen, dass du das Kerbenpaar nicht mehr “Meter” nennen würdest?

    > Dein Weltbild scheint irgendwie zu sein, dass man ohne das Gedankengebäude der SRT keine Physik betreiben könnte

    Eher: dass, wenn man versucht, Physik zu betreiben, man zwangsläufig auf die RT kommt.

    > (aber irgendwie auch nicht, weil man ja auch alles im Newtonschen Weltbild erklären könnte, wenn ich dich richtig verstehe …)

    Ich kann nur vermuten, wie meine Bemerkungen so missverstanden werden konnten.
    Sicher: es gibt in unserer ziemlich kalten, flachen und neutralen Welt viele (mehr oder weniger auffällige) Kerbenpaare, deren chrono-geometrische Distanz zueinander mit recht hoher Genauigkeit gleich bleibt, selbst dann, wenn sie zwischendurch transportiert wurden. Aber das lässt sich eben erst durch Anwendung der chrono-geometrischen Distanzdefinition feststellen.

    > Mir scheint, dass allein die Geschichte der Physik deine Ansicht widerlegt: Man hat den Wechsel von Newton zur SRT gemacht, und zwar durch Experimente getrieben.

    Aber eben nicht getrieben von vorhandenen Messwerten, sondern von der Aussicht, durch nachvollziehbare experimentelle Methoden Messwerte zu erhalten.

    > was du eigentlich sagen willst.

    Gerne nochmal:
    Dass die chrono-geometrische Distanzdefinition und Einsteins Gleichzeitigkeitsdefinition und darauf aufbauende Messoperationen nicht experimentell falsifizierbar sind; weder durch Messwerte, die durch diese Messoperationen erhalten werden, und erst recht nicht durch andere.
    Und dass fraglich ist, was du unter “SRT” verstehst, wenn du das trotzdem behauptest.

  478. #480 MartinB
    17. Januar 2011

    @Frank
    Einmal versuche ich’s noch, vielleicht ja diesmal konkret genug.

    Eine Einheit von Dauer (dem Maß von Zeit) kommt darin nicht vor.
    Was darin vorkommt, ist ein Buchstabe (sofern man dieser Konvention
    überhaupt folgt), “c”.
    Unter Einsatz der Messgrößen “Distanz” und “Dauer” lässt sich die Messgröße
    “Geschwindigkeit” definieren;
    und man erhält insbesondere für den Wert der Geschwindigkeit des
    Ping-Signal-Austausches des Wert “1.0 c”.

    Wie meinst du es denn nun? Gibt es die Messgrößen Distanz und Dauer nun unabhängig von c oder nicht? Erst schreibst du, beim “ping” kommt Dauer als Maß von zeit nicht vor, dann sagst du, unter Einsatz von Dauer und Distanz lässt sich die Geschwindigkeit definieren.

    Überlegungen zur Statik allein geben wohl kaum Anlass, “1/Sqrt[ eps0 mu0 ]” auszuwerten.

    Nein, aber sie geben Werte für eps0 und mu0, die zunächst mal mit c nichts zu tun haben. Die Maxwellgleichungen stellen dann den Zusammenhang her. Die SRT allein kann deshalb den Wert von c nicht festlegen, es sei denn, du betrachtest die Maxwellgleichungen als Bestandteil der SRT.

    Ich kann nur vermuten, wie meine Bemerkungen so missverstanden werden konnten.

    Du hattest selbst geschrieben, dass man die Newtonsche Theorie nicht widerlegen kann.

    Aber eben nicht getrieben von vorhandenen Messwerten

    Kannst du das belegen? Eine Triebfeder war doch Michelson-Morley, also vorhandene Messwerte.

    Dass die chrono-geometrische Distanzdefinition und Einsteins Gleichzeitigkeitsdefinition und darauf aufbauende Messoperationen nicht experimentell falsifizierbar sind; weder durch Messwerte, die durch diese Messoperationen erhalten werden, und erst recht nicht durch andere.

    Das mag richtig sein, wenn man annimmt, dass es keine andere Möglichkeit gibt, Distanzen udn Dauern zu messen, aber das ist eben nicht richtig.

    Kannst du dir so viel Ladung in den Kerben vorstellen, dass du das Kerbenpaar nicht mehr “Meter” nennen würdest?

    Ja, kann ich. Aber was beweist das? Nehmen wir an, es gäbe ein einziges Urmeter auf der Welt und bei jedem Physikexperiment würden alle Physiker ihre Messgeräte an diesem eichen. Und eines nachts manipulierst du das Urmeter so, dass es sene Länge um 1% ändert. Glaubst du wirklich, dass man das nicht merken könnte? Plötzlich ändert sich die Schallgeschwindigkeit, die Größe von Atomen und so ziemlich alles. Es dürfte den Physikern nicht besonders schwer fallen, zu merken, dass da etwas manipuliert wurde – es würden zu viele Inkonsistenzen entstehen. (Genauso wie es möglich wäre, Änderungen von c zu detektieren, auch wenn c zur SI-Definition verwendet wird. – ist in deiner Logik eigentlich eine Änderung der Naturkonstanten mit der Zeit überhaupt möglich? Das wird ja von einigen Theorien angenommen.)

  479. #481 perk
    17. Januar 2011

    Dass die chrono-geometrische Distanzdefinition und Einsteins Gleichzeitigkeitsdefinition und darauf aufbauende Messoperationen nicht experimentell falsifizierbar sind; weder durch Messwerte, die durch diese Messoperationen erhalten werden, und erst recht nicht durch andere.
    Und dass fraglich ist, was du unter “SRT” verstehst, wenn du das trotzdem behauptest.

    ich war zwar nicht gefragt aber misch mich mal ein:
    ich verstehe unter der srt: die definition von größen (distanz dauer geschwindigkeit..) und relationen zwischen diesen.. zb ist die relation zwischen geschwindigkeiten in der srt eine andere als zwischen geschwindigkeiten in der newtonschen kinematik, und diese vorhersage wie geschwindigkeiten komponieren ist definitiv falsifizierbar (und wird von allen außer dir als bestandteil der srt angesehen)

    Sicher: es gibt in unserer ziemlich kalten, flachen und neutralen Welt viele (mehr oder weniger auffällige) Kerbenpaare, deren chrono-geometrische Distanz zueinander mit recht hoher Genauigkeit gleich bleibt, selbst dann, wenn sie zwischendurch transportiert wurden. Aber das lässt sich eben erst durch Anwendung der chrono-geometrischen Distanzdefinition feststellen.

    nein, damit überschätzt du die srt deutlich und unterschätzt die metallurgie ebenso… es ist möglich dass eine quantentheorie der gravitation in raumbereichen hinreichend großer krümmung eine bisher unbekannte wellenlängenabhängige kopplung des lichts an die gravitation enthält, wodurch die chronogeometrische distanzdefinition genauso zu einem spezialfall würde der nur für hinreichend flache raumbereiche gilt.. es gibt keine möglichkeit zu beweisen ab wann eine messdefinition zwangsläufig immer funktioniert und es gibt ebenso keine herleitung, dass die chronogeometrische die einzig anwendbare ist

    wir haben hier zwei möglichkeiten distanzen zu messen und für beide gigantisches erfahrungswissen aus konsistenz von ergebnissen wo diese korrekt anwendbar sind.. aber innerhalb der jeweiligen festlegung keine theoretische grundlage aus der wir schlussfolgern können wo sie scheitern werden

    aus 2 kerben im blech kann man nicht schlussfolgern dass in bewegten bezugssystemen probleme mit der gleichzeitigkeit und damit der nichtproperen maßdistanz entstehen.. aus der chronogeometrischen distanzdefinition kann man nicht schlussfolgern ob real existierendes licht nicht unter irgendwelchen umständen anders propagiert als nullgeodäten in einer pseudoriemannschen mannigfalt

    aber aus einer jeweils neuen theorie heraus kann man die ergebnisse besser erklären, die messdefinitionen ablösen und damit die alte theorie falsifizieren

    (soviel zu prinzipiellen falsifizierbarkeit des theorieanteils den du an der srt siehst.. zu der falsifizierbarkeit des theorieanteils den martin und ich noch sehen (die beziehung der definierten größen untereinander) hab ich ja vorhin schon geschrieben dass sie gegeben ist)

  480. #482 MartinB
    17. Januar 2011

    @perk
    Danke. Beruhigt mich, dass ich anscheinend nicht allein dastehe.

  481. #483 Name auf Verlangen entfernt
    17. Januar 2011

    @ Frank Wappler: “Eine Einheit von Dauer (dem Maß von Zeit) kommt darin nicht vor.
    Was darin vorkommt, ist ein Buchstabe (sofern man dieser Konvention
    überhaupt folgt), “c”.
    Unter Einsatz der Messgrößen “Distanz” und “Dauer” lässt sich die Messgröße
    “Geschwindigkeit” definieren;
    und man erhält insbesondere für den Wert der Geschwindigkeit des
    Ping-Signal-Austausches des Wert “1.0 c”.”

    ist, meine ich, “äquivalent”:

    ” … aus der Definition von Masse als Proportionalitätskonstante aus Kraft und Beschleunigung?- : m/s² – so ist der Meter in der SRT vorausgesetzt, während er gleichzeitig von ihr definiert wird.”

    Besonders bemerkenswert ist Ihre Definition der “Dauer” als “Maß der Zeit”.

  482. #484 nocheinPoet
    18. Januar 2011

    Hallo Martin, erstmal meine Annerkennung, sehr guter Text, und dann auch ein Lob für Deine Engelsgeduld im Umgang mit „weniger Wissenden“. Ich denke mal es interessiert hier keinen, aber der Text und die Kommentare haben bei der größten Logikerin aller Zeiten eine gewisse Verstimmung ausgelöst, wie man hier nachlesen kann: https://www.mahag.com/neufor/viewtopic.php?f=15&t=350

    Aber da die Dame eh bekannt ist, dient das wohl eher dem Amüsement.

  483. #485 Bullet
    18. Januar 2011

    die “Dame”? Na … das kann ja nur eine bestimmte sein. 🙂
    Und der Eintrag, der da verlinkt wird, ist wirklich lesenswert.
    Ich möchte das übrigens unterstützen: wer sich, wie “Die Dame”, so eindeutig überall im deutschsprachigen Netz auf diese Weise präsentiert, muß sich nicht über Kommentare in Blogs wundern, in denen den Aussagen “Der Dame” nicht mehr Relevanz zugeordnet wird als einem Fleck, den eine Fliege nach dem Zerdrücktwerden hinterläßt. “Die Dame” ist offensichtlich nicht in der Lage, bei vernünftigen Menschen den Eindruck der Seriosität zu erzeugen. Üblicherweise ist solches aber ein Kinderspiel.
    Ich habe mal den Namen entfernt, damit sich niemand bestimmtes angesprochen fühlt

  484. #486 MartinB
    18. Januar 2011

    @Bullet, nochEinPoet
    Es hat einen grund, warum gewisse namen unseren Spamfilter aktivieren.
    Es wäre deshalb nett, wenn ihr dies nicht unterlaufen würdet…

    PS: Aber danke für den Hinweis (nächstes Mal vielleicht per mail?) auf die Beschwerde an den Uni-Präsidenten, davon wusste ich bisher nichts.

  485. #487 Bullet
    18. Januar 2011

    Okay.
    Ich habe darauf geachtet, korrekte Formulierungen zu verwenden. Aber du darfst natürlich editieren, wie du magst. Nicht daß es noch Folgen hat. 🙂

  486. #488 nihil jie
    19. Januar 2011

    dieser “offene” brief, den ich da auf der verlinkten seite gelesen habe ist absolut lächerlich. aber was will man gegen menschen machen die alle hebel in bewegung setzen wenn sie sich nicht genug ernst genommen fühlen. sie haben doch wohl unter dem begriff “meinungsfreiheit” nichts falsch verstanden.. oder doch ? heisst das bei denen, dass man inzwischen auch die justiz bemühen darf wenn man für seinen mist, den man ungeniert in der öffentlichkeit verbreitet, nicht gebührend gelobt und ernstgenommen wird ?

  487. #489 nihil jie
    19. Januar 2011

    aber keine panik… die besagte person schreibt wohl gerne breife an diverse unis *gg den guten Thilo Kuessner von Mathlog (hier bei ScienceBlogs) auch schon ereilt 😉

    Re: Demagogie im T. Küssner-Blog?

  488. #490 Bruno
    19. Januar 2011

    Bravo

  489. #491 MartinB
    19. Januar 2011

    @alle
    Lasst uns hier keine Meta-Diskussionen führen, das bringt ja eh nichts.

  490. #492 Frank Wappler
    20. Januar 2011

    MartinB schrieb (17.01.11 · 08:40 Uhr):

    > [Frank Wappler schrieb (17.01.11 · 00:35 Uhr):
    > > MartinB schrieb (15.01.11 · 10:42 Uhr):
    > > > Z.B. definierst du Strecken über ein c-Ping und eine Zeiteinheit.

    > > … “c 1/2 PingDauer” …]
    > > Eine Einheit von Dauer (dem Maß von Zeit) kommt darin nicht vor.

    > Erst schreibst du, beim “ping” kommt Dauer als Maß von zeit nicht vor

    Hab ich nicht (s.o.).
    Du hast eine (bestimmte Maß-) Einheit unterstellt, und ich habe dem widersprochen.
    Verstehst du den Unterschied zwischen einem bestimmten (Mess-)Wert und einer bestimmten Maßeinheit, “in” der ein Messwert ggf. ausgedrückt werden kann?

    > > [Die Entwicklung der SRT war] eben nicht getrieben von vorhandenen Messwerten[, sondern von der Aussicht, durch nachvollziehbare experimentelle Methoden Messwerte zu erhalten.]

    > Kannst du das belegen? Eine Triebfeder war doch Michelson-Morley, also vorhandene Messwerte.

    Welche(n) Messwert(e) wäre(n) das denn gewesen?
    Etwa: Brechungsindex (unter der Haube mit M&Ms Versuchsanordnung) n = 1? …
    Und in jedem Fall: wieso hätte ein bestimmter Messwert (sofern dieser überhaupt erhalten wurde) die Theorie in Frage gestellt, durch deren Anwendung er erhalten worden wäre??

    Nein — die RT setzt (letztlich) wesentlich gründlicher an:
    Ausgehend davon, dass jeder Beteiligte (“wie du und ich”) zumindest im Prinzip entscheiden kann ob “das Zeigen des kleinen Zeigers meiner Uhr auf 7 und das Ankommen des Zuges” entweder koinzident wahrgenommen wurde, oder “das eine nach dem anderen”,
    ist zu untersuchen, wie Beteiligte in Anwendung ihrer beobachterischen Fährigkeit einvernehmliche Aussagen über ihre Beziehungen untereinander finden können.

    (Und falls sich jemand überlegt: im Prinzip alles gut und schön, aber die Auflösung zwischen “koinzident” oder “eins-nach-dem-anderen” ist doch nicht beliebig scharf — der kommt dann offenbar auf die DSR.)

    > > Kannst du dir so viel Ladung in den Kerben vorstellen, dass du das Kerbenpaar nicht mehr “Meter” nennen würdest?

    > Ja, kann ich. Aber was beweist das?

    Dass sich Artefakte wie “das Urmeter” kaum eignen, um (einvernehmliche, reelle) Messwerte in Experimenten zur Elektrostatik zu erhalten.

    > Nehmen wir an, es gäbe ein einziges Urmeter auf der Welt und bei jedem Physikexperiment würden alle Physiker ihre Messgeräte an diesem eichen. Und eines nachts manipulierst du das Urmeter so, dass es sene Länge um 1% ändert.

    … heißt das: die chrono-geometrische Distanz zwischen den beiden Kerben soll sich verringert haben? …

    > Glaubst du wirklich, dass man das nicht merken könnte? Plötzlich ändert sich die Schallgeschwindigkeit, die Größe von Atomen und so ziemlich alles. Es dürfte den Physikern nicht besonders schwer fallen, zu merken, dass da etwas manipuliert wurde – es würden zu viele Inkonsistenzen entstehen.

    Du scheinst in der Erwartung befangen, dass in verschiedenen Versuchen gleiche Werte von Schallgeschwindigkeit zu finden sein sollten, oder gleiche Werte der Feinstrukturzahl usw.
    Der Zweck nachvollziehbarer Messdefinitionen ist aber, Messwerte auch unter Umständen zu ermitteln, die nicht “innerhalb der Alltagserwartungen/-vorurteile” liegen; sich Messwerten auch dann sicher zu sein, falls sie unerwartet wären.

    > (Genauso wie es möglich wäre, Änderungen von c zu detektieren, auch wenn c zur SI-Definition verwendet wird.

    Sicherlich ist der Brechungsindex zwischen gegebenen Beteiligten Versuch zu Versuch messbar, und nicht unbedingt gleich.
    Hängt deine Behauptung, die SRT sei experimentell falsifizierbar, etwa davon ab?

  491. #493 TheBug
    20. Januar 2011

    @FW: Du schmeißt ja immer noch Alles durcheinander.

    Messwerte für eine bestimmte Größe sind in Ihrer Bedeutung unabhängig davon wie die Maßeinheit definiert ist. Wir können auch Kellicams pro Centronen messen und kommen dann bei der Lichtgeschwindigkeit zwar auf einen anderen Zahlenwert, aber der bedeutet immer noch das Gleiche wie der in m/s oder Furlongs per Fortnight.

    Was hat Michelson Morley mit Brechungsindex zu tun?

    Das Ergebnis von Michelson Morley wich von der Erwartung ab, falsifizierte den Äther und führte zur SRT.

    Ich messe Gewicht nicht mit dem Thermometer, könntest Du jetzt bitte aufhören das Urmeter sinnlos unter Strom zu setzen?

    Versuche unter gleichen Bedingungen sollten zu identischen Resultaten kommen, andernfalls sollte man den Messaufbau und die Bedingungen überprüfen. Das gilt sogar für Versuche im Quantenbereich, so fern man eine statistisch signifikante Anzahl an Einzelversuchen bzw. Quanten betrachtet. Nennt sich Reproduzierbarkeit, sind viele interessante Dinge dran gescheitert, wie z.B. die “kalte Fusion”.

    Ein reproduzierbarer Messwert der den Erwartungen widerspricht ist dagegen eine grundlegend interessante Sache, weil der zu neuen Entdeckungen führen kann.

  492. #494 Frank Wappler
    20. Januar 2011

    perk schrieb (17.01.11 · 15:51 Uhr):

    > zb ist die relation zwischen geschwindigkeiten in der srt eine andere als zwischen geschwindigkeiten in der newtonschen kinematik

    Die Messdefinition, gemäß der “geschwindigkeit in der srt” ausgewertet wird, ist jedenfalls nachvollziehbar:
    Start-“Tor” und Ziel-“Band” stellen einvernehmlich ihre (chrono-geometrische) Distanz zueinander fest,
    und beide stellen einvernehmlich fest, welche Anzeige des Ziel-“Bands” gleichzeitig zur Start-Anzeige war, bzw. welche Anzeige des Start-“Tors” gleichzeitig zur Ziel-Anzeige war;
    d.h. beide stellen einvernehmlich ihre Dauer des Austausches “von Start- bis Ziel-Anzeige” fest.

    Die Geschwindigkeit (dessen, was zwischen Start-“Tor” und Ziel-“Band” ausgetauscht wurde) ist dann als
    “Distanz / Dauer_des_Austausches” ==
    “c 1/2 PingDauer / Dauer_des_Austausches”
    auszuwerten.

    Aber was soll “geschwindigkeit in der newtonschen kinematik” sein? —
    etwa: “Geschwindigkeit in der SRT, sofern die Näherung v viel kleiner als c eingehalten ist”??

    > es gibt keine möglichkeit zu beweisen ab wann eine messdefinition zwangsläufig immer funktioniert

    Eine Messdefinition (d.h. ein bestimmter Messoperator) funktionierte in einem bestimmten Versuch, falls aus den in diesem Versuch gegebenen Beobachtungsdaten durch Anwendung des betreffenden Operators ein bestimmter Messwert erhalten wird. Was sonst?

    > es gibt ebenso keine herleitung, dass die chronogeometrische die einzig anwendbare ist

    Gibt es denn andere Messdefinitionen, die nicht mehr voraussetzen, als dass sich bestimmte Beteiligte (Enden) gegenseitig beobachten können (so wie alle anderen Beteiligten im Prinzip auch)?

    > aus der chronogeometrischen distanzdefinition kann man nicht schlussfolgern ob real existierendes licht nicht unter irgendwelchen umständen anders propagiert als nullgeodäten in einer pseudoriemannschen mannigfalt

    Zweifellos kann “Brechungsindex” gemessen werden.
    (Und das ist nicht einfach. Ist das hier ein Thema?)

    > aus einer jeweils neuen theorie heraus kann man die ergebnisse besser erklären, die messdefinitionen ablösen und damit die alte theorie falsifizieren

    Stimmst du zu, dass die jeweilige Messdefinition, “wie” Ergebnisse zu erhalten sind,
    von allen Erklärungs-Modellen zu unterscheiden ist, “warum” die gefundenen Ergebnisse (z.B. in Korrelation zueinander) so gefunden wurden, und welche Werte in weiteren Versuchen zu erhalten wären?

    Stimmst du zu, dass Messdefinition und auch die damit erhaltenen konkreten Ergebnisse nachvollziehbar sein und bleiben sollten, unabhängig davon, ob man sie erwartet hat oder sie (in Korrelation mit anderen Messwerten) “gut erklären” könnte?

  493. #495 nihil jie
    20. Januar 2011

    @MartinB

    da hast Du wohl recht… lieber keine metadiskusionen. aber der umstand den ich hier kurz angesprochen habe ergab sich als ich mir die links genauer angeschaut habe und da auf den seiten herumgestöbert hatte 😉

  494. #496 MartinB
    20. Januar 2011

    @FW
    “Du scheinst in der Erwartung befangen, dass in verschiedenen Versuchen gleiche Werte von Schallgeschwindigkeit zu finden sein sollten, oder gleiche Werte der Feinstrukturzahl usw.”
    Jawohl. Andernfalls wäre die Konsistenz der Messergebnisse bei Verwendung verschiedener Methoden nicht zu erklären. Es *sollten* dieselben Ergebnisse herauskommen – wenn sie das nicht tun, dann wird man unterschiedliche Messmethoden anwenden, um zu sehen, ob das Problem in der Messmethode oder einer veränderlichen Natur”konstante” liegt.

    “Und in jedem Fall: wieso hätte ein bestimmter Messwert (sofern dieser überhaupt erhalten wurde) die Theorie in Frage gestellt, durch deren Anwendung er erhalten worden wäre??”

    Ein bestimmter Messwert (keine Änderung des Interferenzmusters) *hat* die Theorie in Frage gestellt, die hier überprüft werden sollte (nämlich die Äthertheorie), da ist kein Konjunktiv vonnöten.

    In der Praxis misst man Längen übrigens ja selten über Lichtlaufzeiten, jedenfalls nicht auf kleiner Skala – da gibt es Mikrometerschrauben oder Rasterkraftmikroskope, die können auf Nanometer genau Oberflächen vermessen (was mit Licht wegen der Wellenlänge gar nicht geht – auf der Skala sollte nach deiner Logik Länge gar nicht definiert sein können, es sei denn ich pinge mit harter Gamma-Strahlung, beim Durchmesser des Protons dann mit kosmischer Strahlung…)

    Unsere Auffassungen von Physik unterscheiden sich meiner Ansicht nach fundamental in folgender Weise: Du gehst davon aus, dass jede physikalische Messgröße genau eine einzige Definition mit einem einzigen Messverfahren besitzt. Ich dagegen sehe viele unterschiedliche Messmethoden, die alle zum Verständnis der Messgröße beitragen – wenn eines der Verfahren problematisch ist, wird man das an den anderen merken. Für dich sitzt die Messgrößendefinition an der Spitze eines Betonpfeilers, für mich sitzt sie oben auf einem Tragwerk aus vielen einzelnen Pfeilern, von denen keiner so fest ist wie dein Betonpfeiler, die aber untereinander eine große Redundanz besitzen.

  495. #497 Basilius
    20. Januar 2011

    @MartinB
    Danke Martin, das war sehr schön veranschaulicht. So sehe ich das auch, aber ich hätte es jetzt nicht so schnell so klar formulieren können.

  496. #498 Frank Wappler
    24. Januar 2011

    MartinB schrieb (20.01.11 · 17:31 Uhr):

    > [Ich erwarte] Konsistenz der Messergebnisse bei Verwendung verschiedener Methoden […]
    > Es *sollten* dieselben Ergebnisse herauskommen – wenn sie das nicht tun, dann wird man unterschiedliche Messmethoden anwenden, um zu sehen, ob das Problem in der Messmethode oder einer veränderlichen Natur”konstante” liegt.

    Die Nachvollziehbarkeit jeder einzelnen Messmethoden, und die daraus resultierende Kommensurabilität von (durch Anwendung dieser selben Messoperation gewonnenen) Messwerten aus verschieden Versuchen, und die darauf basierende Möglichkeit, Messwerte von verschiedenen Messmethoden über mehrere Versuche hinweg überhaupt auf ihre Konsistenz oder Inkonsistenz hin beurteilen zu können,
    kann und muss von vornherein gedanken-experimentell nachgewiesen werden.

    Wenn du also verschiedene Messmethoden hinreichend durchdacht und ausgewählt hast (hättest?), und du aus dem Vergleich einiger Versuche die Konsistenz der entsprechenden Messwerte erhalten hast, und du im folgenden Versuch Werte ermittelst, die der Konsistenz-Erwartung widerprechen,
    dann wäre es absurd, die eine oder andere oder diese beiden Messmethoden für “falsch” zu erklüren und zu verwerfen, und damit alle entsprechenden bisher erhaltenen Werte und insbesondere die Feststellung von bisheriger Konsistenz einzustampfen.

    Ansonsten, sofern wirklich unterschiedliche Messmethoden betrachtet werden, dann lassen sich eben schlicht und einfach Beobachtungsdaten (von vornherein) denken bzw. simulieren bzw. ggf. auch (tatsächlich) entsprechend auffinden,
    aus denen die unterschiedliche Messmethoden ungleiche Messwerte ermitteln, oder aus denen sich mit der einen Messmethode sich ein bestimmter Messwert ermitteln lässt, aber mit der anderen eben nicht.

    Das lässt sich sehr deutlich auch erkennen, wenn man, wie mittlerweile unter Physikern üblich, gegebene Beobachtungsdaten (eines Versuches) als Element (Strahl) eines Hilbertraumes darstellt, bestimmte Messmethoden bzw. Messgrößen als (Hermitesche) Operatoren und die zu erhaltenden Messwert als die entsprechenden Eigenwerte:

    Falls die (beiden) betrachteten Messoperatoren ungleich dahingehend sind, dass sie noch nicht mal vertauschbar sind, dann existiert ein Eigenzustand des einen Operators, der nicht Eigenzustand des anderen ist.
    Und falls die Messoperatoren vertauschbar sind, dann sind sie nur dann auch ungleich, falls mindestens einen gemeinsamer Eingenzustand existiert, zu dem diese Operatoren ungleiche Eigenwerte besitzen.

    > Ein bestimmter Messwert (keine Änderung des Interferenzmusters) *hat* die Theorie in Frage gestellt, die hier überprüft werden sollte (nämlich die Äthertheorie), da ist kein Konjunktiv vonnöten.

    Eine “Äthertheorie“, die diesen Namen verdiente, sollte insbesondere die entsprechende Begriffs- bzw. Messdefinition beinhalten, wie denn zumindest im gedanken-experimentellen Prinzip festzustellen sei, ob in einem bestimmten Versch “Äther vorlag”, oder nicht; und zwar ausdrücklich ohne dabei vorauszsetzen, was noch gemessen werden soll, nämlich: das Vorhandensein oder die Abwesenheit von “Äther“.

    Erst auf Grundlage einer solchen nachvollziehbaren Theorie und Messdefinition wären überhaupt bestimmte entsprechende Modelle formulierbar — z.B. Modelle, die den Messwert enthalten: “in diesem Versuch (M&Ms) lag in der Versuchsregion Äther vor”; oder Modelle, die den Messwert enthalten: “in diesem Versuch (M&Ms) lag in der Versuchsregion kein Äther vor”.

    Ein Nachweis, inwiefern “Newtonsche Physik“, die wir bislang (im Kontrast zur SRT) diskutiert haben, eine solche “Äthertheorie” darstellt, wäre noch zu erbringen.

    Aus Sicht der SRT (mit entsprechenden Messwerten zur Geometrie der Versuchsanordnung von M&M) wäre eine Änderung des Interferenzmusters jedenfalls ein Maß für den Brechungsindex der Versuchsregion.
    Und — “Äther” hin oder her — zum Brechungsindex in ihrer Versuchsregion haben Michelson und/oder Morley offenbar keine Messwerte mitgeteilt.

    > In der Praxis misst man Längen übrigens ja selten über Lichtlaufzeiten, jedenfalls nicht auf kleiner Skala

    Richtig: in der Praxis ist der definitionsgemäße Einsatz von Messoperatoren oft viel zu aufwändig. Man begnügt sich mit Abschätzungen und Bewertungen des entsprechenden “Vertrauensbereiches” des Schätzwertes (d.h. der wahrscheinlichen Genauigkeit des Schätzwertes bzgl. des Messwertes, den man durch Auswertung des entsprechenden Messoperators erhalten würde).

    Entscheidend ist: man muss sich dafür eben doch nachvollziehbar und von vornherein festlegen, wie man den entsprechenden Messwert zumindest im gedanken-experimentellen Prinzip ermitteln würde; hier: chrono-geometrisch.

    > Unsere Auffassungen von Physik unterscheiden sich meiner Ansicht nach fundamental in folgender Weise: Du gehst davon aus, dass jede physikalische Messgröße genau eine einzige Definition mit einem einzigen Messverfahren besitzt.

    Ganz genau; und ganz entsprechend der oben skizzierten Darstellung von Messgrößen als Operatoren “im Hilbertraum”.

    > Ich dagegen sehe viele unterschiedliche Messmethoden, die alle zum Verständnis der Messgröße beitragen

    Dann hast du von der Physik der letzten ca. 100 Jahre offenbar noch jede Menge zu lernen …

  497. #499 MartinB
    25. Januar 2011

    @FrankWappler
    “Dann hast du von der Physik der letzten ca. 100 Jahre offenbar noch jede Menge zu lernen ”
    Mag sein (ist sogar ziemlich sicher…) – in diesem konkreten Fall teile ich diese Eigenschaft immerhin mit Feynman, der ja auch explizit gegen eine axiomatische Physik war und die Physik mit der Mathematik der Babylonier, nicht der Griechen, verglichen hat.

  498. #500 Bullet
    25. Januar 2011

    OT: Schön aber, daß wenigstens der GröLaZ sich wieder verzogen hat.

  499. #501 Frank Wappler
    27. Januar 2011

    MartinB schrieb (25.01.11 · 08:40 Uhr):

    > Feynman, der ja auch explizit gegen eine axiomatische Physik war

    Als bald nach Feynmans Ableben sein Büro in Augenschein genommen wurde, fand sich auf der Wandtafel (u.a.) die Bemerkung

    what I cannot create
    I cannot understand.

    (Die Fotografie dieser Bürotafel, wie sie nach Feynmans Tod vollgeschrieben war, ist z.B. enthalten in “Physics Today” irgendwann Ende der 80er, WIMRI. Wär’ nett, wenn jemand das genaue Heft wüsste und mitteilen würde; sonst kram ich vielleicht nochmal in meinen Stapel …)

    what I cannot create I cannot understand.” … das scheint mir gradezu die Maxime der axiomatischen Physik zu sein.

    Aber dennoch könnte deine Einschätzung Feynmans richtig sein (und sonst wäre unsere Diskussion wohl kaum so mühsam) —
    denn, wie ich in diesem Zusammenhang schon immer (seit Anfang der 90er) hervorgehoben habe:
    es ist wohl bisher keineswegs erwiesen/gemessen, dass es tatsächlich Feynman war, der diese interessante Bemerkung auf Feynmans Bürotafel hinterließ.

  500. #502 MartinB
    27. Januar 2011

    @FW
    Lies übrigens mal das erste Kapitel vom Misner Thorne Wheeler zum Thema, wie man Zeit definiert: “Time is defined to make motion look simple”. Auch da ist wenig axiomatisches dabei, man definiert die Zeit immer so, dass möglichst viele Phänomene möglichst einfach werden…

    PS: Du hast ein in meinen Augen ungewöhnliches Verständnis des Wortes “create”.

  501. #503 Frank Wappler
    27. Januar 2011

    MartinB schrieb (27.01.11 · 07:47 Uhr):

    > Lies übrigens mal das erste Kapitel vom Misner Thorne Wheeler zum Thema, wie man Zeit definiert: “Time is defined to make motion look simple”.

    Ja — die drei kochen eben auch nur mit Wasser.
    (Allerdings habe ich einen ganz bestimmten Verdacht, wer davon ganz besondere Funken sprühte.)

    Wenn schon (Korinthenkacken), dann eher:
    “It looks good to make simple motion look simple.”

    Ansonsten halte ich eben am meisten davon, wie Einstein Zeit definiert (s.o.);
    und wie er durch weitere Festsetzungen auf eine demgemäße Maßzahl (Verhältnisse von Dauern) kommt.

    > man definiert die Zeit immer so, dass möglichst viele Phänomene möglichst einfach werden…

    Hmm …
    Wir denken möglicher Weise an (knallhart) verschiedene Begriffe:
    Du wohl an “Zeit” in der Dynamik, d.h. insbesondere wie “Zeit” im Prinzip der stationären Wirkung auftritt, durch das man (mathematisch optimal) unterscheidet, welche demnächst zu erlangenden Befunde “einfach so zu erwarten” wären, oder welche “eher überraschend” wären.

    Ich denke dagegen ganz sicher an “Zeit” in der Kinematik (sowie darauf aufbauend natürlich auch an “Dauer” in der Kinematik usw.) — MBMN so wie Einstein;
    also insbesondere daran, wie man aus bloßen Beobachtungs-Phänomenen eines bestimmten Versuches überhaupt ersteinmal (geometrische) Bewertungen gewinnt, die man für anderen (kommende) Versuchen (die wiederum ihre ganz eigenen Beobachtungs-Phänomene mit sich bringen) überhaupt zum Vergleich einsetzen könnte, um überhaupt zu beurteilen “ob man _so_ erwartet hat”, oder “gerade nicht _so_”.

    Der Clou ist aber:
    ohne “meinen” Begriff ernstzunehmen und einzusetzen, käme “dein” Begriff gar nicht erst zum Zuge.
    (D.h., solang es eben nicht nur um bloße Rechnerei geht, einschließlich des Nachweises der Optimalität des Vorhersage-Verfahrens.)

    p.s.
    > Du hast ein in meinen Augen ungewöhnliches Verständnis des Wortes “create”.

    Also wenn ich mir einen Physiker vorstelle
    (dessen Arbeit im Wesentlichen darin besteht, Gedanken-Experimente zu entwickeln damit sich seine Kollegen zunehmend deutlicher mitteilen können, “was sie gemacht, und was sie gefunden haben”)
    — mal ganz unabhängig davon, ob es sich dabei um Feynman gehandelt haben sollte, oder um einen hypothetischen anderen Jemand —
    und dass dieser Physiker in Feynmans Büro ansetzt, mit ‘nem Stück Kreide an die Tafel das Wort “create” zu malen
    (damit er sich sein Vorhaben auch merkt, und sich ggf. mit Hilfe des Tafelschwamms berichtigen könnte, falls er sich bei der Umsetzung seines Vorhabens doch wider Erwarten vermalen sollte) …
    … passt schon.

  502. #504 MartinB
    27. Januar 2011

    Sorry, den teil nach dem Hmm… kann ich wieder nicht nachvollziehen.

    Bezüglich des “create”: Aus dem Wort zu schließen, dass jemand Axiomatiker ist, scheint mir halt etwas ungewöhnlich, mehr wollte ich nicht sagen.

  503. #505 Frank Wappler
    27. Januar 2011

    MartinB schrieb (27.01.11 · 18:17 Uhr):

    > den teil nach dem Hmm… kann ich wieder nicht nachvollziehen.

    Soso:

    Frank Wappler schrieb (27.01.11 · 11:34 Uhr):
    >> Wir denken möglicher Weise an (knallhart) verschiedene Begriffe:
    >> Du wohl an “Zeit” in der Dynamik […]
    >> Ich denke dagegen ganz sicher an “Zeit” in der Kinematik

    Bist du (wenigstens schon mal) bis
    https://de.wikipedia.org/wiki/Kinematik und darin

    […] Ihr Gegenstück ist die [[Dynamik (Physik)|Dynamik]]

    gekommen,
    oder nicht mal so weit?

    > Aus dem Wort zu schließen, dass jemand Axiomatiker ist, scheint mir halt etwas ungewöhnlich

    Worte “an sich” zu setzen, ohne sie (gedanklich) zu verlinken, erschiene mir etwas ungewöhnlich. Viel mehr ist dazu kaum zu sagen …

  504. #506 Name auf Verlangen entfernt
    28. Januar 2011

    @ MartinB: “Time is defined to make motion look simple” – warum haben Sie sich dann so dagegen gewehrt, als ich behauptete, Zeit existiert nicht, sondern nur Bewegung?

  505. #507 MartinB
    28. Januar 2011

    @MT
    Lesen Sie den MTW, Kap. 1, da wird das erklärt.

    @FW
    “Viel mehr ist dazu kaum zu sagen …”
    Prima, dann können wir diese sinnlose “Diskussion” ja vielleicht beenden.

  506. #508 Name auf Verlangen entfernt
    28. Januar 2011

    @ MartinB: so sinnlos war sie nicht. Wir haben immerhin gelernt, das die RT nicht mit theoretischen Mitteln der RT selbst falsifizierbar ist, wozu sogar die Längendefinition gehört. Dabei ist ja Frank Wappler ein ganz offener Anhänger ihres Systems. Doch in dem Augenblick, wo Sie so freundlich sein könnten, aus dieser durchaus vernünftigen Sicht heraus eine erkenntnistheoretische Lehre zu ziehen, und sich also zu fragen: “Was ist dann die RT?- ein hinreichendes Welterklärungssystem auch außerhalb unserer Maßstäbe oder ein geschickt geknüpfter magischer Kurzschluss?” – bezeichnen sie lieber ihre eigene Diskussion als sinnlos. Für eine sinnlose Diskussion haben wir hier zuviel interessantes Argument und Gegenargument gelesen.

  507. #509 MartinB
    28. Januar 2011

    @MT
    Nein, wir haben gelernt, dass Frank Wappler das so sieht – damit steht er allerdings ziemlich allein da. Aber es war ja zu erwarten, dass Sie diese Diskussion so hindrehen, dass sie das aussagt, was Sie gern hören wollen – wenn Sie das mit Planetenkonstellationen können, ist es mit Diskussionen natürlich ein Kinderspiel.

  508. #510 MartinB
    28. Januar 2011

    @MT
    Vielleicht habe ich Ihren Text falsch gelesen. Sie haben geschrieben:
    “das die RT nicht mit theoretischen Mitteln der RT selbst falsifizierbar ist”
    Wenn Sie das wirklich gemeitn haben, dann ist es trivialerweise richtig (ich hatte das Wort “theoretisch” übersehen) – keine physikalische Theorie kann durch theoretische Mittel widerlegt werden (es sei denn, sie wäre logisch inkonsistent). Auch wenn ich selbst theoretischer Physiker bin – die Physik bleibt eine experimentelle Wissenschaft.
    Irgendein erkenntnistheoretischer Gewinn ergibt sich aus dieser Trivialität allerdings auch nicht.

  509. #511 rolak
    28. Januar 2011

    /Irgendein…nicht/ Aber sicher doch – nur eben nicht für den Bereich der Naturwissenschaften.

  510. #512 MartinB
    28. Januar 2011

    @rolak
    Ist mein Humordetektor im Eimer? Hab ich gerade nicht verstanden…

  511. #513 rolak
    28. Januar 2011

    Wenn auch nicht aus der Trivialität an sich, so doch allein aus ihrer Verwendung läßt sich imho einiges an Erkenntnis über das gewinnen, was der Verwender für sinnvolle Argumentation bzw Möglichkeit zum Erkenntnisgewinn hält.

  512. #514 MartinB
    28. Januar 2011

    @rolak
    O.k., aber ob ich das gleich einen “erkenntnistheoretischen Gewinn” nennen würde, die 100000te Bestätigung zu erhalten, dass MT eine eigenwillige Vorstellung von Logik und Argumentation hat…

  513. #515 rolak
    28. Januar 2011

    Oh Mist, natürlich kein “Gewinn für die Erkenntnistheorie” im Sinne einer Päzisierung oder wie auch immer. Wenn es so gemeint war, bedarf dieser Kommentar von mir einer Korrektur. Ich hatte es als “Gewinn im Sinne der Erkenntnistheorie” gelesen á la

    Von Interesse ist hier vor allem, welche Art von Zweifel an welcher Art von Wissen grundsätzlich bestehen kann.

  514. #516 rolak
    28. Januar 2011

    Auch das noch^^ s/Päzisierung/Präzisierung/
    Zuckerrausch wg gespendeter Nappo 😉

  515. #517 erik
    3. Februar 2011

    Kommentar-Direktlink Bullet· 15.01.11 · 22:04 Uhr

    “Ach was. Hältst du dich für einen Propheten?”

    Für die Antwort auf deine Frage habe ich mir Zeit genommen. Hier meine Antwort:

    Es ist nicht schwer Dinge vorher zu sehen, die von einem selbst geplant werden. Ich habe einige Dinge in “petto” die ich der Physik antuen möchte.
    Jeder Mensch hat so seine Dinge im Leben die ihn beschäftigen. Mein Thema ist die Physik und im speziellen die Ordnung aller Dinge im Universum.

    Ist deine Frage ausreichend beantwortet?

  516. #518 Wolfgang Graßmann
    24. Februar 2012

    Ein super Artikel, keine Frage.

    Es gibt halt mehr als die drei oder vier ‘klassischen Tests’
    War mir so detailiert auch nicht klar.

  517. #519 MartinB
    25. Februar 2012

    @Wolfgang
    Nein, so explizit hatte ich das vorher auch nicht gewusst – auch wenn einem natürlich klar ist, dass die SRT in vielen Theorien drinsteckt.

  518. #520 Frank Wappler
    27. Februar 2012

    Wolfgang Graßmann schrieb (24.02.12 · 22:04 Uhr):
    > […] die drei oder vier ‘klassischen Tests’

    Nämlich ?
    “Die SRT ist falsch, falls …” — was ??

  519. #521 MartinB
    27. Februar 2012

    @FW
    Jaja, wissen wir alle, man kann die SRT gar nicht widerlegen, blablabla, Austausch von Ping-Signalen, blabla Messoperator, blabla, zwei Beobachter erzielen Einigkeit blablabla – been there, done that, got the T-shirt…

  520. #522 rolak
    27. Februar 2012

    So einfach kannst Du das aber nicht abtun, MartinB, da fehlt noch ‘laber, keine Gleichzeitigkeit, schwall..’.

  521. #523 MartinB
    27. Februar 2012

    @rolak
    Das war doch impliziert im “Austausch von Ping-Signalen” – außerdem heißt das nicht einfach “keine Gleichzeitigkeit” – du musst schon sagen, dass “zwei an verschiedenen Orten befindliche Beobachter keine Einigkeit über den einvernehmlichen Wert des Gleichzeitigkeitsoperators erzielen können” oder so ähnlich…

  522. #524 rolak
    27. Februar 2012

    /du musst schon sagen/ Da kannste ma sehn, was bei mir tumbem Laienspieler von dem langen Satz ankommt: Eben das eben intonierte ‘laber, keine Gleichzeitigkeit, schwall..’

    Vielleicht lohnt es sich langsam, ein neues Akronym einzuführen: RTUT. Also nicht R-TUT wie realtime Ägyptologie, sondern RT-U-T wie Relativitätstheorie-Unverständnis-Theorie. Einerseits betrachtend die Geschwindigkeit des Erklärungsflusses relativ zum gesunden Menschenverstand, die Korrelation der dort auftretenden Beschleunigungen zum Zeitpunkt des Absturzes in absurdes Behauptungstheater (da fallen dann zB JL mit ihrem starren Stab ohne Längenkontraktion oder HT mit seinen stündlich nachgestellten Uhren beim GPS drunter), andererseits staunend über die anderen Techniken.
    Speziell die von FW meisterlich vertretene Zunft derer, die ob es Grausens über das, was aus der Realität über deren Erklärung folgt lässig schließen, daß die Realität nicht existent sein kann. Zumindest nicht innerhalb der uns bekannten Physik, nein, alles falsch, her mit etwas Neuem. Hamwa jrad nich da? Egal, denken wir uns halt schnell was aus und verpacken es unter möglichst vielen Schichten fein ziselierter, mit Buzzword-Rosinen durchsetzter Unverständlichkeit (das Konzept ist freilich nicht neu, Standardmethodik zB auch bei CAM [nicht CAD/CAM]). OK, es ist einzusehen, daß Menschen wie FW oder Heim sich ernsthaft Mühe geben und höchstwahrscheinlich ein ungemein hohes Arbeitspensum erledigen – doch das gilt auch für Sisyphos.

    Könnte wg Homophonie auch zu einem Default-Kommentar verwertet werden:

    RTUT es schon wieder^^

    😉

  523. #525 Basilius
    27. Februar 2012

    @rolak
    Also eigentlich fand ich ja Wapplers Kommentar/Frage bezogen auf seine übliche Vorgehensweise geradezu unverschämt kurz gefasst. Man könnte schon fast geneigt sein zu sagen: geradezu provokativ.

    Könnte wg Homophonie auch zu einem Default-Kommentar verwertet werden: RTUT es schon wieder^^;-)

    Die Idee finde ich prinzipiell sehr charmant. Ich glaube, das würde ich auch gerne benutzen. Allerdings sollten wir uns schon einvernehmlich darüber einigen, bevor wir eine solche Aussage treffen können…
    °_^

  524. #526 rolak
    27. Februar 2012

    bezogen auf seine übliche

    1. Tagesform
    2. eigentlich nur fragend indirekt verlinkend

    Und das mit dem default-comment bezog sich nicht auf FW speziell, sondern auf RTUT­relevante Phänomene im Allgemeinen. Wobei noch anzumerken wäre, daß mir dieses (nicht ganz unernst gemeinte) Konzept erst vor einer guten Stunde beim Resteessen einfiel. Viel Bohnen Paprika Zwiebeln Kartoffeln mit einer Kalbsrippe als Grundge­schmacks­geber – das hatte ich schon den ganzen Tag vorfreudig in der Nase — ich schweife ab.

    Die Krux ist ja, daß die Antwort auf die titelige Frage im letzten Abschnitt des posts längst gegeben worden ist: Ein klares eriwansches Ja, aber… Alles klar erklärt. Alles? Neihein, in einer kleinen wapplerschen Welt etc etc Und statt auf die Argumente bzw Aussagen des posts einzugehen, werden Strohmanner aufgebaut, nur um sie im nächsten Satz ermahnend abzufackeln. ‘Da müssen wir uns aber erst einmal einigen, was dieses Komma an jener Stelle für eine Bedeutung..’-‘DieseUndJene’-‘Ist »Und« ein angemessener, falls überhaupt existenter Sprachoperator?’ ad lib. diffusum

  525. #527 Basilius
    27. Februar 2012

    @rolak·

    Und das mit dem default-comment bezog sich nicht auf FW speziell, sondern auf RTUT­relevante Phänomene im Allgemeinen.

    Ja, schon klar. Ich hätte das nicht anders verstehen wollen. Ist wie mit dem Bullet’schen Form&Inhalt. Das will ich ja auch für alle Fälle benutzen können, wo es passt und nicht ausschließlich für den Kommentator, von dem ausgehend das Prädikat mal ursprünglich appliziert wurde. RT-U-T können ja doch noch ein paar mehr Menschen von sich geben, als nur der werte Herr Wappler.

  526. #528 rolak
    27. Februar 2012

    will ich ja auch für alle Fälle benutzen können

    Falls das eine Frage sein soll: Klar doch!

  527. #529 Frank Wappler
    28. Februar 2012

    MartinB schrieb (27.02.12 · 15:18 Uhr):

    > Jaja, wissen wir alle, man kann die SRT gar nicht widerlegen

    Wolfgang Graßmann (24.02.12 · 22:04 Uhr) wusste das wohl (noch) nicht;
    und was er seitdem dazugelernt haben sollte, drückt er hier bisher durch (eher disharmonisches) Schweigen aus.

    p.s.
    > been there, done that, got the T-shirt…

    So you’ve been to geometry class — and all you’ve got is a lousy T-shirt (?) …

  528. #530 gant
    Deutschland
    1. Februar 2013

    Es gibt keine Eigenschaft der Welt, die man vom Standpunkt des gesunden Menschenverstandes aus nicht begreifen könnte
    Relativitätstheorie, die spezielle wie die allgemeine, verworfen und vergessen werden müssen, obwohl die moderne Physik darauf aufbaut (…) Die RT hat eine neue, relativistische Weltsicht geschaffen. Halten wir drei ihrer Kennzeichen fest: sie leugnet den gesunden Menschenverstand, verwirft jeden Zweifel an der Richtigkeit der RT und hat eine imaginäre Welt geschaffen. Die gesamte Physik der Mikro- und Makrowelt ist auf dieser Weltsicht aufgebaut. Weil man sich laut RT nicht an den gesunden Menschenverstand halten darf, sind die Wahrheitskriterien verloren…Wir (hingegen) sind überzeugt, daß es auf der Welt nichts Jenseitiges und Unerklärliches gibt

  529. #531 MartinB
    1. Februar 2013

    Mit anderen Worten: Wir verwenden das Pippi-Langstumpf-Prinzip “Ich mach mir die Welt,….”

    Ich find’a ja traurig, wenn man so sehr an seinem ach so gesunden menschenverstand hängt, dass man sich nicht von Fakten überzeugen lassen mag, aber zumindest belegt es die These, die ich hier diskutiert habe:
    https://scienceblogs.de/hier-wohnen-drachen/2012/11/22/angst-vor-der-wissenschaft-i-was-kann-ich-wissen/

  530. #532 gant
    21. Februar 2013

    warum die Relativitätstheorie unlogisch ist
    1. Physikalisches Grundprinzip der speziellen Relativitätstheorie ist die Hypothese der invarianten Lichtgeschwindigkeit. Jeder Beobachter soll die Lichtgeschwindigkeit stets mit konstant c messen, gleich ob er sich auf die Lichtquelle zu bewegt oder sich von ihr entfernt. Doch nach mathematischer und physikalischer Logik kann eine Geschwindigkeit in Bezug auf unterschiedlich bewegte Beobachter niemals den selben Wert haben. Die Invarianz der Lichtgeschwindigkeit stützt sich allein auf das berühmte Michelson-Morley-Experiment, welches jedoch unterschiedliche Deutungen zulässt und das durch andere Experimente relativiert wird.

    2. Einstein betrachtet zwei Punkte A und B im bewegten System. Aus der Invarianz der Lichtgeschwindigkeit folgt, dass ein ruhender Beobachter einen von A ausgehenden Lichtimpuls später in B eintreffen sieht als der bewegte Beobachter, das heißt, dass der ruhende Beobachter alle physikalischen Vorgänge im bewegten System langsamer ablaufen sieht. Die scheinbare Verzögerung bemisst sich nach dem Lorentzfaktor.
    Diese Überlegung setzt voraus, dass sich das Licht für den ruhenden und den bewegten Beobachter nach unterschiedlichen Naturgesetzen ausbreitet. Für den bewegten Beobachter soll das Licht im bewegten System mitgeführt werden, so dass er die Lichtgeschwindigkeit mit c misst. Dagegen soll aus Sicht des ruhenden Beobachters das Licht im bewegten System nicht mitgeführt werden, so dass er ebenfalls die Lichtgeschwindigkeit mit c feststellt. Es gibt aber nur eine Wirklichkeit. Warum sollten für die Lichtausbreitung unterschiedliche, vom Beobachter abhängige Naturgesetze gelten?

    3. Es ist logisch und tatsächlich ausgeschlossen, dass ein und derselbe Lichtimpuls an einem konkreten Ort B zu unterschiedlichen Zeiten eintrifft. Sowohl der bewegte als auch der ruhende Beobachter beobachten das selbe Ereignis, nämlich das Eintreffen des Lichtimpulses in B. Der einzige Unterschied besteht darin, dass der ruhende Beobachter den Ort B als C bezeichnet.

    4. Wenn wir trotzdem unterstellen, dass die Invarianz der Lichtgeschwindigkeit zutrifft, was wäre dann? Dem ruhenden Beobachter wäre bekannt, dass er die Vorgänge im bewegten System infolge der für ihn längeren Lichtsignaldauer zwischen A und B langsamer ablaufen sieht. Er würde, sofern er die Relativgeschwindigkeit des bewegten Systems kennt, seine Beobachtungen durch Anwendung des Lorentzfaktors korrigieren. Das Ergebnis: in beiden Systemen ist der Verlauf physikalischer Vorgänge und damit auch der Gang der Uhren gleich.

    5. Wenn der ruhende Beobachter die Relativgeschwindigkeit des bewegten Systems nicht kennt, so kann er zwar im konkreten Fall die “Zeitdilatation” nicht berechnen. Aber er weiß, dass nach der Relativitätstheorie zwischen bewegten Inertialsystemen stets eine scheinbare Verzögerung nach Maßgabe des Lorentzfaktors zu beobachten ist, und dass dies ein reziproker (wechselseitiger) Effekt ist. Daraus folgt, dass in Wirklichkeit in keinem der Systeme die physikalischen Vorgänge langsamer ablaufen.

    6. Infolge der aus der invarianten Lichtgeschwindigkeit resultierenden unterschiedlichen Lichtlaufzeiten scheint eine bewegte Uhr für den ruhenden Beobachter langsamer zu gehen. Doch Einstein behauptet, dies sei ein wirklicher Effekt. Die Umdeutung von Sinneseindrücken zur Wirklichkeit ist insofern entscheidend, als daraus folgt, dass von zwei unterschiedlich bewegten Uhren jede gegenüber der anderen tatsächlich nachgeht. Weil dies logisch und tatsächlich ausgeschlossen ist (Langevin 1911), ist die Relativitätstheorie rein logisch und ohne Experiment auf einfache Weise offenkundig widerlegt. Um die Theorie zu retten, wurde der Begriff Uhrenparadoxon bzw. Zwillingsparadoxon erfunden.

    7. Einstein macht die Zeit von Sinneseindrücken abhängig. Weil die Sinneseindrücke des ruhenden und des bewegten Beobachters infolge der invarianten Lichtgeschwindigkeit unterschiedlich sind, soll Zeit relativ sein. Zeitrelationen, das heißt Zeitspannen, sind aber von Natur aus nicht relativ, weil die Abstände in der Aufeinanderfolge von Ereignissen feste Größen sind, die lediglich durch unterschiedliche Sinneseindrücke relativ erscheinen.

    Zum Schluß kann man feststellen: Aus so vielen konstitutiven (grundlegenden) logischen Fehlern der Relativitätstheorie folgt zwangsläufig, dass sie mit dem gesunden Menschverstand nicht zu verstehen ist.

  531. #533 MartinB
    22. Februar 2013

    Ach geh woanders spielen, das ist jetzt echt zu doof (Punkt 1: Nach Logik kann die Welt nicht so sein, wie sie ist – dann musst du wohl mal deine Logik überprüfen…)
    Und wenn dein angeblich so gesunder menschenverstand nicht ausreicht, dann musst du den auch mal auf den Prüfstand stellen.

  532. #534 gant
    22. Februar 2013

    Ein Gedankenexperiment: „Wenn durch hohe Geschwindigkeiten die Zeit sich dehnt und zusätzlich die Längen sich kürzen dann müsste sich doch die Geschwindigkeit erhöhen (durch kürzeren Weg) und wenn die Geschwindigkeit sich erhöht verkürzen sich weiter automatisch die Längen und so weiter und so fort“. Diese Methode ist nicht nur zeit- sondern auch noch energiesparend um das Universum zu erforschen. Wenn die NASA von diesem Prinzip erfährt hat sie in keiner Hinsicht mehr Probleme und könnten Heute noch zum nächsten Fixstern oder auch zur nächsten Galaxie fliegen und am Wochenende zur Grillparty wieder zu Hause sein.

  533. #535 gant
    22. Februar 2013

    “Sie stellen es sich so vor, dass ich mit stiller Befriedigung auf ein Lebenswerk zurückschaue. Aber es ist ganz anders von der Nähe gesehen. Da ist kein einziger Begriff, von dem ich überzeugt wäre, dass er standhalten wird, und ich fühle mich unsicher, ob ich überhaupt auf dem rechten Wege bin.”
    Albert Einstein, Nobelpreisträger

  534. #536 Eeerich
    27. Februar 2013

    Relativitätstheorie als unrichtig entlarvt

    Manch einer hat es schon immer geahnt. Aber nun ist eine neue Prämisse aufgetaucht, die die Relativitätstheorie in einem sehr schlechten Licht erscheinen lässt. Auf leicht verständliche Art werden einem die Verrücktheiten der Relativitätstheorie vermittelt und eine völlig neue Erklärung geboten, warum für das Licht stets die gleiche Geschwindigkeit gemessen wird. So einfach, dass schlaue Köpfe schon früher auf diesen Gedanken kommen hätten müssen. Nachzulesen in der Neuerscheinung ISBN 978849039173-0: „Die Welt der Relativität – alles falsch? Korrekturen zur Relativitätstheorie“ mit Leseprobe unter Amazon

  535. #537 MartinB
    27. Februar 2013

    @gant
    Das “Gedankenexperiment” ist ein schlechtes Wortspiel, mehr nicht.
    Und ein Einstein-Zitat gibt es auch für alles – das beweist genau nichts. (Und dass die SRT möglicherweise irgendwann als Grenzfall in einer anderen Theorie aufgehen wird, wird an den experimentellen Befunden zur Zeitdilatation etc. nichts ändern.)

    @Eerich
    Oh, toll, ich soll ein Buch kaufen?
    Und da stehen so tolle Sachen drin wie die, dass Einstein die Existenz schwarzer Löcher ablehnte? (Was hat das mit der Richtigkeit der Theorie zu tun?)
    Das unverstandene Zwillingsparadoxon (nein, es ist nicht relativ, wer beschleunigt) macht die Sache auch nicht besser. Wer soll denn für so einen Blödsinn 20 Euronen ausgeben? (Gut dass Amazon den Blick ins Buch erlaubt…)

  536. #538 gant
    28. Februar 2013

    Das ist die Rückkehr zu einem vorphilosophischen mythischen Denken, “als der bildhaften Artikulation einer Erfahrung … als der Versuch, durch eine bewußt bildhafte Deutung der Welt dem bereits mythenlosen, aufgeklärten Denken einen neuen “mythischen” Sinn mit aller Verbindlichkeit entgegenzusetzen.”* Demgemäß bekannte Einstein, dass bei seinem Denken “Worte dabei keine Rolle spielen” und er eher “ein assoziatives Spiel mit mehr oder weniger klaren Bildern” treibt.

  537. #539 koi
    28. Februar 2013

    @gant, @Eeerich
    Danke für die Kommentare zur RT und den Link zu dem Buch bei Amazon. (Eigentlich meide ich solche Links wegen Zeitverschwendungsgefahr, aber diese Dunning Kruger Interpretation hat echten Unterhaltungswert) Manchmal brauch ich das, wenn ich einen schlechten Tag gehabt habe und denke dass es nimmer schlimmer geht: alles ist relativ. Auch das Universum.

  538. #540 MartinB
    1. März 2013

    Es ist immer wieder faszinierend, dass gerade die RT-Leugner glauben, dass Einstein-Zitate irgendeine tiefere Bedeutung hätten. Es ist vollkommen egal, ob Einstein irgendwann gesagt hat, dass der Mond aus grünem Käse besteht, das ändert an der Richtigkeit der Theorie genau nichts, weil diese experimentell verifiziert ist (dazu gibt es ja oben einen Artikel).

  539. #541 Slobo
    1. März 2013

    Zu dem Buch: Die Welt der Relativität – alles falsch?

    Diese neuen Thesen sehen tatsächlich richtig aus.
    Keine logischen Fehler zu finden.
    Alles passt zusammen. Großartig!

    Moderationshinweis: Die IP-Adresse dieses Kommentars ist mit der von Eerich identisch. Sockenpuppen sind nun wirklich peinlich.

  540. #542 gant
    1. März 2013

    @MartinB” Es ist immer wieder faszinierend, dass gerade die RT-Leugner glauben, dass Einstein-Zitate irgendeine tiefere Bedeutung hätten.”

    Das klingt ja wie ein Titel des neuen Artikels!

  541. #543 der grosse schweiger
    8. März 2013

    Warum machte Einstein solche Aussagen, wenn sie ohnehin nicht stimmen?
    Und diese Formel mit der relativistischen Wurzel ist nichts anderes als der Lehrsatz von Pythagoras. Was soll denn dieser Irrsinn, den Einstein hier angerichtet hat? Einstein kann doch diesen Lehrsatz nicht auf Geschwindigkeiten anwenden! Der gilt doch nur für Längen! Da kann ja nichts Richtiges dabei herauskommen! Das ist doch völlig meschugge!

  542. #544 Erich S.
    8. März 2013

    Ganz genau. Der Cosinus-Satz c² = a² + b² – 2 ab . cos Gamma entspricht dem pythagoräischen Satz, wenn der Winkel 90 Grad ist. Bei 90 Grad ist der Cosinus null. Deshalb bleibt c² = a² + b² übrig. a, b und c sind Strecken, während v und c bei der relativistischen Wurzel Geschwindigkeiten sind. Deshalb kann jedes Rechnen mit der relativistischen Wurzel nur kompletter Wahnwitz sein. Dass dies über 100 Jahre niemandem aufgefallen ist, ist ein Armutszeugnis für die gesamte Physikgemeinde.

  543. #545 MartinB
    8. März 2013

    “Dass dies über 100 Jahre niemandem aufgefallen ist, ist ein Armutszeugnis für die gesamte Physikgemeinde.”

    Ein Geisterfahrer? Nein, Hunderte!!!

    Das mit dem Satz von Pythargoras ist jetzt echt so doof, dass es wieder lustig ist.

  544. #546 rolak
    8. März 2013

    Rechnen mit der relativistischen Wurzel

    Wie schnell ist die Möhre?

    Ja, MartinB, eindeutig lustig.

  545. #547 Not-Einstein
    8. März 2013

    @MartinB #15
    Ich bin etwas ratlos.
    Wenn das Licht einer gekrümmten Bahn entlangläuft, kann man dann sagen, der Raum ist gekrümmt? Wo kommen wir mit so einer Aussage hin?

    Dann kann man dann ja auch bei einem beliebigen krummen Gegenstand sagen, der Raum ist an dieser Stelle gekrümmt!

  546. #548 MartinB
    9. März 2013

    @Not-Einstein
    Wenn das eine ernst gemeinte Frage sein soll – rechts bei Artikelserien gibt es einen Link zu meiner Serie über Raumkrümmung, wo das ausführlich erklärt ist.
    Falls das nur wieder dämliche Wortspiele eines REalitätsverweigerers sind – geh woanders spielen.

  547. #549 der grosse schweiger
    11. März 2013

    Albert Einstein hatte kaum mathematische Fähigkeiten, hatte nicht mal ein Abitur und durfte in Deutschland nicht studieren. Er schrieb die Berechnungen Poincares zur Äthertheorie ab, übernahm Theorien anderer Personen und behauptete danach, dass alle Überlegungen von ihm selbst stammten. Eine Lügen- und Plagiatsorgie, ein Mixtum aus verschiedenen Theorien auf dem Gerippe der Äthertheorie, die nachweislich falsch ist. Sogar Guttenberg wirkt wie ein Unschuldslamm und verblasst neben diesem überheblichen Ignoranten.

  548. #550 Erich S.
    11. März 2013

    Dass Einstein so hochgejubelt wurde, liegt ja am Röntgen, der Einstein immer als Genie des 20. Jh. bezeichnet hat. Röntgen soll ja eine Wette laufen gehabt haben, indem er ….. Zitat: „einen dahergelaufenen Spinner zum Nobelpreisträger hochmanipulieren“ kann. Röntgen hat die Ideen Einsteins nie geglaubt, ihm nur Honig ums Maul geschmiert.

  549. #551 MartinB
    11. März 2013

    @schweiger, Erich
    Es ist vollkommen egal, ob Einstein zum Frühstück kleine Hundebabies gegessen hat. Entscheidend ist, dass seine Theorien überprüft und bestätigt wurden, siehe den Artikel oben.
    Warum machen eigentlich Kreationisten und RT-Leugner imm so einen Anti-Personen-Kult? Weil ihr die grundlegenden Regeln der Wissenschaft nicht versteht? Autoritäten sind egal.

    Hier ein sehr schönes Zitat von Feynman (nicht, weil er eine Autorität ist, sondern weil es einfach stimmt):
    “In general we look for a new law by the following process. First we guess it. Then we compute the consequences of the guess to see what would be implied if this law that we guessed is right. Then we compare the result of the computation to nature, with experiment or experience, compare it directly with observation, to see if it works. If it disagrees with experiment it is wrong. In that simple statement is the key to science. It does not make any difference how beautiful your guess is. It does not make any difference how smart you are, who made the guess, or what his name is – if it disagrees with experiment it is wrong.”

  550. #552 Erich S.
    11. März 2013

    Nichts von Einstein ist bestätigt. Es treten Effekte auf, die wie eine Raumkrümmung erscheinen. Aber diese Effekte lassen sich ganz anders erklären.

    Die Zeitdehnung existiert zwar, hat aber mit der Einsteinschen Relativitätstheorie nur am Rande zu tun.

  551. #553 MartinB
    11. März 2013

    @Erich
    Ja klar, Und das können wir in dem tollen Buch bei Amazon nachlesen, das schon in der Leseprobe völlig falsch ist.

  552. #554 Erich S.
    11. März 2013

    Völlig egal, ob der Inhalt dieses dämlichen Buchs richtig oder falsch ist. Der Raum ist generell leer. Warum sollte der Raum, das All gekrümmt sein? Da müssten viel eher alle Objekte auf der Erde wegen der enormen Erdkugel unter unseren Füßen gekrümmt sein. Warum erscheint jeder Würfel, jedes Gebäude auf der Erde perfekt und regelmäßig? Antwort: Weil es keine Krümmung des Raums gibt.

    Aber weit draußen im All, wo gähnende Leere herrscht, weit weg von einer Galaxie soll sich der Raum krümmen, weil das Licht angeblich gebogen, gekrümmt wird (siehe Gravitations-Linseneffekt), obwohl weit und breit nichts existiert, um das ein Lichtstrahl gebogen werden könnte.

    Licht hat keine Masse. Warum sollte sich Licht wegen einer weit entfernten Masse (Galaxie) krümmen oder biegen? Dies ergibt keinen Sinn.

    Es stellt niemand in Frage, dass das Licht die bekannten Bewegungen nicht ausführt. Aber die Einsteinsche Erklärung dazu kann nicht richtig sein.

  553. #555 Adent
    11. März 2013

    @Erich S.
    Nicht labern, belegen.
    @Martin B
    Mit dem Anti-Personenkult gegen Einstein geht aber komischerweise immer einher, daß er so oft zitiert wird (aus dem Zusammenhang gerissen), daß er eigentlich Zeit seines Lebens nur Zitate abgesondert haben muß, um diese Frequenz zu erklären.
    Und noch zwei Gemeinsamkeiten bei den Einstein Widerlegern sind, KEINER hat auch nur ansatzweise eine besser erklärende und veröffentlichte Hypothese. Der grosse Schweiger nicht, der Erich S. nicht, der Thim nicht, der Derksen nicht und wie sie alle heißen, dafür haben sie meistens, wie du schon sagtest, ihren geistigen Sondermüll bei Amazon abgeladen, seltsam seltsam.

  554. #556 MartinB
    11. März 2013

    @Erich
    Und siehe da, der Raum auf der Eroberfläche (besser gesagt, die Raumzeit) ist tatsächlich gekrümmt, wenn auch sehr wenig.
    “Licht hat keine Masse.”
    Falsch. Licht hat keine Ruhemasse, aber Photonen haben selbstverständlich eine effektive Masse gemäß E=mc².
    “Dies ergibt keinen Sinn.”
    Ist aber experimentell belegt. Wer gewinnt wohl, wenn dutzende experimentelle Belege gegen das “Sinngefühl” eines ErichS stehen?

    @Adent
    naja, ein guter Teil der Zitate sind oft falsch, und die anderen sind immer wieder dieselben (oft aus dem Zusammenhang gerissenen). Ist bei Darwin auch so.

  555. #557 Adent
    11. März 2013

    @Martin B
    Exakt das meinte ich, soviele Zitate konnte nicht mal Einstein liefern 😉

  556. #558 Not-Einstein
    11. März 2013

    @MartinB #548
    Ja, stimmt schon, ich bin bzgl. Raum-Zeit-Krümmung “ratlos”, aber zugegeben, war schon etwas polemisch.
    Sorry
    ————
    Ich wäre sehr dankbar, wenn mir jemand sagen könnte, wie man diese Kommentare formatieren kann, Italic, Fett, Zitate, Hyperlink etc. ?
    Besten Dank im Voraus!

  557. #559 MartinB
    12. März 2013

    @Not-Einstein
    wie gesagt, wenn’s ernst gemeint ist, dann gibt es rechts die Serie zur Raumkrümmung.
    Für die Formatierung gelten irgendwelche html-codes; es gibt auch ein greasemonkey-plugin, dazu findest du den Link irgendwo bei astrodicticum.

  558. #560 Erich
    12. März 2013

    „Licht hat keine Ruhemasse.“ Richtig, weil Licht immer in Bewegung ist.

    „Aber Photonen haben eine effektive Masse gemäß E=mc².“ Falsch, weil E=mc² keine klassische Formel ist. Etwas, das Masse hat, kann Lichtgeschwindigkeit nicht erreichen. Deshalb ergibt mc² keinen Sinn.

    Die ursprüngliche Formel war: E = m . v . v (wobei: Impuls = m . v )
    beziehungsweise: E = m . v²

    Ausschließlich, wenn Masse eines Objekts in Energie übergeht, ergibt E = mc² einen Sinn. Aber nur, wenn man wissen will, wie viel Energie ein Objekt besitzt.

    z. B: Ein Objekt mit der Masse von 1 kg hat die Energie von

    E = 1 kg . 299792458 m/s . 299792458 m/s = 8,987551e+16 Newtonmeter

    E=mc² ist nur eine Umrechnung, um den Energiewert eines Objekts zu erhalten. Oder: Wenn ein Objekt ganz in Energie übergeht, erhält man über E=mc² seine Energiemenge. Ausschließlich dazu dient E=mc². Dies bedeutet aber nicht, dass Photonen Masse haben. Ich wüsste nicht, dass jemals eine Masse für Photonen gemessen worden wäre. Es wäre auch unlogisch.

  559. #561 MartinB
    12. März 2013

    “Etwas, das Masse hat, kann Lichtgeschwindigkeit nicht erreichen. ”
    Ruhemasse und Masse sind nicht dasselbe. Das sollte man nicht verwechseln.

    “Es wäre auch unlogisch.”
    Ach so, dann ist ja gut, wenn’s der Erich sich nicht vorstellen kann und es unlogisch findet.
    Das war sicher auch früher schon so:
    “Wie soll denn Holz Wärme erzeugen? das ist doch unlogisch, Holz ist doch nicht warm.”
    “Wie soll denn ein Stück Holz ein anderes schneller schleudern können als ich es selbst kann? Das ist doch unlogisch.”
    “Wie soll denn etwas ohne Beine sich schnell fortbewegen können? Räder sind unlogisch!”
    usw. ad nauseam.

  560. #562 Erich
    13. März 2013

    Wenn die Photonen eines Lichtstrahls laut Martin B. Masse haben, wie soll dann das Licht Lichtgeschwindigkeit erreichen? Da Licht bewiesenermaßen mit Lichtgeschwindigkeit unterwegs ist, ergibt sich nach der Formel

    m‘ = m/SQRT (1 – v²/c²) =
    m/SQRT (1 – c²/c²) =
    m/SQRT (1 – 1) = m/0 = unendlich

    eine unendlich große Masse für die Photonen. Wenn aber die Masse unendlich groß ist, kann die Lichtgeschwindigkeit nicht erreicht werden. Die Schlussfolgerung: Entweder haben die Photonen keine Masse oder die Formel ist falsch. Und wenn die Formel falsch ist, wäre die Relativitätstheorie falsch.

    Wenn man aber weiß, dass die Formel m‘ = m/SQRT (1 – v²/c²) aussagt, dass mit zunehmender Geschwindigkeit die Masse m‘ ansteigt, wird es noch absurder. Obwohl sich an der Materie (die die Masse repräsentiert) nichts geändert hat, soll sich aus unerklärlichen Gründen die Masse erhöhen. Es ist egal, ob eine Rakete mit 50 km/s oder mit 298.000 km/s durch den Kosmos fliegt, weil es an der Menge der Materie, aus der die Rakete besteht, nichts ändert. Warum soll um Einsteins Willen die Masse zunehmen? Mit steigender Geschwindigkeit nimmt die Wirkung oder die Druckkraft (bei einem kosmischen Zusammenstoß) zu, aber nicht die Masse. Die Masse der Rakete ist unverändert.

    Wenn sich jemand auf der Erde auf eine Waage stellt, wird die Gewichtskraft gemessen ….. z. B. 784 Newton. Weil auf der Erde alle Waagen gleich geeicht sind, wird der Newton-Wert durch 9,8 m/s² dividiert. Für die Masse ergibt sich der Wert von 80 kg (m = F/a).

    Wenn sich dieselbe Person auf dem Mond auf eine Waage stellt, wird eine Gewichtskraft von 129,76 Newton gemessen. Eine Waage von der Erde ist auf dem Mond nutzlos, weil sie falsch geeicht ist. Eine richtig geeichte Mondwaage muss den Gewichtswert durch 1,622 m/s² dividieren, was der Fallbeschleunigung auf dem Mond entspricht. Als Ergebnis erhält man wieder die Masse von 80 kg. Die Masse ist überall gleich, egal ob auf dem Mond, im Kosmos, bei Flügen mit 50 km/s oder mit 298.000 km/s.

    Als nächstes wird behauptet, dass auch die Temperaturen relativistisch beeinflusst sind. Es gibt keine relativistischen Einflüsse. Nirgendwo im Kosmos. Und die Zeitdilatation hat mit der Relativitätstheorie nichts zu tun. Das ist es, was die Einstein-Freaks nicht verstehen. Einstein ist deren Gott und die Relativitätstheorie ist deren Religion. Alle Experimente, die die Relativitätstheorie angeblich bestätigt haben, sind falsch (interpretiert) oder bestätigen nur eine richtige Tendenz …….. meist im Zusammenhang mit der Zeitdilatation, die auch niemand abstreitet. Aber dadurch ist die Relativitätstheorie nicht bestätigt.

    Ebenso beim Gravitations-Linseneffekt: Martin B. argumentiert, dass wir die Krümmung des Raums auf der Erde nicht messen können, weil sie so minimal ist. Obwohl wir von anderen Planeten, Sternen und schwarzen Löchern umgeben sind, uns mitten in einer Galaxis befinden, sind wir nicht fähig, diese angebliche Krümmung des Raums nachzuweisen. Aber wenn ein Lichtstrahl weit entfernt von einer Galaxis um diese Galaxis ‚gebogen‘ wird, obwohl sich kein einziges Gestirn in der Nähe dieses Lichtstrahls befindet, dann soll dies ein Beweis für Einsteins Raumkrümmung sein. „Einstein hatte Recht“ oder „Einsteins Raumkrümmung ist bestätigt“ klingen die Äußerungen, ohne eine einzige Berechnung. Man macht eine Beobachtung eines um eine Galaxis gebogenen Lichtstrahls und erklärt Einsteins Theorie „als weiteres Mal bestätigt“. So einfach ist das. Dabei lässt sich dies auf ganz andere Weise erklären. Aber bei Einstein-Fanatikern ist jedes weitere Wort zwecklos. Und darum wird es diese Theorie noch in 100 Jahren geben, damit sich Menschen in 200 Jahren darüber krumm und schief lachen können ……. wie bei der Erde, die angeblich eine Scheibe gewesen sein soll.

  561. #563 MartinB
    13. März 2013

    Masse ist nicht Ruhemasse und dmit bricht leider das ganze tolle Argument zusammen
    Aber Glüchwunsch, dass du den Unterschied zwischen Masse und Gewicht kennst, immerhin…
    Der Rest enthält nur Behauptungen, keine Argumente (ja, man kann nachrechnen, dass der Gravitationslinseneffeklt messbar ist während ich von der Lichtkrümmung auf der Erde nichts sehe, das geben die Formeln tatsächlich her).
    Komisch, die RT-Leugner sind genau so langweilig vorhersehbar wie die ET-Leugner drüben im anderen Thread…

  562. #564 Not-Einstein
    13. März 2013

    @MartinB
    “Aber schon prinzipiell ist mir nicht klar, warum die Raumzeitstruktur nun mathematisch ausgerechnet durch eine ungekrümmte Mannigfaltigkeit beschrieben werden soll.”

    “Die allgemeine Relativitätstheorie beschreibt …”(Wiki)
    … ist eine mathematische Beschreibung. Dass die ART „in diesem Sinne“ richtig ist, richtige Ergebnisse liefert, wird wohl kein ernsthafter Mensch bezweifeln wollen (ich tue es auf jedenfalls nicht). Die Erkärung, die nachgeliefert wurde, “Raumkrümmung” stammt nicht von Einstein. Einstein hatte eine andere Erklärung – “Äther”. (sicherheitshalber: Äther und Relativitätstheorie und Über den Äther; übrigens verstehe ich nicht, dass von den „Zweiflern“ hier niemand auf den Äther hingewiesen hat, kennt man diese Dokumente nicht?). Einstein: „in diesem Sinne gibt es einen Äther“.

    Mein Eindruck ist der, dass gegen diese Erklärung niemals sachliche Argumente vorgebracht wurden. Wiki: „Nun ist diese Übereinstimmung mit dem klassischen Äther zu gering, als dass sich dieser neue Ätherbegriff in der Fachwelt hätte durchsetzen können…“ ? Einstein weist unmißverständlich darauf hin, dass es sich bei dem „neuen“ Äther um den „klassischen“ Lorentz-Äther handelt, marginal abgeändert an die Anforderungen der ART. „Das prinzipiell Neuartige des Äthers der allgemeinen Relativitätstheorie gegenüber dem Lorentzschen Äther …“. Mein Eindruck ist vielmehr der, dass nachdem der Äther „abgeschafft“ worden ist, es nicht mehr dem „Geschmack“ der Zeitgenossen entsprochen hat, ihn wieder aufzustehen lassen.

    Man hat, um den Äther endgültig loszuwerden, sich eine alternative Erklärung ausgedacht, die Erklärung, dass der völlig(!) leere Raum durch Materie gekrümmt werden kann. Vor allem hat man alles vermieden, was Hinweise darauf geben könnte, dass Einstein seine Position bzgl. der Existenz eines Äthers geändert hatte (an dieser Taktik hält man noch heute fest!).

    Die Geometrie des völlig(!) leeren Raumes wird (durch die Anwesenheit einer Materie) verändert.

    Für meine Begriffe ist diese Erklärung völlig absurd: Es werden hier zwei gedankliche Gegenstände betrachtet; der eine ist die Geometrie, der andere der leere Raum, wobei de facto der zweite Gegenstand nicht vorhanden ist, weil er ein Nichts darstellt. Auf Deutsch, wie es die Allgemeinheit verwendet, bedeutet diese Aussage, die Geometrie des völlig(!) leeren Raums, des „Nichts“ wird verändert; auf gut Deutsch stellt dies eine sinnlose, eine völlig absurde Aussage dar.

    „Weil der Äther schon seit Jahrzehnten als wissenschaftlicher Irrtum gilt, …“ (Wikipedia/Äther) und deshalb implizit dieser auch Einstein unterstellt wird, was ihm wahrscheinlich nicht gefallen haben dürfte und er von der alternativen Erklärung wohl auch nichts gehalten hat, hatte er für sie nur Hohn und Spott übrig, Einstein 1954:

    „Der Raum, ans Licht gebracht durch das körperliche Objekt, zur physikalischen Realität erhoben durch em>Newton, hat in den letzten Jahrzehnten den Äther und die Zeit verschlungen und scheint im Begriffe zu sein, auch das Feld und die Korpuskeln zu verschlingen, so daß er als alleiniger Träger der Realität[!!!] übrig bleibt“

    P.S. Danke für den Hinweis für die Formatierungen!

  563. #565 Not-Einstein
    13. März 2013

    Mit dem Hyperlink hat’s nicht funktioniert:
    “Äther und Relativitätstheorie” , Äther und Relativitätstheorie
    https://wikilivres.ca/wiki/%C3%84ther_und_Relativit%C3%A4tstheorie

    ” Über den Äther”, Über den Äther, https://wikilivres.ca/wiki/%C3%9Cber_den_%C3%84ther#cite_ref-1

  564. #566 Niels
    13. März 2013

    Not-Einstein spielt dieses Spielchen übrigens in mehreren Blogs, zum Beispiel auch hier:
    https://scienceblogs.de/astrodicticum-simplex/2009/11/24/wie-man-dunkle-materie-nachweist/#comment-192134

    Dort hat er die Antworten dann schlicht ignoriert. Das wird hier vermutlich auch nichts anders laufen.

    @Not-Einstein

    Für meine Begriffe ist diese Erklärung völlig absurd

    Wenn du es dir nicht vorstellen kannst, muss es natürlich falsch sein. Millionen Physiker müssen irren.
    Klingt vernünftig. Ist schließlich völlig unvorstellbar, dass du es bist, der falsch liegt.

    und er von der alternativen Erklärung wohl auch nichts gehalten hat, hatte er für sie nur Hohn und Spott übrig, Einstein 1954

    Hast du einen Link zur Originalquelle?
    Soviel ich weiß ist dieses Zitat nämlich durchaus ernst gemeint und keineswegs etwas, für das Einstein “Hohn und Spott” übrig hatte.
    Außerdem geht es sehr wahrscheinlich um die Frage nach der Gültigkeit des Machschen Prinzips in der ART. Raumkrümmung wird hier als Gegeben vorausgesetzt.

    Allgemein:
    Wenn man ”Über den Äther” tatsächlich liest und nicht nur einzelne Zitate aus dem Zusammenhang reißt und falsch interpretiert, kann man das eigentlich nicht so interpretieren, wie du es tust.

    Einstein weist unmißverständlich darauf hin, dass es sich bei dem „neuen“ Äther um den „klassischen“ Lorentz-Äther handelt, marginal abgeändert an die Anforderungen der ART. „Das prinzipiell Neuartige des Äthers der allgemeinen Relativitätstheorie gegenüber dem Lorentzschen Äther …“

    Marginal abgeändert?
    Du hast den Rest des Zitats vergessen:
    Das prinzipiell Neuartige des Äthers der allgemeinen Relativitätstheorie gegenüber dem Lorentzschen Äther besteht darin, daß der Zustand des ersteren an jeder Stelle bestimmt ist durch gesetzliche Zusammenhänge mit der Materie und mit dem Ätherzustände in benachbarten Stellen in Gestalt von Differentialgleichungen, während der Zustand des Lorentzschen Äthers bei Abwesenheit von elektromagnetischen Feldern durch nichts außer ihm bedingt und überall der gleiche ist.

    Oder an anderer Stelle:
    Dieser Ätherbegriff, auf den die Machsche Betrachtungsweise führt, unterscheidet sich aber wesentlich vom Ätherbegriff Newtons, Fresnels und H.A. Lorentz. Dieser Machsche Äther bedingt nicht nur das Verhalten der trägen Massen, sondern wird in seinem Zustand auch bedingt durch die trägen Massen.

    Das ist ein ganz fundamentaler Unterschied. So fundamental, dass das eine eigentlich nichts mehr mit dem anderen zu tun hat. Wenn es keine Gemeinsamkeiten mehr gibt, ist es nicht hilfreich oder sinnvoll, beides Äther zu nennen.

    Einstein: „in diesem Sinne gibt es einen Äther“.
    Mein Eindruck ist der, dass gegen diese Erklärung niemals sachliche Argumente vorgebracht wurden.

    Das Zitat lautet vollständig:
    Nach der allgemeinen Relativitätstheorie ist der Raum mit physikalischen Qualitäten ausgestattet; es existiert also in diesem Sinne ein Äther.

    Nochmal an anderer Stelle dazu:
    Man könnte statt von „Äther“ also ebensogut von „physikalischen Qualitäten des Raumes“ sprechen.

    Übrigens ist Einstein generell freizügig mit dem Äther-Begriff:
    In Newtons Bewegungslehre besitzt der „Raum“ physikalische Realität — im Gegensatz zu Geometrie und Kinematik. Wir wollen dies physikalisch Reale, welches neben den beobachtbaren ponderabeln Körpern in das Newtonsche Bewegungsgesetz eingeht, als „Äther der Mechanik” bezeichnen.

    Es ist einfach nicht sinnvoll, wenn man „Äther“ sagt, aber Raumzeit oder sogar Newtons absoluten Raum meint. Deswegen hat sich das nicht durchgesetzt.

    Die Erkärung, die nachgeliefert wurde, “Raumkrümmung” stammt nicht von Einstein.
    […]
    Man hat, um den Äther endgültig loszuwerden, sich eine alternative Erklärung ausgedacht, die Erklärung, dass der völlig(!) leere Raum durch Materie gekrümmt werden kann

    Doch, die Erklärung „Raumkrümmung“ stammt von Einstein. Wenn man sich die Komponenten anschaut, aus denen die Einsteingleichungen aufgebaut sind, kann man das eigentlich auch nicht ernsthaft abstreiten.
    https://de.wikipedia.org/wiki/Allgemeine_Relativit%C3%A4tstheorie#Einsteinsche_Feldgleichungen

    Aber Einstein sagt das auch selbst. Sogar in den Texten, die du selbst als Gegenbeleg verlinkt hast. Genau darum geht es doch in der Hauptsache:

    Die allgemeine Relativitätstheorie beseitigt einen Übelstand der klassischen Dynamik: nach letzterer erscheinen Trägheit und Schwere als ganz verschiedene, voneinander unabhängige Erscheinungen, trotzdem sie beide durch dieselbe Körperkonstante, die Masse, bedingt werden. Die Relativitätstheorie überwindet diesen Mangel, indem sie das dynamische Verhalten des elektrisch neutralen Massenpunktes durch das Gesetz der geodätischen Linie festlegt, in welchem die Trägheits- und Schwerewirkungen nicht mehr auseinandergehalten sind. Dabei legt sie dem Äther von Punkt zu Punkt variable, die Metrik und das dynamische Verhalten materieller Punkte bestimmende Eigenschaften bei, welche ihrerseits durch physikalische Faktoren, nämlich durch die Verteilung von Masse bezw. Energie bestimmt sind. Der Äther der allgemeinen Relativitätstheorie unterscheidet sich also von demjenigen der klassischen Mechanik bezw. der speziellen Relativitätstheorie dadurch, dass er nicht „absolut“, sondern in seinen örtlich variablen Eigenschaften durch die ponderable Materie bestimmt ist.

    Und hier:
    Das prinzipiell Neuartige des Äthers der allgemeinen Relativitätstheorie gegenüber dem Lorentzschen Äther besteht darin, daß der Zustand des ersteren an jeder Stelle bestimmt ist durch gesetzliche Zusammenhänge mit der Materie und mit dem Ätherzustände in benachbarten Stellen in Gestalt von Differentialgleichungen, während der Zustand des Lorentzschen Äthers bei Abwesenheit von elektromagnetischen Feldern durch nichts außer ihm bedingt und überall der gleiche ist.

    Und hier:
    Dieser Machsche Äther bedingt nicht nur das Verhalten der trägen Massen, sondern wird in seinem Zustand auch bedingt durch die trägen Massen.

    Vor allem hat man alles vermieden, was Hinweise darauf geben könnte, dass Einstein seine Position bzgl. der Existenz eines Äthers geändert hatte

    Da musste man nichts vermeiden. Einstein hat seine Position nie geändert.

  565. #567 PDP10
    13. März 2013

    Mein lieber @Not-Einstein: Dein selektives Zitieren aus der Rede “Über den Äther” von 1924 habe ich doch vorvorgestern im Forum nebenan bei Astrodicticum Simplex auseinander genommen.

    Nochmal: Einstein verwirft den Äther-Begriff von Lorentz in dieser Rede.

    Und auch nochmal: Im Verlaufe des 20sten Jahrhunderts ist der Äther-Begriff aus guten Gründen ad Acta gelegt worden.

    Man braucht ihn einfach nicht mehr.

  566. #568 PDP10
    13. März 2013

    @Niels:

    Uuups … du tippst offenbar verdammt schnell … 🙂

  567. #569 Not-Einstein
    19. März 2013

    Im Verlaufe des 20sten Jahrhunderts ist der Äther-Begriff aus guten Gründen ad Acta gelegt worden.
    Man braucht ihn einfach nicht mehr.

    Klar, schließlich kann sich ja das Licht im Vakuum fortpflanzen. Ja, und dass das Licht sich in der Nähe von Massen auf einer gekrümmten Bahn fortbewegt, ist eine Illusion, sozusagen eine „optische“ Täuschung. In Wahrheit ist es nämlich so, dass in der Nähe von Massen der Raum gekrümmt ist, das Licht sich tatsächlich aber immer geradlinig fortbewegt.

    Na ja, so klar ist das auch wieder nicht. Klingt irgendwie nach „Total Verrückt!“. Jedenfalls verstehe ich es nicht in dem Moment, indem ich das niederschreibe.

    Doch, die Erklärung „Raumkrümmung“ stammt von Einstein. Wenn man sich die Komponenten anschaut, aus denen die Einsteingleichungen aufgebaut sind, kann man das eigentlich auch nicht ernsthaft abstreiten.
    https://de.wikipedia.org/wiki/Allgemeine_Relativit%C3%A4tstheorie#Einsteinsche_Feldgleichun

    (Mir verschlägt es den Atem über diese Art der Beweisführung! Aber Zähne zusammenbeißen, cool bleiben!)

    Mir ist nicht bekannt, dass Einstein das Wort „Raumkrümmung“ jemals auch nur in den Mund genommen hat. Wenn er das Wort überhaupt nicht verwendet hat, wie soll er es dann als eine Erklärung benutzt haben. Dei einfachsten Regeln der Logik besagen, dass man für eine Erklärung das entsprechende Wort verwenden muss.

    Weil Einstein das Wort „Raumkrümmung“ nie verwendet hat, kann er (logischerweise) „Raumkrümmung“ nie als eine Erklärung für (seine!) Allgemeine Relativitätstheorie verwendet haben. Er hatte eine andere Erklärung und dafür verwendet er das Wort(!), dass ich im nachfolgendem Zitat fett hervorgehoben habe:

    (egal, ich habe jetzt, wo ich das nocheinmal durchlese, Zweifel, ob es noch Sinn macht, sich zu wiederholen – ich habe es nun mal schon geschrieben …)

    „Die allgemeine Relativitätstheorie beseitigt einen Übelstand der klassischen Dynamik: nach letzterer erscheinen Trägheit und Schwere als ganz verschiedene, voneinander unabhängige Erscheinungen, trotzdem sie beide durch dieselbe Körperkonstante, die Masse, bedingt werden. Die Relativitätstheorie überwindet diesen Mangel, indem sie das dynamische Verhalten des elektrisch neutralen Massenpunktes durch das Gesetz der geodätischen Linie festlegt, in welchem die Trägheits- und Schwerewirkungen nicht mehr auseinandergehalten sind.  Dabei legt sie dem Äthervon Punkt zu Punkt variable, die Metrik und das dynamische Verhalten materieller Punkte bestimmende Eigenschaften bei, welche ihrerseits durch physikalische Faktoren, nämlich durch die Verteilung von Masse bezw. Energie bestimmt sind. Der Äther der allgemeinen Relativitätstheorie unterscheidet sich also von demjenigen der klassischen Mechanik bezw. der speziellen Relativitätstheorie dadurch, dass er nicht „absolut“, sondern in seinen örtlich variablen Eigenschaften durch die ponderable Materie bestimmt ist.“

    „Der Äther der allgemeinen Relativitätstheorie unterscheidet sich also von demjenigen der klassischen Mechanik bezw. der speziellen Relativitätstheorie dadurch, dass er nicht „absolut“, sondern in seinen örtlich variablen Eigenschaften durch die ponderable Materie bestimmt ist.“

    Dieser Satz ist eigentlich so klar, dass man ihn nicht mehr interpretieren bräuchte, „…kann man das eigentlich auch nicht ernsthaft abstreiten …“, dass „der Äther der allgemeinen Relativitätstheorie „in seinen örtlich variablen Eigenschaften durch die ponderable Materie bestimmt ist“ (Einstein!), dass der Äther der allgemeinen Relativitätstheorie sich durch die Anwesenheit von Materie krümmt.

    Gemäß Einstein „krümmt“ sich nicht der Raum; davon ist bei Einstein nie die Rede – er hat das Wort „Raumkrümmung“ in seinem ganzen Leben kein einziges Mal benutzt!; der Äther „krümmt“ sich durch die Anwesenheit von Materie!!
    Einstein, letzter Absatz in „Über den Äther“, 1924

    “Aber selbst wenn diese Möglichkeiten zu wirklichen Theorien heranreifen, werden wir des Äthers, d. h. des mit physikalischen Eigenschaften ausgestatteten Kontinuums, in der theoretischen Physik nicht entbehren können; denn die allgemeine Relativitätstheorie, an deren grundsätzlichen Gesichtspunkten die Physiker wohl stets festhalten werden, schliesst eine unvermittelte Fernwirkung aus; jede Nahewirkungs-Theorie aber setzt kontinuierliche Felder voraus, also auch die Existenz eines „Äthers“.

    ….werden wir des Äthers, d. h. des mit physikalischen Eigenschaften ausgestatteten Kontinuums, in der theoretischen Physik nicht entbehren können“ – Für Einstein ist der Ätherbegriff unentbehrlich in der theoretischen Physik!

    Nach der Auffassung von Einstein setzt „…die allgemeine Relativitätstheorie…“ „…die Existenz eines „Äthers…“ voraus. – Dies ist die Auffassung von Einstein!

    „Soviel ich weiß [woher, von wem, wieso, wohin?] ist dieses Zitat nämlich durchaus ernst gemeint und keineswegs etwas, für das Einstein “Hohn und Spott” übrig hatte.“

    Ok, nehmen wird dieses Zitat ernst und greifen wir die Idee auf, „ Feld und die Korpuskeln“ in den Raum zu integrieren, warum eigentlich nicht? – Immerhin ist das Quantenfeld ein universelles Feld, ein Feld, das den ganzen Raum durchzieht. Also, warum soll man nicht sagen, das Quantenfeld ist eine Eigenschaft des Raums? – Man hätte dann ALLES im Raum vereinigt, man hätte dann eine Theory of Everything, eine ToE, in Form einer „Spacial(!) Relativistic Theory“ – alles unter Dach und Fach des „Raumes“ !

    „Unter diesen Bedingungen muss man keine Kraft postulieren…“ – Zitat aus https://www.einstein-online.info/einsteiger/allgRT/geomGravitation

    Unter diesen Bedingungen muss man auch nicht mehr das Vorhandensein von Materie postulieren!

    „Im Verlaufe des 20sten Jahrhunderts ist der „Materie“-Begriff aus guten Gründen ad Acta gelegt worden. Man braucht ihn einfach nicht mehr.“

  568. #570 Niels
    19. März 2013

    Ich hab keine Ahnung, was du mit dem letzten Teil deines Beitrags sagen willst.
    Wie passt es, dass du auf einmal auf einen Text verlinkst, in dem die Raum-(Zeit)-Krümmung als Tatsache vorausgesetzt wird?
    (Außerdem fällt mir auf, dass du die Quelle des angesprochenen Zitates offenbar nicht offenlegen willst? Warum nicht?)

    Gemäß Einstein „krümmt“ sich nicht der Raum; davon ist bei Einstein nie die Rede – er hat das Wort „Raumkrümmung“ in seinem ganzen Leben kein einziges Mal benutzt!; der Äther “krümmt“ sich durch die Anwesenheit von Materie!!
    […]
    Mir ist nicht bekannt, dass Einstein das Wort „Raumkrümmung“ jemals auch nur in den Mund genommen hat. Wenn er das Wort überhaupt nicht verwendet hat, wie soll er es dann als eine Erklärung benutzt haben. Dei einfachsten Regeln der Logik besagen, dass man für eine Erklärung das entsprechende Wort verwenden muss.

    Hast du ein Zitat, in dem Einstein das Wort „Ätherkrümmung“ verwendet?
    Wenn nicht, folgt offensichtlich nach den einfachsten Regeln der Logik…
    Übrigens bezweifle ich irgendwie, dass du alle Briefe, wissenschaftliche Veröffentlichungen, Reden und sonstigen Aufzeichnungen von und über Einstein derart genau überprüft hast, dass deine Meinung als Beleg für deine Behauptungen gelten könnte.

    Aber mal ernsthaft:
    Ich hab doch schon oben folgendes zitiert:
    Man könnte statt von „Äther“ also ebensogut von „physikalischen Qualitäten des Raumes“ sprechen.
    Statt von der Krümmung des „Äthers der Allgemeinen Relativitätstheorie“ kann man also laut Einstein ebensogut davon sprechen, dass der Raum die „physikalische Qualität“ besitzt, dass er eine Krümmung aufweist.
    Vulgo: Der Raum ist gekrümmt.
    (Wobei natürlich eigentlich die Raumzeit gekrümmt ist, nicht nur der Raum.)

    Davon abgesehen sind wir hier in der Physik, nicht in der Literaturwissenschaft. Selbst wenn er tatsächlich in einer Rede geäußert hätte, dass in der ART eine Art Lorentzscher Äther zwingend notwendig wäre (das war übrigens deine Einstiegsthese, schon vergessen?):
    Dann hätte Einstein sich in diesem Fall eben geirrt. Autoritätsargumente sind in der Physik wenig wert.

  569. #571 Adent
    19. März 2013

    @Niels
    Hat er nicht, im Gegenteil, unser Not-Einstein liest nur sehr selektiv.

  570. #572 PDP10
    19. März 2013

    Hmmmpf … jetzt habe ich Not-Einstein doch schon einen Artikel auf das wunderschöne Gravity Probe B Exeriment im Forum nebenan verlinkt …

    Und dann kommt er wieder hier her um nochmal zu sagen, dass er unsere Förmchen doof findet und lieber mit seinen eigenen spielt.

    Von mir aus ….

  571. #573 Not-Einstein
    23. März 2013

    „Klar, schließlich kann sich ja das Licht im Vakuum fortpflanzen. Ja, und dass das Licht sich in der Nähe von Massen auf einer gekrümmten Bahn fortbewegt, ist eine Illusion, sozusagen eine „optische“ Täuschung. In Wahrheit ist es nämlich so, dass in der Nähe von Massen der Raum gekrümmt ist, das Licht sich tatsächlich aber immer geradlinig fortbewegt.
    Na ja, so klar ist das auch wieder nicht. Klingt irgendwie nach „Total Verrückt!“. Jedenfalls verstehe ich es nicht in dem Moment, indem ich das niederschreibe.“

    Die Schwierigkeit, das zu verstehen, besteht darin, dass sich der völlig leere Raum krümmen soll. Aber genau das wird behauptet; an dieser Behauptung kann nicht der geringste Zweifel bestehen: „Stellen Sie sich leeren Raum vor…“ Zitat aus https://www.einstein-online.info/einsteiger/allgRT/geomGravitation . Was dabei besonders erstaunlich ist, dass diese Behauptung von Wissenschaftlern stammt, von denen man eigentlich nicht annehmen kann, dass sie sich mit so einer Behauptung einen Jux erlauben.

    Andererseits: Wenn man davon ausgeht, dass Licht sich im im Vakuum, im völlig leeren Raum fortpflanzt – es ist kein Medium, kein Träger vorhanden – was hat man für Möglichkeiten, einen Richtungswechsel bei der Fortpflanzung des Lichts zu erklären? – Man hat gar keine Möglichkeiten! Man muss dabei bleiben, dass sich Licht immer geradlinig ausbreitet. Dann gibt es eben nur noch den Ausweg, dass man sagt: „Der Raum ist gekrümmt“. Man nimmt in Kauf, dass man eine Aussage macht, deren Ausmass an Absurdität nicht mehr überboten werden kann.

    Wenn man so eine Aussage zulässt, dann kann man mit dem gleichen Recht behaupten, der „Schiefe Turm von Pisa“ ist gar nicht schief, es hat sich nur der Raum an dieser Stelle verzogen.

    Aber solange ein MartinB auf so einen Einwand gereizt reagiert, „ – geh woanders spielen.“, besteht noch Hoffnung. Er hat die Problematik von solchen Aussagen wenigstens grundsätzlich erkannt.

    P.S. Das Einstein Zitat stammt aus: Einstein, Albert (1954): Raum, Äther und Feld in der Physik.

  572. #574 rolak
    23. März 2013

    Selbst wenn irgendwo der Raum tatsächlich komplett leer sein sollte, Not-Einstein, so hat er doch noch immer seine Metrik. Die trägt zwar nichts zu seinem Inhalt bei, ist dennoch mit ihm untrennbar verbunden – womit etwas Krümmbares vorhanden ist.

  573. #575 dsch64
    Wörth am Rhein
    23. März 2013

    @MB
    Sie haben mein Hochachtung wie sie immer wieder versuchen, Leuten die mit dem Prinzip Wisschenschaft nix am Hut haben mit ihrer physikalischen Weltsicht zu bekehren!!

    @Alle Einstein Leugner
    Was regen sie sich nur über diese Theorie auf??? Nach meiner bescheidenen Ingenieurssicht ist eine Theorie nur ein Hilfmittel um Aspekte der Wirklichkeit zu beschreiben. Was wohl jeder zugeben muss, ist, die SRT beschreibt den Bereich der Wirklichkeit für den sie sich zuständig fühlt, überwältigend genau.
    Was ich immer lustig finde, wenn die 2 Grundprämissen der SRT mit der Konstanz der Lichtgeschwindigkeit und der Gleichheit der physikalischen Gesetzen in Inertialsystemen angezweifelt werden, denn wenn man sagt, diese Prämissen nicht gelten, dann ist es auch keine SRT.
    Es steht jedem frei eine Theorie zu entwickeln mit den Prämissen die Lichtgeschwindigkeit ist abhängig von der Lage des Beobachters und die physikalischen Gesetze sind abhängig vom Bewegungszustand des Systems. Diese Theorie muss dann nur noch in Einklang mit jedweden Messdaten gebracht werden und schon wird die SRT nicht mehr benötigt. Denn wie schon Feyman sinngemäß sagte, wichtig sind nicht die Theorien sondern
    alleine die Messdaten. Allerdings ist dies eine Jahrhundertaufgabe denn da extrem viele Theorien seit 108 jahren die SRT als Basis haben, sollten diejenigen
    sich eine paar Physiker suchen, die diese Theorien umformulieren und mit der neuen Theorie in Einklang bringen.

    Viel Spass dabei!!!

  574. #576 PDP10
    23. März 2013

    @rolak:

    “Selbst wenn irgendwo der Raum tatsächlich komplett leer sein sollte, Not-Einstein, so hat er doch noch immer seine Metrik.”

    Bah! Komm dem doch nicht mit sowas! Das sieht doch nur für uns “mathematisch verblödete” (Markus T.) Menschen so aus!

    Aber mal im Ernst, mein lieber Not-Einstein:

    Das Universum wurde nicht zu dem Zweck geschaffen unserer Vorstellung zu entsprechen.
    Wir müssen, im Gegenteil, unsere Vorstellung dem anpassen, was wir messen.

    Wenn Sie das Konzept der Raumkrümmung, das uns sowohl die ART, als auch Experimente nahelegen nicht verstehen, so ist das nicht die Schuld des Universums und schon gar nicht die Schuld Martin Bäkers.

  575. #577 rolak
    24. März 2013

    Mal ganz allgemein: Was für ein Notfall liegt eigentlich vor, daß wir uns hier mit einem Not-Einstein abgeben müssen?

  576. #578 Spritkopf
    24. März 2013

    @Not-Einstein

    Wenn man davon ausgeht, dass Licht sich im im Vakuum, im völlig leeren Raum fortpflanzt – es ist kein Medium, kein Träger vorhanden – was hat man für Möglichkeiten, einen Richtungswechsel bei der Fortpflanzung des Lichts zu erklären? – Man hat gar keine Möglichkeiten! Man muss dabei bleiben, dass sich Licht immer geradlinig ausbreitet. Dann gibt es eben nur noch den Ausweg, dass man sagt: „Der Raum ist gekrümmt“. Man nimmt in Kauf, dass man eine Aussage macht, deren Ausmass an Absurdität nicht mehr überboten werden kann.

    https://yourlogicalfallacyis.com/personal-incredulity

  577. #579 Adent
    24. März 2013

    Ja das ist schon interessant, die Legionen von Einstein Leugnern oder Widerlegern sind alle in der Lage zu sagen, hey das kann nicht stimmen, mehr leider nicht. Kein einziger kam bisher mit einer überprüfbaren oder nachrechenbaren Hypothese, die das was wir täglich in der Physik erleben können auch nur ansatzweise so gut und funktional beschreibt wie die Relativitätstheorien.
    Aber immer schön krakeelen.
    “Ne das glaube ich nicht, das versteh ich nämlich nicht,” das ist ja soooo langweilig….. seufz.

  578. #580 Mampf
    7. Mai 2013

    Es stimmt nicht, daß keine überprüfbare und nachrechenbare Hypothese existiert. Die These, die ich gelesen habe, beschreibt sehr gut und funktional, daß die Relativitätstheorie falsch sein muß. Man braucht nur im Internet nach den neuesten Ausgaben zu suchen.

  579. #581 Martin B.
    17. Juni 2013

    Viele Messungen und Behauptungen sind falsch

    Die Behauptung von Experten, das Licht einer 13 Milliarden Lichtjahre entfernten Galaxie brauche 13 Milliarden Jahre bis zu uns, ist [im Normalfall] in zweifacher Hinsicht ein Trugschluss.

    Erstens: Wenn das Licht von jener anderen Galaxie abgegeben wird, reist auch unsere Milchstraße weiter, weshalb die Distanz zu unserer Galaxie immer größer wird. Bis dieses Licht bei uns eintrifft, ist letzten Endes zusätzliche Zeit vergangen. Und man möge sich auch nicht auf den ausgleichenden Effekt der Zeitdilatation rausreden, der nur zwei Spezialfälle betrifft, die (eigentlich) nie vorkommen. [Die einzige Ausnahme, die fast zu 100 % greift, betrifft unsere Nachbargalaxie Andromeda, die aber nur etwa 2,3 Millionen Lichtjahre entfernt ist.] Außerdem wäre mit den Formeln der Relativitätstheorie diese von Experten getätigte Aussage sogar zu 100 % falsch.

    Zweitens: Durch seine tatsächliche Verhaltensweise reist das Licht auf einer ‘gezackten’ Bahn, weshalb man zwischen der Brutto-Geschwindigkeit des Lichts, die immer 299.792,458 km/s ausmacht, und seiner Netto-Geschwindigkeit unterscheiden muss. Und weil das Licht von anderen Galaxien (fast) ausschließlich mit Netto-Geschwindigkeit reist, kann man eine Distanz von 13 Milliarden Lichtjahren nicht eins zu eins auf eine Reisedauer von 13 Milliarden Jahre umlegen.

    Überdies sind Aussagen “Man hat die Geschwindigkeit des Lichts aus anderen Galaxien (bei uns) gemessen und keinen Unterschied zur Geschwindigkeit des Lichts unserer Galaxie festgestellt” eine derartige Dreistigkeit, die die schräge Denkweise der Relativisten untermauert. – Wenn Lichtstrahlen aus anderen Galaxien bei uns ankommen, dann haben sie sich längst an die ‘Gepflogenheiten’ unserer Galaxie angepasst. Warum sollen sich Lichtstrahlen aus einer anderen Galaxie bei uns anders verhalten als unsere ‘eigenen’ Lichtstrahlen?

    Ebenso sind Aussagen von Experten wie “eine Reise zu einem 150 Lichtjahre entfernten Planeten wird nie möglich sein bzw. Menschen würden altersbedingt zuvor sterben” blanker Unsinn. Wegen der Zeitdilatation, die nachweislich existiert, vergeht die Zeit bei Hochgeschwindigkeitsreisen viel langsamer. Auch wenn auf der Erde 150 Jahre vergehen, altern die Raumfahrer bei über 90 % der Lichtgeschwindigkeit weitaus langsamer.

    Außerdem ist die Relativitätstheorie ein Unding, was sogar das US-amerikanische Wunderkind Jacob Barnett erkannt hat, der mit 14 Jahren einen IQ von nahezu 170 erzielte und als 15-Jähriger in Quantenphysik promoviert. Das Ziel von Barnett ist es, die Relativitätstheorie zu widerlegen. Was Barnett nicht weiß: Diese Theorie wurde bereits widerlegt. Falls er eines Tages seine Ergebnisse präsentiert, wird es interessant sein, sie mit den vorliegenden Erkenntnissen zu vergleichen. Dann hätte nach einem Europäer auch ein Amerikaner (hoffentlich unabhängig davon) Einsteins Thesen widerlegt.

  580. #582 MartinB
    17. Juni 2013

    @MartinB
    Bitte nicht mein Kürzel verwenden, das führt zu Schwierigkeiten.
    Auf diesem Blog gibt es bitte nur einen MartinB, das bin ich.

    Ansonsten ist das, was du schreibst, ziemlicher Blödsinn, sorry.

  581. #583 Alexxx
    HH
    18. Juni 2013

    Was ist Mengenlehre?
    Wenn drei Personen in einem Raum gehen, vier wieder rauskommen, und einer zurück geht, damit keiner mehr drinne ist.

  582. #584 Alderamin
    18. Juni 2013

    @Martin B. (nicht MartinB)

    Die Behauptung von Experten, das Licht einer 13 Milliarden Lichtjahre entfernten Galaxie brauche 13 Milliarden Jahre bis zu uns, ist [im Normalfall] in zweifacher Hinsicht ein Trugschluss.

    Hätrtest Du mal eine Referenz, wo so etwas behauptet wird? Die Experten kennen sehr wohl den Unterschied zwischen der Laufzeitentfernung und der mitbewegten Entfernung, den Du hier ansprichst. Lediglich manche Medienverteter nichtwissenschaftlicher Zeitschriften verwechseln diese des öfteren.

    Durch seine tatsächliche Verhaltensweise reist das Licht auf einer ‘gezackten’ Bahn, weshalb man zwischen der Brutto-Geschwindigkeit des Lichts, die immer 299.792,458 km/s ausmacht, und seiner Netto-Geschwindigkeit unterscheiden muss.

    Wie soll es das denn bewerkstelligen? Umlenkung in Gravitationsfeldern? Da sind nur kleine Winkel im Spiel, die meiste Zeit kann sich das Licht geradlinig ausbreiten. Da kommt nicht viel Umweg zusammen, ganz bestimmt kein Faktor 10, wie er schon als Rotverschiebung von Galaxien gemessen wurde.

    Warum sollen sich Lichtstrahlen aus einer anderen Galaxie bei uns anders verhalten als unsere ‘eigenen’ Lichtstrahlen?

    Warum sollen sie sich in einer anderen Galaxie anders verhalten als bei uns? Was man z.B. genau kennt, ist die Wellenlänge, mit der das Licht ursprünglich erzeugt wurde, z.B. für die Wasserstofflinien im Spektrum aller Sterne einer entfernten Galaxie (kann man aus den Energieniveaus im Atom ausrechnen). Die kommen dann um einen gewissen Faktor frequenzverschoben bei uns an. Um den gleichen Faktor frequenzverschoben erscheinen aber auch andere Prozesse in der gleichen Galaxie, z.B. Supernovaexplosionen des Typs Ia, die stets gleich ablaufen und deren Helligkeitsentwicklung einen charakteristischen Verlauf hat (der resultiert aus der stets gleichen Startmasse und dem radioaktiven Zerfall der erzeugten Isotope). Das bedeutet, dass das Licht der gleichen Zeitdilatation unterworfen sein muss wie der Verlauf der Supernovaexplosion, eine Folge der kosmologischen Dopplerverschiebung. Rechnet man den heraus, folgt aus der bekannten Gleichung Lichtgeschwindigkeit = Frequenz * Wellenlänge die Lichtgeschwindigkeit in der fernen Galaxie.

    Man kann aber z.B. auch versuchen, die Feinstrukturkonstanten aus dem Abstand gewisser Spektrallinien zu bestimmen. In der Feinstrukturkonstanten stecken die Lichtgeschwindigkeit, die Elektronenladung, die Dielekttrizitätskonstante und das Plancksche Wirkungsquantum. Falls sich eine dieser Größen über kosmische Entfernungen / Zeiträume geändert haben sollte, würde man das an den Spektren erkennen können. Dergleichen wurde bisher nicht nachgewiesen.

    Ebenso sind Aussagen von Experten wie “eine Reise zu einem 150 Lichtjahre entfernten Planeten wird nie möglich sein bzw. Menschen würden altersbedingt zuvor sterben” blanker Unsinn. Wegen der Zeitdilatation, die nachweislich existiert, vergeht die Zeit bei Hochgeschwindigkeitsreisen viel langsamer.

    Wissen wir. Das Problem ist, annähernd auf solche Geschwindigkeiten zu kommen.

    Außerdem ist die Relativitätstheorie ein Unding,

    Öh, sprachst Du nicht gerade von der Zeitdilatation? Ist nur einer der Effekte der speziellen Relativitätstheorie, die hängen aber alle miteinander zusammen und folgen direkt aus der Konstanz der Lichtgeschwindigkeit für bewegte und ruhende Beobachter, die messtechnisch bestens belegt ist. Ansonsten würde Dein Navi gar nicht funktionieren.

  583. #585 MartinB
    18. Juni 2013

    @Alexxx
    Was soll das jetzt? Der Witz ist schon so alt, den hat mir mein Großvater als Gutenachtgeschichte erzählt…

  584. #586 Alderamin
    18. Juni 2013

    @MartinB

    Öh, wärest Du so freundlich, #584 aus der Moderation zu befreien? Mir war klar, dass ich mit den vielen Links dort lande, aber ich mag halte keine unbelegten Behauptungen machen…

  585. #587 MartinB
    18. Juni 2013

    Sorry, hatte ich nicht gesehen.

  586. #588 Quercus
    18. Juni 2013

    Kleine Rückmeldung von einem, dessen letzte Physikstunde 26 Jahre zurückliegt:

    Danke! Für sowas lese ich als Laie Wissenschaftsblogs.

    Ich hatte immer so grob im Kopf: Für den Mesokosmos reicht der olle Newton, für den Mikrokosmos braucht man die Quantenmechanik, für den Makrokosmos die RT.

    Dass letztere auf der atomaren oder subatomaren Ebene eine Rolle spielt, z.B. für die Farbe von Gold, war mir bisher entgangen.

    Gibt es so eine Beziehung denn auch umgekehrt? Braucht man Quantenmechanik, um, sagen wir mal, bestimmte Beobachtungen im Zusammenhang mir Supernovae zu erklären?

  587. #589 MartinB
    19. Juni 2013

    @Quercus
    Aber Hallo…
    Ohne zum Beispiel die massive Entstehung von Neutrinos (die viel Energie wegtragen) gäbe es keine Supernovae. Generell kann man Sterne nur mit den Mitteln der QM verstehen – auch weiße Zwerge z.B. sind durch quantenmechanische Effekte stabil (die Elektronen weigern sich, dieselben Energieniveaus einzunehmen); die Kernfusion in der Sonne funktioniert auch nur, weil die teilchen eine gewisse quantenmechanische Wahrscheinlichkeit haben, die elektrische Abstoßung zu überwinden. Ohne QM funktiioniert die Astronomie nicht.

  588. #590 Alderamin
    19. Juni 2013

    @Quercus

    Braucht man Quantenmechanik, um, sagen wir mal, bestimmte Beobachtungen im Zusammenhang mir Supernovae zu erklären

    Die Helligkeitsabnahme einer Supernova (hier: Supernova 1987A) wird wesentlich durch den Zerfall von radioaktiven Isotopen (Ni-56, Co-56) bestimmt, die bei der Explosion entstanden. Der Zerfall wie auch die Entstehung durch Fusion sind natürlich quantenmechanische Prozesse.

    Auch die Fusion im Sterninneren ist QM, wie Martin schon sagte. Es ist der Tunneleffekt, der die Fusion überhaupt erst möglich macht, denn klassisch betrachtet sollten die Kerne beim Druck und der Temperatur in der Sonne noch gar nicht zusammenstoßen. Sie tun es dennoch, aber extrem selten: die mittlere Zeit für ein Proton, mit einem anderen zu fusionieren, beträgt 14 Milliarden Jahre (!). Die Sonne erzeugt in ihrem Kern gerade einmal 140 W pro Kubikmeter, nicht mehr als ein Komposthaufen (siehe deutschen Wiki-Artikel zur Sonne). Aber sie ist halt ein sehr, sehr großer Komposthaufen, und die Leistung summiert sich auf insgesamt 3,9*10^26 W, entsprechend 620 Millionen Tonnen fusionierten Wasserstoffs pro Sekunde (siehe englischen Wiki-Artikel zur Sonne).

    Darüber hinaus ist der Urknall bis zur Entstehung der Elemente (Baryogenese) QM.

  589. #591 Alderamin
    19. Juni 2013

    @myself

    Darüber hinaus ist der Urknall bis zur Entstehung der Elemente (Baryogenese) QM.

    Ergänzung: Wobei es hier nur um Wasserstoff und Helium sowie ein wenig Lithium geht, alles andere entstand später in Sternen.

  590. #592 Quercus
    19. Juni 2013

    @MartinB

    Aber Hallo…
    (…)
    Ohne QM funktiioniert die Astronomie nicht.

    ok, dann war mein Beispiel nicht ganz durchdacht gewählt. Dass da draußen Dinge geschehen, für deren Verständnis man die QM braucht, war mir schon klar, auch wenn ich die konkreten Beispiele nicht hätte nennen können. Aber diese Dinge passieren ja zwar in von uns aus gesehen riesigen Entfernungen, dort aber auf atomaren oder subatomaren Ebene, und addiert kann dann z.B. sowas rauskommen wie eine Supernova (bitte korrigiert mich, wenn ich einen grundsätzlichen Denkfehler begehe).

    Mal ein anderes Beispiel: Entfernungs- oder Geschwindigkeitsbestimmungen in der Astronomie funktionieren nur, wenn man relativistische Effekte berücksichtigt. Muss man nun ggf. dort auch quantenmechanische Effekte berücksichtigen?

  591. #593 MartinB
    20. Juni 2013

    @Quercus
    Direkt würde ich erst einmal sagen, nein – solange es nur um die Ausbreitung des Lichts geht, kommt man mit der klassischen Elektrodynamik aus, denke ich. Aber spätestens, wenn man an so etwas wie Spektrallinien denkt, dann geht’s nicht ohne QM – die sind klassisch nicht gut zu erklären.

  592. #594 Alderamin
    20. Juni 2013

    @Quercus

    Sehe ich auch so wie Martin. Zwar werden Spektrallinien zur Messung der Dopplerverschiebung und damit der klassischen Radialgeschwindigkeit bzw. der kosmologischen Expansion verwendet, aber ansonsten sind, soweit ich das sehe, keine QM-Effekte beteiligt. Entfernungen werden geometrisch (Parallaxe), optisch (Standardkerzen: Hauptreihensterne mit bekannter Helligkeit, Cepheiden oder Supernovae, hellste Mitglieder von Galaxienhaufen u.ä.) oder über die kosmologische Rotverschiebung (kalibriert an anderen Verfahren) gemessen. Geschwindigkeiten ebenfalls geometrisch (Eigenbewegung in Winkel pro Zeiteinheit), optisch (Radialgeschwindigkeit aus Dopplereffekt) oder bei Pulsaren auch aus der zeitlichen Variation ihrer Pulse abgeleitet. Und dann gibt es indirekte Methoden, wie etwa bei Transits, wo ein Planet oder Stern einen anderen partiell bedeckt und man wenigstens einen Durchmesser kennt (z.B. aus der Spektralklasse des Sterns geschätzt). Dazu verwendet man dann die Lichtkurve des Systems. Schließlich fallen mir noch Gravitationslinsen ein. ART aber keine QM.

  593. #595 Elektron
    19. August 2013

    Hallo Leute,

    ich hätte da mal eine Frage, die zumindest ansatzweise zum Thema dieses Artikels passt. Es geht dabei um ein Wunderkind namens Jacob Barnett aus den USA , laut deutschen Zeitungen soll dieser Einstein mit seinen Theorien blamiert haben bzw. im Begriff sein dies zu tun. Dies ist natürlich ein “gefundenes Fressen” für RT-Kritiker aber um diese geht es mir nicht. Was ich wissen will ist, was hat dieser Junge denn nun erdacht, sprich welche Theorien hat er entwickelt, weiß vielleicht jemand von euch etwas genaues?

    MfG Elektron

  594. #596 MartinB
    19. August 2013

    @Elektron
    Keine Ahnung, eine kurze google-Suche gab nichts außer nen Link auf einen youtube-Video und ein paar Online-Meldungen.
    Solange das, was immer er behauptet, nicht irgendwo zum Nachlesen steht, kann man es sicher getrost ignorieren.

  595. #597 Alderamin
    19. August 2013

    @MartinB

    Hat mich neugierig gemacht.

    Er hat eine Hompage und von da aus gibt’s einen Link auf ein Paper: https://pra.aps.org/abstract/PRA/v84/i2/e024103 . Geht bei ihm wohl hauptsächlich um Quantenmechanik

    Was ich sonst gefunden habe ist, dass er ausgerechnet haben will, das Universum müsse 21 Milliarden Jahre alt sein, um die Menge an Kohlenstoff generiert zu haben, die es im All gibt, und ich denke, da irrt er.

    Außerdem soll er behaupten, das Geschwindigkeitslimit der Relativitätstheorie habe einen etwas höheren Wert als die Lichtgeschwindigkeit:

    “So personally I believe that there would be a different universal speed limit, that isn’t the speed of light, but probably slightly larger than it. Maybe point-9 times larger. Haven’t quite worked it out yet, to the point that it is actually related to the density of the universe.”

    (“Point-9 times” wäre aber doch wohl kleiner?)

    Jedenfalls ein bemerkenswerter junger Mann. Ein paar Jahre weiter und mal schauen.

  596. #598 Elektron
    19. August 2013

    @MartinB, Alderamin:

    Vielen Dank für eure schnellen Antworten!

    MfG Elektron

  597. #599 Erik
    26. November 2013

    Erklärung der Relativitätstheorie an zwei kurzen Beispielen:

    1. Zwei Astronauten driften im Weltall voneinander weg. Der erste Astronaut denkt sich: “Weil der andere von mir wegschwebt, muss er einer höheren Geschwindigkeit ausgesetzt sein und deshalb langsamer altern als ich.” Der zweite Astronaut denkt genau dasselbe über den ersten. Und laut Relativitätstheorie haben beide recht.

    2. Zwei Raumschiffe schweben antriebslos im All mit 1000 km/h aufeinander zu. Der Kommandeur des ersten Raumschiffs teilt dem zweiten Raumschiff mit: “Ihr fliegt mit 1000 km/h in unsere Richtung. Geht endlich auf einen anderen Kurs!” Da meldet sich jemand vom zweiten Raumschiff: “Nein, ihr fliegt mit 1000 km/h auf uns zu.” Wieder liegen beide richtig. Falls noch eine dritte Person behauptet: “Wir fliegen mit 200 km/h und ihr da drüben mit rund 800 km/h”, dann liegt laut Relativitätstheorie auch diese Person mit ihrer Aussage richtig, weil alles relativ(istisch) ist. Wer es nicht glaubt, ist selbst schuld.

  598. #600 Erik
    26. November 2013

    Stimmt’s? Das ist doch die Relativitätstheorie!

  599. #601 MartinB
    27. November 2013

    @Erik
    Nein. Punkt 2 ist die handelsübliche Galilei-Invarianz – dass Geschwindigkeiten relativ zueinander sind, gilt schon in der klassischen Newtonschen Physik.
    Punkt 1 ist nicht “die Relativitätstheorie”, sondern eine sehr vereinfachte Darstellung eines Aspekts.

  600. #602 MB
    6. Dezember 2013

    z. Hd. Alderamin (zu #584): Vielen Dank für die Antwort. Gerne bin ich bereit, auf die Fragen einzugehen.

    Punkt 1:
    MB: „Die Behauptung von Experten, das Licht einer 13 Milliarden Lichtjahre entfernten Galaxie brauche 13 Milliarden Jahre bis zu uns, ist im Normalfall in zweifacher Hinsicht ein Trugschluss.“

    Alderamin: „Hättest Du mal eine Referenz, wo so etwas behauptet wird? Die Experten kennen sehr wohl den Unterschied zwischen der Laufzeitentfernung und der mitbewegten Entfernung, den Du hier ansprichst.“

    Ich kann amerikanische Physiker anbieten, die so etwas in Dokus über das Universum auf ntv gesagt haben.

    Punkt 2:
    MB: „Durch seine tatsächliche Verhaltensweise reist das Licht auf einer ‘gezackten’ Bahn, weshalb man zwischen der Brutto-Geschwindigkeit des Lichts, die immer 299.792,458 km/s ausmacht, und seiner Netto-Geschwindigkeit unterscheiden muss.“

    Alderamin: „Wie soll es das denn bewerkstelligen?“

    Licht reagiert auf andere Geschwindigkeiten. Wenn eine Galaxie mit 1/3 c von A nach B unterwegs ist, kann das Licht nur noch mit 2/3 c autonom reisen.

    A
    .. |
    .. |________X
    B \
    … \
    … . \
    … … \ Y

    Da die Strecke AB 100.000 km misst, kann das Licht nur noch 200.000 km (je Sekunde) eigenständig zurücklegen (z. B. die Strecke BX oder BY). Sonst würde sich eine Überlichtgeschwindigkeit ergeben. Natürlich bewegt sich das Licht nicht zuerst von A nach B und dann nach X (bzw. Y), sondern es bewegt sich entlang der Strecke AX (bzw. AY). Das Licht ‘wandert’ um den Weg AX (bzw. AY) hin und her, wodurch es brutto 300.000 km zurücklegt, netto reist es aber von A nach X (bzw. nach Y). Es entsteht eine ‘Zick-zack’-Bahn mit den kleinsten Weg- und Zeiteinheiten (wie nach Max Planck).
    Für Lebewesen eines Planeten, die die Bewegung von A nach B mitmachen, legt das Licht nur 200.000 km von B nach X (bzw. nach Y) zurück. Weil die Lebewesen der Geschwindigkeit von 1/3 c ausgesetzt sind, unterliegen sie der Zeitdilatation, wobei eine Sekunde auf 0,6666 Sekunden gedehnt wird. Aus diesem Grund misst man für das Licht immer den Wert c. Sollten weitere Geschwindigkeiten auftreten, reagiert das Licht auch auf diese, und die Zeitdilatation sorgt wieder dafür, dass für das Licht der Wert c gemessen wird.

    Punkt 3:
    MB: „Warum sollen sich Lichtstrahlen aus einer anderen Galaxie bei uns anders verhalten als unsere ‘eigenen’ Lichtstrahlen?“

    Alderamin: „Warum sollen sie sich in einer anderen Galaxie anders verhalten als bei uns?“

    Weil andere Galaxien anderen Geschwindigkeiten ausgesetzt sind. Wenn eine andere Galaxie mit 3/4 c unterwegs ist, steht dort für das Licht nur 1/4 c frei zur Verfügung. Und die Zeitdilatation sorgt dafür, dass Lebewesen auch in jener Galaxie für das Licht immer den Wert c messen.

    Punkt 4:
    MB: „Ebenso sind Aussagen von Experten wie ‘eine Reise zu einem 150 Lichtjahre entfernten Planeten wird nie möglich sein bzw. Menschen würden altersbedingt zuvor sterben’ blanker Unsinn. Wegen der Zeitdilatation, die nachweislich existiert, vergeht die Zeit bei Hochgeschwindigkeitsreisen viel langsamer.“

    Alderamin: „Wissen wir. Das Problem ist, annähernd auf solche Geschwindigkeiten zu kommen.“

    Hier herrscht Konsens. Als ‘Belege für die SRT’ wurden angeführt: Messungen der Zeitdilatation in Teilchenbeschleunigern, an Myonen oder mit Uhren in Flugzeugen.
    Das sind nur Belege für die Zeitdilatation. Auch die Aberration oder die Auswirkungen der Kräfte ließen sich widerspruchsfrei erklären. Anziehungskräfte bewirken Anziehungsgeschwindigkeiten, auf die das Licht reagiert.

    Ebenso wurde erwähnt: „Die Lichtablenkung (Aberration), der Dopplereffekt und die Lorentzkraft. Für sich genommen sind diese Effekte und ihre Erklärung durch die SRT sicherlich schon gute Argumente und jeder Kritiker der SRT täte gut daran, sie alle ernst zu nehmen, aber sind das nicht eher sehr spezielle Effekte? Könnte man die nicht auch wirklich anders erklären.“

    Genau das wurde versucht: Die Erklärung dieser Effekte auf eine andere Weise.

  601. #603 ArnaudAntoineA
    Atommodelle
    21. Dezember 2013

    Feynman-Diagramme VERALTET !

    Bitte besuchen Sie mein BLOG –> arnaudantoineandrieu.wordpress.com

    Danke.

  602. #605 MartinB
    22. Dezember 2013

    @Antoine
    Feynmandiagramme sind keine netten Bildchen, die man durch andere Bildchen ersetzen kann – Feynmandiagramme sind in Bildsprache aufgeschriebene Formeln. Ersetzen kann man die nur durch Formeln, nicht durch Bildchen ohne Erklärung.

    @alle
    Die Mühe des Anklickens könnt Ihr euch sparen, auf dem Blog gibt es eine Handvoll Diagramme mit Linien ohne jeder Erklärung; keine Formeln, keine Rechnungen, keine Vorhersagen, ergo: Keine Physik.

  603. #606 rolak
    22. Dezember 2013

    könnt Ihr euch sparen

    Zu spät – und eigentlich doch nicht, bin ich doch genau davon ausgegangen, MartinB, ist doch hinter einer solchen Betextung analog zu, passend zum Thema des threads relativ viel Falsches zu erwarten.
    ‘Holomovement’ ist imho nicht nur spaßig konstruiert, sondern läßt auch Entspannendes aus alten Tagen anklingen. Um dann doch aufs Neue enervierend zu sein.

  604. #607 ArnaudAntoineA
    Feynman-diagramme veraltet
    23. Dezember 2013

    @MartinB
    Guten Morgen. Die spezifische Prinzip meine Grafik, Ist genau alle netten Bildchen zu einem großen Bild.

  605. #608 MartinB
    23. Dezember 2013

    @Antoine
    Das sagt nichts aus. Wie gesagt, Physik braucht Formeln.

  606. #609 ArnaudAntoineA
    Das sagt nichts aus. Wie gesagt, Physik braucht Formeln.
    28. Dezember 2013

    Nichts ändert sich. Es hält sich immer auf das gleiche Prinzip der Vektorausdruck.
    Ich habe einfach angeordnet, die Geometrie … und resultierte aus dem Puzzle wird in “all in ein”. Ein bisschen wie ein Kurzschluss. Ansonsten und In der absoluten einfachsten Formeln werden sein: Energie im Quadrat. (E²) ..und Zeit bleibt proportional zur Entfernung und / oder das Volumen für jede Masse, wie das Photon. Es gibt nur die Masse und Energie ; und dies Masse / Energie können so einen Raum in Zeit zu besetzen. Diejenigen, die Zeit sagen wollen existierte vor dem Urknall. Das ist, was meine Grafik zeigt durch, die Entstehung.
    Bitte entschuldigen Sie mein Deutsch,
    Herzlich.

  607. #610 ArnaudAntoineA
    28. Dezember 2013

    PostScriptum:
    Die vektoriell annihilation zwischen eines Elektrons / Photon kann nicht anders sein. Der einzige Unterschied zwischen Feynman-Diagramme und mich ist, dass ich setzen könnte ein tailed und ein kopf. Würde ich falsch?

  608. #611 MartinB
    28. Dezember 2013

    @Arbaud
    Vielleicht liegt es an der Sprache, aber ich habe kein wort verstanden.

  609. #612 ArnaudAntoineA
    Feynmandiagramme veraltet
    29. Dezember 2013

    @MartinB
    Guten Morgen. Das ist, warum ich Zeichnungen. Ich Suche nach einem Partner, um ein Wort auf meiner Grafiken setzen. Wären Sie daran interessiert? In allen dem Fall, ich danke Ihnen für mich zu beantworten.

  610. #613 Cryptic
    29. Dezember 2013

    @MartinB

    ich wundere mich, dass Sie immer noch an “relativistische Effekte” glauben (in der letzten Zeit ist die Kritik an der RT erheblich zugenommen). Welche Effekte wären das, die nicht “klassisch” genauso gut erklärt werden könnten?

  611. #614 MartinB
    29. Dezember 2013

    @Antoine
    Nein, ich habe kein Interesse.

    @Cryptic
    Wie wäre es, den Artikel oben zu lesen?

  612. #615 Cryptic
    29. Dezember 2013

    @MartinB

    Den Artikel habe ich gelesen. Die Beweise überzeugen mich nicht weil keine klare Verbindung zur SRT besteht bzw. die Betrachtung ein bevorzugtes Bezugssystem voraussetzt (z.B. Myonen-Experiment, Hafele-Keating usw.). Oder konkret, die “relativistische” Energiezunahme der bewegten geladenen Teilchen wurde bereits gegen Ende des 19-ten Jahrhunderts von Heaviside, Searle ua. “klassisch” berechnet.

  613. #616 MartinB
    29. Dezember 2013

    @Cryptic
    Wie ich oben schon schrieb:
    “You cannot reason people out of a position that they did not reason themselves into.”

    Falls du jetzt erwartest, dass ich dieselben Argumente immer wieder wiederhole, bis du auch überzeugt bist – dazu fehlen mir zeit und Lust.

    Lege bessere Erklärungen für die Vielzahl der beschriebenen Phänomene vor (aber nicht hier im Blog, sondern in entsprechenden Fachzeitschriften, dafür sind die da) – auf eine nicht-relativistische Berecnung des g-Faktors oder des Myonenstreuquerschnitts wäre ich ja sehr gespannt.

  614. #617 Cryptic
    29. Dezember 2013

    Sie verlangen das Unmögliche! Seit wann können Mainstreamkritische Erklärungen in Fachzeitschriften publiziert werden? Sie wissen bestimmt was mit KPK passiert ist nachdem dort versucht wurde von einem Äther (Lichtträger) zu sprechen.

  615. #618 MartinB
    29. Dezember 2013

    Ach ja, die gute alte Verschwörungstheorie obendrauf. Wie absurd das ist, habe ich ja oben schon geschrieben.

  616. #619 Cryptic
    30. Dezember 2013

    Von einer Verschwörung zu sprechen ist überhaupt nicht absurd. Absurd ist dass man auf der einen Seite von leerem “Raum mit Eigenschaften” spricht und auf der anderen die Existenz des Äthers leugnet. “Absolute Leere mit Eigenschaften” – wie absurd ist das?
    Und dann erfindet man “virtuelle” Teilchen die aus dem Nichts herausspringen und wieder in Nichts verschwinden. Für die moderne Physik ist überhaupt kein Problem dass dabei alle Erhaltungssätze verletzt werden.

  617. #620 rolak
    30. Dezember 2013

    in der letzten Zeit ist[!] die Kritik an der RT erheblich zugenommen

    Manchmal frage ich mich, ob dergleichen einer durch was auch immer (¿Weihrauch?) ausgelösten verstärkten Eigenweltdiskussion zuzurechnen ist oder doch etwas Realerem, einem Zusammenschluß der Napoleons der Abteilung.

  618. #621 Harti
    Münster
    30. Dezember 2013

    Wie sollen bei einer raumzeitlichen Betrachtung(Vereinigung von Raum und Zeit) relativistische Effekte (z.B. Zeitdilatation) zustande kommen ?
    Beispiel Zwillingsparadoxon: Beide Zwillinge bewegen sich von der Abreise (Ereignis A) zum Wiedersehen (Ereignis B).
    Der Erdzwilling, räumlich ruhend vorgestellt, bewegt sich mit mit dem Vektor v1 nur in der Zeit von A nach B, der Reisezwilling bewegt sich in Raum und Zeit mit dem Vektor v2 von A zum Wendepunkt und von dort mit dem Vektor v3 zum Wiedersehen (Ereignis B).
    v1 = v2 +v3
    Die Bewegungen beider Zwillinge erfolgen vom Ereignis A zum Ereignis B. Sie sind raumzeitlich(ohne Trennung von Raum und Zeit) nicht zu unterscheiden.
    Ergebnis: Die relativistischen Effekte sind eine Folge der getrennten Betrachtung von Raum und Zeit. Ihr Auftreten hängt von der Geometrie ab, die man der Betrachtung zugrunde legt.

  619. #622 MartinB
    30. Dezember 2013

    @Cryptic
    Was die Wortspiele angeht – “Äther” war ein wohldefiniertes Konzept (eine Substanz, die den Raum erfüllen sollte und die nicht Galilei-Invariant war). Dieses Konzept wurde widerlegt. Die Tatsache, dass die Raumzeit Eigenschaften hat (alles worüber man sprechen kann, muss Eigenschaften haben…), hat damit nix zu tun.

    “Für die moderne Physik ist überhaupt kein Problem dass dabei alle Erhaltungssätze verletzt werden.”
    Wenig überrraschend, dass du das mit den virtuellen teilchen auch nicht verstanden hast – nein, Prozesse mit virtuellen Teilchen verletzen keine Erhaltungssätze, auch wenn das manchmal in übermäßig vereinfachten Darstellungen so aussieht.

    @rolak
    Ich denke, das liegt einfach daran, dass die “Kritiker” dank Internet besser auf sich aufmerksam machen können als früher, wo die RT-Leugner vor den Physikhörsälen standen und dort Zettelchen verteilten. Und ansonsten kann man natürlich viel behaupten – ob die Behauptung stimmt, ist was anderes.

    @Harti
    Du hast nicht verstanden, was die Raumzeit ist. Mit der gleiche Logik könnte ich sagen: Es ist egal, ob ich von hier zum Bäcker auf dem direkten Weg oder über Rom fahre. Das zwillingsparadoxon in der Raumzeit ist übrigens in meiner Serie zur Raumzeitkrümmung ausführlich diskutiert, falls du ernsthaft verstehen willst, was es damit auf sich hat.

  620. #623 rolak
    30. Dezember 2013

    besser auf sich aufmerksam machen können

    Unbestritten, MartinB, doch mir ging es nicht um eine verstärkte Wahrnehmung bei mir, sondern um das Erleben der Betroffenen, das vermeintliche Erkennen des Erstarkens der eigenen Seite, das des Schwächelns der anderen. Www gibt es zwar schon 20 Jahre, doch es kann selbstverständlich durch den erleichterten und damit automatisch stärker genutzten Schnellweg zur Öffentlichkeit bedingt sein, kann ebenso selbstverständlich schon deswegen von mir nicht richtig eingeschätzt werden können, daß mir eben jener Hang zum unreflektierten Eigenweltbau fehlt, doch all das scheint mir halt nicht Erklärung genug.
    Nu, vielleicht ist mit ‘in der letzten Zeit’ ja auch ‘in den letzten 108 Jahren’ gemeint.

    dort Zettelchen verteilten

    Kann sein, daß ich die damals nicht wahrgenommen habe, doch afaik standen dort keine Häretiker sondern Agitatoren, also nicht Eris’Speerspitze sondern Marx’Zeugen und Nachfolger Adolfs, später dann auch Jünger Muns.

  621. #624 MartinB
    30. Dezember 2013

    Also bei uns stand vor dem Physik-Hörsaal alle 1-2 Wochen einmal einer, der Zettel gegen die SRT verteilte. Bei den Mathematen gab es einen, der Zettel hatte, das Pi in Wahrheit eine rationale Zahl ist (3,1428, IIRC).

  622. #625 Cryptic
    30. Dezember 2013

    @MartinB

    Sie schreiben: ““Äther” war ein wohldefiniertes Konzept (eine Substanz, die den Raum erfüllen sollte und die nicht Galilei-Invariant war). Dieses Konzept wurde widerlegt. ”

    Frage: Wie kommen Sie darauf, dass der Äther nicht Galilei-Invariant war und dass “dieses Konzept” widerlegt wurde?

    Es wurde nichts widerlegt und der Äther ist bezüglich mechanischer Vorgänge selbstverständlich Galilei-Invariant.

  623. #626 MartinB
    30. Dezember 2013

    @Cryptic
    Wie schon gesagt, ich habe kein Interesse an dieser Art Diskussion, schon gar nicht mit Leuten, die von Physik keinen Schimmer haben. Wie das ursprüngiche Ätherkonzept funktionierte, kann man in diversen Büchern nachlesen, das werde ich jetzt hier nicht nachbeten.

  624. #627 Cryptic
    31. Dezember 2013

    Aber lieber Kollege, Sie haben mich falsch eingeschätzt. Ich bin Physiker genauso wie Sie. Die moderne Physik ist für mich ein baufälliger Turm von Babel (wie einer sagte). Diese ganze Mystik, welche auch Sie hier vertreten, hat mit der Physik überhaupt nichts zu tun! Was die SRT betrift, nicht einmal eine saubere Ableitung der Lorentz-Transformation existiert bis heute! Und wie gesagt, die absolute Leere kann keine Eigenschaften haben, außer leer zu sein.

  625. #628 Cryptic
    31. Dezember 2013

    rolak: “Manchmal frage ich mich, ob dergleichen einer durch was auch immer (¿Weihrauch?) ausgelösten verstärkten Eigenweltdiskussion zuzurechnen ist oder doch etwas Realerem, einem Zusammenschluß der Napoleons der Abteilung.”

    Ich meinte die Kritik wie beispielsweise von Alexander Unzicker

  626. #629 MartinB
    31. Dezember 2013

    Zu Unzicker
    https://scienceblogs.de/astrodicticum-simplex/2010/09/10/durchgefallen-vom-urknall-zum-durchknall/

    Dass Unzicker die Relativitätstheorie ablehnt, ist mir aber neu – mag ja durchaus sein, gibt’s dafür ne Quelle?

  627. #630 rolak
    31. Dezember 2013

    Schönen Dank für die Aufklärung – also trifft der zweite Teil zu. Realitätsverweiger fühlen sich von sich untereinander unterstützt. Geht doch nichts über einen unhinterfragten Binnenkonsens…

  628. #631 MartinB
    31. Dezember 2013

    @rolak
    So groß ist die Einigkeit gar nicht, laut dieser Quelle
    https://www.spektrum.de/rezension/vom-urknall-zum-durchknall/1040493
    hält Unzicker die RT für gesichertes Wissen:
    “Was Unzicker als (halbwegs) gesichert erachtet, ist der Kern der klassischen Physik, die klassische Quantenmechanik und -elektrodynamik sowie die spezielle und die allgemeine Relativitätstheorie (SRT und ART).”
    Er wird also von den RT-Leugnern zu Unrecht vereinnahmt, wohl nach dem Motto: “Der Feind meines Feindes ist mein Freund” oder so.

  629. #632 Harti
    31. Dezember 2013

    @MartinB
    Ist es raumzeitlich nicht tatsächlich egal, ob ich mich direkt oder über Rom zum Bäcker begebe ? Lediglich die Beziehung zwischen Raum und Zeit (Geschwindigkeit in einem allgemeineren Sinn) muss ich ändern, um mich selbst dort zu treffen. Auf direktem Weg:, wenig Raum und viel Zeit (langsam) auf dem Weg über Rom: viel Raum und wenig Zeit (schnell). Eine Änderung der Geschwindigkeit ist bei einer Vereinheitlichung von Raum und Zeit nicht darstellbar, es sei denn, man greift auf die Trennung von Raum und Zeit zurück. Raumzeitlich gibt es nur Bewegungen von Ereignis A zu Ereignis B, genannt Weltlinie.
    Möglicherweise erscheint die Vorstellung, dass es raumzeitlich egal ist, ob ich mich direkt oder über Rom zum Bäcker begebe, nur deshalb so abstrus, weil wir so sehr in der Vorstellung der Trennung von Raum und Zeit verhaftet sind ?

  630. #633 MartinB
    31. Dezember 2013

    @Harti
    Dass Raum und Zeit zur Raumzeit vereint sind, heisst nicht, dass raum und zeit dasselbe sind – so wie unterschiedliche Himmelsrichtungen auch nicht dasselbe sind.

    “Eine Änderung der Geschwindigkeit ist bei einer Vereinheitlichung von Raum und Zeit nicht darstellbar, es sei denn, man greift auf die Trennung von Raum und Zeit zurück. Raumzeitlich gibt es nur Bewegungen von Ereignis A zu Ereignis B, genannt Weltlinie.”
    Aber die Weltlinie des Weges über Rom ist eine andere als die des direkten Weges???

    Oder willst du auf so etwas wie das prinzip der maximalen Eigenzeit hinaus? ich habe ehrlich gesagt keine Ahnung, was du eigentlich zu sagen versuchst.

  631. #634 rolak
    31. Dezember 2013

    von den RT-Leugnern zu Unrecht vereinnahmt

    Gerne würde ich jetzt anführen, mich undeutlich ausgedrückt zu haben, MartinB – doch dazu wurde überhaupt nichts gesagt. Schlimmer noch: Vorsätzlich 😉
    Ok, das mit den Napoleons war vielleicht etwas irreführend, soviel Gleichheit sollte garnicht angedeutet werden. Das Geschehene ist allerdings der typische Schulterschluß unter den Renitenten, ach, Sie sind auch gegen den etablierten Wissenschaftsbetrieb? Nein, erzählen Sie nichts, will ich garnicht so genau wissen, wir kämpfen ab sofort gemeinsam. In Fällen, da ein Torso vom SchwarzenRitter™ nicht ausreicht oder sich schlicht nur die Gelegenheit bietet, schließen sich mehrere zusammen – und sind erstaunlicherweise der Meinung, daß sie fürderhin etwas mit Hand und Fuß zustande brächten.

    Eines allerdings kann aus diesem Verhalten gelernt werden, dem vorgeturnten Beispiel entsprechend: Die Menge an Aufmerksamkeit, die den Thesen eines aus diesem Kreise als Unterstützer Herbeigezogenen gewidmet werden sollte, ist mit ‘Null’ noch freundlich umschrieben.

    btw: Gibt es eigentlich eine Hitliste der wahnhaft Impersonierten, x%Napoleon, y%Jesus etc? Das allererste Bild in mir ist aus welchem Grunde auch immer der Kleine mit der Hand in der Weste.

  632. #635 Cryptic
    31. Dezember 2013

    MartinB: “So groß ist die Einigkeit gar nicht, laut dieser Quelle
    https://www.spektrum.de/rezension/vom-urknall-zum-durchknall/1040493
    hält Unzicker die RT für gesichertes Wissen:…”

    Lesen Sie einfach weiter und Sie finden mnidestens zwei kritische Stellen.

  633. #636 MartinB
    31. Dezember 2013

    @Cryptic
    Und keine davon bezieht sich auf die SRT, um die es hier geht…
    Auf seiner jomepage hat Unzicker sogar ein ganzes Dokument zur RT, da sagt er soweit ich sehe auch nirgends, dass er die SRT nicht akzeptiert
    http://www.alexander-unzicker.de/Einstein25.pdf‎

  634. #637 Cryptic
    31. Dezember 2013

    Und was ist damit?

    Schließlich lesen wir: “Einsteins Relativitätstheorie (beruht) gerade darauf, dass man mit keinem Experiment … unterscheiden kann, ob man ›ruht‹ oder sich mit gleichmäßiger Geschwindigkeit bewegt. Seit Kurzem gibt es dieses Experiment doch. Die Signale des Mikrowellenhintergrunds sagen ganz klar, dass wir nicht ruhen, sondern uns mit 370 Kilometern pro Sekunde in Richtung des Sternbilds Becher bewegen … Fakt ist, dass der kosmische Mikrowellenhintergrund ein absolutes Bezugssystem definiert.”

  635. #638 MartinB
    31. Dezember 2013

    @Cryptic
    Das ist nichts, was der “Mainstream”-Physik widerspricht – so schlau, dass man den Mikrowellenhintergrund als bezugssystem verwenden kann, sind auch andere Physikerinnen. An der Gültigkeit der SRT ändert das nichts, ebensowenig wie wenn man die Erde oder die Sonne oder sonst etwas als Bezugssystem verwendet.
    Und wie gesagt, du musst belegen, dass Unzicker die SRT für falsch hält, wenn du ihn als zeugen heranziehst.

  636. #639 Cryptic
    1. Januar 2014

    Doch, das widerspricht der “Mainstream”-Physik weil man nach Auffassung Unzickers die absolute Bewegung feststellen kann. Steht oben schwarz auf weiß auch wenn Sie das nicht anerkennen wollen!


    Einsteins Relativitätstheorie (beruht) gerade darauf, dass man mit keinem Experiment … unterscheiden kann, ob man ›ruht‹ oder sich mit gleichmäßiger Geschwindigkeit bewegt. Seit Kurzem gibt es dieses Experiment doch…

  637. #640 MartinB
    1. Januar 2014

    @Cryptic
    Das ist genau so sinnvoll wie zu sagen, dass man absolute Bewegung feststellen kann, wenn man die Bewegung gegen den Schwerpunkt der Milchstraße oder den aller Materie im Universum misst. Klar kann man das – aber die Hintergrundstrahlung ist letztlich genauso ein Objekt wie die Erde oder der materieschwerpunkt; an der Gültigkeit der Gleichungen der Relativitätstheorie ändert das nichts.
    Und auch Unzicker zweifelt die SRT ja nicht an – wie ich oben klar belegt habe. Ich warte nach wie vor auf den Gegenbeleg.

  638. #641 Cryptic
    1. Januar 2014

    Ich rede davon was Unzicker geschrieben hat (steht oben). Ob das sinnvoll ist oder nicht ist eine andere Frage!

  639. #642 Niels
    1. Januar 2014

    @Cryptic
    Das Relativitätsprinzip der SRT postuliert aber nur, dass die Physik in allen Inertialsystemen gleich ist.
    Oder direkt nach Einstein:
    Die Gesetze, nach denen sich die Zustände der physikalischen Systeme ändern, sind unabhängig davon, auf welches von zwei relativ zueinander in gleichförmiger Translationsbewegung befindlichen Koordinatensystemen diese Zustandsänderungen bezogen werden.
    (Für die ART ersetzt man dann einfach Inertialsystem durch Bezugssystem.)

    Wo findest du da einen Widerspruch zur Existenz des Mikrowellenhintergrundes?

    Nochmal ausführlicher:
    https://www.xn--relativittsprinzip-ttb.info/faq/cmb-ausgezeichnetes-inertialsystem.html

    Ich rede davon was Unzicker geschrieben hat (steht oben). Ob das sinnvoll ist oder nicht ist eine andere Frage!

    Als Beleg für in der letzten Zeit ist die Kritik an der RT erheblich zugenommen ist das aber ein bisschen wenig.
    Da fällt dir nur etwas ein, dass du selbst für nicht stichhaltig hältst?

    Ich bin Physiker genauso wie Sie.

    Erstaunlicherweise haben es hier immer nur die Unsinn erzählenden Kommentatoren nötig, ihre (übrigens nicht einmal überprüfbare) Fachkompetenz als Scheinargument anzuführen.

  640. #643 Cryptic
    1. Januar 2014

    Niels: “Das Relativitätsprinzip der SRT postuliert aber nur, dass die Physik in allen Inertialsystemen gleich ist…”

    Und dieses Postulat ist eben falsch.

    Beweis: Schallwellen breiten sich mit einer konstanten Geschwindigkeit (lokal) in alle Richtungen (isotrop) nur in einem einzigen Bezugsystem aus. Alle anderen Bezugssysteme sind nicht “gleichberechtigt”.

    Niels: “Als Beleg für in der letzten Zeit ist die Kritik an der RT erheblich zugenommen ist das aber ein bisschen wenig…”

    Dass die Kritik zugenommen hat, zeigt z.B. dieser Link

    The list includes more than 7000 names…

    Niels: “Erstaunlicherweise haben es hier immer nur die Unsinn erzählenden Kommentatoren nötig, ihre (übrigens nicht einmal überprüfbare) Fachkompetenz als Scheinargument anzuführen.”

    Was sollte ich Ihrer Meinung nach tun? Mich mit dem richtigen Namen anmelden?

  641. #644 Harti
    2. Januar 2014

    @Martin B
    Dass Raum und Zeit nicht dasselbe sind, ist für mich begrifflich klar. Es sind die Physiker, die Raum und Zeit zwecks mathematischer (geometrischer) Darstellung zu Strecken vereinheitlichen. Mir ist klar, dass der Gegensatz zwischen beiden Begriffen mathematisch dadurch gebildet wird, dass Raum und Zeit geometrisch senkrecht aufeinander stehen.
    Mein Problem beim Verständnis der SRT sind begrifflichen Unklarheiten, die zum Teil aus den zugrunde gelegten geometrischen Modellen resultieren.
    Beispiel der Begriff “Ruhe”:
    In einem raumzeitlichen Modell gibt es keine Ruhe. Alles bewegt sich, halt nur in verschiedene Richtungen. Wer in herkömmlicher Vorstellung in seinem Sessel ruht, bewegt sich mit “Lichtgeschwindigkeit” allein in Zeitrichtung.
    In einem Modell, das Raum und Zeit trennt, wird “Ruhe” als Fehlen einer allein räumlichen Veränderung aufgefasst.
    ich könnte viele Beispiele begrifflicher Unklarheiten anführen.
    Konkret bin ich zu der Überzeugung gekommen, dass es in einem vierdimensionalen /raumzeitlichen Modell die Effekte der Zeitdilatation und Längenkontraktion nicht gibt, weil alle Objekte sich in einem Zustand der Gleichwertigkeit von Raum und Zeit befinden, was bei Rückgriff auf die herkömmlichen Raum- und Zeiteinheiten als Bewegung mit Lichtgeschwindigkeit erscheint.
    Ein Beleg dafür sind auch die Darstellungen, die unter dem Stichwort “Gamov`s Radfahrer” angestellt worden sind.
    Mir geht es im Prinzip um das Verständnis der Veränderungen in der Natur und das intellektuelle Instrumentarium, das wir dafür verwenden.

  642. #645 MartinB
    2. Januar 2014

    @Harti
    Raum und Zeit stehen nicht einfach senkrecht aufeinander – denn man kann sie nicht durch eine gewöhnliche Rotation ineinander überführen.

    “In einem raumzeitlichen Modell gibt es keine Ruhe.”
    Das gilt schon in der klassischen Physik bezüglich der Galilei-Invarianz. Newton hat zwar einen absoluten Raum postuliert, aber tatsächlich geben die Gleichungen der klassischen Mechanik das gar nicht her. Siehe auch den aktuellen Text “Ein Teilchen fliegt vonA nach B”, erster Teil.

    “ich könnte viele Beispiele begrifflicher Unklarheiten anführen.”
    Diese ganzen Begrifflichen Unklarheiten rühren daher, dass in der Physik Begriffe eigentlich nie perfekt scharf definiert sind. Man braucht immer sowohl die zugrundeliegenden Gleichungen als auch eine konkrete Anbindung an ein Experiment, um zu verstehen, was die Begriffe bedeuten. Physik ist weder Philosophie noch Mathematik.

    “Konkret bin ich zu der Überzeugung gekommen, dass es in einem vierdimensionalen /raumzeitlichen Modell die Effekte der Zeitdilatation und Längenkontraktion nicht gibt,”
    Das ist sowohl experimentell widerlegt als auch konzeptionell nicht richtig – denn man kann die Zeitdilatation ja mathematisch aus den raumzeitlichen Modellen ableiten.

    Ich kenne zwar das Buch von Gamov, aber wo das Problem mit seinem Radfahrer liegen soll, weiß ich nicht. Hast du bitte eine Quelle?

    @Niels
    Siehste: Weil Schallwellen in einem Medium nicht in allen Bezugssystemen isotrop sind, gilt die SRT nicht. Da hätten wir aber auch wirklich selbst drauf kommen können….

  643. #646 ArnaudAntoineA
    ‘Holomovement’ ist imho nicht nur spaßig konstruiert, sondern läßt auch Entspannendes aus alten Tagen anklingen. Um dann doch aufs Neue enervierend zu sein.
    2. Januar 2014

    @Rolak
    ‘Holomovement’ bedeuten, ein ‘Holobewegung’ (hologramm).
    Ein gutes Beispiel ist: Die Implizite Ordnung nach Bohm.

    https://arnaudantoineandrieu.wordpress.com/gravity-entanglement-and-decoherence/

  644. #647 rolak
    Das ist ein Feld zur Wohnortsangabe, keines um den Versuch einer Bezugnahme zu unternehmen. DIOBohm war übrigens ein sinnleerer Ablenkungsversuch.
    2. Januar 2014

    <A> bedeutet <A>

    Thx, Cpt Obvious, so einen hätte ich auch noch: Geschwafel ist Geschwafel.

  645. #648 Harti
    2. Januar 2014

    @MartinB
    Über Google findet man einiges zum Stichwort “Gamovs Radfahrer”. Danach wird ein Objekt nicht einfach dünner (Längenkontraktion) bei Erhöhung der Geschwindigkeit im Verhältnis zum Bezugssystem, sondern es erscheint lediglich in Bewegungsrichtung verkürzt, weil es seine Raumrichtung ändert, was in Bewegungsrichtung nicht darstellbar ist. Einen Radfahrer, den man ursprünglich in Seitenansicht fahren sieht, sieht man bei Annäherung an die Lichtgeschwindigkeit immer stärker in eine um 90° versetzte Richtung fahren, also in Rückenansicht. Der Radfahrer ist also tatsächlich nicht dünner, egal wie schnell er im Verhältnis zum Bezugssystem fährt. Entsprechendes gilt natürlich für die Zeitdilatation. Den Computersimulationen, die diese Effekte zeigen, liegen wohl raumzeitliche Modelle zugrunde.

    Raum und Zeit sind nach meiner Meinung in unserer Vorstellung ( begrifflich) als Gegensätze konzipiert. Ihre geometrische Darstellung erfolgt deshalb korrekterweise in einem kartesischen Koordinatensystem. Die Beziehung zwischen Raum und Zeit (Geschwindigkeit) erscheint als Funktion und die Vereinheitlichung von Raum und Zeit, die für die vollständige Beschreibungen von Bewegungen erforderlich ist, wird mit Hilfe des Satzes von Pythagoras ermöglicht. In diesem Sinne war meine Äußerung gemeint, dass Raum und Zeit senkrecht aufeinander stehen.

  646. #649 MartinB
    2. Januar 2014

    @Harti
    ” Der Radfahrer ist also tatsächlich nicht dünner, egal wie schnell er im Verhältnis zum Bezugssystem fährt. ”
    Da speieln zwei Effekte zusammen: Das eine ist die Längenkontraktion, das andere die Frage, wie das Licht läuft und was man tatsächlich wegen der unterschiedlichen Lichtlaufwege sieht. Nichts davon wider spricht dem Bild der Raumzeit oder der SRT (die Effekte werden mit Hilfe der SRT berechnet).

    “Ihre geometrische Darstellung erfolgt deshalb korrekterweise in einem kartesischen Koordinatensystem.”
    Nicht wirklich, weil man kartesische Koordinatensysteme einfach rotieren kann; die Rotation in der Raumzeit sieht aber anders aus.

  647. #650 Niels
    2. Januar 2014

    @Cryptic
    Wow, eine Liste mit 7000 Namen von irgendwelchen Menschen, die entweder die SRT, die ART, die Kosmologie, den Urknall, die Maxwell-Gleichungen, die Quantenmechanik, … ablehnen oder für unvollständig halten.
    Prima.
    Aber was hat das damit zu tun, ob die Kritik an der SRT zugenommen hat? Wo versteckt sich der Beleg?

    PS: Der Hinweis auf Ihre Ausbildung bleibt auch dann ein Scheinargument, wenn Sie dafür einen Nachweis liefern würden.
    Das ganze ist nämlich nur ein Autoritätsargument, ein äußerst Schwaches noch dazu. So etwas unglaublich Atemberaubendes ist es nämlich auch nicht, Physiker zu sein.
    Sie haben von SRT aber offensichtlich trotzdem absolut keine Ahnung. Es gibt sehr viele Laien, die sich hier wesentlich besser auskennen.
    Wenn sie die ganze moderne Physik für Mystik und einen “baufälligen Turm von Babel” halten, sieht es beim Rest dieses Fachgebietes anscheinend auch nicht besser aus.
    Ein Nachweis für Ihre angebliche Ausbildung kann daran auch nichts ändern. Da Sie Ihn aber nicht einmal liefern, wird Ihr sowieso schon unglaublich schwaches Autoritätsargument noch ein ganzes Stück schwächer.

    Zur Sache mit den Schallwellen fällt mir echt keine Erwiderung ein, da bin ich sprachlos.

    @Harti
    Vierdimensional ist nicht gleichbedeutend mit dem euklidischen R4. Da gibt es noch unzählige weitere Möglichkeiten, auch wenn man in der Schule in der Regel keine anderen Räume kennenlernt.
    Der vierdimensionale Raum der SRT ist der Minkowski-Raum.
    https://de.wikipedia.org/wiki/Minkowski-Raum
    Da muss man sich aber ein bisschen einlesen, dass wird niemand intuitiv klar, der mit Schulmathematik ein bisschen über Vierdimensionalität philosophiert.

    @MartinB
    Ich behandle die Sache mit den Schallwellen einfach mal als zusätzliche anekdotische Evidenz für meine These. Unsinn erzählen und die eigene (angebliche) Ausbildung als Argument einsetzen sind anscheinend positiv korreliert.

  648. #651 Cryptic
    2. Januar 2014

    @MartinB
    Sie haben oben Myonen-Experiment und Hafele-Keating Experiment als Beweise für die Zeitdilatation aufgeführt. Wikipedia schreibt aber: “Die Zeitdilatation fällt also – wie vom Relativitätsprinzip gefordert – in allen Inertialsystemen symmetrisch aus: Jeder misst, dass die Uhr des jeweils anderen langsamer läuft als seine eigene. ”

    Bemerkung: Was Wikipedia schreibt entspricht genau der Auffassung von Herbert Dingle.

    Frage: Mit welchem Trick wird diese Symmtrie der Lorentz-Transformation “ausgespielt”?

  649. #652 Cryptic
    2. Januar 2014

    Niels: “Das Relativitätsprinzip der SRT postuliert aber nur, dass die Physik in allen Inertialsystemen gleich ist…”

    Wenden Sie das auf Schallwellen, Wasserwellen, Lichtwellen, Phononen usw. an.

  650. #653 rolak
    3. Januar 2014

    bin ich sprachlos

    Kein Wunder, wenn jemand anderes die Schallwellen annektiert, Niels.

  651. #654 MartinB
    3. Januar 2014

    @Cryptic
    “Mit welchem Trick wird diese Symmtrie der Lorentz-Transformation “ausgespielt”?”
    Mit gar keinem.
    Und wenn Sie diese Antwort nicht verstehen, dann haben Sie nicht mal die Grundzüge der SRT verstanden.

  652. #655 Cryptic
    3. Januar 2014

    @MartinB
    Begründung?

    Zu Aberration des Lichts

    Lichtwellen besitzen mechanischen Impuls (Rückstoß bei Emission, Compton-Effekt usw.). Wegen Impulserhaltung ist eine Änderung der Ausbreitungsrichtung von Lichtwellen nicht möglich, auch dann nicht wenn sich das quasi-“masselose” Medium (Äther) bewegt (Mitführung des Äthers durch die Gravitationskraft der Erde).

    Bemerkung: Die immer vorhandenen elektromagnetischen Felder im Äther geben dem Äther eine “Masse”.

  653. #656 stone1
    3. Januar 2014

    Bin zwar nur relativ stark interessierter Laie, solche Diskussionen sind aber immer wieder für den ein- oder anderen Schmunzler gut, wie z.B. o.a. Bemerkung… massiv-ätherisch… das gefällt mir, kommt in mein interessante Wortkombinationen-Archiv.

  654. #657 MartinB
    3. Januar 2014

    @stone1
    Ja, aber außer zum Schmunzeln und als Beleg für’s Dunning-Krüger-Theorem sind diese Diskussionen wenig nützlich. Außerdem sieht man, wie schön die Torpfosten verschoben werden, jetzt geht es plötzlich um die Aberration, die vorher gar kein Thema war…

  655. #658 Harti
    4. Januar 2014

    @MartinB
    Ich maße mir nicht an, die Relativitätstheorien widerlegen zu wollen. Es gibt nur in den üblichen Darstellungsweisen Widersprüchlichkeiten, die ich durch Fragen aufzuklären versuche.
    Beispiel: Es wird gesagt, bei einer raumzeitlichen Betrachtung bewegen sich alle Objekte mit c, also bei Anwendung der üblichen Einheiten mit ca. 300 000 km/sec oder bei Annahme der Gleichwertigkeit von Raum und Zeit mit der dimensionslosen “Geschwindigkeit” 1. Die Anteile der Bewegung in Raum bzw. Zeit sind lediglich unterschiedlich. Wie kann es dann in der Raumzeit zu Längenkontraktion und Zeitdilatation kommen ?

  656. #659 JolietJake
    5. Januar 2014

    Ich maße mir nicht an, die Relativitätstheorien widerlegen zu wollen. Es gibt nur in den üblichen Darstellungsweisen Widersprüchlichkeiten, die ich durch Fragen aufzuklären versuche.

    Das klingt so, als läge der Fehler bei den Erklärern.
    Ehrlicher, und höflicher, wäre zu schreiben: “Ich habe die RT nicht (ganz) verstanden, bitte beantwortet mir meine Fragen. Es gibt da einiges, was mir nicht klar ist.”
    Auf eine ehrlich gestellte Frage, ohne die Implikation eines (ganz allein!) ausgedachten persönlichen Weltbildes, hat, meiner Erfahrung nach, bisher jeder Kommentator eine fundierte Antwort erhalten.

  657. #660 MartinB
    5. Januar 2014

    @Harti
    “Es wird gesagt, bei einer raumzeitlichen Betrachtung bewegen sich alle Objekte mit c,”
    Wer sagt das? (Außer, wenn man den Higgs-mechanismus diskutiert, da ergibt diese Aussage Sinn.) Wo stammt diese Aussage her?

  658. #661 Harti
    5. Januar 2014

    Ich habe das schon in versc hiedenen Darstellungen gelesen, z.B. in Wikipedia zu “Vierergeschwindigkeit” unter der Rubrik “Interpretation”

  659. #662 MartinB
    5. Januar 2014

    @Harti
    Das ist die Norm der Vierergeschwindigkeit, die entpsricht nicht dem Wert der Geschwindigkeit, wie man sie anschaulich versteht. Objekte bewegen sich nicht mit dieser Geschwindigkeit – mit welcher Geschwindigkeit sie sich bewegen, hängt vom Bezugssystem ab, aber das heißt nicht, dass es eine Raumzeitbetrachtung gibt, in der alle Objekte immer Lichtgeschwindigkeit haben.

  660. #663 Rainer Zufall
    5. Januar 2014

    Ich hab mir jetzt nochmal den Artikel von oben durchgelesen.
    Dumme Frage: Kann es sein, das man den einfachsten Beweis dafür, das die SRT nicht falsch sein kann ganz einfach sieht, wenn man nach draußen schaut und die Sonne sieht?
    Ich meine, ohne E=mc² würde schließlich die Sonne nicht leuchten. Wer die SRT kritisiert, muss also auch erklären, warum die Sonne leuchtet und da habe ich bisher noch keine gelesen…?

  661. #664 MartinB
    5. Januar 2014

    @RainerZufall
    Naja, dass Energien ineinander umgewandelt werden können, ist ja auch in der nicht-relativistischen Physik so. Und messen kann man den Masseverlust in der Sonne ja nicht (wohl aber den bei verschiedenen Kernreaktionen auf der Erde; das ist also ein gutes Argument für E=mc^2).
    Aber man kann ja vielleicht auch E=mc^2 akzeptieren, aber Dinge wie Längenkontraktion nicht.

  662. #665 Rainer Zufall
    5. Januar 2014

    ja das war mir schon klar, das man das theoretisch auch mit nichtrelativistischen Effekten wie Energieumwandlung erklären könnte, auch wenn man die Erklärung bisher nicht gefunden hat. Aber das gilt ja für die Belege aus dem Artikel (z.B. Myonen oder die Farbe des Golds) genauso.
    Ich habe das Beispiel nur gebracht, weil es etwas naheliegender ist. Die Sonne kann ich mir nun mal besser vorstellen als Myonen 😉

  663. #666 MartinB
    5. Januar 2014

    @Rainer
    Es ging mir nicht so sehr darum, dass man die Erklärung nicht gefunden hat, sondern darum, dass man aus der Tatsache allein, dass die Sonne viel Energie abstrahlt, nicht auf die Energie-Masse-Äquivalenz schließen kann.

  664. #667 Cryptic
    5. Januar 2014

    MartinB: “…Naja, dass Energien ineinander umgewandelt werden können, ist ja auch in der nicht-relativistischen Physik so. Und messen kann man den Masseverlust in der Sonne ja nicht (wohl aber den bei verschiedenen Kernreaktionen auf der Erde; das ist also ein gutes Argument für E=mc^2)….”

    Und was passiert mit diesem angeblichen Massenverlust später? Diese verloren geglaubte Masse wird von einem anderen Körper (oder von mehreren) wieder “absorbiert” und die Masse von diesem (diesen) wird um genau diesen Betrag größer. Da ging also keine Masse verloren. Das ist das Gleiche wie wenn von einem schnell rotierenden Rad Wassertropfen wegfliegen. Weit und breit keine SRT.

  665. #668 Harti
    6. Januar 2014

    @ MartinB
    “Das ist die Norm der Vierergeschwindigkeit, die entpsricht nicht dem Wert der Geschwindigkeit, wie man sie anschaulich versteht. Objekte bewegen sich nicht mit dieser Geschwindigkeit – mit welcher Geschwindigkeit sie sich bewegen, hängt vom Bezugssystem ab, aber das heißt nicht, dass es eine Raumzeitbetrachtung gibt, in der alle Objekte immer Lichtgeschwindigkeit haben.”

    Muss man daraus nicht die Konsequenz ziehen, dass der Begriff “Geschwindigkeit” in der raumzeitlichen (vierdimensionalen) Betrachtung anders zu definieren ist als in einem Modell, das Raum und Zeit trennt ?
    Mit anderen Worten. Stellt die Angabe einer Beziehung zwischen Raum und Zeit, die herkömmlicherweise aus der Perspektive des Raumes (Strecke/Zeit) definiert ist, nicht automatisch einen Rückgriff auf ein anderes geometrisches Modell als die Raumzeit dar ?
    Welche Bedeutung hat dies gegebenfalls für Zeitdilatation und Längenkontraktion in der Raumzeit ? Sind diese Effekte bei einer “Bewegung” (auch dieser Begriff wäre in der Raumzeit eventuell anders definiert) vom Ereignis A zu Ereignis B darstellbar ?

  666. #669 MartinB
    6. Januar 2014

    @Harti
    “Muss man daraus nicht die Konsequenz ziehen, dass der Begriff “Geschwindigkeit” in der raumzeitlichen (vierdimensionalen) Betrachtung anders zu definieren ist als in einem Modell, das Raum und Zeit trennt ?”
    Nein, daraus muss man nur die Konsequenz ziehen, dass Physiker meist schlampig mit Worten umgehen und Dingen ungeschickte Namen geben. Es ist nie eine gute Idee, aus den bloßen Begriffen der Physik ohne Rückgriff auf die zugrundeliegenden Modelle und Formeln irgendwelche Schlüsse ziehen zu wollen.

    Den letzten Absatz verstehe ich nicht. Begriffe wie “bewegung” sind in der SRT nicht plötzlich etwas ganz anderes, auch wenn die Bewegung natürlich vom Beobachter abhängt (das tut sie in der Newtonschen Physik aber auch).

  667. #670 Harti
    6. Januar 2014

    @MartinB
    “Das ist die Norm der Vierergeschwindigkeit, die entpsricht nicht dem Wert der Geschwindigkeit, wie man sie anschaulich versteht.”
    Aber sie wird doch aus dem Raumzeitmodell errechnet. Bedeutet die Formulierung “anschaulich” in diesem Zusammenhang nicht einfach Rückgriff auf Trennung von Raum und Zeit mit der Definition Strecke/Zeit ?
    “Objekte bewegen sich nicht mit dieser Geschwindigkeit – mit welcher Geschwindigkeit sie sich bewegen, hängt vom Bezugssystem ab,”
    Meines Erachtens kann man für eine raumzeitliche Betrachtung keine anschauliche Geschwindigkeit in deinem Sinne definieren, weil der Gegensatz von Raum und Zeit in dem Modell des Raumzeitkontinuums aufgehoben ist. Raum und Zeit verschmelzen zu einer Einheit.

    Ich möchte noch mal auf meine Ausgangsfrage zurückkommen und sie bildlich formulieren.
    Es wird gesagt, Längenkontraktion und Zeitdilatation seien zwei Seiten derselben Medaille. Bedeutet die Betrachtung von Bewegungen in einem raumzeitlichen Modell nicht die Betrachtung der gesamten Medaille ohne Unterscheidung von Vorder- und Rückseite ? Und die Angabe einer Geschwindigkeit im herkömmlichen Sinn die Herstellung einer Beziehung zwischen diesen beiden Seiten ?

  668. #671 Harti
    6. Januar 2014

    @MartinB
    Ich habe vergessen auf Deine Frage zur Definition des Begriffes “Bewegung” zu antworten.

    Herkömmlicherweise wird “Bewegung” als rein räumliche Veränderung verstanden und als Gegensatz das Wort “Ruhe” gebildet. In einem Raumzeitmodell bewegt sich auch derjenige, der im Sessel sitzt, allerdings, wenn man auf die Trennung von Raum und Zeit zurückgreift, nur in der Zeit.
    Die Grenzen der Bewegungsmöglichkeit in einem Raumzeitmodell sind so bestimmt:
    Keine Bewegung im Raum, nur Bewegung in der Zeit = herkömmlich Ruhe
    und
    Nur Bewegung im Raum, keine Bewegung in der Zeit = herkömmlich Lichtgeschwindigkeit.

  669. #672 MartinB
    6. Januar 2014

    @Harti
    “Rückgriff auf Trennung von Raum und Zeit mit der Definition Strecke/Zeit ?”
    Ja. Raum und Zeit sind ja nicht dasselbe, auch nicht im Bild der Raumzeit. Das raumzeitbild erlaubt es nur, eine Beschreibung zu finden, die es einfach macht, zwischen den unterschiedlichen Ansichten von Raum und Zeit unterschiedlicher Beobachter hin- und herzutransformieren.

    “Raum und Zeit verschmelzen zu einer Einheit. ”
    Jein. Raum und Zeit verschmelzen zwar, aber jeder Beobachter nimmt trotzdem Raum und Zeit unterschiedliche wahr. Wenn du dich drehst, wird aus der Richtung Vorn die Richtung rechts – trotzdem ist es ein Unterschied, ob du geradeaus oder zur Seite gehst.

    “Es wird gesagt, Längenkontraktion und Zeitdilatation seien zwei Seiten derselben Medaille. ”
    Ja, das ist richtig, das eine geht nicht ohne das andere.

    “Bedeutet die Betrachtung von Bewegungen in einem raumzeitlichen Modell nicht die Betrachtung der gesamten Medaille ohne Unterscheidung von Vorder- und Rückseite ”
    Würde ich so nicht sagen. Nimm wieder das Bild eines normalen raumes, zum Beispiel auf der Erdoberfläche. Man kann verschiedene Koordinatensysteme nehmen, um einen Punkt auf der Erde zu beschreiben – aber ganz ohne Koordinaten kommt man nicht aus, wenn man tatsächlich eine konkrete Angabe machen will.

    genauso kann man sich zwar ein Raumzeit-Bild z.B. einer Weltlinie machen, aber wenn man etwas konkretes beschreiben will, dann braucht man ein konkretes bezugssystem.

    Noch ein Tipp: In der Physik ist es immer gut, Fragen an einem konkreten Fall zu diskutieren, nicht an den ganz abstrakten Begriffen. Wenn du konkret an einem Beispiel sagen kannst, was du nicht verstehst, würde das weiterhelfen.

  670. #673 MartinB
    6. Januar 2014

    Den zweiten Kommentar verstehe ich wieder gar nicht – schon in der Newtonschen Physik kann man nicht sagen, wer sich bewegt und wer nicht (Galilei-Invarianz).

  671. #674 Cryptic
    6. Januar 2014

    Walter Orlov: “…Die Lichtgeschwindigkeit als Grenze ist in der Äthertheorie eine Selbstverständlichkeit. Und E = mc^2 hat Hasenöhrl noch vor Einstein ohne jede Relativität hergeleitet… ”

    Woher E=mc² kommt hat Maxwell 1965 “klassisch” erklärt. Das steht auch im Einklang mit Äquipartitionstheorem: Kinetische Energie der Translation + Schwingungsenergie des Photons (ein bewegter harmonischer Oszillator).

    E=mc²/2 + hf/2

    …the energy must have been half in the form of motion of the medium and half in the form of elastic resilience…

  672. #675 Cryptic
    6. Januar 2014

    Korrektur: … Maxwell 1865 …

    Link

  673. #676 Cryptic
    6. Januar 2014

    Harti: “…Nur Bewegung im Raum, keine Bewegung in der Zeit = herkömmlich Lichtgeschwindigkeit.”

    Wie kann sich etwas im Raum bewegen ohne dass Zeit vergeht? Ist die Geschwindigkeit nicht als v=dr/dt definiert? Was ist das für Esoterik?

  674. #677 Niels
    6. Januar 2014

    @Harti

    weil der Gegensatz von Raum und Zeit in dem Modell des Raumzeitkontinuums aufgehoben ist

    Nein, Raum und Zeit sind in der mathematischen Beschreibung durchaus getrennt. Die vierdimensionale Raumzeit besitzt genau eine zeitartige und drei raumartige Dimensionen. Ein euklidischer vierdimensionaler Raum dagegen ausschließlich raumartige Dimensionen.

    Raum und Zeit verschmelzen zu einer Einheit.

    Ich weiß nicht, ob diese Sichtweise besonders hilfreich ist. Das ist zwar wahrscheinlich nicht falsch, vor allem, weil gar nicht klar ist, was “verschmelzen” genau bedeutet. Aber hätte es dir für das Verständnis des dreidimensionalen euklidischen Raumes weitergeholfen, wenn dir jemand erzählt hätte, dass dort Länge, Breite und Höhe “verschmelzen”?

  675. #678 Harti
    7. Januar 2014

    @Niels
    Dass die Raumzeit nichteuklidisch ist, gilt m.E. nur für die ART, soweit ich es verstanden habe.
    Du hast allerdings recht, dass in der SRT die Zeitkoordinate der Raumzeit imaginär vorgestellt werden muss. Der Grund dafür liegt in der begrifflichen Gegensätzlichkeit von Raum und Zeit. Beim Rechnen führt dies dazu, dass der Betrag einer Bewegung eine komplexe Zahl ist, die in der Gauß`schen Zahlenebne aus einem Realteil (Raum) und einem Imaginärteil (Zeit) gebildet wird. Grundlage für die Vereinheitlichung von Raum und Zeit bei einer Bewegung ist auch hier der Satz des Pythagoras, mit dem geometrisch dargestellte Gegensätze vereinheitlicht werden.
    Soweit ich mich hier unfachmännisch ausdrücke, bitte ich um Nachsicht

  676. #679 Harti
    7. Januar 2014

    @Cryptik
    Die Ansicht, dass Photonen sich nur im Raum bewegen, für sie also keine Zeit vergeht, ist wohl herrschende Meinung.
    Im Gegensatz dazu wird für die Bewegung des Lichts eine konkrete Geschwindigkeit von ca. 300 000 km/sec gemessen.

    Diesen Gegensatz aufzuklären ist mein Anliegen. Es liegt m.E. an den verschiedenen Modellen, die den Betrachtungen zugrunde gelegt weden.

  677. #680 Harti
    7. Januar 2014

    @ MartinB
    ich verstehe konkret nicht, wie es möglich ist, dass in der SRT unter Geltung also des Relativitätsprinzips, aus meiner Sicht Myonen langsamer altern und aus Sicht der Myonen ich langsamer altere.
    Dass dies als relativistischer Effekt so ist und keinen Widerspruch darstellt, verstehe ich sehr wohl. In der Realität erscheint mir dies als Widerspruch.

  678. #681 MartinB
    7. Januar 2014

    @Harti
    “soweit ich es verstanden habe.”
    Dann hast du es falsch verstanden. Die Raumzeit hat eine Minkowksi-Metrik, sonst könntest du dich rumdrehen und deine Nase würde in Richtung gester zeigen.

    ” dass in der SRT die Zeitkoordinate der Raumzeit imaginär vorgestellt werden muss.”
    Das ist eine ziemlich veraltete Art, die SRT aufzufassen, besser, man arbeitet mit einer Minkowski-metrik, sonst entsteht genau die Vewirrung, unter der du leidest.

    “Die Ansicht, dass Photonen sich nur im Raum bewegen, für sie also keine Zeit vergeht, ist wohl herrschende Meinung.”
    Nein, so kann man das nicht sagen. Verwendet man die Gleichungen der SRT für ein Photon, dann vergeht zwar für das Photon keine Zeit, aber es legt auch keine Strecke zurück (weil die Längenkontraktion unendlich ist). Letztlich zeigt das nur, dass man sich eben nicht sinnvoll ins Ruhesystem eines Photons setzen kann.

    ” In der Realität erscheint mir dies als Widerspruch.”
    Ich erklärs mal mit ner Analogie: Wenn du 100 Meter weit weg stehst, erscheinst du mir sehr klein – ich dir aber auch. Um unsere Größen tatsächlich zu vergleichen, müssen wir aufeinander zu gehen.
    Genauso beim zwillingsparadoxon: um zu sehen, wer schneller oder langsamer altert, muss einer der Beteiligten umdrehen, und dabei gibt es Beschleunigungen.
    Steht übrigens im 4. teil der Raumzeitserie erklärt, wenn ich mich recht entsinne.

  679. #682 Rainer Zufall
    7. Januar 2014

    “vielleicht kann man ja E=mc² akzeptieren, aber Dinge wie die Längenkontraktion nicht”

    @ -> Kritiker der SRT:
    Akzeptiert ihr E=mc² ?
    Und wenn nicht, warum leuchtet die Sonne???

  680. #683 Niels
    7. Januar 2014

    @Harti @MartinB

    Die Ansicht, dass Photonen sich nur im Raum bewegen, für sie also keine Zeit vergeht, ist wohl herrschende Meinung.

    Harti hat seine Quelle ja schon genannt, nämlich den deutschen Wikipediaeintrag zur Vierergeschwindigkeit:
    Photonen und andere, masselose Teilchen bewegen sich immer mit Lichtgeschwindigkeit durch den Raum und ruhen dafür in der Zeit (Vierergeschwindigkeit nicht definiert).
    https://de.wikipedia.org/wiki/Vierervektor#Interpretation

    Das ist nur leider ziemlicher Mist.
    Das Ruhesystem eines Photons kann schon per Definition kein Inertialsystem sein. Tatsächlich kann man wie von MartinB erwähnt gar kein sinnvolles Ruhesystem finden. Die Eigenzeit ist für masselose Teilchen wie Photonen nicht sinnvoll definierbar. Damit ist auch die Frage, ob für ein Photon Zeit vergeht, sinnlos.

  681. #684 Harti
    8. Januar 2014

    @ MartinB
    “Dann hast du es falsch verstanden. Die Raumzeit hat eine Minkowksi-Metrik, sonst könntest du dich rumdrehen und deine Nase würde in Richtung gester zeigen.”

    Eigentlich müsste in meinen Äußerungen, wenn auch unfachmännisch, durchaus klar geworden sein, dass ich verstanden habe, weshalb die Minkowski-Metrik für die Darstellung von Bewegungen benötigt wird. Ob ich die Zeitachse mit ct oder ict bezeichne, ist wohl nicht entscheidend, jedenfalls haben die Raum- und Zeitkoordinaten entgegengesetzte Vorzeichen.

    In dem von Dir verwendeten Wort “gestern” kommt eine der wesentlichen begrifflichen Unklarheiten der Physik zum Ausdruck. Es betrifft den Begriff der “Zeit”.
    Einstein hat definiert: Zeit ist das, was die Uht anzeigt.
    Dieser Zeitbegriff beinhaltet die reine Dauer, also keine kausalen Abläufe.
    Beispiel: Ich kann auf einer Uhr zwar ablesen, dass ein Fußballspiel 90 Minuten dauert, nicht aber ob es gestern stattgefunden hat oder morgen stattfinden wird.
    Dieser Zeitbegriff hat keine Richtung. Man braucht sich also nicht zu wundern, dass die Zeit in mathematisch formulierten Naturgesetzen zeitinvariant, ich würde sagen richtungsneutral ist.
    Demgegenüber beinhaltet der allgemein sprachliche Zeitbegriff, wie wir Zeit intuitiv vestehen, auch die kausale Abfolge von Vorgängen. Dieser Zeitbegriff hat eine Richtung. Ich kann mit ihm Vergangenheit und Zukunft, vorher-nachher usw. definieren.
    Mir ist bekannt, dass es auch Ansichten gibt, die dem naturwissenschaftlichen Zeitbegriff eine Richtung geben, indem man z.B. die Entwicklung der Entropie als Bestandteil des Zeitbegriffs aufgefasst wird.

    “Genauso beim zwillingsparadoxon: um zu sehen, wer schneller oder langsamer altert, muss einer der Beteiligten umdrehen, und dabei gibt es Beschleunigungen.”
    Muss man die Tatsache, dass der Erdzwilling sich inertial bewegt (nur in der Zeit) und der Reisezwilling sich nichtinertial bewegt, weil er wendet, nicht so interpretieren, dass die Bewegungen im Rahhmen der SRT nicht vergleichbar sind und deshalb das Zwillingsparadoxon auf der Grundlage der SRT nicht lösbar ist ?

  682. #685 MartinB
    8. Januar 2014

    “Muss man die Tatsache, dass der Erdzwilling sich inertial bewegt (nur in der Zeit) und der Reisezwilling sich nichtinertial bewegt, weil er wendet, nicht so interpretieren, dass die Bewegungen im Rahhmen der SRT nicht vergleichbar sind und deshalb das Zwillingsparadoxon auf der Grundlage der SRT nicht lösbar ist ?”
    Nein. Entgegen weit verbreiterter Ansicht kann man auch Beschleunigungen mit der SRT berechnen, siehe z.B. das Buch von MisnerThorneWheeler, da ist das gut erklärt.

    Der Rest ist mir zu schwammig, als dass ich dazu etwas sagen könnte. (Und da es auch gestern Uhren gab, kan ich natürlich feststellen, ob das Fussballspiel gestern stattfand….)

    Und wenn Zeit das ist, was die Uhr anzeigt, dann hat der Zeitbegriff natürlich eine Richtung.

  683. #686 Harti
    8. Januar 2014

    @MartinB
    “Der Rest ist mir zu schwammig, als dass ich dazu etwas sagen könnte. (Und da es auch gestern Uhren gab, kan ich natürlich feststellen, ob das Fussballspiel gestern stattfand….)”
    Ich kann nachvollziehen, dass Du als Physiker kein so richtiges Interesse an der Klärung von Begriffen hast, weil die “Sprache” der Physik die Mathematik ist und man sich dort nicht mit begrifflichen Unklarheiten herumschlagen muss.
    Beim Begriff “Zeit” liegt die Sache aber etwas anders. Dafür gibt es nämlich keine mathematische Beschreibung. Der Buchstabe t leistet dies jedenfalls nicht. Wenn man die Kausalität aus der Begriffsbestimmung herausläßt und die “reine Dauer” als Zeit bestimmt, dies tun die Physiker jedenfalls faktisch,wenn sie Naturgesetze zeitinvariant fassen. Es würden sich auch Diskussionen über Zeitreisen etc. im Zusammenhang mit Zeitvorstellungen erledigen; denn solche Fragen müssten als Kausalitätsfragen erörtert werden, ob z.B. aus der Gravitation als Anziehungskraft eine Abstoßungskraft werden kann.

    “Und wenn Zeit das ist, was die Uhr anzeigt, dann hat der Zeitbegriff natürlich eine Richtung.”
    Das sehe ich anders. Grundlage für die Bestimmung von Dauer bei Uhren sind periodische Vorgänge (Pendel, Unruh, Quarz, Atom). Solche Vorgänge haben keine Richtung, weil nach Durchlaufen einer Periode die räumliche Veränderung 0 ist, zeigen solche Uhren die “reine Dauer” an. Zeiger etc. sind nur eine Visualisierung dieser Dauer.

    Falls Du an weiteren Überlegungen und begrifflichen Klärungsversuchen meinerseits kein Interesse hast, ist es kein Problem mir dies mitzuteilen.

  684. #687 MartinB
    8. Januar 2014

    “Das sehe ich anders. ”
    Dann schau mal deine Pendeluhr genau an und du wirst feststellen, dass es da eine eindeutige Richtung gibt, weil das Pendel sich nach unten bewegt.
    Dass es einen Zeitpfeil gibt und dass man ihn leich tbestimmen kann, ist nicht das Problem – die Frage ist, wie er sich aus fundamentalen Naturgesetzen ableiten lässt.

    “Falls Du an weiteren Überlegungen und begrifflichen Klärungsversuchen meinerseits kein Interesse hast, ist es kein Problem mir dies mitzuteilen.”
    Ich habe prinzipiell nichts gegen begriffliche Klärungsversuche – aber, wie oben schon angemerkt, bitte möglichst an einem konkreten Beispiel, nicht mit abstrakten Wortspielen wie “Für Zeit gibt es keine mathematsische Beschreibung”. Die gibt es auch nicht für Ort oder für elektrisches Feld oder für sonstwas in der Physik. Physikalische Begriffe brauchen immer die Anbindung an ein konkretes Experiment, um sie fassen zu können.

    Für ein bisschen Grundlagenverständnis, wie Physik funktioniert, empfehle ich feynmans “Character of physical law” – gibt’s als Buch und auch irgendwo als Video (habe ich im Sommer 2011 hier auf dem Blog besprchen, da sind auch irgendwo die Links).

  685. #688 Cryptic
    8. Januar 2014

    MartinB: ” Entgegen weit verbreiterter Ansicht kann man auch Beschleunigungen mit der SRT berechnen,…”

    Damit wäre die SRT sofort “erledigt” und zwar schon durch das Magnet-Leiter Beispiel von Einstein.

  686. #689 Cryptic
    9. Januar 2014

    Rainer Zufall:

    “@ -> Kritiker der SRT:
    Akzeptiert ihr E=mc² ?”

    Selbstverständlich, das gilt aber nur für die elektromagnetischen Wellen (wie von Maxwell erklärt).

    mc² = mc²/2 + hf/2 =hf

    Oder Gesamtenergie = kinetische Energie + Schwingungsenergie. Das ist auch “klassisch” richtig.

  687. #690 Georg Haberler
    23. November 2014

    Habe mit einem befreundeten Physiker (Festkörperphysik) eine kleine Arbeit verfasst. Sie zeigt, dass c nicht als Absolutgeschwindigkeit betrachtet werden muß. Unter
    https://www.rapidshare.com/share/CCB8BA4C702DD6E875FC7E00E4B87640 ist sie abrufbar.

  688. #691 MartinB
    24. November 2014

    @Georg
    Ich zitiere mal aus dem text:
    “Ebenso müsste man unendlich viel Energie aufwenden um die Geschwindigkeit eines massehaltigen Objektes auf 0 zu reduzieren.”
    ??? Was soll den der Satz bedeuten???

  689. #692 Georg Haberler
    25. November 2014

    @ MartinB
    Die Teilchen können nicht ganz zum Stillstand gebracht werden, solange nicht unendlich viel Energie aufgewendet wird. Der absolute Nullpunkt, c und v unendlich können nur in einem unendlichen Universum, welches sich selbst unendliche Energiequelle ist erreicht werden. Laut ART ist dies in der der Endsingularität der Fall, da das Universum in York Zeit gegen plus unedlich divergiert. Zwischen der Anfang und der Endsingularität, muss aber laut Frank Tipler tatsächlich unendlich viel Information verarbeitet werden.

  690. #693 MartinB
    25. November 2014

    @Georg
    “Die Teilchen können nicht ganz zum Stillstand gebracht werden, solange nicht unendlich viel Energie aufgewendet wird. ”
    Sorry, aber das ist – zumindest in der klassischen Physik, schlicht falsch. Der Rest hat mit Physik nur noch wenig zu tun (und da beziehe ich den auf Wunschdenken beruhenden Blödsinn von Tipler mit ein).

  691. #694 Georg Haberler
    1. März 2015

    @ MartinB

    siehe https://de.wikipedia.org/wiki/Absoluter_Nullpunkt: “Nach dem dritten Hauptsatz der Thermodynamik ist der absolute Nullpunkt eine ideale Messgröße, welche nicht erreichbar ist, …”

  692. #695 Georg Haberler
    1. März 2015

    @ MartinB

    Da die Universen eben mächtig sind, lassen sie sich auch als ein einziges Universum beschreiben; das würde bedeuten, dass der Absolute Nullpunkt sowohl mit c als auch mit v unendlich im Gleichgewicht steht.
    Gemeinsam mit einem befreundeten Mathematiker (Richard Grabner) entwickeln wir eine auf dem Kontinuum basierende Mathematik unter der Annahme, dass es möglich ist, in endlicher Zeit eine unendliche Informationsmenge (beispielsweise die Zahl pi) zu verarbeiten (in ihrer unendlichen Gesamtheit zu erfassen).
    Die Everett Interpretation der Quantenmechanik legt nahe – dies der Unterschied zur Newtonschen Mechanik – das ein zu untersuchendes Teilchen in Wahrheit eine unendliches (ununterscheidbares) Ensemble an Teichen darstellt, auch wenn wir bei einer Messung immer nur eines wahrnehmen.
    Daraus folgt, dass tatsächlich unedlich viele Teilchen existieren, die numerisch eins (einunddasselbe) Teilchen sind.

  693. #696 MartinB
    1. März 2015

    @Georg
    Ein thermodynamisches System lässt sich nicht auf den absoluten Nullpunkt abkühlen; ein einzelnes teilchen kann aber problemlos beliebig langsam gemacht werden (und schon aus der klassische galilei-Invarianz folgt, dass man sich problemlos ins Ruhesystem eines teilchens setzen kann).

  694. #697 kosmo-erik . . . ..
    3. März 2015

    Interessanter Gedanke, in endlicher Zeit (beschrieben durch die Physik) eine unendliche Informationsmenge (verkörpert durch die mathematische Konstante Pi) in einen Zusammenhang zu stellen . . . ..eine begrenzt, messbare Physik erfährt durch die unendliche Informationsmenge von mathematischen Konstanten (e, pi, i , 1 & 0) einen vollständigen Formalismus . . . .. Ein besseres Wort fällt mir im Moment nicht ein . . . . ..
    Die Thermodynamik erlaubt uns nicht hinter die Grenze von NullgradKelvin zu schauen . . . .. Aber der Formalismus von PhysikMathematik als System von Informationsverarbeitung im Universum bringt neue Denkanstöße für eine vollständige Beschreibung unserer Physik . . . ..

  695. #698 Georg Haberler
    29. April 2015

    Aus unserer Arbeit geht nicht hervor, dass die SRT falsch ist, denn c im einen entspricht ja v unendlich im anderen Universum. Man kann also von einem Doppeluniversum sprechen, wobei unser Universum von jenem identen analogen Universum aus emuliert wird (diese Theorie wird von Mag. Dr. Gebhard Grübel https://www.uibk.ac.at/th-physik/fth/people/gruebl_gebhard.html vertreten (dass unser Universum NUR von einem analogen identen Universum aus emuliert werden könnte). Unsere Arbeit bestätigt diese Theorie.
    Werde in kürze (einigen Wochen) unsere Arbeit ins Netz stellen. Sie wird dann als MP3 zu hören sein.
    Mitarbeiter:
    Dr. Alexander Friedrich (Festkörperphysik), Mag. Richard Grabner (Zahlentheorie, Mengenlehre) und Mag.Dr. Hans-Günter Löw (Physik, Biophsik https://homepage.univie.ac.at/hans.guenter.loew/) ins Netz stellen

  696. #699 Vereinfacher
    19. Juli 2015

    Ein interessanter Artikel!

    Ich hab’ nix in die Richtung studiert, mache mir einfach nur Gedanken. Absolut auf Kriegsfuß stehe ich mit der Zeitdilatation. So wie ich alle bis 2015 gemachten wissenschaftlichen Erkenntnisse verstehe, wäre es, wenn man das “Ganze” betrachtet doch nicht im Sinne des Erfinders, wenn irgendein Schnickschnack die sogenannte Zeit beeinflussen könnte. Ich glaube fest daran, das, wenn es wirklich so wäre, entweder ein heilloses Chaos da draußen herrschen würde oder schon längst alles in einem großen Knall explodiert wäre.

    Wissenschaftlich gesehen kann man doch eine Uhr nicht wirklich ernstnehmen für die großen Fragen. Ich habe gestern lange nachgedacht, wie ich Zeit wissenschaftlich nennen würde und kam auf “Ereignisvolumen”. Vom Loslassen einer Vase bis zum Aufprall am Boden “entsteht” ein Ereignisvolumen. Dieses ist bei entstehung des Universums bis 2015 halt sehr groß.

    Mir ergibt sich nun das große Problem, das alle Beobachtungen und Erkenntnisse von Menschen stammen. Hierzu gehört leider auch die Mathematik. Ich befürchte eine Art Festfahren, die ein freieres Denken und auch eine Vereinfachung behindern.

    Ein Ereignisvolumen sehe ich als bestätigt an, da man nur eine kaputte Vase erhält, wenn man sie auch vorher fallen lässt.

    Weitaus schlimmer ist für mich Einsteins Schachzug, Raum & Zeit derart zu verschmelzen, dass sich die Wissenschaft so gut wie gar nicht mehr bewegen kann. Ich würde mich darüber freuen, wenn in X Jahren irgendeine Erkenntnis dazu führt, zu erkennen, dass die Bindung von Raum an Zeit seine eigentliche Eselei war. Ich sehe es jedenfalls so.

    Die Zeitdilatation kann aber durchaus hilfreich sein, wenn man sie einmal von der anderen Seite betrachtet: Was führt im Endeffekt zu den unterschiedlichen Messungen? Ich denke, da geht noch was.

    Besonders extrem finde ich es auf Grund der Zeitdilatation auch noch auf mögliche Zeitreisen zu spekulieren. Auch hier würde ein heilloses Chaos entstehen.

    Gestern hieß es mal wieder, ein Raumfahrer würde im All weniger schnell altern, als auf der Erde. Das halte ich für absoluten schwachsinn. Das würde bei den x³ Billiarden Sternen, Galaxien und Universen etwa genauso viele Zeitebenen bedeuten. Wo das Ganze doch recht vernünftig konstruiert ist (von eiernden Planeten mit irren Umlaufbahnen und anderen exotischen Beobachtungen einmal abgesehen), wäre es doch absurd so ein “Zeitchaos” eingebunden zu sehen.

    Natürlich fühlt man sich als “Normalo” etwas seltsam wissenschaftliches anzuzweifeln, aber jedesmal, wenn es um solche Dinge geht, werde ich stutzig und da ich ein Freund einfacher Lösungen bin, wird das nicht besser.

    Komischerweise habe ich z. B mit Teilchen die sich an 2 Orten zugleich befinden absolut kein problem, ich bin also nicht gegen alles oder will mich mit einer kontroversen Meinung besser fühlen.

    So, das musste ich mal loswerden, egal wie falsch oder richtig ich liege. Vielleicht bekomme ich es ja noch mit. 😉

  697. #700 MartinB
    19. Juli 2015

    @Vereinfacher
    Der Natur ist es herzlich egal, was wir für plausibel halten oder uns vorstellen können – deswegen machen wir Experimente und schauen, wie die Natur sich tatsächlich verhält.
    Und da führt an der zeitdilatation kein Weg vorbei.

    “Gestern hieß es mal wieder, ein Raumfahrer würde im All weniger schnell altern, als auf der Erde. Das halte ich für absoluten schwachsinn.”
    Eine raumfahrerin altert schneller, je geringer das Schwerefeld ist, in dem sie sich bewegt (und je langsamer sie sich relativ zu mir bewegt). Dadurch entsteht kein Chaos, weil da nirgends eine absolute Uhr mitläuft, die da irgendetwas wieder zusammenbringen müsste, wenn zwei Objekte mit unterschiedlichem Zeitablauf wieder zusammentreffen. Wenn du da Chaos siehst, dann bedeutet es, dass du die Theorie nicht verstanden hast.

  698. #701 Vereinfacher
    19. Juli 2015

    Wow… Ist ja superaktuell hier, hätte nicht so schnell mit einer Reaktion gerechnet.

    Gut, ich habe grad’ mal schnell ein Gedankenexperiment gebastelt.

    Wir haben Eineiige Zwillinge, einer der Zwillinge wird im Raumschiff des Raumfahrers geboren, der andere auf der Erde. Du beurteilst das Altern auf der Erde, ich das Altern des anderen Zwillings im Raumschiff (sorry, ich will da unbedingt mal hin, deshalb musst Du auf der Erde bleiven!).

    Wirst Du nun einen anderen Alterungsprozess neben dem Zwilling stehend beobachten, als ich?

    Tauschen wir nach 40 Jahren die Zwillinge. Wird der Raumfahrerzwilling auf der Erde seinen Altersvorteil einbüßen und der Erdenzwilling dazugewinnen?

  699. #702 MartinB
    19. Juli 2015

    @Vereinfacher
    Google mal “Zwillingsparadoxon” – das ist das Standard-beispiel für Zeitdilatation. Es gibt vermutlich kaum ein Buch über die SRT, in dem genau dieses Problem nicht diskutiert wird.
    Bevor du die Theorie kritisierst, solltest du dir vielleicht doch ein bisschen Mühe geben, sie zu verstehen.

  700. #703 Vereinfacher
    19. Juli 2015

    Danke Dir, das kannte ich noch gar nicht.

  701. #704 MartinB
    19. Juli 2015

    Eine gute Adresse, wenn du die SRT wirklich verstehen willst, ist Einstein online:
    https://www.einstein-online.info/einsteiger/spezRT
    Das Zwillingsparadoxon wird auch erklärt:
    https://www.einstein-online.info/vertiefung/Zwillinge

    Die Seite hat viele schöne beispiele und Erklärungen und ist generell zu empfehlen. (Auch die Seiten über die allg. RT sind gut – da habe ich auch diverse Animationen her.)

  702. #705 Vereinfacher
    19. Juli 2015

    Trotzdem wäre es nett, wenn Du auf mein Gedankenexperiment eingehst. 😉

  703. #706 MartinB
    19. Juli 2015

    Welches? Das mit den Zwillingen? Warum soll ich die etwa 100000 Erklärung des Zwillingsparadoxons schreiben? Ich habe dir Links gegeben, wo du die Erklärung nachlesen kannst, das sollte doch reichen.

  704. #707 Vereinfacher
    19. Juli 2015

    Sorry, da fehlte der 2. Teil des Textes. Schlechtes Internet.

    Mir ging es nur um 2 zusätzliche Dinge, die die Zwillinge dabei haben, aber ich seh’ schon, ich nerve Dich eher.

    Das schaute beim durchlesen etwas aufgeschlossener aus.

    Keine Angst, ich bin hier weg. 😉

  705. #708 MartinB
    20. Juli 2015

    Warum soll es etwas ändern, was die Zwillinge dabeihaben?

  706. #709 Hartmut Schüppler
    Münster
    21. Juli 2015

    Kann es sein, dass ein unterschiedliches Altern der Zwillinge in einem Raumzeitmodell nicht darstellbar ist ?

    Beide Zwillinge bewegen sich in der Raumzeit von Ereignis 1 (Abreise) zu Ereignis 2 (Wiedersehen). Dies entspricht einem invarianten Raumzeitintervall.
    Vektotriell betrachtet: Erdzwilling “bewegt” sich allein auf der Zeitachse (Vektor a). Reisezwilling bewegt sich bis zum Umkehrpunkt (Vektor b) und von dort zurück zum Wiedersehen (Vektor c). a = b+c

    In einem Raumzeitmodell ist tatsächliches Geschehen in Form von Raumzeitintervallen invariant. Folglich unterliegt biologisches Wachstum und Altern nicht den relativistischen (perspektivischen) Effekten.

    Woraus ergibt sich nach SRT, dass die relativistischen Effekte nicht nur die Maßstäbe, sondern auch tatsächliches Geschehen verändern ???

  707. #710 MartinB
    21. Juli 2015

    @Hartmut
    “Kann es sein, dass ein unterschiedliches Altern der Zwillinge in einem Raumzeitmodell nicht darstellbar ist ?”
    Nein, kann es nicht.
    “Dies entspricht einem invarianten Raumzeitintervall”
    Ja, aber die beiden Intervalle sind eben nicht gleich. Genauso wie wir beide nicht dieselbe Strecke zurücklegen, wenn ich von Braunschweig direkt nach Berlin fahre und du den Umweg über Hamburg machst- und das, obwohl auf der Erde lokale Distanzen auch invariant sind.

  708. #711 Hartmut Schüppler
    Münster
    21. Juli 2015

    Hallo MartinB,
    kann es sein, dass unterschiedliche Geschwindigkeiten in der Raumzeit nicht darstellbar sind, weil Raum und Zeit untrennbar miteinander verbunden sind ? Raum und Zeit sind in einem raumzeitlichen Modell als gleichwertig anzusehen, weshalb sich alle Objekte mit der einheitenfreien “Geschwindigkeit” 1 bewegen, was häufig als Vierergeschwindigkeit “c” bezeichnet wird.

    MfG
    Hartmut

  709. #712 MartinB
    21. Juli 2015

    @Hatrtmut
    Nein, das ist nicht richtig.
    Mal ehrlich: Meinst du nicht, in den letzten 110 Jahren hätte es irgendne Physikerin bemerkt, dass man im Rahmen der Relativität keine unterschiedlichen Geschwindigkeiten darstellen kann?
    Die Vierergeschwindigkeit ist ein invariantes Maß, während Geschwindigkeiten immer relativ zur jeweiligen Beobachterin sind.
    Ich empfehle nochmal die Einstein-online-Seite, insbesondere die Analogie mit dem Wanderer halte ich für sehr nützlich.

  710. #713 Hartmut Schüppler
    Münster
    21. Juli 2015

    @MartinB
    Natürlich haben die Physiker schon sehr früh bemerkt, dass Raum und Zeit in der Raumzeit ihre Selbständigkeit verlieren und deswegen die Darstellung einer Geschwindigkeit als Beziehung zwischen Raum und Zeit in der Raumzeit nicht möglich ist. Es war wohl Minkowski der sich entsprechend geäußert hat.
    Ich stelle nicht die Geltung der speziellen Relativitätstheorie infrage; sondern weise lediglich darauf hin, dass die Naturgesetze, z.B. über Wachstum und Altern, unabhängig vom Bewegungszustand eines Systems gleichermaßen gelten und man nicht durch langsameres Altern in einem System feststellen kann, dass man sich in einem bewegten System befindet (Relativitätsprinzip).

    MIt der Angabe einer Geschwindigkeit befindet man sich automatisch außerhalb eines Raumzeitmodells in der herkömmlichen getrennten Betrachtung von Raum und Zeit.

  711. #714 MartinB
    21. Juli 2015

    @Hartmut
    ” man nicht durch langsameres Altern in einem System feststellen kann, dass man sich in einem bewegten System befindet (Relativitätsprinzip).”
    Natürlich nicht – das behauptet die SRT ja auc nicht.
    “MIt der Angabe einer Geschwindigkeit befindet man sich automatisch außerhalb eines Raumzeitmodells in der herkömmlichen getrennten Betrachtung von Raum und Zeit.”
    In dem Sinne, dass geschwindigkeiten nur jeweils für ein bestimmtes Bezugssystem gelten – ja natürlich.

    Das heißt aber nicht, dass es keine Geschwindigkeiten in der Raumzeit gibt o.ä. – ist genau wie mit dem Wanderer bei Einstein online: Auch bei einem Vektor hängt es vom Bezugssystem ab, welche Komponenten er hat, aber der Vektor ist derselbe.

    Genauso legt ein Objekt eine Strecke von A nach B zurück, und darüber sind sich dann auch alle Beobachterinnen einig. Uneinig sind sie sich nur darüber, wie man diesen Weg in räumliche und zeitliche Komponente aufteilt (Dilatation/Längenkontraktion).

    Und biologisches Wachstum findet jeweils im Ruhesystem des biologischen Systems statt – jemand, der sich relativ dazu bewegt, mag das Wachstum entsprechend anders wahrnehmen.

  712. #715 Gerd
    Phnom Penh
    26. Oktober 2015

    Ich glaube die Herren haben sich nie mit der kaum beachteten Begründung der Relativitätstheorie aus dem Jahre 1905 zu Prag beschäftigt.
    Zu finden in Kultur der Gegenwart Part 3
    DIE RELATIVITÄTSTHEORIE von Albert Einstein
    Einstein bezieht sich auf den genialen Physiker Fizeau, der ein Versuch von fundamentaler Bedeutung durchgefürt hat.
    In Wikisource: Fizeau Ueber die Hypothese vom Lichtäther
    Fizeau hat Licht durch bewegte Flüssigkeiten geleitet und dabei eine Mitnahme des Lichtes durch die Bewegung erkannt.
    Leider hat er die Messwerte falsch interpretiert. Durch den Look In Effekt wird bei geringen Geschwindigkeiten weniger angezeigt, als tatsächlich vorliegen.
    Er hat daraufhin dem Licht nur eine Teilmitnahme durch bewegte Materie zugestanden.Nach seiner Darstellung ist ein Teil des Lichtes an die Materie gebunden und ein anderer Teil an ein Äther.
    Er favorisiert die Frensnellsche Mitnahmeformel. In dieser steckt die Teilmitnahme des Lichtes und damit die Abhängigkeit vom Äther.
    Einstein wendet nun genau diese Formel zur Begründung seiner konstanz der Lichtgeschwindigkeit an und schfft den Äther als solchen ab.
    Wie blind muss man sein, um diesen Trick nicht zu durchschauen.
    Es wird weiter getrickst. Die erste, ja die erste Gleichung in seiner Rede zu Prag ist falsch. Er spricht von der Geschwindigkeit des Lichtes in Wasser. Wo bleibt der Brechungsindex !
    Gute Frage. Aber stände dort der Brechungsindex des Wassers, so würde jeder halbwegs belastete Leser erkenne, dass hier der Brechungsindex gleichgesetzt wird mit einem etwas komlizierteren Ausdruck n = (1 – 1/n)

    Da lachen die optischen Gesetze und die Mathematik wird verhöhnt.

    Und das am Anfang einer hochwissenschaftlichen Arbeit.
    Wenn der Anfang nicht richtig ist, was sollte dann im Nachfolgenden richtig sein?
    Alle Esotheriktheorien bauen auf unbegründeten Vorstellungen auf, die nicht wissenschaftlich begründet werden können.
    Damit ist die Relativitätstheorie nur eine von vielen Esotheriktheorien.
    Zugegeben sie hat einen wesentlchen Vorteil: Die Relativitötstheorie ist FAST richtig, aber eben nur fast.
    Sie wäre richtig, wenn das Licht tatsächlich eine unendliche Geschwindigkeit aufweisen würde.

  713. #716 MartinB
    26. Oktober 2015

    @Gerd
    Es spielt überhaupt keine Rolle, ob die ursprüngliche Erklärung der SRT fehlerhaft war (war sie nicht, ist aber egal). Die Theorie in ihrer heutigen Form ist extrem gut bestätigt, wie ja im Artikel thematisiert wird. Autoritätsgläubigkeit ist in der Wissenschafrt wenig nützlich – egal ob Einstein fehlr gemacht hat oder nicht, sene Theorie ist inerhalb der aktuelen Messgrenzen korrekt.

  714. #717 Gerd
    27. Oktober 2015

    So So, also korrekt! So blind kann man doch gar nicht sein.

    Eine Theorie, welche falsch ist, hat zumeist seinen Grundfehler in den ersten drei Seiten. Einstein war noch besser. Bei ihm steht der Grundfehler auf der ersten Seite.
    Bereits die Invarianz des Lichtes ist ein Ding der Unmöglichkeit. Die Frequenzverschiebungen zeigen die unterschiedlichen Geschwindigkeiten bei bewegten Lichquellen. Dieser Tatbestand wird von der Relativitätstheorie einfach unter den Tisch gekehrt.
    Was ist mit Cäsium, welches einen Brechungsindex von kleiner 1 aufweist !
    Was ist mit Rönntgenlinsen, die als Sammellinsen nicht konkav, sondern konvex geformt sind.
    In beiden Fällen wirkt eine Geschwindigkeit des Lichtes, welche oberhalb der Vakuumlichtgeschwindigkeit liegt. Also Hypergeschwindigkeit

    Gruß Gerd

  715. #718 Bjoern
    27. Oktober 2015

    Oh, schön – der nächste Kerl, der nix kapiert hat, aber felsenfest davon überzeugt ist, dass er schlauer ist als alle Physiker der letzten 100 Jahre…

  716. #719 Krypto
    27. Oktober 2015

    @Gerd:
    Du bist ein prima Kandidat für den Crackpot of the day 😀
    Du grabbelst irgendwelche unbelegten Informationen zusammen, kennst offenbar noch nicht einmal SRT und ART, machst aber ordentlich auf wichtig…hmm…
    Ist das Argument von Martin überhaupt zu Dir durchgedrungen?
    Bist Du Dir im Klaren darüber, dass die ART zu den bestbelegtesten Theorien gehört?
    Ist Dir bekannt, wie Wissenschaft funktioniert?

    Grundsätzlich ist es völlig egal, ob eine Theorie aus einem Haufen Mist hervorgeht, ob nach etwas ganz anderem geforscht wurde und die Theorie nur ein Nebenprodukt oder eine Zufallsentdeckung ist, oder eben auf gezielter, exzellenter Forschung und Auffassungsgabe beruht!

    Im Ergebnis bleibt eine Theorie, welche von der Fachleuten akribisch geprüft wird, welche man widerlegen oder verbessern will und bei der man schaut, ob sie sich mit anderen Theorien vereinigen lässt usw.

    Eine Theorie sollte dann eine Bereicherung für die Wissenschaft sein, wenn sie Beobachtungen mindestens genauso gut erklärt wie konkurrierende Theorien.

    Kannst Du Dir jetzt einigermaßen ausmalen, wie sinnfrei Deine Argumentation ist?
    Wie prinzipiell piepegal es ist, was Einstein mal irgendwann gesagt oder nicht gesagt hat?
    Er könnte sogar gesagt haben, er hätte das Geheimnis der Bananenkrümmung entdeckt und dadurch wäre er auf die Raumkrümmung gekommen.
    Völlig wurscht.
    Durch das Diskreditieren des Entwicklers löst sich sein Werk nicht in Luft auf, sondern steht weiterhin im Raum.

    Du kannst natürlich weiterhin versuchen, an irgendwelchen ominösen Stuhlbeinen von Einsteins Theorien zu sägen.
    Abgesehen davon, dass Du mitleidig belächelt wirst, ändert sich nix.

  717. #720 rolak
    27. Oktober 2015

    Hypergeschwindigkeit

    Ein + für Gerd: Hübsche Benamungen, tief verwurzelt in der Zeit, als Rechner noch Dosen waren…

  718. #721 Gerd
    28. Oktober 2015

    Es ist unfassbar. Die Relativitätstheorie baut auf dem Äther auf und verleugnet ihn gleichzeitig.
    Kann es einen größeren Widerspruch geben?

    Was ist mit Röntgenstrahlung und deren Bündelung oder mit dem Brechungsindex des Cäsium.

    Beide Tatsachen widerspechen nicht nur der Einsteinschen Theorie, sondern sie beweisen deren Nichtrichtigkeit, denn einen Brechungsindex kleiner 1 kann es nach der RT nicht geben. Es gibt ihn aber dennoch.

    Dazu natürlich von Ihnen kein Wort. Statt dessen nur oberflächliches Pla pla Pla. So auch, dass Wissenschaftliche Theorien nicht unbedingt ein sauberes Fundament benötigen, sondern wenn das Luftschloss darauf halt findet, auch richtig sein muss.
    Diese Darstellung ist mir neu und ich werde sie bei passender Gelegenheit zum Besten geben.

    Einstein wird in die Geschichte der Menschheit eingehen, als der größte und erfolgreichste Gaukler aller Zeiiten. Es ist nur eine Frage der Zeit.
    Er gehört zu den großen unter den Zauberkünstlern und wenn man seine Ausführungen kritisch betrachtet , so fallen solche Begriffe wie die Geschwindigkeit des Wassers im Sinne der Lichtausbreitung auf.
    Das Ist das Vokabular von Zauberkünstlern, aber keine wissenschaftliche Darlegung.
    Bitte denken sie selbst darüber nach und lassen sie sich nicht die Meinung anderer aufprägen. Auch wenn diese in der Mehrzahl sind, so müssen sie nicht unbedingt Recht haben.

    So nun können sie mir Schreibverbot verordnen und die Löschung aller Beiträge durchführen. Es war mir trotzdem ein Vergnügen.
    Gruß an den Verstand, wenn es geht an den Eigenen, wenn man den diesen auch hat

  719. #722 MartinB
    28. Oktober 2015

    @Gerd
    Warum soll ich deinen Blödsinn löschen – den der anderen RT-Leugner, die in jeder zeile ihre Ahnungslosigkeit der Physik offenbaren, habe ich ja auch stehen gelassen. Jeder muss selbst wissen, wie er sich im Netz lächerlich machen möchte, und die RT ficht das Geschreibsel von ahnungslosen Kommentatorinnen nicht an.

    Und was das Cäsium angeht, ist das hier erklärt (wirst du aber nicht verstehen, ist echte Physik)
    https://physicsworld.com/cws/article/news/2003/mar/20/good-news-for-negative-index-materials
    https://www.physicsforums.com/threads/speed-of-light-in-cesium-vapors.424652/

  720. #723 Hartmut Schüppler
    Münster
    28. Oktober 2015

    Hallo MartinB

    zur Klarstellung, ich stelle nicht die Richtigkeit der SRT infrage.
    Meine Kritik gilt der Art und Weise, wie Physiker sie Laien zu erklären versuchen, ohne zuvor begiffliche Klarheit zu schaffen.

    Einige Beispiele dazu:
    Die Bewegungsgesetze werden als “zeitinvariant” bezeichnet; dies soll bedeuten, dass die Zeit keine Richtung (Zeitpfeil) hat.
    Dieser Zeitbegriff bedeutet aber eigentlich, dass kausale Geschehensabläufe nicht Bestandteil dieses Zeitbegriffs sind.
    Beispiel: Für die Dauer (reine Zeit ohne Kausalität) spielt es keine Rolle, ob sich der Apfel vom Baum auf die Erde oder von der Erde an den Baum bewegt.
    Weil man keine begriffliche Klarheit schafft und den allgemeinsprachlichen Zeitbegriff verwendet, der kausales Geschehen (einen Zeitpfeil) beinhaltet, muss man dann komplizierte Wahrscheinlichkeitsüberlegungen anstellen, warum die Bewegungsgesetze eine Bewegung von der Erde an den Baum nicht ausschließen, dies aber trotzdem nicht passiert.

    Es gibt weitere begriffliche Unklarheiten, die das verständnis der SRT für Laien erschweren. Möglicherweise ist das mit ein Grund, weshalb die SRT von Laien immer wieder angezweifelt wird.

    MfG
    H.

  721. #724 Bjoern
    28. Oktober 2015

    @Hartmut:

    Die Bewegungsgesetze werden als “zeitinvariant” bezeichnet; dies soll bedeuten, dass die Zeit keine Richtung (Zeitpfeil) hat.

    Habe ich so noch nicht gehört. Meines Wissens wird normalerweise der Begriff “zeitumkehrinvariant” benutzt. Und auch das eigentlich vor allem in Fachkreisen, selten im Gespräch mit Laien. Und mit der SRT hat das übrigens auch nichts zu tun.

    Weil man keine begriffliche Klarheit schafft und den allgemeinsprachlichen Zeitbegriff verwendet, der kausales Geschehen (einen Zeitpfeil) beinhaltet, muss man dann komplizierte Wahrscheinlichkeitsüberlegungen anstellen, warum die Bewegungsgesetze eine Bewegung von der Erde an den Baum nicht ausschließen, dies aber trotzdem nicht passiert.

    Das hat wenig mit der Verwendung des allgemeinsprachlichesn Zeitbegriffs zu tun, und rein gar nichts mit der SRT.

  722. #725 Physik-Fan
    28. Oktober 2015

    @Hartmut Schüppler
    Die Bewegungsgesetze werden als “zeitinvariant” bezeichnet; dies soll bedeuten, dass die Zeit keine Richtung (Zeitpfeil) hat.
    Dieser Zeitbegriff bedeutet aber eigentlich, dass kausale Geschehensabläufe nicht Bestandteil dieses Zeitbegriffs sind.
    Beispiel: Für die Dauer (reine Zeit ohne Kausalität) spielt es keine Rolle, ob sich der Apfel vom Baum auf die Erde oder von der Erde an den Baum bewegt.
    Weil man keine begriffliche Klarheit schafft und den allgemeinsprachlichen Zeitbegriff verwendet, der kausales Geschehen (einen Zeitpfeil) beinhaltet, muss man dann komplizierte Wahrscheinlichkeitsüberlegungen anstellen, warum die Bewegungsgesetze eine Bewegung von der Erde an den Baum nicht ausschließen, dies aber trotzdem nicht passiert.

    Es bedeutet doch nur dass, wenn man einen physikalischen Prozess zeitl. umgekehrt betrachtet, wieder ein physikalisch möglicher Prozess herauskommt. Wo soll dann bei dem Beispiel das Problem sein? Wenn der Apfel z.B. auf der Erde aufprallt hat er dort einen bestimmten Impuls. Dreht man den Impuls um (z.B. indem man ihn hochwirft), fliegt er nach oben und kommt an der Stelle zum Stehen, wo er gehangen hat.

    Die Zeitumkehrinvarianz gilt tatsächlich nicht immer, aber da muss man tief in die Teilchenphysik eintauchen (Zerfall der neutralen K-Mesonen).

  723. #726 Physik-Fan
    28. Oktober 2015

    @Gerd
    DIE RELATIVITÄTSTHEORIE von Albert Einstein
    Einstein bezieht sich auf den genialen Physiker Fizeau, der ein Versuch von fundamentaler Bedeutung durchgefürt hat.
    In Wikisource: Fizeau Ueber die Hypothese vom Lichtäther
    Fizeau hat Licht durch bewegte Flüssigkeiten geleitet und dabei eine Mitnahme des Lichtes durch die Bewegung erkannt.
    Leider hat er die Messwerte falsch interpretiert. Durch den Look In Effekt wird bei geringen Geschwindigkeiten weniger angezeigt, als tatsächlich vorliegen.
    Er hat daraufhin dem Licht nur eine Teilmitnahme durch bewegte Materie zugestanden.Nach seiner Darstellung ist ein Teil des Lichtes an die Materie gebunden und ein anderer Teil an ein Äther.
    Er favorisiert die Frensnellsche Mitnahmeformel. In dieser steckt die Teilmitnahme des Lichtes und damit die Abhängigkeit vom Äther.
    Einstein wendet nun genau diese Formel zur Begründung seiner konstanz der Lichtgeschwindigkeit an und schfft den Äther als solchen ab.
    Wie blind muss man sein, um diesen Trick nicht zu durchschauen.

    Es kann ja sein, dass Einstein diese Form der Begründung gewählt hat. Das war dann ein Fehlgriff. Nichtsdestotrotz hatte er sich schon vorher, in grundlegender Art, die LG-Invarianz erschlossen. Gemäß den Maxwellschen Gleichungen gibt es keine stehenden elektromagn. Wellen. In der Newtonschen Mechanik ist das unverständlich. Es ist z.B. immer möglich, sich im Wasser so zu bewegen, dass eine Wasserwelle zu einem ruht. Warum ist das mit Licht nicht möglich? Die einzig sinnvolle Konsequenz war für ihn, dass die LG invariant ist und die Obergrenze für Relativgeschw. darstellt.

    Sind Sie ein Anhänger des Äthers? Der ist völlig unplausibel. Das Michelson-Morley- Experiment zeigt ja, dass auf der Erdoberfläche die LG immer dieselbe ist, ganz gleich, wie man das Messinstrument dreht. Das bedeutet, dass die Erdoberfläche zum Äther ruhen muss, anders geht’s nicht. Aber die Erdoberfläche ist in Rotation befindlich und das mit unterschiedl. Rotationsgeschw. Wie soll das mit einem plausiblen Äther-Modell zu erklären sein? Der Äther muss auch supersteif sein, da sich die Transversalwelle Licht in ihm beliebig weit ausbreitet und gleichzeitig völlig transparent. Wie ist das möglich?

  724. #727 MartinB
    29. Oktober 2015

    @Hartmut
    Zeitinvarianz bedeutet, dass die Gesetze heute dieselben sind wie morgen und gestern. Wie Bjoern schon schrub ist Zeitumkehrinvariant was anderes.

  725. #728 Hartmut Schüppler
    29. Oktober 2015

    Warum weigern sich die Physiker so hartnäckig gegen eine begriffliche Klärung des Zeitbegriffs, der den Bewegungsgesetzen der SRT zugrundeliegt ?
    Dies ist schlicht ein anderer Zeitbegriff, als der des allgemeinen Sprachgebrauchs. Die Bewegungsgesetze als “zeitumkehrinvariant” zu bezeichnen, trägt zum Verständnis nicht bei; denn man muß dann die Frage beantworten, warum sind die Bewegungegestze “zeitumkehrinvariant” ? Es ist auch nicht sinnvoll dem physikalischen Zeitbegriff dadurch eine Richtung geben zu wollen, dass man den Entropiesatz als Bestandteil des Zeitbegriffs auffasst. Besser wäre es “Zeit” im Sinne der SRT als reine Dauer zu definieren, wie sie von Uhren, deren Gang auf richtungsfreien periodischen Bewegungen beruht, angezeigt wird.

  726. #729 MartinB
    29. Oktober 2015

    @HArtmut
    Ich habe keine Ahnung, wovon du redest.
    “Dies ist schlicht ein anderer Zeitbegriff, als der des allgemeinen Sprachgebrauchs”
    Nein, das sehe ich nicht.
    “Besser wäre es “Zeit” im Sinne der SRT als reine Dauer zu definieren, wie sie von Uhren, deren Gang auf richtungsfreien periodischen Bewegungen beruht, angezeigt wird.”
    Genau dasist doch was Einstein sagt “Zeit st das, was eine Uhr anzeigt”.

  727. #730 Physik-Fan
    29. Oktober 2015

    @Hartmut Schüppler
    Warum weigern sich die Physiker so hartnäckig gegen eine begriffliche Klärung des Zeitbegriffs, der den Bewegungsgesetzen der SRT zugrundeliegt ?
    Dies ist schlicht ein anderer Zeitbegriff, als der des allgemeinen Sprachgebrauchs.

    Nochmals, Du kritisierst, dass bei der Zeitumkehr die Kausalität verloren geht, so habe ich es verstanden. Aber das stimmt nicht, s. meine Antwort #725. Die Umkehrung ist genauso kausal, nur ist die Ursache eine andere, weil der Prozess dort beginnt, wo der andere aufgehört hat. Die Ursache kann z.B. ein Wurf des Apfels nach oben sein.

  728. #731 Gerd
    29. Oktober 2015

    Es kann ja sein, dass Einstein diese Form der Begründung gewählt hat. Das war dann ein Fehlgriff.

    Ich nehme auch an, dass er sich im Nachhinein über die Rede in Prag geärgert hat.
    Aber dort sprach er die Wahrheit, was sie als Fehlgriff bezeichnen. Dennoch genau dieser Fehlgriff steckt in der Relativitätstheorie. Daraus wird die Systeminvarinaz des Lichtes begründet und die gibt es nicht.
    Licht hat immer zu dem aussendendem Objekt genau Lichtgeschwindigkeit. Bei jedem Kontakt mit Materie erscheint das Licht so, als wenn es von diesem erneut ausgesendet wäre.
    Das Ende vom Lied: Nach jedem Kontakt hat das Licht genau zu dem kontaktierten Material Lichtgeschwindigkeit.
    Wenn wir die Lichtgeschwindigkeit messen, so messen wir immer nur die Geschwindigkeit zu der Materie, welche aus der Messvorrichtung besteht.
    Die Natur zeigt uns aber durch die Rot- bzw. Blauverschiebung bewegter Objekte die Geschwindigket dieser Objekte. Und genau zu dieser Geschwindigkeit wird die EM Welle in den Raum gesendet.
    Haben wir die gleiche Geschwindigkeit, wie das aussendende Objekt, so gibt es keine Frequenzverschiebung.
    Könnten wir die Lichtgeschwindigkeit der aussendenden Lichtquelle messen, so käme zur Lichtgeschwindigkeit die Bewegungsgeschwindigkeit hinzu.
    Physki-Fan
    Gemäß den Maxwellschen Gleichungen gibt es keine stehenden elektromagn. Wellen. In der Newtonschen Mechanik ist das unverständlich. Es ist z.B. immer möglich, sich im Wasser so zu bewegen, dass eine Wasserwelle zu einem ruht. Warum ist das mit Licht nicht möglich?

    Es ist schon möglich. Genau dann, wenn ich mich mit Lichtgeschwindigkeit zu der Lichtquelle fortbewege, kommt die Welle zur Ruhe. Die Frequenz zu mir wird Null.

    Zu Martin B
    Und was das Cäsium angeht, ist das hier erklärt (wirst du aber nicht verstehen, ist echte Physik)
    https://physicsworld.com/cws/article/news/2003/mar/20/good-news-for-negative-index-materials
    https://www.physicsforums.com/threads/speed-of-light-in-cesium-vapors.424652/

    Es findet sich seltsamerweise immer irgendeine Erklärung, dass die RT richtig sei und dies sei nur ein weiterer Beweis für die Vielfältigkeit und allumfassenden Aussagen, welche die RT bereithält.
    Glauben sie wirklich an diese angesprochenen Reflexionen.
    In der anderen Veröffentlichung ist das Licht bereits da, bevor es abgesendet wurde. Lachhaft.
    Vielleicht bekommt dafür noch jemand einen Nobelpreis.
    Und wie sollte es bei Röntgenlicht sein. Je nach Ferquenz ändert sich dabei kontinuierlich der Brechungsindex. Von einem Brechungsindex größer 1 geht er durch Null um danach negative Werte anzunehmen.
    Ich nehme doch an, dass in beiden Fällen die gleiche Physik gilt. Nach Einstein düfte es keinen negativen Brechungsindex geben. Dieser Tatbestand falsifiziert damit seine Theorie.
    Aber Einstein hat ja eine Mauer des Nichtbegreifens aufgebaut, indem er behauptet, die Relativitätstheorie sei nur durch die konsequente Anwendung der Relativitätsprinzipien beschreibbar (jedoch nicht logisch erfassbar).

  729. #732 Hartmut Schüppler
    29. Oktober 2015

    Indem Einstein sagt “Zeit ist das, was die Uhr anzeigt” schafft er einen vom allgemeinen Sprachgebrauch abweichenden Zeitbegriff. Dies gilt es zu realisieren und begrifflich zu erfassen. Kausale Abläufe gehören nicht zum Inhalt dieses Zeitbegriffs. Auf einer Uhr kann ich nicht ablesen, ob ein Ereignis gestern stattgefunden hat oder morgen stattfinden wird. Die Zeit im Sinne der Relativitätstheorien hat keine durch kausales Geschehen festgelegte Richtung (Zeitpfeil). Vergangenheit und Zukunft sind unter Geltung dieses Zeitbegriffs keine speziellen Zeitbegriffe.
    Mit einer Uhr kann ich feststellen, dass ein Film 30 Minuten dauert, nicht aber ob er vorwärts oder rückwärts abgespielt wurde (dient nur zur Veranschaulichung der Begriffsbestimmung von Zeit in der SRT).
    Damit erledigen sich alle Überlegungen zur Verletzung der Kausalität im Zusammenhang mit Bewegungsvorgängen.

  730. #733 Bjoern
    29. Oktober 2015

    Indem Einstein sagt “Zeit ist das, was die Uhr anzeigt” schafft er einen vom allgemeinen Sprachgebrauch abweichenden Zeitbegriff.

    Echt? Im allgemeinen Sprachgebrauch werden die Uhrzeit und Zeitdauern _nicht_ an Uhren abgelesen?!? Das ist mir jetzt aber irgendwie neu…

    Die Zeit im Sinne der Relativitätstheorien hat keine durch kausales Geschehen festgelegte Richtung (Zeitpfeil). Vergangenheit und Zukunft sind unter Geltung dieses Zeitbegriffs keine speziellen Zeitbegriffe.

    Stimmt. Und?!? Der Zeitpfeil kommt durch die Thermodynamik (Entropie) erst in die Physik, mit der Relativitätstheorie hat der Zeitpfeil nichts zu tun!

  731. #734 MartinB
    29. Oktober 2015

    @Hartmut
    Ich habe nach wie vor keinen Schimmer, was du eigentlich sagen willst. Nein, die SRT legt keinen Zeitpfeil fest. Warum soll das ein Problem für irgendwas sein?
    Auch das mit der Kausalität verstehe ich nicht – Kausalität allein unter Betrachtung der SRT kann’s eh nicht geben, weil Kausalität eine Theorie braucht in der Dinge etwas bewirken, während die SRT ja quasi nur Buchführung ist. Dass z.B. ein Lichtsignal irgrndwo ausgesandt und dann anderswo detektiert wird, wird von einer anderen theorie geklärt (QM und E-Dynamik), die sagt, wie die kausale Reihenfolge ist.

  732. #735 Hartmut Schüppler
    30. Oktober 2015

    @MartinB
    Offensichtlich sind wir uns in der Sache einig. Ihr sagt: die Bewegungsgesetze sind “zeitumkehrinvariant”, ich sage: Der Zeitbegriff der Bewegungsgesetze hat im Gegensatz zum allgemeinsprachlichen Zeitbegriff keine Richtung (Zeitpfeil). Ich finde es ansonsten schon interessant, dass nach meiner Ansicht die Richtung des Zeitpfeils im allgemeinen Sprachgebrauch dadurch zustande kommt, dass man (unbewußt) die kausalen Abläufe als Bestandteil des Zeitbegriffs auffasst. Ob man insoweit einen begrifflichen Klärungsbedarf sieht, ist Ansichtssache. Zumindest erklärt meine Begriffsbestimmung, warum die Bewegungsgesetze kausalitätswidrige Bewegungen nicht ausschließen.

    Auch der Begriff der “Bewegung” ist m.E. klärungsbedürftig.
    Allgemeinsprachlich wird er als Gegensatz zu Ruhe verwendet und beinhaltet insoweit eine rein räumliche Veränderung (Zeit vergeht immer). In einem raumzeitlichen Modell, das ja den Relativitätstheorien zugrunde liegt, passt dieser allgemeinsprachliche Bewegungsbegriff aber nicht, weil auch ein im herkömmlichen Sinn ruhendes Objekt sich “bewegt” nämlich allein in der Zeit. Mir ist bekannt, dass die Physiker dem auch Rechnung tragen, indem sie von Weltpunkten (Ereignissen), Weltlinien und Raumzeitintervallen sprechen, wenn sie Veränderungen mit einem Raumzeitmodell beschreiben. Mir erscheint es zum Verständnis der Zusammenhänge nur sinnvoll, den begrifflichen Unterschied von “Bewegung” im herkömmlichen Sinn ( nur räumliche Veränderung) und “Bewegung” in einem Raumzeitmodell (Veränderung in Raum und Zeit) deutlich zu machen.
    ,@Bjoern
    Ob es sinnvoll ist, dem Begriff der Zeit dadurch eine Richtung ( Zeitpfeil) zu geben, dass man den Entropiesatz als Bestandteil des Zeitbegriffs auffasst, erscheint mir zweifelhaft. Zum Zeitbegriff der Bewegungsgesetze passt dies genauso nicht, wie der allgemeinsprachliche Zeitbegriff.

  733. #736 MartinB
    30. Oktober 2015

    @Hartmut
    Ich denke, die Argumentation vom Alltag her ist insofern problematisch, weil der Alltagsbegriff der Zeit ganz unterschiedliche Aspekte beinhaltet – Zeitdauern, Zeitpunkte, der qualitativ erlebte Unterschied von Vergangenheit, Gegenwart und Zukunft (ein “jetzt” hat in keiner physikalischen Theorie eine Entsprechung), usw.

    “Allgemeinsprachlich wird er als Gegensatz zu Ruhe verwendet und beinhaltet insoweit eine rein räumliche Veränderung (Zeit vergeht immer).”
    Naja, das ist aber eine Schlampigkeit des Alltagsbegriffs einerseits und andererseits der Tatsache geschuldet, dass wir keine Möglichkeit haben, Objekte instantan zu bewegen. Wenn ich mich als Teleporterin in Nullzeit von hier nach da versetze, ist das dann eine “Bewegung”? Wäre ich mir nicht sicher – im Alltag beinhaltet Bewegung eigentlich imemr auch eine Geschwindigkeit, und damit ist die Zeitkomponente wieder drin.

  734. #737 MartinB
    30. Oktober 2015

    PS: Um Jonathan Livingston Seagull zu zitieren:”Perfect speed is to be there.”

  735. #738 Niels
    30. Oktober 2015

    @Bjoern @MartinB
    Sicher, dass die Relativitätstheorie keinen Zeitpfeil festlegt?
    Man beschränkt sich doch auf zeitorientierte Mannigfaltigkeiten. Dann ergibt sich durch den zugehörigen Lichtkegel für jeden Beobachter eindeutig Zukunft und Vergangenheit, oder?

  736. #739 Gerd
    30. Oktober 2015

    Werde das Gespräch fortsetzen, wenn sich die Zeit verflüchtigt hat.

  737. #740 Physik-Fan
    30. Oktober 2015

    Hartmut Schüppler
    Auch der Begriff der “Bewegung” ist m.E. klärungsbedürftig.
    Allgemeinsprachlich wird er als Gegensatz zu Ruhe verwendet und beinhaltet insoweit eine rein räumliche Veränderung (Zeit vergeht immer). In einem raumzeitlichen Modell, das ja den Relativitätstheorien zugrunde liegt, passt dieser allgemeinsprachliche Bewegungsbegriff aber nicht, weil auch ein im herkömmlichen Sinn ruhendes Objekt sich “bewegt” nämlich allein in der Zeit. Mir ist bekannt, dass die Physiker dem auch Rechnung tragen, indem sie von Weltpunkten (Ereignissen), Weltlinien und Raumzeitintervallen sprechen, wenn sie Veränderungen mit einem Raumzeitmodell beschreiben.

    Du hast in dem Sinne recht, dass es in der Raumzeit keinen zeitlichen Ablauf gibt. Das ist schon Einsteins Zeitgenossen aufgefallen, etwa Ehrenfest, der dafür den Begriff “Blockuniversum” geprägt hat. Es würde eine äußere Zeit erfordern. Ein anschauliches Bild dazu ist ein Beobachter, der von “außen”, mit seiner eigenen Zeit in die Raumzeit hinein sieht und dort Bewegung feststellt. Aber diesen Beobachter und die externe Zeit gibt es nicht. Die Raumzeit ist schon alles und die Zeit ist als Koordinatenachse (prinzipiell äquivalent zu einer räuml. Achse) darin enthalten. Man kann eigentl. nur von der Weltlinie als “Bahn” sprechen, die zur Gänze da ist. Die Weltpunkte sind quasi alle auf einmal da, wie z.B. die Punkt einer Ebene. Da ist auch keiner “vorher” oder “nachher”.

    Aber, das hat meine Ausdrucksweise schon angedeutet und wie es auch Martin gesagt hat, in der RT (SRT und ART) haben wir es rein mit Geometrie zu tun. Sie macht geometrische Aussagen, nichts weiter. Eine Kräftedynamik gibt es nicht. In der ART ist die Gravitation keine Kraft. Körper bewegen sich gravitativ rein kräftefrei und folgen nur der Raumzeitkrümmung entlang einer Geodäten.

    Man kann auch die Raumzeit als ein reines Kunstkonstrukt auffassen, das hilft, die Dinge einfacher, in einer geschlossen Weise darzustellen.

  738. #741 MartinB
    30. Oktober 2015

    @Niels
    Aber ändert sich irgendwas, wenn ich überall die Orientierung umdrehe? Welcher Teil des Kegels Zukunft und welcher Vergangenheit ist, wird doch durch die Theorie selbst nicht festgelegt, oder?

  739. #742 Niels
    30. Oktober 2015

    @MartinB
    Na ja, wenn es in der Raumzeit Ereignishorizonte oder Singularitäten gibt, klappt das mit dem Umdrehen nicht mehr wirklich, oder?

  740. #743 Bjoern
    30. Oktober 2015

    @Niels: Ereignishorizonte oder Singularitäten? Also, ich habe eigentlich bisher nur von der SRT geredet (und Hartmut Schüppler anscheinend auch), wo so etwas ja wohl nicht vorkommt…?

    In der ART kann man damit wohl einen Zeitpfeil definieren (,,kosmologischer Zeitpfeil”). Vor langen Jahren habe ich mal was dazu gelesen, erinnere mich aber kaum daran.

  741. #744 MartinB
    30. Oktober 2015

    @Physikfan
    “Eine Kräftedynamik gibt es nicht.”
    In der ART gibt es schon eine Dynamik, immerhin haben wir auch Gleichungen für die Raumzeitkrümmung, Gravitationswellen etc.

    @Niels
    Ich dachte bisher auch wie Bjoern, dass wir in Sachen Zeitpfeil nur über die SRT reden.

  742. #745 Niels
    30. Oktober 2015

    Ach so, sorry.
    Dann nehme ich alles zurück und behaupte das Gegenteil.

  743. #746 Physik-Fan
    30. Oktober 2015

    @MartinB
    In der ART gibt es schon eine Dynamik, immerhin haben wir auch Gleichungen für die Raumzeitkrümmung, Gravitationswellen etc.

    Schon klar, es ist eine Geometrodynamik.

  744. #747 Zweisteine
    29. Dezember 2015

    ” Schaut man einmal bei Wikipedia, welche Belege dort für die SRT angeführt werden, so gibt es da Messungen der Zeitdilatation in Teilchenbeschleunigern oder an Myonen der Höhenstrahlung, ähnliche Messungen mit Uhren, die in Flugzeugen bewegt werden, dann eine Erklärung der Lichtablenkung (Aberration) und des Dopplereffektes, sowie schließlich die Erklärung der Lorentzkraft, also der magnetischen Kraft auf ein bewegtes Teilchen.”

    Um an die RT zu glauben du muss zwei grundsätzliche Konzeptfehler machen :
    Dass die Zeit eine physikalische und dass der Raum eine abstrakte grösse ist.
    Dazu muss du, konsequent, auch grundsätzliche Fragen meiden. Wie z.B. :

    Warum kann ich überhaupt bewegen wenn sich die Zeit dehnt?
    Oder:
    Wieviel Zeit braucht die Zeit um sich zu dehnen?

    Im Bezug auf die “Belege” die angeblich die RT bestätigen, gut. Solange du nicht weisst ob der Raum ein Zustand der Materie ist oder nicht, solange sind diese Belege keine Belege.
    Die Folgen für deine Überzeugung von der Richtigkeit der RT muss ich dir nicht erklären, wenn der Raum sich, nach einen einfachen Schlussfolgerung, als materiel herausstellt. (und die Zeit bloss als ein Vergleich von Wandlungszustände. was in der Tat sie auch ist)

  745. #748 rolak
    29. Dezember 2015

    Wieviel Zeit braucht die Zeit um sich zu dehnen?

    Nichts gegen Sprachspielereien, ganz gewiß nicht von meiner Seite – aber doch nicht als Argument^^ Fällt Dir die Problematik vielleicht auf, wenn ein Subjekt ausgetauscht wird, Zweisteine?

    Wieviel Raum braucht der Raum um sich zu dehnen?

  746. #749 Zweisteine
    29. Dezember 2015

    “Nichts gegen Sprachspielereien, ganz gewiß nicht von meiner Seite – aber doch nicht als Argument^^ Fällt Dir die Problematik vielleicht auf, wenn ein Subjekt ausgetauscht wird, Zweisteine?”

    Schwierig?
    Nimm doch die andere Frage dann.
    Wie kannst du dich überhaupt bewegen wenn die Zeit sich dehnt?

  747. #750 Zweisteine
    29. Dezember 2015

    “Fällt Dir die Problematik vielleicht auf, wenn ein Subjekt ausgetauscht wird, Zweisteine?””

    Und dies könntest du mir ein wenig deutlicher auslegen.
    Von welcher Problematik sprichst du?
    Braucht die Ausdehnung der Zeit keine Zeit denn?
    Instantan?
    Ohne dass alles explodiert?

  748. #751 rolak
    29. Dezember 2015

    ein wenig deutlicher

    Kein Problem, Zweisteine (hoffe ich, der alte optimistische Pragmatiker): Bloß weil irgendeine Buchstabengruppe identisch ist zu einer anderen, kann nicht automatisch von identischer Bedeutung ausgegangen werden. Im konkreten Falle kommt in Denem Satz

    Wieviel Zeit¹ braucht die Zeit² um sich zu dehnen?

    zweimal die Buchstabengruppe ‘Zeit’ vor, ohne daß auch nur ansatzweise davon ausgegangen werden kann, daß Zeit² sich in-sich-selber-meßbar ausdehnt, mithin Zeit² und Zeit¹ bedeutungsidentisch wären. Welches Zeit¹Maß benötige ich, um das sich verändernde Vergehen von Zeit² zu bemaßen?

    Etwas konkreter bitte. Bisher fällt das unter ‘bedeutungsschwanger, doch inhaltsleer’.

  749. #752 Zweisteine
    29. Dezember 2015

    “” Kein Problem, Zweisteine (hoffe ich, der alte optimistische Pragmatiker): Bloß weil irgendeine Buchstabengruppe identisch ist zu einer anderen, kann nicht automatisch von identischer Bedeutung ausgegangen werden. Im konkreten Falle kommt in Denem Satz

    Wieviel Zeit¹ braucht die Zeit² um sich zu dehnen?

    zweimal die Buchstabengruppe ‘Zeit’ vor, ohne daß auch nur ansatzweise davon ausgegangen werden kann, daß Zeit² sich in-sich-selber-meßbar ausdehnt, mithin Zeit² und Zeit¹ bedeutungsidentisch wären. ”

    Das ist nicht mein Problem. Die Zeitdehnung ist eine Fluktuation, also, sie braucht auch eine bestimmte Zeit um zu geschehen.
    Was ist so unverständlich?
    Du merkst sehr wahrscheinlich dass du in eine unendliche Progression der Zeitausdehnung kommst, so, dass die Bewegung gar nicht stattfinden kann.

    ” Welches Zeit¹Maß benötige ich, um das sich verändernde Vergehen von Zeit² zu bemaßen? ”

    Diese Antwort muss du mir geben !

  750. #753 Spritkopf
    29. Dezember 2015

    @Zweisteine

    Erstens: “Messung” ist immer ein Vergleich zweier Grössen.

    Herzlichen Glückwunsch zu dieser Erkenntnis. Wenn meine Kristalluhr zu einem bestimmten Zeitpunkt x Schwingungen hinter sich gebracht hat und zu einem späteren Zeitpunkt x + y Schwingungen, dann habe ich gerade zwei Größen verglichen.

    Du kannst sagen dass ein Kristall konstant schwingt will du es mit einer anderen Schwingung vergleichen kannst. Sonst kannst du nicht wissen dass es konstant ist.

    Na und? Für Uhren haben wir haufenweise unterschiedliche Körper und Atome, die wir schwingen lassen können. Und wenn wir das unter gleichen und reproduzierbaren Bedingungen tun, dann werden wir feststellen, dass sie zwar mit unterschiedlichen Frequenzen schwingen, aber dass die Frequenzverhältnisse zwischen ihnen immer gleich bleiben.

    Und, konstant, nur im vergleich zum Maßstab. Aber das ist eine andere Frage.

    Und was, wenn alle Maßstäbe, die wir verwenden, diese Konstanz bestätigen? Und bevor du jetzt weinst, das wäre ja genau das Gegenteil dessen, was die Relativitätstheorie besagt: Deswegen habe ich “unter gleichen und reproduzierbaren Bedingungen” geschrieben (also insbesondere im Ruhezustand und bei gleichem Schwerefeld).

    Was du unbedingt Zeit nennen willst entsteht nur aus der Vergleich zweier Fluktuationen oder Bewegungen. Und diesen Vergleich erstellen wir, willkürlich. Es ist eine Abstraktion und nicht etwas materielles und eigenständiges.

    Was ist daran willkürlich, Schwingungen zu zählen? Oder die Anzahl Schwingungen eines Kristalls mit der Anzahl Schwingungen eines anderen Kristalls im gleichen Zeitraum zu vergleichen? Nichts.

    Und was ist so Schlimmes daran, dass es sich um eine Abstraktion handelt? Wenn das deine Kritik an der RT sein soll, dann ist das ziemlich dürftig.

  751. #754 Spritkopf
    29. Dezember 2015
  752. #755 Braun Josef / IVE
    home: ive.xyz
    2. Januar 2016

    Die spezielle Relativitätstheorie ist falsch, wirklich. Widerlegt an einer deutschen Hochschule siehe: https://ive.xyz/wp-content/uploads/2015/08/Widerlegung-Relativit%C3%A4tstheorie.pdf und aus dem Michelson-Morley-Experiment: https://ive.xyz/wp-content/uploads/2014/01/Michelson.pdf.pdf.
    Und somit ist die ART auch falsch. MfG

  753. #756 Alderamin
    2. Januar 2016

    @Braun Josef / IVE

    Nö, das liegt nur daran, dass die Beobachter in den System S und S’ nicht einig sind über die Gleichzeitigkeit der Ereignisse “Licht erreicht x bzw. -x” und “Licht erreicht x’ bzw. -x’ “. Guckst Du hier.

    Meinst Du ernsthaft, so ein Bock, wenn es einer wäre, wäre in 100 Jahren niemandem aufgefallen?

  754. #757 Zweisteine
    2. Januar 2016

    +++Alderamin
    2. Januar 2016
    Nö, das liegt nur daran, dass die Beobachter in den System S und S’ nicht einig sind über die Gleichzeitigkeit der Ereignisse “Licht erreicht x bzw. -x” und “Licht erreicht x’ bzw. -x’ “+++

    Bis zur RT, Theorien wurden immer erarbeitet um Klarheit und Einigkeit in der Beobachtung der Natur zu schaffen.
    Eistein schafft aber eine wo sich die Beobachter nicht mehr einigen können.
    Was ist der Sinn denn, für eine solche Theorie ?

  755. #758 Braun Josef / IVE
    home: ive.xyz
    2. Januar 2016

    Es ist eine Momentaufnahme, der Wirklichkeit, von mir aus ohne Beobachter (wäre z.B. einer in z-Richtung sehr weit weg, er würde evtl. gar nichts beobachten).
    Zum “Bock” – es gab früher schon falsche Theorien, die über mehrere 100 Jahre bestand hatten. MfG

  756. #759 Krypto
    2. Januar 2016

    @Braun Josef:

    Zum “Bock” – es gab früher schon falsche Theorien, die über mehrere 100 Jahre bestand hatten. MfG

    Na dann nenn mal Ross und Reiter…auch bitte für die von Dir erwähnte Hochschule…

    Im Übrigen ist Dir schon klar, dass wahre Herrscharen von Wissenschaftlern seit eben diesen 100 Jahren versuchen, die SRT zu widerlegen?

    Kleiner Tipp:
    Versuche erst einmal, die SRT zu verstehen. Und fang dabei ganz am Anfang bei den Postulaten an…

  757. #760 Zweisteine
    2. Januar 2016

    +++Krypto
    2. Januar 2016
    Im Übrigen ist Dir schon klar, dass wahre Herrscharen von Wissenschaftlern seit eben diesen 100 Jahren versuchen, die SRT zu widerlegen?+++

    Und viele haben es auch geschafft….

    +++Kleiner Tipp:
    Versuche erst einmal, die SRT zu verstehen. Und fang dabei ganz am Anfang bei den Postulaten an…+++

    Ich versuche ja, aber keiner kann erklären wie Bewegung möglich ist wenn die Zeit sich dehnt. (weil ohne Bewegung brauchen wir doch keine RT, nicht wahr Krypto?)

  758. #761 MartinB
    2. Januar 2016

    @Zweisteine
    Erklär du doch mal, wie Bewegung möglich ist, wenn die Zeit sich nicht dehnt.

  759. #762 Zweisteine
    2. Januar 2016

    +++MartinB
    2. Januar 2016
    @Zweisteine
    Erklär du doch mal, wie Bewegung möglich ist, wenn die Zeit sich nicht dehnt.+++

    Ein Gegenstand kann dann, sein Standort ändern.
    Warum sollte nicht möglich sein, wenn sich die Zeit nicht dehnt?

  760. #763 Braun Josef / IVE
    home: ive.xyz
    3. Januar 2016

    @MartinB – “Bewegung ohne Zeitdehnung”
    Könntest Du es mir erklären, einfach / kurz? MfG

  761. #764 MartinB
    3. Januar 2016

    @Zweistein
    Und auch wenn die Zeit sich – von unterschiedlichen Betrachtern aus gesehen relativ zueinander – dehnt, kann ein gegenstand seinen Standort ändern.

  762. #765 Adent
    3. Januar 2016

    @Braun
    Darf ich sie auch bitten hier etwas peer referiertes zu ihrer Behauptung zu verlinken?

  763. #766 Zweisteine
    3. Januar 2016

    +++ MartinB
    3. Januar 2016
    @Zweistein
    Und auch wenn die Zeit sich – von unterschiedlichen Betrachtern aus gesehen relativ zueinander – dehnt, kann ein Gegenstand seinen Standort ändern.+++

    Also, du meinst “wenn es so aussieht” …..

    In der Tat aber, hat sich die Zeit gar nicht gedehnt. Sonst gäbe keine Bewegung.
    Jetzt haben wir einen Massstab gefunden um zwischen Fantasie und Realität zu unterscheiden.

  764. #767 MartinB
    3. Januar 2016

    @Zweisteine
    “In der Tat aber, hat sich die Zeit gar nicht gedehnt”
    Warum nicht?
    Natürlich merkt ein einzelnes Objekt allein nie etwas von einer Zeitdehnung – wie auch. Dilatationseffekte kann man nur durch den Vergleich unterschiedlicher Beobachter erkennen.
    Und tu bitte nicht so, als ginge es hier darum, etwas zu “finden” – hier geht es darum, dir nachhilfe in den elementarsten Physikgrundlagen zu erteilen, sonst nichts.Deine Wortspiele und unbegründeten Behauptungen haben mit Physik nichts zu tun-

  765. #768 Alderamin
    3. Januar 2016

    @Zweisteine

    Also, du meinst “wenn es so aussieht” …..

    Ja, genau, wenn man Myonen am Erdboden nachweisen kann die in über 10 km Höhe entstehen und deren Lebensadauer bei fast Lichtgeschwindigkeit ohne Zeitdilatation nur für 450 m gut wäre, dann sieht das nur so aus. GPS sieht auch noch so aus, als wenn man damit navigieren könnte. Du bist ein großer Held der Wissenschaft, der uns alle bekehren wird. Nur ein bisschen Mathematik müstest Du noch lernen, dann wird es uns wie Schuppen aus den Haaren fallen.

    Du bekommst gar nichts mit, was? Z.B. wie verloren der Posten ist, auf dem Du stehst? (Na gut, Herr Braun steht gleich nebenan).

  766. #769 Zweisteine
    3. Januar 2016

    +++ MartinB
    3. Januar 2016
    @Zweisteine
    “In der Tat aber, hat sich die Zeit gar nicht gedehnt”
    Warum nicht?+++

    Genau darum, weil sonst Bewegung nicht möglich wäre.
    Und wenn du darauf bestehen willst dass trotz Zeitdehnung Bewegung möglich ist, dann erkläre bitte wie soll das gehen. (und nicht, wie soll das aussehen)

    +++ Und tu bitte nicht so, als ginge es hier darum, etwas zu “finden” – hier geht es darum, dir Nachhilfe in den elementarsten Physikgrundlagen zu erteilen, sonst nichts.+++

    Bin voll dabei und mit ganz offenen Augen. Kläre mich auf.

    +++ Deine Wortspiele und unbegründeten Behauptungen haben mit Physik nichts zu tun+++

    Sobald du es bewiesen hast.

  767. #770 Zweisteine
    3. Januar 2016

    +++ Alderamin
    3. Januar 2016
    @Zweisteine

    ((( Also, du meinst “wenn es so aussieht” …..))))

    Ja, genau, wenn man Myonen am Erdboden nachweisen kann die in über 10 km Höhe entstehen und deren Lebensadauer bei fast Lichtgeschwindigkeit ohne Zeitdilatation nur für 450 m gut wäre, dann sieht das nur so aus.+++

    So. Weil du keinen anderen Grund für die Lebensverlängerung der Myonen finden kannst, willst du diese Verlängerung unbedingt einen Phänomen zuschreiben der logisch gar nicht funktionieren kann?
    Verstehe ich nicht.

    +++ Du bekommst gar nichts mit, was? Z.B. wie verloren der Posten ist, auf dem Du stehst?.+++

    Offensichtlich nicht. Aber bitte, du kannst mich jeder Zeit aufklären.

  768. #771 Braun Josef / IVE
    home: ive.xyz
    3. Januar 2016

    @MartinB Welche Behauptung?

  769. #772 Braun Josef / IVE
    home: ive.xyz
    3. Januar 2016

    Sorry MartinB es ging um @Adent – Welche Behauptung von mir?

  770. #773 MartinB
    3. Januar 2016

    @Zweisteine
    “dann erkläre bitte wie soll das gehen. ”
    Indem ein Objekt zu zwei unterschiedlichen Zeiten an unterschiedlichen orten ist.
    Ich hatte dich genau deswegen selbst gefragt, weil ich keine Idee habe, was du dir überhaupt für eine Antwort vorstellst.
    Auch mit Zeitdilatation kann ein Objekt zu zwei zeiten an unterschiedlichen Orten sein – wenn du nicht in der Lage oder willens bist, das zu verstehen, ist das nicht mein Problem.

  771. #774 Zweisteine
    3. Januar 2016

    +++ MartinB
    3. Januar 2016
    @Zweisteine
    “dann erkläre bitte wie soll das gehen. ”
    Indem ein Objekt zu zwei unterschiedlichen Zeiten an unterschiedlichen orten ist.
    Ich hatte dich genau deswegen selbst gefragt, weil ich keine Idee habe, was du dir überhaupt für eine Antwort vorstellst.
    Auch mit Zeitdilatation kann ein Objekt zu zwei zeiten an unterschiedlichen Orten sein – wenn du nicht in der Lage oder willens bist, das zu verstehen, ist das nicht mein Problem.+++

    Dass ein Objekt zu zwei unterschiedlichen Zeiten an unterschiedlichen Orten sein kann ist selbstverständlich. Was du noch erklären muss, und das ist dein Problem, ist wie dieses Objekt von einem Ort zum anderen kommen kann wenn sich die Zeit dehnt.
    Das geht gar nicht.
    Du meinst es geht, dann bitte erkläre es. (und ohne Wortspiele. Die Frage ist dafür zu einfach)

  772. #775 Krypto
    3. Januar 2016

    @MartinB:
    OMG, hast Du jetzt ein Problem 😀
    “Ich versteh was nicht und stelle deshalb absurde Behauptungen auf. Das ist aber nicht mein Problem, sondern ein Problem aller anderen!”
    Wenn Zweisteine freundlicher wäre und überdies noch einfach mal einräumen würde, dass er ein absoluter Laie ist, könnte man ja gerne versuchen, ihm seinen Denkfehler klar zu machen.
    Vielleicht mit einem stark vereinfachten Beispiel.
    Z.B. anhand eines Gummibandes, auf dem jemand läuft, welches langsam gedehnt wird.
    Oder noch einfacher: Anhand einer Slowmotion-Wiedergabe.

  773. #776 Zweisteine
    3. Januar 2016

    +++ Krypto
    3. Januar 2016
    Z.B. anhand eines Gummibandes, auf dem jemand läuft, welches langsam gedehnt wird.+++

    Ist das dein Ernst? Hat man dir an der Uni die Zeitdehnung anhand eines Gummibandes beigebracht?

  774. #777 Alderamin
    3. Januar 2016

    @Zweisteine

    So. Weil du keinen anderen Grund für die Lebensverlängerung der Myonen finden kannst, willst du diese Verlängerung unbedingt einen Phänomen zuschreiben der logisch gar nicht funktionieren kann?

    Es passiert aber. Weißt Du, Du argumentierst gar nicht gegen Einstein. Einstein könnte völlig falsch liegen. Das einzigste was am Ende zählt, sind die Beobachtungen. Die kann im Prinzip jeder mit ein bisschen Kleingeld nachprüfen. Hat einer gemacht. Ist mit seinen Kindern (so einen Vater hätte ich Dir gewünscht!) und drei Cäsiumuhren auf einen Berg gefahren. Als er zurück kam, gingen sie vor. Alle drei. Um den von Einstein vorhergesagten Betrag.

    https://www.leapsecond.com/great2005/tour/

    Und jetzt kannst Du noch hundertmal behaupten, Hummeln könnten nicht fliegen, weil das angeblich keiner erklären kann, aber sie fliegen halt doch. Gegen die Realität zu argumentieren, ist relativ sinnfrei.

    Und damit habe ich Dir genug Aufmerksamkeit gewidmet, natürlich hat einen Crank noch kein Faktum interessiert, das in kognitiver Dissonanz mit seinen Vorstellungen war, das wird hier nicht anders sein. Werd’ halt glücklich mit Deinem Aberglauben. Ist Dein gutes Recht. Und wir haben alle mal geschmunzelt. 🙂

  775. #778 Zweisteine
    3. Januar 2016

    +++ Alderamin
    3. Januar 2016
    @Zweisteine

    ((( So. Weil du keinen anderen Grund für die Lebensverlängerung der Myonen finden kannst, willst du diese Verlängerung unbedingt einen Phänomen zuschreiben der logisch gar nicht funktionieren kann? )))

    Es passiert aber. Weißt Du,+++

    Ja es passiert. Und weil es passiert muss unbedingt eine Zeitdehnung im spiel sein, die du nicht mal erklären kannst?
    Ich bitte dich, halt mich nicht auf mit einem solchen Kram.

    +++ Du argumentierst gar nicht gegen Einstein. Einstein könnte völlig falsch liegen. Das einzigste was am Ende zählt, sind die Beobachtungen.+++

    Da liegst du leider wieder falsch. Es sind nicht die Beobachtungen entscheidend aber wie du sie miteinander verbindest.

    +++ Die kann im Prinzip jeder mit ein bisschen Kleingeld nachprüfen. Hat einer gemacht. Ist mit seinen Kindern (so einen Vater hätte ich Dir gewünscht!) und drei Cäsiumuhren auf einen Berg gefahren. Als er zurück kam, gingen sie vor. Alle drei. Um den von Einstein vorhergesagten Betrag.+++

    2 Nanoseks?
    Hat das Einstein vorhergesagt? Und der toller Vater dreier Kinder hat es bewiesen?
    Bitte keine mexikanische Telenovelas hier.
    Ich diskutiere diesen kram sowieso nicht.

    Kannst du erklären wie Bewegung möglich ist wenn die Zeit sich dehnt, oder nicht?
    Das is wohl alles was ich von dir will. Den Rest kannst du dir sparen.

  776. #779 Alderamin
    3. Januar 2016

    Siehe #773. Wie gesagt, niemand hier versteht Dein “Problem”. Da Du ja auch nicht in der Lage bist, vorzurechnen, was das Problem sein soll, wird das wohl auch so bleiben. Jetzt aber wirklich “tschüß”.

  777. #780 Zweisteine
    3. Januar 2016

    +++ Alderamin
    3. Januar 2016
    Siehe #773. Wie gesagt, niemand hier versteht Dein “Problem”. Da Du ja auch nicht in der Lage bist, vorzurechnen, was das Problem sein soll, wird das wohl auch so bleiben. Jetzt aber wirklich “tschüß”.+++

    Mein Problem?
    Ich bin ja kein Anhänger der RT.
    Was für ein Problem sollte ich haben wenn die Grundlage der RT bloss nicht funktioniert?
    Und wenn du so eine einfache Frage nicht kapieren kannst dann frage ich mich, wieso du glauben kannst die Theorie überhaupt zu verstehen?

  778. #781 Krypto
    3. Januar 2016

    Ich weiß nicht, wie Ihr das seht, aber der Anflug von Mitleid für nen Ahnungslosen ist bei mit vorbei, die Trollfütterung ist beendet.

  779. #782 Alderamin
    3. Januar 2016

    @Krypto

    Ein bisschen Mitleid und vor allem Unverständnis bleibt darüber, was in manchen Köpfen so vorgeht (noch dazu unter dem Text dieses Artikels, der vor Belegen für die RT nur so strotzt, hat er wahrscheinlich weder gelesen noch verstanden) aber ja, die Fütterung ist von meiner Seite beendet.

  780. #783 Zweisteine
    4. Januar 2016

    +++ (noch dazu unter dem Text dieses Artikels, der vor Belegen für die RT nur so strotzt, hat er wahrscheinlich weder gelesen noch verstanden)+++

    Und dir juckt gar nicht, dass gerade die Grundlage der Theorie gar nicht stimmt, obwohl es so von Belegen “strotzt”?

  781. #784 MartinB
    4. Januar 2016

    @Zweistein
    ” Was du noch erklären muss, und das ist dein Problem, ist wie dieses Objekt von einem Ort zum anderen kommen kann wenn sich die Zeit dehnt.”
    Aha, die Antwort, die du selbst auf dieselbe Frage an dich zurückgegeben gegeben hast, stellt dich also nicht zufrieden, wenn ich sie gebe.
    Also wieder, da ich keine Ahnung habe, was für eine Art Antwort du dir vorstellst:
    Wie kommt deiner Vorstellung nach ein Objekt von einem Ort zum anderen?

  782. #785 Adent
    4. Januar 2016

    @Braun Josef
    Diese hier:

    Die spezielle Relativitätstheorie ist falsch, wirklich.Und somit ist die ART auch falsch.

    Ist das so schwer sich die eigenen Behauptungen zu merken?
    Und nein, an einer deutschen Hochschule widerlegt ist kein Peer Review…

  783. #786 Adent
    4. Januar 2016

    @Zweisteine
    Ich bitte erneut um die Verlinkung aussagekräftiger und geprüfter Texte zu:

    Und dir juckt gar nicht, dass gerade die Grundlage der Theorie gar nicht stimmt,…

    Behaupten kann man viel, belegen muß man es aber auch und zwar nicht durch erneute Behauptungen, das scheinen Sie nicht verstehen zu können oder zu wollen? Haben sie schon mal irgendwas in der Forschung gemacht?

  784. #787 Braun Josef / IVE
    home: ive.xyz
    4. Januar 2016

    @Adent
    Es gibt kein Gutachten, die 2 Seiten sind doch wirklich nachzuvollziehen.
    Und ich finde es nicht gut gleich nachteilig über meine Person zu schreiben, nur weil ich was nicht sofort verstanden habe! MfG

  785. #788 Zweisteine
    4. Januar 2016

    +++ MartinB
    4. Januar 2016
    @Zweistein
    ” Was du noch erklären muss, und das ist dein Problem, ist wie dieses Objekt von einem Ort zum anderen kommen kann wenn sich die Zeit dehnt.”
    Aha, die Antwort, die du selbst auf dieselbe Frage an dich zurückgegeben gegeben hast, stellt dich also nicht zufrieden, wenn ich sie gebe.
    Also wieder, da ich keine Ahnung habe, was für eine Art Antwort du dir vorstellst:+++

    Ich habe dich doch darum gebeten die Wortspiele beiseite zu lassen und dich auf die Aufgabe zu konzentrieren.

    +++ Wie kommt deiner Vorstellung nach ein Objekt von einem Ort zum anderen?+++

    In der guten alten Manier : Er bewegt.

    Aber die Frage hier ist immer noch : Wie kann er bewegen wenn sich die Zeit dehnt?
    Der Objekt kann doch, durch die Zeitdehnung, sein Zeitpunkt nicht überwinden.
    (ein Problem für dich dürfte sein dass, in deinem Sinne, du gar nicht weisst was Zeit ist, eigentlich. Und das macht eine Antwort ziemlich schwierig wenn nicht unmöglich)

  786. #789 Braun Josef / IVE
    home: ive.xyz
    4. Januar 2016

    @MartinB @Zweisteine
    Vielleicht kann ich helfen bevor es noch zu polemisch wird. Aus der Vorlesung über die spezielle Relativitätstheorie (Skript: https://ive.xyz/wp-content/uploads/2015/08/zueb11-5.pdf) beginnt auf Seite 8 die Zeitdilatation doch vorher ist auf Seite 5 Mitte, der Bruch, der Gedankenfehler vgl. #755

  787. #790 MartinB
    4. Januar 2016

    @Zweisteine
    “Ich habe dich doch darum gebeten die Wortspiele beiseite zu lassen und dich auf die Aufgabe zu konzentrieren.”
    Es geht nicht um Wortspiele, es geht schlicht arum, dass du sehr unpräzise Fragen stellst und ich durch Rückfragen versuche, zu verstehen, was du eigentlich wissen willst.

    “In der guten alten Manier : Er bewegt.”
    Und das bleibt in der SRT so. Auch da bewegen sich Objekte in der guten alten Manier -s ie sind zu einem Zeitpunkt an einem Ort, zu einem anderen zeitpunkt an einem anderen. Unterschiedliche Beobachter haben unterschiedlicher Ansichtend arüber, wie groß der räumliche und zeitliche Abstand zwischen den beiden Punkten ist, aber das ist alles.

    “Der Objekt kann doch, durch die Zeitdehnung, sein Zeitpunkt nicht überwinden.”
    Warum sollte das so sein? Du hast anscheinend schlicht eine falsche Vorstellung davon, was die Dehnung der Zeit ist. Für das Objekt bleibt eine Sekunde eine Sekunde, ein Objekt allein merkt nichts von der Dilatation. Die bemerkt man erst, wenn man zwei Objekte vergleicht, deswegen heißt es ja auch Relativitätstheorie.

  788. #791 Adent
    4. Januar 2016

    @Braun Josef
    Wenn es so einfach nachzuvollziehen ist und die SRT widerlegt (und gleich die ART mit), dann ist es mit Sicherheit einen wenn nicht zwei Nobelpreise wert und würde die gesamte Physik revolutionieren, haben Sie daran kein Interesse?
    @Zweisteine
    Schon mal überlegt ob du vielleicht nicht ganz richtig verstanden hast was Zeit ist und dich daher keiner versteht?

  789. #792 Adent
    4. Januar 2016

    @Martin B

    Die bemerkt man erst, wenn man zwei Objekte vergleicht, deswegen heißt es ja auch Relativitätstheorie.

    Daumen hoch für diese einfache wie geistreiche Erklärung 😉
    Allein, ich befürchte Zweisteine lebt nicht in dieser einfachen Welt …

  790. #793 Braun Josef / IVE
    home: ive.xyz
    4. Januar 2016

    @Adent
    Die ART hat ja als Voraussetzung die SRT, somit ist die ART nicht eigenständig widerlegt. Mir geht es um die Sache und ich bin kein Physiker (ich werde keine Prüfung mit Einsteininhalten schreiben). Und was wird revolutioniert, die heutigen/zukünftigen Computer z.B. laufen auch ohne RT.
    Meine Hoffnung wäre es, da ja die Elektrodynamik mit den widerlegten Maxwell – Gleichungen zusammenhängt einen kleinen Beitrag zu leisten in dem jemand ohne Hindernisse eine neue Kraft findet die zur Lösung u.a. der Energiefrage was beiträgt. MfG

  791. #794 Zweisteine
    4. Januar 2016

    +++ MartinB
    4. Januar 2016
    (((( Der Objekt kann doch, durch die Zeitdehnung, sein Zeitpunkt nicht überwinden.)))
    Warum sollte das so sein? Du hast anscheinend schlicht eine falsche Vorstellung davon, was die Dehnung der Zeit ist. Für das Objekt bleibt eine Sekunde eine Sekunde, ein Objekt allein merkt nichts von der Dilatation. Die bemerkt man erst, wenn man zwei Objekte vergleicht, deswegen heißt es ja auch Relativitätstheorie.+++

    Du solltest deine Argumente koordiniert darlegen. Andernfalls du kannst sagen was du willst und anschliessend behaupten du hättest eine Erklärung geliefert.

    +++ Für das Objekt bleibt eine Sekunde eine Sekunde,+++

    Wenn sich die Zeit dehnt dann läuft für dieses Objekt nie eine Sekunde ab.

    Wenn es nur so aussieht dass die Zeit sich dehnt dann gibt es keine tatsächliche Zeitdehnung und wir brauchen diese Unterhaltung gar nicht zu führen.
    In anderen Worten, wenn die Zeitdehnung nur ein virtuelles Phänomen ist, dann kann diese angebliche Zeitdehnung keine Auswirkungen auf physikalische Abläufe haben: Myonen, Uhren usw…. Relativisten aber, behaupten das Gegenteil.

    Also, noch einmal. Wenn du eine reale Antwort geben willst dann, keine Wortspiele mehr.

    ,

  792. #795 Hartmut Schüppler
    Münster
    4. Januar 2016

    Hallo zusammen,
    mein Vorschlag wäre, die Bewegung von Myonen ohne Uhren, die sich im Verhältnis zu einander bewegen, zu betrachten; denn Myonen führen keine Uhr mit. Dies ist mit Hilfe eines Raumzeitmodells möglich.
    Die Entstehung der Myonen in der Atmosphäre ist das Ereiginis 1 (Weltpunkt), das Eintreffen auf der Erde ist Ereignis 2. Das tatsächliche Geschehen, die Bewegung des Myons von Ereignis1 zu Ereignis 2 ist ein Raumzeitintervall und wird wie folgt berechnet:
    sqrt (Zeitdauer^2 – Raumentfernung^2). Relativistische Effekte treten dabei nicht auf. Man kann sich das Raumzeitkoordinatensystem mit dem Beobachter (seiner Uhr und seinem Längenmaßstab) verbunden vorstellen. Das Raumzeitintervall, also das tatsächliche Geschehen, kann als Vektor aufgefasst werden, der den Zeitanteil und den Raumanteil der Bewegung repräsentiert.

    MfG
    H.

  793. #796 Zweisteine
    4. Januar 2016

    +++ Adent
    4. Januar 2016
    Wenn es so einfach nachzuvollziehen ist und die SRT widerlegt (und gleich die ART mit), dann ist es mit Sicherheit einen wenn nicht zwei Nobelpreise wert und würde die gesamte Physik revolutionieren, haben Sie daran kein Interesse? +++

    Aden. Einstein hat keinen Nobel für seine “revolutionierende” Theorie bekommen. Warum sollte es einen geben wenn man sie widerlegt?

    +++ @Zweisteine
    Schon mal überlegt ob du vielleicht nicht ganz richtig verstanden hast was Zeit ist und dich daher keiner versteht?+++

    Wenn ich nicht verstanden habe dann kann ich mich mit der Tatsache trösten dass du auch keinen Schimmer hast davon, was Zeit ist.
    Oder hast du?
    Wenn schon, dann tobt dich aus und widerlege meine Argumente.

  794. #797 MartinB
    4. Januar 2016

    @Zweistein
    “Du solltest deine Argumente koordiniert darlegen”
    Danke, jetzt ist mir schon ein Ironiemeter selbst hinter einer eigentlich komplett ironiefesten Abschirmung explodiert.

    “Wenn sich die Zeit dehnt dann läuft für dieses Objekt nie eine Sekunde ab.”
    Diese Behauptung wird durch beliebig häufiges Wiederholen nicht wahrer. Wenn du wirklich glaubst, dass das so ist, dann hast du etwas nicht verstanden.

    “Wenn es nur so aussieht dass die Zeit sich dehnt dann gibt es keine tatsächliche Zeitdehnung und wir brauchen diese Unterhaltung gar nicht zu führen.”
    Es “sieht nicht nur so aus” – aber nur der Vergleich zweier Messungen kann die – relative – Dehnung nachweisen. Wenn wir beide jeder einen Maßstab haben, aber meiner ist aus Gummi und kann sich dehnen, dann merke ich die Dehnung an dem Maßstab selbst nicht – wohl aber, wenn ich meine Messergebnisse mit deinen Vergleiche.

    So, damit war’s das jetzt – falls du tatsächlich noch etwas neues oder sinnvolles zu sagen hast, dann gern, aber ich habe keine Lust, immer wieder zu sehen, wie du wütend mit dem Fuß aufstampfst und immer wieder denselben falschen Satz wiederholst. Du hast nicht verstanden, wie die Zeitdehnung funktioniert – entweder du lernst es oder halt nicht, das ist deine Sache (und nein, dein Unverständnis ist kein Problem der Physik).

  795. #798 Zweisteine
    4. Januar 2016

    +++ Hartmut Schüppler
    Münster
    4. Januar 2016
    Hallo zusammen,
    mein Vorschlag wäre, die Bewegung von Myonen ohne Uhren, die sich im Verhältnis zu einander bewegen, zu betrachten; denn Myonen führen keine Uhr mit. Dies ist mit Hilfe eines Raumzeitmodells möglich…….
    ……………….
    Das Raumzeitintervall, also das tatsächliche Geschehen, kann als Vektor aufgefasst werden, der den Zeitanteil und den Raumanteil der Bewegung repräsentiert.+++

    Und das erklärt was genau im Bezug auf die angebliche Zeitdehnung?

  796. #799 Braun Josef / IVE
    home: ive.xyz
    4. Januar 2016

    @Hartmut Schüppler
    Wie geht es dann mit den Einheiten? Sekunde im Quadrat minus Meter im Quadrat? MfG

  797. #800 Zweisteine
    4. Januar 2016

    +++ MartinB
    4. Januar 2016
    @Zweistein

    ((( Wenn sich die Zeit dehnt dann läuft für dieses Objekt nie eine Sekunde ab.)))

    +++ Diese Behauptung wird durch beliebig häufiges Wiederholen nicht wahrer. Wenn du wirklich glaubst, dass das so ist, dann hast du etwas nicht verstanden.+++

    Ich warte noch auf deine Erklärung dass es NICHT so ist.

    ((( Wenn es nur so aussieht dass die Zeit sich dehnt dann gibt es keine tatsächliche Zeitdehnung und wir brauchen diese Unterhaltung gar nicht zu führen.)))

    +++ Es “sieht nicht nur so aus” – aber nur der Vergleich zweier Messungen kann die – relative – Dehnung nachweisen. +++

    Das Phänomen der Myonen ist absolut und es geschieht nicht erst durch irgendwelche Vergleiche.
    Wenn eine Uhr vor oder nachgeht, nach einem Pic-Nic in den Bergen, das ist auch ein absolutes Ereignis. Nur der Beobachter braucht den Vergleich um den Unterschied zu merken. Also, die “Effekte” der Zeitdehnung sind in der Tat, nach deiner Theorie, tatsächlich und absolut.
    Wenn aber bei einer Zeitdehnung die Zeit nicht vorbei gehen kann, bleibt dir noch zu erklären wie das vor sich gehen soll.
    Wie kann ein Objekt in die Zeit bewegen wenn sein Ausgangspunkt sich stetig und unendlich dehnt?!

    +++Wenn wir beide jeder einen Maßstab haben, aber meiner ist aus Gummi und kann sich dehnen, dann merke ich die Dehnung an dem Maßstab selbst nicht – wohl aber, wenn ich meine Messergebnisse mit deinen Vergleiche.+++

    Du hast da keine Ahnung was du sagst!
    Ein Gummiband mit der Zeit und ihre Ausdehnung vergleichen und erklären wollen?
    Tut mir Leid aber das ist weit unter mein Niveau. Da bräuchte ich eine halben Tag um dir deiner horrenden Konzept Fehlern zu erklären, bis du sie kapiert hast. Falls du sie Kapierst.
    Keine Gummis bitte.

  798. #801 MartinB
    4. Januar 2016

    “Du hast da keine Ahnung was du sagst!..Tut mir Leid aber das ist weit unter mein Niveau. ”
    ROFL.
    Danke, nichts fördert die Verdauung so gut wie herzhaftes Gelächter.
    Nein, was Physik angeht ist nichts unter deinem Niveau – die spielt sich komplett oberhalb deiner Reichweite ab.

  799. #802 Zweisteine
    4. Januar 2016

    /// MartinB
    4. Januar 2016
    “Du hast da keine Ahnung was du sagst!..Tut mir Leid aber das ist weit unter mein Niveau. ”
    ROFL.
    Danke, nichts fördert die Verdauung so gut wie herzhaftes Gelächter.
    Nein, was Physik angeht ist nichts unter deinem Niveau – die spielt sich komplett oberhalb deiner Reichweite ab.///

    Oberhalb meiner Reichweite?
    Wenn du mir andrehen willst dass objektive, direkt beobachtbare Veränderungen in der Natur, nur durch vergleich als Veränderung wahrnehmbar sind?!
    Das soll oberhalb meiner Reichweite sein?

    Ok.
    Du hast keine Antwort auf die angebliche Zeitdehnung und die daraus resultierende Unmöglichkeit der Bewegung in der Relativistische Auffassung der Zeit.
    Das ist schon in Ordnung.
    Musst du aber nicht deswegen mich persönlich angreifen.

  800. #803 MartinB
    4. Januar 2016

    @Zweisteine
    “Du hast keine Antwort auf die angebliche Zeitdehnung und die daraus resultierende Unmöglichkeit der Bewegung in der Relativistische Auffassung der Zeit.”
    Doch, du bist nur entweder zu dumm oder zu vernagelt, um die Antwort zu verstehen.

    “Musst du aber nicht deswegen mich persönlich angreifen.”
    Tue ich nicht – ich stelle lediglich beobachtbare Tatsachen fest, und deine Ahnungslosigkeit in Sachen Physik hast du hier eindrucksvoll unter Beweis gestellt.

  801. #804 Zweisteine
    4. Januar 2016

    //// MartinB
    4. Januar 2016
    @Zweisteine
    Doch, du bist nur entweder zu dumm oder zu vernagelt, um die Antwort zu verstehen.///

    Ich bin zu dumm oder zu vernagelt wenn du ständig aus dem Handbuch rezitierst und den unterschied zw. einen absoluten und einen relativen Phänomen durcheinander bringst?
    Du kannst partout nicht kapieren dass in einer sich dehnende Zeit keine Bewegung möglich ist, weil die Zeit einfach nicht vorbei gehen kann und du willst mich, als dummer darstellen?

    Natürlich wirst du in eine Krise geraten wenn du ständig von einer Zeit sprichst die, so, in der Natur nicht existiert. Und viel schlimmer wird wenn du diese inexistente Zeit noch dehnen willst und dazu glauben dass darin Bewegung möglich ist.
    DAS nenne ich eine Vernagelte Wahrnehmung.

    Und die Frage der Zeit ist nicht komplizierter als das. So hör auf, ständig um den heissen Brei herum zu reden.

    “Musst du aber nicht deswegen mich persönlich angreifen.”
    /// Tue ich nicht – ich stelle lediglich beobachtbare Tatsachen fest, und deine Ahnungslosigkeit in Sachen Physik hast du hier eindrucksvoll unter Beweis gestellt.///

    Habe ich dir schon gesagt, meine Ahnungslosigkeit muss du zuerst beweisen um sie mir vorwerfen zu können. So, mit billige, unbegründete Beleidigungen kommst du hier nicht weiter .

  802. #805 Hartmut Schüppler
    4. Januar 2016

    Hallo Josef Braun,
    man stellt die Zeit geometrisch in Form einer Strecke dar, indem man sie mit der Lichtgeschwindigkeit multipliziert.
    Im kartesischen Koordinatensystem wird die y-Achse dann häufig als ct-Achse bezeichnet.
    MfG
    H.

  803. #806 MartinB
    4. Januar 2016

    @Zweisteine
    ” meine Ahnungslosigkeit muss du zuerst beweisen ”
    Nein, das hast du selbst zur Genüge getan.
    Und nein, ich werde nicht weiter mit dir diskutieren (und bitte auch alle anderen zu überlegen, ob sie die Diskussion nicht einstellen wollen, da sie zu nix führt) – eine Diskussion, in der einer immer nur sagt “Das kann aber nicht sein, weil ich es mir nicht vorstellen kann, und deswegen ist es auch nicht so” ist wenig ergiebig. Meinetwegen kannst du gern glauben, dass alle Physikerinnen doof sind und sich nie darüber Gedanken gemacht haben, was die Zeitdilatation genau bedeutet – wie ich schon im Eingangstext schrieb “You cannot reason people out of a position that they did not reason themselves into.”
    So, von mir war’s das.

  804. #807 Adent
    4. Januar 2016

    @Zweisteine

    So, mit billige, unbegründete Beleidigungen kommst du hier nicht weiter .

    Wieso, du zeigst doch sehr schön, dass man immer weiter kommt, wenn man so wie du nur billige unbegründete Annahmen und Beleidigungen postet.
    DU hinterfragst die belegte Theorie, also musst auch DU was bringen (und zwar einiges), kein anderer ist hier in der Bringschuld.

  805. #808 Peter GausL
    4. Januar 2016

    @Zweisteine
    Stimmst Du folgendem Satz zu?

    Wenn sich eine Strecke dehnt, dann wird sie unendlich lang.

    Wenn nein, was ist denn der Unterschied zu:

    Wenn sich die Zeit dehnt dann läuft für dieses Objekt nie eine Sekunde ab

  806. #809 Zweisteine
    4. Januar 2016

    /// MartinB
    4. Januar 2016
    @Zweisteine
    ” meine Ahnungslosigkeit muss du zuerst beweisen ”
    Nein, das hast du selbst zur Genüge getan.////

    Das nennt man, ohne Beweise, “Behauptung”.

    /// Und nein, ich werde nicht weiter mit dir diskutieren (und bitte auch alle anderen zu überlegen, ob sie die Diskussion nicht einstellen wollen, da sie zu nix führt) – eine Diskussion, in der einer immer nur sagt “Das kann aber nicht sein, weil ich es mir nicht vorstellen kann, und deswegen ist es auch nicht so” ist wenig ergiebig.////

    Nein mein Lieber. Diese Rhetorik ist von dir.
    Ich habe dir deutlich dargestellt warum in eine sich dehnende Zeit keine Bewegung möglich ist.
    Du hast bisher keine Gegendarstellung gebracht und sagst ständig meine Auffassung sei falsch. Einfach so.
    Das ist natürlich wenig ergiebig.

    /// Meinetwegen kannst du gern glauben, dass alle Physikerinnen doof sind und sich nie darüber Gedanken gemacht haben, was die Zeitdilatation genau bedeutet – wie ich schon im Eingangstext schrieb “You cannot reason people out of a position that they did not reason themselves into.”///

    Exakt. Du hast dir nie Gedanken darüber gemacht was Zeit eigentlich ist. Also, du wirst auch Mühe haben zu verstehen was sie NICHT ist.

    ///So, von mir war’s das.///

    Right from the beginning.

  807. #810 Zweisteine
    4. Januar 2016

    /// Peter GausL
    4. Januar 2016
    @Zweisteine
    Stimmst Du folgendem Satz zu?

    Wenn sich eine Strecke dehnt, dann wird sie unendlich lang.///

    Wenn du mir zuerst erklärst WAS für eine Strecke ist das die sich dehnt, kann ich mir dann Gedanken darüber machen.
    Oder wenn du mir erklärst, was würde diese Strecke daran hindern, sich unendlich lang zu dehnen.
    Ok?

  808. #811 Peter GausL
    4. Januar 2016

    Oder wenn du mir erklärst, was würde diese Strecke daran hindern, sich unendlich lang zu dehnen.

    Andersherum: Erklär mal warum sich die Zeit unendlich dehnen sollte. Du bist immerhin derjenige der das behauptet.

  809. #812 Zweisteine
    4. Januar 2016

    /// Peter GausL
    4. Januar 2016
    Andersherum: Erklär mal warum sich die Zeit unendlich dehnen sollte. Du bist immerhin derjenige der das behauptet.////

    Genau. So fragt man das!

    Sie würde sich unendlich ausdehnen weil dieser Prozess sich unendlich fortsetzen würde. Eine unendliche Progression.
    Es ist nicht so dass sich eine ganze Sekunde Dehnt aber jeder Teil dieser Sek. und wiederum jeder Teil der Teile. usw.
    D.h. Nichts kann sich von seinem Zeitpunkt fortbewegen.
    Und niemand konnte hier, bisher, erklären WARUM es nicht so sein sollte.

    Fazit. In der RT ist Bewegung unmöglich. Und ohne Bewegung ist die RT auch hinfällig.

  810. #813 Hartmut Schüppler
    4. Januar 2016

    Hallo Zweisteine,
    in einem Raumzeitmodell gibt es keine relativistischen Effekte und damit auch keine Zeitdehnung. Es gibt nur ein Koordinatensystem und das ist die Raumzeit, die man zur Vereinfachung auch räumlich eindimensional als Minkowski-Diagramm darstellen kann. Tatsächliche Vorgänge erscheinen als Vektoren, die durch Bezug auf eine oder mehrere Raumachsen und auf die Zeitachse den Raum- und Zeitanteil der Bewegung repräsentieren.
    Relativistische Effekte erhält man nur, wenn man zumindest zwei, im Verhältnis zueinander bewegte Objekte, jeweils mit einem Koordinatensystem aus Raum und Zeit verbindet. Die Betrachtung des einen Koordinatensystems aus dem anderen heraus (dem Bezugssystem) führt zu einer perspektivischen Verzerrung von Raum und Zeit (Zeitdilatation und Längenkontraktion) genannt. Tatsächlich verzerrt oder gedehnt wird auch bei dieser Betrachtung nichts (Relativitätsprinzip).
    MfG
    H.

  811. #814 Peter GausL
    4. Januar 2016

    Wenn sich eine Zeitspanne um x Prozent dehnt, dehnt sich jeder beliebige Teil der Spanne auch um x Prozent. Und jeder beliebige Teil dieser Teile auch. Wie ändert das die Dehnung der gesamten Spanne? Die beträgt immer noch x Prozent.

    Und niemand konnte hier, bisher, erklären WARUM es nicht so sein sollte.

    Erst einmal müsstest Du erklären warum eine endliche Dehnung einer Zeitspanne unmöglich sein sollte.

  812. #815 Braun Josef / IVE
    home: ive.xyz
    4. Januar 2016

    @Hartmut Schüppler “Einheiten”
    Ja, Zeit mal (Licht-)Geschwindigkeit ergibt den Weg in Meter, dann hab ich statt der angegebenen Sekunde Meter? MfG

  813. #816 Zweisteine
    4. Januar 2016

    //// Hartmut Schüppler
    4. Januar 2016
    Hallo Zweisteine,
    in einem Raumzeitmodell gibt es keine relativistischen Effekte und damit auch keine Zeitdehnung.////

    ///Peter GausL
    4. Januar 2016
    Erst einmal müsstest Du erklären warum eine endliche Dehnung einer Zeitspanne unmöglich sein sollte.///

    Und hier eine typische Situation die man oft bei Relativisten begegnet :

    Jeder hat seine Version der Dinger.
    Der eine sagt dir: “nä, da gibt es keine Zeitdehnung” und der anderer versucht gerade seine Erklärung durchzusetzen dass die Zeit sich doch dehnt.

    wo soll man ansetzen?

  814. #817 Zweisteine
    4. Januar 2016

    //// Peter GausL
    4. Januar 2016
    Wenn sich eine Zeitspanne um x Prozent dehnt, dehnt sich jeder beliebige Teil der Spanne auch um x Prozent. Und jeder beliebige Teil dieser Teile auch. Wie ändert das die Dehnung der gesamten Spanne? Die beträgt immer noch x Prozent.////

    1°) Es ist nicht der xProzent bestimmend aber die unendliche Wiederholung dieses xProzents.
    2°) Wenn die Zeit still steht weil die Zeit sich unendlich dehnt, du hast überhaupt noch keine “gesamte Spanne”!

    /// Erst einmal müsstest Du erklären warum eine endliche Dehnung einer Zeitspanne unmöglich sein sollte.///

    Woher hast du jetzt das die Dehnung “endlich” ist?
    “Endlich” deutet auf Zeitablauf, wir haben aber bei der Zeitdehnung kein Zeitablauf.
    Da kommt nichts vom Fleck.
    Ausser du willst jetzt eine Zeit erfinden die die Dehnung bestimmt, die sich wiederum dehnen wird usw. Ad Infinitum…ich komme dich gerne in der Klapsmühle besuchen dann….

  815. #818 Zweisteine
    4. Januar 2016

    /// Hartmut Schüppler
    4. Januar 2016
    Die Betrachtung des einen Koordinatensystems aus dem anderen heraus (dem Bezugssystem) führt zu einer perspektivischen Verzerrung von Raum und Zeit (Zeitdilatation und Längenkontraktion) genannt. Tatsächlich verzerrt oder gedehnt wird auch bei dieser Betrachtung nichts (Relativitätsprinzip).////

    Also, das ganze ist wie eine Fata Morgana. Ja?
    Ok.

  816. #819 Peter GausL
    4. Januar 2016

    Es ist nicht der xProzent bestimmend aber die unendliche Wiederholung dieses xProzents.

    Wie kommst Du auf die unendliche Wiederholung?

  817. #820 Zweisteine
    4. Januar 2016

    /// Peter GausL
    4. Januar 2016
    Es ist nicht der xProzent bestimmend aber die unendliche Wiederholung dieses xProzents.

    Wie kommst Du auf die unendliche Wiederholung?///

    Es ist ein Bild, der veranschaulichen soll dass 0 Zeit sich nicht dehnen kann.
    Wo keine Zeit ist gibt es auch keinen Prozess oder Ablauf.
    Ausser du willst jetzt sagen dass die Dehnung sich aus einer Vermehrung der 0 Zeiten ergibt. Aber das würde wiederum die Bewegung verunmöglichen, weil der Augenblick sich auch unendlich dehnen würde. In null Zeit und das wäre tatsächlich unendlich.

    Du kannst hiermit langsam sehen wo die Konzeptlosigkeit der Relativisten hinführt.

  818. #821 Peter GausL
    4. Januar 2016

    Es ist ein Bild, der veranschaulichen soll dass 0 Zeit sich nicht dehnen kann.

    Dass ein gedehnter Zeitraum automatisch unendlich lang wird (was Du ohne Beleg oder Begründung behauptest), zeigt dass sich eine Zeitspanne der Länge 0 nicht dehnen kann? Hältst Du das für Logik?

    Du kannst hiermit langsam sehen wo die Konzeptlosigkeit der Relativisten hinführt.

    Ich sehe etwas anderes …

  819. #822 Zweisteine
    5. Januar 2016

    //// Peter GausL
    4. Januar 2016
    ((( Es ist ein Bild, der veranschaulichen soll dass 0 Zeit sich nicht dehnen kann.)))

    Dass ein gedehnter Zeitraum automatisch unendlich lang wird (was Du ohne Beleg oder Begründung behauptest), zeigt dass sich eine Zeitspanne der Länge 0 nicht dehnen kann? Hältst Du das für Logik? ////

    Auch wenn ich es nicht exakt so gesagt habe es ist im Endeffekt das gleiche.

    ((( Du kannst hiermit langsam sehen wo die Konzeptlosigkeit der Relativisten hinführt.))))))

    /// Ich sehe etwas anderes ///

    Du siehst gar nichts. Du rezitierst einfach aus dem Handbuch und versuchst mit Rhetorik hier Klug auszusehen. Sonst wärest du schon draufgekommen dass eine Zahl sich gar nicht dehnen kann.

  820. #823 Peter GausL
    5. Januar 2016

    Sonst wärest du schon draufgekommen dass eine Zahl sich gar nicht dehnen kann.

    Wer hat denn behauptet das sich Zahlen dehnen?

  821. #824 Zweisteine
    5. Januar 2016

    Peter GausL
    5. Januar 2016
    (((Sonst wärest du schon draufgekommen dass eine Zahl sich gar nicht dehnen kann.)))

    /// Wer hat denn behauptet das sich Zahlen dehnen?///

    Relativisten tun das. Zeit ist nicht mehr als ein Faktor, eine Zahl. Relativisten sind felsenfest überzeugt aber, dass die Zeit eine “Dimension” ist….und das diese Dimension sich dehnt…..

  822. #825 Peter GausL
    5. Januar 2016

    Zeit ist nicht mehr als ein Faktor, eine Zahl.

    Noch eine unbelegte Behauptung von dir.

    Was ist jetzt eigentlich deiner Meinung nach das Problem? Dass Zeit sich nicht dehnen kann da sie nur eine Zahl ist oder das sie ins Unendliche gedehnt wird?

  823. #826 Zweisteine
    5. Januar 2016

    //// Peter GausL
    5. Januar 2016
    ((( Zeit ist nicht mehr als ein Faktor, eine Zahl.)))

    /// Noch eine unbelegte Behauptung von dir.///

    Dann weisst du nicht wie man Zeit misst. Aber du willst wissen dass sie sich dehnt.

    /// Was ist jetzt eigentlich deiner Meinung nach das Problem? Dass Zeit sich nicht dehnen kann da sie nur eine Zahl ist oder das sie ins Unendliche gedehnt wird?///

    Das ist das interessante hier. Du kannst es drehen und betrachten wie es dir beliebt, die Sache mit der Dehnung geht nicht auf. Zeitdehnung bringt nur Stillstand. Auch logisch ist das, weil die Dehnung zuerst den Augenblick dehnt und der lässt sich nicht mehr überwinden.

  824. #827 Adent
    5. Januar 2016

    @Zweisteine

    Auch logisch ist das, weil die Dehnung zuerst den Augenblick dehnt und der lässt sich nicht mehr überwinden.

    Und noch eine unbelegte Behauptung, schon 100 erreicht?

    Zeit ist nicht mehr als ein Faktor, eine Zahl.

    Was denn nun, Faktor oder Zahl? Und seit wann kann man eine Zahl nicht dehnen? Hast du als Kind nie mit den Gummizahlenteppichen gespielt Zweistein?
    Du hältst deine kläglichen Versuche Zeit zu definieren also für logisch, sonst niemand, hmmmmm woran erinnert mich das bloß…?

  825. #828 Hartmut Schüppler
    5. Januar 2016

    Hallo Zweisteine,
    man kommt wohl nicht umhin, zunächst zu klären, was “Zeit” ihrem Wesen nach ist und entsprechend begrifflich zu beschreiben. Allerdings haben sich daran schon andere Geistesgrößen, als wir es sind, mit mehr oder weniger Erfolg versucht.
    Der allgemeinsprachliche Zeitbegriff ist diffus und man muss ihn deshalb zunächst auf auf seinen Kern reduzieren. Den sehe ich im Begriff “Dauer”. Unsere Vorstellung von Dauer kommt dadurch zustande, dass wir einen Vorgang zu einem anderen Vorgang in Beziehung setzen, also zumindest zwei Vorgänge miteinander vergleichen. Wenn der zweite Vorgang periodischer Natur ist, haben wir eine Normierung und können Zeit zählen. Dies ist das Prinzip von Uhren.
    Beispiel: Vorgang1: Fußballspiel
    Vorgang2: Gang einer Uhr mit 90 Minuten.

    Nach meiner Meinung ist Zeit daher im Prinzip nichts anderes als eine Vorstellung, die Menschen zur Beschreibung von Vorgängen entwickelt haben; sozusagen ein Programm (software) unseres Gehirns, das bei der Programmierung des Gehirns von Kindern gelernt werden muss.
    Zeit ist daher ihrem Wesen nach nicht als Zahl beschreibbar. Als Dimension wird sie bei ihrer mathematisch/geometrischen Beschreibung aufgefasst.
    MfG
    H.

  826. #829 Zweisteine
    5. Januar 2016

    /// Hartmut Schüppler
    5. Januar 2016
    Hallo Zweisteine,
    man kommt wohl nicht umhin, zunächst zu klären, was “Zeit” ihrem Wesen nach ist und entsprechend begrifflich zu beschreiben. Allerdings haben sich daran schon andere Geistesgrößen, als wir es sind, mit mehr oder weniger Erfolg versucht.///

    Die Zeit wie sie in der Physik gebraucht wird ist, entsteht aus dem Vergleich zweier Prozesse und daraus ergibt sich eine Zahl. Diese Prozesse finden immer noch in eine der bekannten drei Räumliche Dimensionen statt.
    Einstein hat daraus eine Vierte Dimension gemacht, in etwa, eine der dreien noch dazu gezählt. (sehr wahrscheinlich sieht deswegen das RT Bild der Welt so verzerrt aus).
    Und diese Zahl, die wir Zeit nennen, soll sich noch dehnen?

  827. #830 Hartmut Schüppler
    5. Januar 2016

    Hallo Zweisteine,
    man kann Vorgänge (Veränderungen, Bewegungen) mathematisch/geometrisch mit den Kategorien Raum und Zeit in verschieden vielen Dimensionen beschreiben.
    Lediglich unser Vorstellungsvermögen (die Leistungsfähigkeit unseres Gehirns) ist normalerweise auf drei Dimensionen beschränkt.
    Man kann die Darstellung auch vereinfachen, indem man für eine Raumzeitmodells lediglich zwei Dimensionen verwendet; z.B. ein kartesisches Koordinatensystem mit einer Raumachse (x-Achse = Strecke) und Zeitachse (y-Achse).
    Ein Objekt, das ruhend vorgestellt wird (keine räumliche Veränderung) bewegt sich in diesem Modell nur auf der Zeitachse. Elektromagnetische Wellen (Licht) bewegen sich in diesem Modell in einem 45°-Winkel, d.h. mit gleichen Anteilen in Raum und Zeit. Die Lichtgeschwindigkeit (Beziehung zwischen Raum und Zeit) hat in diesem Modell den Wert 1.
    Wahrnehmbar sind nur Bewegungen zwischen Punkten, die zwischen y-Achse (Zeitachse) und 45°-Linie liegen, weil nur bei ihnen der Zeitanteil der Bewegung größer ist als der Raumanteil. Anders ausgedrückt: Das Licht hat genügend Zeit des Raum zu überbrücken.
    Die Strecke zwischen zwei Punkten, die eine Veränderung (Bewegung) in Raum und Zeit repräsentiert (Raumzeitintervall, Weltlinie), wird mit Hilfe des Satzes des Pythagoras berechnet. Dabei muss man allerdings der Tatsache Rechnung tragen, dass Raum und Zeit begrifflich Gegensätze sind und nicht einfach addiert werden können. Dies ist mit komplexen Zahlen möglich, weil diese aus einem reelen und (gegensätzlich) imaginären Teil bestehen. Der imaginäre Anteil (i) löst sich bei Anwendung des Satzes des Pythagoras allerdings auf, weil i^2 =-1 ist.
    Das Raumzeitintervall kann man auch als Vektor auffassen, der durch seinen Bezug auf die y-Achse den Zeitanteil und durch Bezug auf die x-Achse den Raumanteil der Veränderung repräsentiert.
    Mein Problem beim Verständnis der Dinge war, dass die verschiedenen mathematischen Modelle nicht klar auseinander gehalten werden.
    Deshalb habe ich hier einmal ein Raumzeitmodell relativ umfangreich begrifflich dargestellt.
    Sobald man zwei im Verhältnis zueinander bewegte Uhren betrachtet, ist dieses Modell nicht mehr brauchbar, weil man dann zwangsläufig mit den bewegten Objekten jeweils ein Koordinatensystem verbindet und es treten die relativistischen Effekte auf, wenn man eines der Systeme zum Bezugssystem macht.
    mfG
    H.

  828. #831 Zweisteine
    5. Januar 2016

    ///Hartmut Schüppler
    5. Januar 2016// Dabei muss man allerdings der Tatsache Rechnung tragen, dass Raum und Zeit begrifflich Gegensätze sind und nicht einfach addiert werden können.///

    Das ist erfunden. Was du Zeit nennst ist immer noch Raum.
    Wieso sollten denn R u. Z gegensätzlich sein? einfach so?

    /// Ein Objekt, das ruhend vorgestellt wird (keine räumliche Veränderung) bewegt sich in diesem Modell nur auf der Zeitachse.///

    Zeit ist keine Dimension!
    Das ist wieder eine Erfindung.
    Du kannst natürlich alles verdrehen und daraus abstrakte Systeme aufbauen, d.h. aber nicht dass sie real sind.
    Weil du Zeit als Dimension hinstellst heisst nicht das Zeit dadurch zur Dimension wird.
    Das ruhende Objekt bewegt sich nicht in der Zeitachse. Er ruht bloss währenddem die Uhr tickt und ihre Zeiger sich im Raum bewegen.
    Du hast den alten guten dreidimensionalen Raum noch nicht verlassen. Du kritzelst einfach unverständliche Zeichen und Symbole an der Wand.
    Zum welchem Zweck?

  829. #832 MartinB
    5. Januar 2016

    @alle
    Bitte aufhören, mit Zweisteine zu diskutieren. Das müllt hier nur die Kommentare voll und Sinn hat es offensichtlich keinen – Zweistein ist vollständig denk- und argumentresistent, wie der Verlauf der Diskussion zeigt.

  830. #833 Zweisteine
    5. Januar 2016

    /// MartinB
    5. Januar 2016
    @alle
    Bitte aufhören, mit Zweisteine zu diskutieren. Das müllt hier nur die Kommentare voll und Sinn hat es offensichtlich keinen – Zweistein ist vollständig denk- und argumentresistent, wie der Verlauf der Diskussion zeigt.///

    Du hast noch nicht erklärt wie sich Zeit dehnen kann.
    Wie kannst du sagen das ich “argumentsresistent” bin, wenn du noch kein Argument gebracht hast?
    Bring mal eine Erklärung und lass uns sehen wie gut du die RT verstehst…….lass dir was einfallen. Nicht aber aus dem Handbuch, da bist du sowieso auf verlorenen Posten. Mal was eigenes.

    (und wenn Hartmut gerne ausschweift ist doch noch lange nicht meine Schuld! Er schwebt gerne in imaginären Welten…)

  831. #834 Braun Josef / IVE
    home: ive.xyz
    6. Januar 2016

    @Krypto
    Von Dir: “Kleiner Tipp:
    Versuche erst einmal, die SRT zu verstehen. Und fang dabei ganz am Anfang bei den Postulaten an…”

    For you, Skript Vorlesung: https://ive.xyz/wp-content/uploads/2015/08/zueb11-5.pdf siehe Seite 2 und Bruch bzw. Denkfehler bei Seite 5 Mitte vgl. #755. MfG

  832. #835 MartinB
    6. Januar 2016

    @Zweisteine und Braun Josef
    Bitte hört auf, hier weiter immer dieselben Dinge zu posten, es nervt wirklich.

  833. #836 Zweisteine
    6. Januar 2016

    ///MartinB
    6. Januar 2016
    @Zweisteine und Braun Josef
    Bitte hört auf, hier weiter immer dieselben Dinge zu posten, es nervt wirklich.///

    Du schreibst ein Artikel mit dem Titel :
    “Kann die Spezielle Relativitätstheorie falsch sein?”

    …und wenn man dir Antwort gibt dass sie tatsächlich falsch ist, das nervt dir.

    Es ist aber so. Ohne Zeitdehnung keine RT.

  834. #837 Braun Josef / IVE
    home: ive.xyz
    9. Januar 2016

    @MartinB
    Zu wieviel Prozent warst Du Dir sicher, das die SRT falsch ist, wie Du den Artikel geschrieben hast. MfG

  835. #838 Braun Josef / IVE
    home: ive.xyz
    10. Januar 2016

    @MartinB
    Ich finde es nicht gut, daß Du auf meine Fragen nicht antwortest. MfG

  836. #839 MartinB
    11. Januar 2016

    @JosefBraun
    Zur Klarstellung: ich muss nicht jede absurde Frage beantworten.

  837. #840 Braun Josef / IVE
    home: ive.xyz
    11. Januar 2016

    @MartinB
    Ich finde, das ist jetzt eine Rausrederei. Egal, da sowieso zu #755 kein sachliches Gegenargument kam ist es wieder klar, daß die SRT falsch ist. MfG

  838. #841 MartinB
    11. Januar 2016

    @Braun Josef
    Du glaubst gar nicht, wie egal mir ist, was du findest…

  839. #842 Braun Josef / IVE
    home: ive.xyz
    11. Januar 2016

    @MartinB
    Warum machst du dann den Blog hier?

  840. #843 Spritkopf
    11. Januar 2016

    Egal, da sowieso zu #755 kein sachliches Gegenargument kam ist es wieder klar, daß die SRT falsch ist.

    Das ist kein Argument, sondern das Genöle eines verzogenen Kleinkindes.

  841. #844 rolak
    11. Januar 2016

    da sowieso zu #755 kein sachliches Gegenargument kam ist es wieder klar, daß die SRT falsch ist

    Heißa, das sticken wir ins nächste Zierdeckchen!

    Das Universalrezept für alles und jedes: Wenn du es schaffst, derart absurden Blödsinn abzusondern, daß keiner mehr Lust hat das auseinanderzuklamüsern, dann hast du recht. Genial^^

    MfG

    Immerhin, die Schlußformel stimmt: Multiple failure guaranteed.

  842. #845 Braun Josef / IVE
    home: ive.xyz
    11. Januar 2016

    @Spritkopf
    Es ist die Widerlegung an einer deutschen Hochschule.
    Zum Kleinkind: https://ive.xyz/?page_id=45 denkst du, du kannst es mit mir aufnehmen??

  843. #846 MartinB
    11. Januar 2016

    @Josef
    Ich mache den Blog für Leute, die an Wissenschaft interessiert sind und in der Lage sind, vernünftig zu denken und zu argumentieren.
    Für argumentresistente Leute wie dich mache ich ihn nicht.

  844. #847 Braun Josef / IVE
    home: ive.xyz
    11. Januar 2016

    @rolak
    Wer behauptet, “das Universalrezept für alles und jedes”?!
    Die im Kern 2 Seiten mit Bildern, da gibts nicht viel zu verstehen.

  845. #848 Braun Josef / IVE
    home: ive.xyz
    11. Januar 2016

    @MartinB
    Jetzt geht es ohne Gründe nur ins persönliche (Facebook läßt wieder grüßen) wo sind den deine Argumente?

  846. #849 Spritkopf
    11. Januar 2016

    @rolak

    Das Universalrezept für alles und jedes: Wenn du es schaffst, derart absurden Blödsinn abzusondern, daß keiner mehr Lust hat das auseinanderzuklamüsern, dann hast du recht.

    Das Traurige ist ja, dass der Herr schon seit mindestens 10 Jahren mit dem gleichen Schmarrn um die Häuser zieht, ob als JotBe, als Josef Braunstein oder unter welchem Alter ego auch sonst (Spionage-Geheule in drei, zwo, eins …!). Und seit diesen 10 Jahren erklären ihm alle möglichen Leute – zuletzt Markus Pössel auf Scilogs – vollkommen umsonst, dass er den Kardinalfehler aller selbsternannten Einstein-Widerleger begeht und von einer absoluten Raumzeit ausgeht.

    Dummerweise ficht ihn das – wie alle anderen Einstein-Widerleger ebenfalls – nicht an. Egal, wie sachlich ihm in diesen zehn Jahren begegnet wurde – Braun widerlegt ja nicht die vom Bezugssystem abhängige Raumzeit, sondern er setzt in dem, was er als Widerlegung bezeichnet, einfach den Newton’schen Raum als fixe Prämisse und alle Versuche, ihn darauf hinzuweisen, scheitern daran, dass er dieses Argument nicht zur Kenntnis nehmen will und es daher standhaft ignoriert. Dass MartinB die Debatte mit so jemandem für fruchtlos hält, ist nachvollziehbar.

  847. #850 Spritkopf
    11. Januar 2016

    @Braun

    Zum Kleinkind: https://ive.xyz/?page_id=45 denkst du, du kannst es mit mir aufnehmen??

    Danke nein, auf Schw***vergleiche habe ich keine Lust.

  848. #851 Braun Josef / IVE
    home: ive.xyz
    11. Januar 2016

    @Spritkopf
    Du vergleichst also nur Kleinkinder?!

  849. #852 Braun Josef / IVE
    home: ive.xyz
    11. Januar 2016

    @Spritkopf @rolak
    “JotBe, als Josef Braunstein” – kenn ich nicht, Ihr tut mir Unrecht das ist alles.

  850. #853 Zweisteine
    11. Januar 2016

    /// MartinB
    11. Januar 2016
    Ich mache den Blog für Leute, die an Wissenschaft interessiert sind und in der Lage sind, vernünftig zu denken und zu argumentieren.
    Für argumentresistente Leute wie dich mache ich ihn nicht.///

    Du könntest sehr schnell und glaubwürdig deine Haltung beweisen in dem du, endlich, meine Frage beantwortest- was du bisher nicht getan hast.

    Wie ist Bewegung möglich wenn sich die Zeit dehnt?
    Oder, wenn diese Frage zu schwierig scheint :
    Wie kann Bewegung den Ruhepunkt überwinden wenn sich die Zeit dehnt?
    Oder:
    Wie kann sich ein Null-Zeitpunkt überhaupt dehnen?
    Diese letzte Frage wird noch einfacher wenn du dich auf den ersten Null-Zeitpunkt des Universums beziehst, ohne störende Nebengeräusche die dich ablenken könnten.

    Diese Frage betrifft die Tragsäule der RT, du kannst sie unmöglich absurd finden. Ausser du findest den ganzen relativistischen Kram absurd, was dein gutes recht wäre.
    Aber die Frage scheint nicht so absurd zu sein, weil, die sonst redseligen Draufgänger der SBs Instantan auf die Zähne gebissen haben als sie sie gelesen haben.

    Also Martin, viel Glück bei deiner Aufgabe.

  851. #854 MartinB
    11. Januar 2016

    @Zweisteine
    Von Leuten wie dir lasse ich mir keine Aufgaben stellen. Dir wurde her zu Hauf erklärt, warum deine Fragestellung unsinnig ist, dass du es nicht verstehen kannst oder willst, ist wirklich dein Problem.

  852. #855 Zweisteine
    11. Januar 2016

    /// MartinB
    11. Januar 2016
    @Zweisteine
    Von Leuten wie dir lasse ich mir keine Aufgaben stellen. Dir wurde her zu Hauf erklärt, warum deine Fragestellung unsinnig ist, dass du es nicht verstehen kannst oder willst, ist wirklich dein Problem.///

    Von Leute wie mir?
    Du kennst mich ja gar nicht.

    Und was die Unsinnigkeit der Frage betrifft, du hast bis Dato keine nachvollziehbare Erklärung geliefert, ausser in deiner Fantasie, dort wo du auch die RT für richtig hältst.
    Sei ehrlich, du hast nie im entferntesten gedacht dass die Widerlegung der RT so einfach sein könnte. Bloss weil Zeitdehnung und Bewegung sich gegenseitig ausschliessen.
    (du kannst dich aber damit trösten im wissen, dass nicht mal Einstein selbst an diesen Unsinn geglaubt hat)

  853. #856 MartinB
    11. Januar 2016

    @Zweisteine
    “Du kennst mich ja gar nicht.”
    Natürlich kenne ich dich – immerhin postest du hier ja einen Blödsinskommentar nach dem anderen.

    “du hast bis Dato keine nachvollziehbare Erklärung geliefert”
    Die Tatsache, dass du etwas nicht nachvollziehen kannst, sagt wenig über die generelle Nachvollziehbarkeit. Für dich magst du selbst ja der Maßstab allen Wissens und Denkens sein, für uns andere gilt das zum Glück nicht.

  854. #857 Zweisteine
    11. Januar 2016

    /// MartinB
    11. Januar 2016
    @Zweisteine
    “Du kennst mich ja gar nicht.”
    Natürlich kenne ich dich – immerhin postest du hier ja einen Blödsinskommentar nach dem anderen.///

    Muss du noch beweisen, ganz nach der wissenschaftliche Methode die du so getreu folgst. Persönliche Meinungen sind hier irrelevant.

    /// Für dich magst du selbst ja der Maßstab allen Wissens und Denkens sein, für uns andere gilt das zum Glück nicht.///

    Was für ein Gelaber. Zeitdehnung und Bewegung sind allgemeine Massstäbe und dass sie sich ausschließen ein Ergebnis barer Logik.

    Also, jetzt wo das geklärt ist sei nicht scheu. Greif zur Tastatur und tobt dich aus.

  855. #858 Braun Josef / IVE
    home: ive.xyz
    13. Januar 2016

    @Zweisteine
    Bitte hilf und schreib bei https://scienceblogs.de/astrodicticum-simplex/2016/01/01/sternengeschichten-folge-162-relativistische-quantenchemie/ daß ich gesperrt bin und es nicht drinsteht, weil ich u.a. eine Antwort auf Karl-Heinz (Zur Rotation/Vektor => Skalarprodukt) hätte.
    Vielen Dank (Braun-Wartenberg@t-online.de)

  856. #859 Zweisteine
    13. Januar 2016

    /// Braun Josef / IVE
    home: ive.xyz
    13. Januar 2016
    @Zweisteine
    Bitte hilf und schreib///

    Ich mache es später, sonst nimmt er es zur Anlass um mich auch zu sperren.

  857. #860 Higgs-Teilchen
    Im Standardmodell oben rechts
    14. Januar 2016

    @Zweisteine und Braun Josef

    “Bitte hilf und schreib… dass ich gesperrt bin…”

    Alter, ist euch das nicht peinlich?

    Ps. Was sollen wir bitte mit deiner E-Mail?

  858. #861 Braun Josef / IVE
    home: ive.xyz
    14. Januar 2016

    @Higgs-Teilchen
    Das ich antworten kann, es geht um einen anderen Artikel, in dem es so ausschaut, daß andere mich niederargumentiert haben aber ich bin nur gesperrt, ohne daß es drinsteht, die machen sich über mich lustig.
    Ich bin für Gerechtigkeit und nicht für Schadenfreude. MfG

  859. #862 Volker
    Berlin
    14. Januar 2016

    aber ich bin nur gesperrt, ohne daß es drinsteht, die machen sich über mich lustig

    Heul doch!

  860. #863 Braun Josef / IVE
    home: ive.xyz
    14. Januar 2016

    @Volker
    Wenn es bei dir so wäre, würdest du dich freuen, wenn du dich nicht wehren kannst?

  861. #864 Zweisteine
    14. Januar 2016

    ////Higgs-Teilchen
    Im Standardmodell oben rechts
    14. Januar 2016
    @Zweisteine und Braun Josef
    “Bitte hilf und schreib… dass ich gesperrt bin…”
    Alter, ist euch das nicht peinlich?///

    Mir sollte hier was peinlich sein?!
    Und wieso das?

    Hier ist ein Open-thread um den Artikel
    § Kann die Spezielle Relativitätstheorie falsch sein? §
    zu kommentieren und wenn einer den Beweis erbringt dass die Theorie doch falsch ist, dann wird man beleidigt und ignoriert und in manchen Threads auch gesperrt.
    Wem sollte hier was peinlich sein, eigentlich? Mir oder denjenigen die nicht widerlegen können dass Zeitdehnung und Bewegung sich gegenseitig ausschliessen und sich peinlich verhalten dazu?
    Also, Ich bitte dich !

  862. #865 MartinB
    14. Januar 2016

    @Josef und Zweisteine
    Das hier ist allerdings nicht das Forum für unverstandene Möchtegern-RT-Widerleger. Wenn ihr beide Kontakt miteinander aufnehmen wollt, könnt ihr mir gern ne mail schicken und ich leite die an den anderen weiter, falls ihr eure mailadressen hier nicht posten wollt. Diskussionen darüber, wer wo auf anderen Seiten gesperrt ist, gehören nicht hierher.
    “wenn einer den Beweis erbringt dass die Theorie doch falsch ist,”
    Das ist bisher nicht passiert.

  863. #866 Alderamin
    14. Januar 2016

    Vielleicht erkennt sich jemand hier wieder:

    https://www.final-frontier.ch/kleiner_einstein

    Ihr reitet Euch nur noch mehr in diese Ecke rein, Jungs.

  864. #867 Braun Josef / IVE
    home: ive.xyz
    14. Januar 2016

    @MartinB
    Warum nicht widerlegt? Der Bruch, Gedankenfehler ist bei der Vorlesung zur SRT https://ive.xyz/wp-content/uploads/2015/08/zueb11-5.pdf auf Mitte Seite 5 wenn man es extra mit 2 Zeichnungen ausführt. Abgesichert noch mit der Ablenkung im Michelson-Morley-Experiment https://ive.xyz/wp-content/uploads/2014/01/Michelson.pdf.pdf – wenn das nicht reicht, habt Ihr überhaupt die Originalschrift dazu gelesen? MfG

  865. #868 Zweisteine
    14. Januar 2016

    ////MartinB
    14. Januar 2016
    @Josef und Zweisteine
    Diskussionen darüber, wer wo auf anderen Seiten gesperrt ist, gehören nicht hierher.///

    Dass ist korrekt. ich habe allerdings keine Diskussion geführt aber nur einmal geantwortet.

    /// “wenn einer den Beweis erbringt dass die Theorie doch falsch ist,”
    Das ist bisher nicht passiert.////

    Nicht passiert?
    Wirf mal ein Blick weiter oben. Das ist doch passiert und du hast es noch nicht widerlegen können.
    Ich verstehe dass es dir peinlich ist dass die ganze RT an einen solchen elementaren Widerspruch scheitern sollte. Aber es ist so, Zeitdehnung ist die Grundlage und wenn das gar nicht gibt , gar nicht geben kann, dann gibt es die RT auch nicht.
    Wenn du mein einfaches Argument nicht verstehst, wie kommst du drauf dass du die RT verstehst?

    “Bewegung und Zeitdehnung schliessen sich gegenseitig aus, weil die Zeit sich nur von Null aus dehnen kann und das ist unmöglich. ”

    Du nennst mich einen “unverstandenen Möchtegern-RT-Widerleger.”
    Glaubst du, ich mache es beruflich?
    Mir ist dieser katastrophaler Konzeptfehler beim frühstücken aufgefallen. Ich beschäftige mich sonst mit wichtigeren Sachen.
    Du bist aber hier der Profi und hast davon keine Ahnung und nimmst deine Zuflucht in eine unsinnige Rhetorik um davon abzulenken.

    Also. Kannst du mein Argument widerlegen oder nicht?
    Alles andere interessiert mich nicht.

    (und die Software dieses blogs ist Schrott. Alle drei Worte blockiert sich und man muss seine Sätze mehrmals korrigieren!!!)

  866. #869 Volker
    Berlin
    14. Januar 2016

    @Josef:

    Wenn es bei dir so wäre, würdest du dich freuen, wenn du dich nicht wehren kannst?

    Du befindest Dich hier nicht in einem öffentlichen Forum, sondern in der Kommentarspalte eines Blogartikels. Das bedeutet hier hat der Blogbetreiber (MB bzw. FF drüben) das Hausrecht und kann Dich sperren, rausschmeissen, etc., ganz nach seinem Belieben.

    Wenn Dir das nicht passt, eröffne Dein eigenes Blog, dort kann und wird Dich niemand sperren. Im Übrigen siehe hier:

    https://xkcd.com/1357/

  867. #870 Volker
    Berlin
    14. Januar 2016

    ^^ und weil ich das für sehr wichtig und richtig halte, hier das xkcd-Comic nochmal auf Deutsch, falls Du dem Englischen nicht mächtig bist:

    https://www.sprachlog.de/2014/04/19/xkcd-meinungsfreiheit/

  868. #871 Volker
    Berlin
    14. Januar 2016

    grrr… *des Englischen* sollte es heissen

  869. #872 Braun Josef / IVE
    home: ive.xyz
    14. Januar 2016

    @Volker
    Danke für das Deutsche, ist einfacher, aber ein Video braucht keinen Speicher. MfG

  870. #873 Braun Josef / IVE
    home: ive.xyz
    14. Januar 2016

    @Alle
    Also hat keiner “Zur Elektrodynamik bewegter Körper gelesen (S. 891-907)”?
    Gut, um S. 898 kann man u.a. wieder extra Zeichnungen machen, prinzipiell ist es aber wie der 1. Link von #755. MfG

  871. #874 zweisteine
    16. Januar 2016

    Und es gibt doch noch eine andere Frage die sich stellt in Bezug auf die Zeitdehnung :
    Wie können wir Objekte überhaupt sehen, die mit einer anderen Zeitdehnung als unsere unterwegs sind?
    Klar.
    Die Zeitdehnung würde nicht bloss eine Sekunde länger machen, sie verschiebt den Zeitpunkt der Gegenwart dazu. Unsere Gegenwart ist in Bezug auf die Gegenwart eines Objekts mit verschiedener Zeitdehnung, verschoben!
    Das heisst: Ein Objekt, das irgendwann schneller bewegt als wir, befindet sich von da an bereits in der Zukunft, in unserer Zukunft. Wir hätten es nicht mehr sehen können. Mit der Zeitdehnung würden wir bereits eine neue Dimension betreten und andere Zeitdimensionen mit verschiedener Zeitdehnung nicht mehr wahrnehmen können. Schon beim Aufstehen am Morgen, würde die Erde unter unseren Füssen verschwinden und alles sonst, was nicht unsere Geschwindigkeit hat. Aufgrund dessen kann man sich leicht vorstellen, wie das Leben in einer Einsteinschen Welt aussehen würde.

    Dass wir aber Objekte, mit einer unterschiedlichen Geschwindigkeit zur unseren, noch sehen und erleben können, ist der unerschütterliche Beweis dafür, dass es Zeitdehnung nicht gibt und dass die RT von Grund auf falsch ist.

  872. #875 Herr Senf
    16. Januar 2016

    Es gibt keine anderen Zeitdimensionen und keine absolute Zeit.
    Lediglich die Uhren gehen unterschiedlich und lassen sich bei Bewegung zueinander nicht dauerhaft synchronisieren, deswegen wird GPS immer wieder ein bißchen nachgeregelt.
    Wie wär’s mal, einfach ein Physikbuch kaufen und auch lesen!

  873. #876 zweisteine
    16. Januar 2016

    Herr Senf
    16. Januar 2016
    /// Wie wär’s mal, einfach ein Physikbuch kaufen und auch lesen!///

    Wie wäre es, einen zu kaufen, zu lesen und zu verstehen dazu?

    ///Es gibt keine anderen Zeitdimensionen und keine absolute Zeit.///

    Dafür muss auch keine absolute Zeit geben. Wenn sich aber die Zeit dehnt das ist auch schon eine andere Dimension. Wenn du weisst was Dimension bedeutet.

    /// Lediglich die Uhren gehen unterschiedlich ///

    Das hat aber mit Zeitdehnung nichts zu tun, aber mit Bewegung und ihre Wirkung auf die Schwingung der Atome.
    Weil wir sie Uhren nennen dann muss die Zeit die Ursache sein wenn sie anderes ticken? Bitte!

  874. #877 MartinB
    16. Januar 2016

    @Zweisteine
    Du hast uns jetzt über Hundert Kommenatre lang deutlich gemacht, dass du die RT nichtverstehst und keinen Schimmer hast, was Zeitdilatation oder ein unterschiedlicher zeitverlauf in unterschiedlichen Bezugssystemen bedeutet. Es reicht – bitte hör auf, hier weiter immer denselben Blödsinn zu posten.

  875. #878 zweisteine
    16. Januar 2016

    //// MartinB
    16. Januar 2016
    @Zweisteine
    Du hast uns jetzt über Hundert Kommenatre lang deutlich gemacht, dass du die RT nichtverstehst und keinen Schimmer hast, was Zeitdilatation oder ein unterschiedlicher zeitverlauf in unterschiedlichen Bezugssystemen bedeutet. Es reicht – bitte hör auf, hier weiter immer denselben Blödsinn zu posten.////

    Es ist aber schon gepostet.
    Und es ist nicht der gleicher Blödsinn, es it ein anderer.
    Du könntest mal als Ausnahme erklären, warum es so ein Blödsinn sein sollte.
    Zu Bewegung und Zeitdehnung, als zwei sich gegenseitig ausschliessende Phänomene hast du gar nichts gesagt.

    Vielleicht könntest du jetzt was über die Verschiebung der Gegenwart bei der Zeitdehnung was sagen und mich sogar widerlegen. Was mich sehr freuen würde. Ich sehe selber nicht den Widerspruch, wenn die Zeit eine Dimension sein sollte.
    Mag sein dass du diese Sachen studiert hast aber die Logik kannst du dir nicht kaufen und anderen verbieten sich Gedanken darüber zu machen.

    Was ich sehr schätze bei dir ist, dass du nicht zur Sperrung greifst wenn jemand dir unangenehm ist.

  876. #879 MartinB
    16. Januar 2016

    @zweisteine
    “Was ich sehr schätze bei dir ist, dass du nicht zur Sperrung greifst wenn jemand dir unangenehm ist.”
    Damit ist es jetzt allerdings vorbei, da du ja nicht gewillt bist, meinen mehrfachen Aufforderungen Folge zu leisten.
    Leb wohl, zweisteine…

  877. #880 Gerd
    29. Januar 2016

    Die Relativitätstheorie wäre richtig, wenn das Licht wirklich und wahrhaftig eine unendliche Geschwindigkeit hätte.
    Dann läge auch eine Systeminvarianz bezüglich der Lichtgeschwindigkeit vor. Jerde würde egal wo er sich befindet oder wie er sich bewegt das Licht genauso wahrnehmen.
    Das Problem ist nur, de Lichtgeschwindigkeit ist immens hoch, jedoch nicht unendlich.
    Bei allen Messungen der Lichtgeschwindigkeit wird immer nur die Geschwindigkeit bezüglich der Messapparatur ermittelt. Dann kommt natürlich immer die gleiche Geschwindigkeit heraus.
    Jedoch zeigt die Rot- und Blauverschiebung bei bewegten Objekten, dass sich das Licht immer genau zu der Lichquelle mit Lichtgeschwindigkeit bewegt.
    Jede Lichtquelle sendet das Licht mit Lichtgeschwindigkeit aus. Sobald das Licht den Bereich der Lichtquelle verlassen hat, nimmt es die Lichtgeschwindigkeit zu der umgebenden Materie an. Die Wellenlänge wird dabei jedoch nicht geändert. Und nun zeigt uns genau die Änderung der Wellenlänge die ursprüngliche Lichtgeschwindigkeit bei der Ausstrahlung.
    Wenn man so will ist die Materie und deren Felder der Äther, wonach gesucht wurde. Wieso man genau diesen Tatbestand nicht aufgenommen hat entzieht sich meiner Kenntnis und ist mir total unverständlich.

  878. #881 MartinB
    30. Januar 2016

    @Gerd
    Die Invarianz der Lichtgeschwindigkeit ist eine experimentelle Tatsache, die kannst dunicht wegleugnen.
    Der rest deines Kommentars funktioniert nicht, da man die Lichtgeschwindigkeit mit Lichtlaufzeiten misst – siehe zum Beispiel Einsteins Experiment mit dem Zug, an dessen Enden zwei Blitze einschlagen.
    “Wieso man genau diesen Tatbestand nicht aufgenommen hat”
    Weil es schlicht falsch ist.

  879. #882 Krypto
    30. Januar 2016

    @Gerd:
    Ich mag Dich nur ungern mit der Nase drauf stoßen, aaaaber: 😉
    Du stellst eine Behauptung auf, die schlicht falsch ist und wunderst Dich, dass niemand zuvor auf Deine Erkenntnis gestoßen ist.

    Die Relativitätstheorie wäre richtig, wenn das Licht wirklich und wahrhaftig eine unendliche Geschwindigkeit hätte.

    Diese Aussage offenbart, dass Du Dich noch nicht einmal mit der Namensgebung der Theorie auseinander gesetzt haben kannst.
    Wenn Du Interesse an der SRT und/oder ART hast, lies Dich doch mal ein und versuche, zu verstehen.

  880. #883 Krypto
    30. Januar 2016

    Oops, vergessen:
    Nehmen wir mal die einfache, zum Allgemeinwissen gehörende Gleichung E=mc^2:
    Du sagst c wäre unendlich und damit wäre die Gleichung korrekt.
    Dein großer Wurf würde bedeuten, dass schon der kleinstvorstellbare Krümel Masse unendlich viel Ernergie hätte.
    Finde den Fehler 😉

  881. #884 Krypto
    30. Januar 2016

    Ernergie

    Fehler selbst gefunden 😀

  882. #885 Hartmut Schüppler
    5. Februar 2016

    Hallo MartinB,
    hast Du schon mal darüber nachgedacht, dass sich die Invarianz der Lichtgeschwindigkeit auch nichtexperimentell aus der Natur der elektromagnetischen Wechselwirkung und der Anwendung der Kategorien von Raum und Zeit auf dieses Phänomen begründen lässt ?
    Die üblichen Einheiten (Meter,Sekunde) sind willkürlich festgesetzt. Daraus folgt, dass die Lichtgeschwindigkeit nicht exakt messbar ist und ein gewisses Missverhältnis von Raum und Zeit (ca. 3000000 :1) entsteht. Wenn man davon ausgeht, dass die elektrische und die magnetische Wirkung gleichwertig sind, ergibt das für die Beziehung zwischen Raum und Zeit bei Anwendung auf elektromagnetische Wellen den Wert 1. Dieser Wert wird ja in den Darstellungen auch häufig verwendet, aber meist nur als vereinfachte Darstellung interpretiert. Ich bin hingegen der Meinung, dass aus der Gleichwertigkeit von Raum und Zeit bei Anwendung auf die elektromagnetische Wechselwirkung die Konstanz der “Lichtgeschwindigkeit” folgt, weil der Elektromagnetismus ein tatsächliches Geschehen ist.

    MfG
    H.

  883. #886 Bullet
    5. Februar 2016

    @Gerd (vielleicht liest er es ja noch, obwohl schon eine Woche vergangen ist):

    Bei allen Messungen der Lichtgeschwindigkeit wird immer nur die Geschwindigkeit bezüglich der Messapparatur ermittelt.

    Dat is so nich janz korrekt. Da gibt es beispielsweise die Möglichkeit, bei Supernovaüberresten die Lichtechos auszuwerten. Da geht das Licht von der Supernova erstma’ rechtwinklig zum Beobachter (also “zur Seite”) weg und wird dann an einer Wolke zum Beobachter hin reflektiert. Wenn man dann weiß, wie weit die Supernova weg ist und den Sichtwinkel zwischen Supernova und den davon entfernten Wolken mißt, dann kann man auch messen, um wieviel länger das Licht mit Umweg braucht, um zu uns zu gelangen. Das Ergebnis ist eine Geschwindigkeitsmessung des Lichtes zwischen zwei entfernten Meßpunkten.

    Martin? Irgendein Denkfehler? Ich war mir sicher, daß das auch schon so gemacht wurde, aber …. sicher bin ich nicht.

  884. #887 Herr Senf
    5. Februar 2016

    Ich denke nicht, daß man so die LG aus der “Ferne” bestimmen kann.
    Unter der Voraussetzung LG=const berechnet man die Geometrie des Lichtechos
    und erhält so auch die Entfernung recht genau.

  885. #888 MartinB
    6. Februar 2016

    @Hartmut
    Bin mirnicht sicher, ob ich dich richtig verstehe – aber letztlich ist Einstein ja unter anderem dadurch auf die SRT gekommen, dass er sah, dass die maxwellgleichungen nicht Galilei-invariant sind (was würde passieren, wenn man auf einer Lichtwelle reitet?).

    Ich glaube, es ist keine echte physikalische Frage, darüber nachzudenken, ob nun die Maxwell-gleichungen grundlegender sind als die Invarianz von c oder andersherum. Die Gesetze beschreiben die Realität und sind miteinander konsistent. Aus praktischen Gründen formulieren wir sie so, dass es Redundanzen gibt. Und solange es keine grundlegendere Theorie gibt, aus der wir alles ableiten, spielt es keine Rolle, welche Fakten wir genau für die Fundamentalsten erklären.
    Wobei natürlich in der QFT die Raumzeit und c als Konzept deutlich fundamentaler eingehen als die Tatsache, dass die em-Wechselwirkung über Photonen vermittelt wird. Von der Warte aus ist c fundamentaler – alle masselosen Teilchen müssen sich nach QFT mit c bewegen.

  886. #889 MartinB
    6. Februar 2016

    @Bullet
    Damit das geht, müsste man aber die Größe der Wolke oder was immer da das Licht reflektiert, schon kennen, oder?

  887. #890 AB
    14. Februar 2016

    Die SRT ist falsch.

    Warum ist sie falsch?
    Weil das Herz der Speziellen Relativitätstheorie die Lorentztransformation ist.

    Und die Lorentztransformation ist definitiv falsch.

    Warum ist sie falsch?

    Weil die Lorentztransformation – ungewollt! – aus einem Lichtstrahl zwei Lichtstrahlen macht!

    Und zwar kommt hier zu dem Lichtstrahl, der aus dem Ursprung des ruhenden Systems eingeschaltet wird, noch – auf wundersame Weise – ein zweiter Lichtstrahl hinzu, der dann obendrein auch noch mit dem bewegten System mitreist, d.h., dessen Lichtquelle im Ursprung des bewegten Systems sitzt.

    Nur für diesen Fall würde gelten: x’=c mal t’.

    Dieser zweite Lichtstrahl wäre dann jedoch komplett unabhängig vom Ersten des ruhenden Systems.

    Dieses Szenario würde überhaupt nichts bringen!

    Die Lorentztransformation soll ja gerade die Korrelation zwischen den beiden Systemen zeigen bzw. berechnen.

    Deshalb gibt es im Szenario der Lorentztransformation nur einen einzigen Lichtstrahl oder besser: soll es nur einen einzigen Lichtstrahl geben und das ist derjenige Lichtstrahl, der im Ursprung des ruhenden Inertialsystems sitzt und von hier aus losgesendet wird.

    Statt x’=c mal t’ gilt daher für die Raum- und Zeitkoordinate des bewegten Systems:

    x’=(c-v) mal t’.

    (x’ ist die Raumkoordinate des bewegten Systems und t’ ist die Zeitkoordinate des bewegten Systems).

  888. #891 Spritkopf
    14. Februar 2016

    Statt x’=c mal t’ gilt daher für die Raum- und Zeitkoordinate des bewegten Systems:

    x’=(c-v) mal t’.

    Ja, Büttgen, wir haben mittlerweile kapiert, dass Sie sich für den zweiten Newton halten und als solcher absoluten Raum und absolute Zeit proklamieren.

    Nein, Büttgen, dass Sie glauben, der zweite Newton zu sein, heißt noch lange nicht, dass Sie das sind.

  889. #892 Krypto
    14. Februar 2016

    @Spritkopf:
    ArthurB hat sich auf telepolis ausgeheult, dass er in den scienceblogs so gebashed wurde 😉

  890. #893 Spritkopf
    14. Februar 2016

    @Krypto
    Lass mich raten. Er hat dort genügend Leute gefunden, die ihm beigesprungen sind und ihm seine Leidensgeschichte des von den bösen Dogmatikern im Wissenschaftsmainstream unterdrückten Genies abgekauft haben?

  891. #894 MartinB
    14. Februar 2016

    @AB
    Ich habe keine Geduld mehr mit Leuten, die wirren Blödsin über ihr Unverständnis der Relativitätstheorie posten. Bitte damit aufhören.

  892. #895 Hartmut Schüppler
    14. Februar 2016

    Hallo AB,
    kann es sein, dass Du zwei Lichtstrahlen, die man durchaus, z.B. aus zwei Taschenlampen, haben kann, mit der Lichtgeschwindigkeit (c) verwechselst ? Die Lichtgeschwindigkeit stellt einen einzigen, konstanten Wert dar, der sich aus der Anwendung der Kategorien von Raum und Zeit auf ein tatsächliches Geschehen, die elektromagnetische Wechselwirkung, ergibt. In die Lorentz-Transformation findet c auch als Konstante Eingang, während allein die Differenzgeschwindigkeit (v) zwischen zwei Systemen variabel ist.
    MfG
    H.

  893. #896 AB
    14. Februar 2016

    @ an alle, die mich kritisieren:
    Von Dopplereffekt haben Sie noch nie etwas gehört, nicht wahr?
    Das Szenario der Lorentztransformation ist nichts Anderes als die Herleitung bzw. Berechnung des Dopplereffektes.
    Und zwar desjenigen Dopplereffektes der ruhenden Signalquelle und des Beobachters, der sich von der Signalquelle wegbewegt.
    Von mir der folgende Tipp:
    Ersetzen Sie bei diesem Dopplereffekt die “Lambda” (Wellenlängen) durch “x” (beliebige Weglängen) und die Periodendauern “T” durch “t” (beliebige Zeitdauern).
    Auf diese Weise kommt man auf x’=(c-v) mal t`.

    Die Spezielle Relativitätstheorie kann den Doppler-Effekt (Frequenzverschiebung) nicht abbilden, sollte sie aber können, denn das Szenario der zwei Inertialsysteme mit einem – einzigen!- Lichtstrahl entspricht, wie gesagt, genau dem Szenario des Dopplereffektes: Lichtquelle in Ruhe und Beobachter entfernt sich davon.
    Denken Sie mal darüber nach bzw. rechnen Sie das mal nach.

  894. #897 Adent
    14. Februar 2016

    @AB
    Nur das der klassische Doppler-Effekt nichts mit der Rotverschiebung zu tun hat, wie Ihnen sicher schon 100mal erklärt wurde, aber halten Sie gern weiter die Finger in die Ohren (obwohl hier müsste man besser schreiben die Hände über die Augen) und singen lalalala, nur zu.

  895. #898 AB
    14. Februar 2016

    @Adent

    Nur das der klassische Doppler-Effekt nichts mit der Rotverschiebung zu tun hat, wie Ihnen sicher schon 100mal erklärt wurde
    Wie kommen Sie darauf?
    Womit hat die Rotverschiebung denn sonst zu tun, wenn nicht mit dem “klassischen” Doppler-Effekt?

  896. #899 AB
    14. Februar 2016

    @Hartmut Schüppler
    Hallo Herr Schüppler,
    das “Du” nehme ich gerne an.
    Wenn ich Sie recht verstehe, wollen Sie sagen, dass die Weg-Zeit-Gleichung “`x’=(c-v) mal t’ dem zweiten Einsteinschen Postulat widerspricht:
    (Wikipedia)

    “Das zweite Prinzip ist die Konstanz der Lichtgeschwindigkeit. Es besagt, dass in allen Inertialsystemen die Vakuumlichtgeschwindigkeit gleich groß ist.”

    In der Tat widerspricht dieses Weg-Zeit-Gesetz nicht dem Einsteinschen Postulat. Mit der Betonung auf “in”, “….in allen Inertialsystemen ist die Vakuumlichtgeschwindigkeit gleich groß”.
    Bei der Lorentztransformation – bzw. dem Doppler-Effekt befindet sich der Lichtstrahl ja sozusagen “zwischen” den beiden Inertialsystemen, also – wenn man so will – außerhalb der beiden Inertialsysteme, er verbindet sozusagen beide Inertialsysteme miteinander. Die Frequenzverschiebung (Rotverschiebung im Falle der Bewegung voneinander weg und Blauverschiebung im Falle der Bewegung zueinander hin) ist ja auch nur für den Beobachter eine scheinbares Phänomen. Zu keinem Zeitpunkt ist dabei die Frequenz (oder die Wellenlänge) des Lichtstrahls tatsächlich verändert. Sie erscheint dem Beobachter nur verändert! (Betonung auf “erscheint”)
    Man denke dabei an den akustischen Dopplereffekt – dieser ist ja auch eine Alltagserscheinung: Der Krankenwagen, der auf einen Beobachter zufährt: Der Ton ist höher.
    Fährt der Krankenwagen unmittelbar an jemandem vorbei, klingt der Ton in einer mittleren Tonhöhe.
    Entfernt sich der Krankenwagen vom Beobachter, erscheint der Ton tiefer.
    Die Frage ist doch nun:
    Welche Tonhöhe von den drei Tönen hat der Ton denn nun wirklich?
    Die Antwort lautet: Der Ton, den das Martinshorn wirklich hat, ist der mittlere, den der Beobachter bei der unmittelbaren Vorbeifahrt hört.
    Das ist auch der Ton, den der Fahrer des Krankenwagens permanent hört. Der Ton erscheint dem (hier: ruhenden) Beobachter lediglich höher bzw. tiefer.
    Genauso verhält es sich auch mit dem Licht, welches dem auf der Erde ruhenden Beobachter rötlicher oder bläulicher – als tatsächlich – erscheint.
    Im Falle der Rotverschiebung (Stern bewegt sich von Erde weg) erreicht den (ruhenden) Beobachter das Licht mit verminderter Geschwindigkeit (c-v), im Falle der Blauverschiebung (Stern bewegt sich auf die Erde zu) erscheint dem ruhenden Beobachter die Geschwindigkeit des Lichts tatsächlich als Überlichtgeschwindigkeit (c+v). Auch hier liegt die Betonung auf “erscheint”.

  897. #900 MartinB
    14. Februar 2016

    @AB
    Welcher Teil von “Bitte damit aufhören” war unverständlich?

  898. #901 AB
    14. Februar 2016

    @MartinB
    Welchen Teil von “Doppler-Effekt” verstehen Sie nicht?

    Na denn, Tschüss. Sie sind ab jetzt hier gesperrt.

  899. #902 Hartmut Schüppler
    14. Februar 2016

    Hallo AB,
    Die Lichtgeschwindigkeit ist von der Farbe des Lichts unabhängig, weil das Produkt Wellenlänge X Frequenz konstant ist.
    @MartinB Sorry, ich werde auf Äußerungen von AB nicht mehr reagieren.
    MfG
    H.

  900. #903 AB
    16. Februar 2016

    Ja, ich habe Sie wirklich hier gesperrt

  901. #904 Wolf
    10. Juli 2017

    zu #899: “Das zweite Prinzip ist die Konstanz der Lichtgeschwindigkeit. Es besagt, dass in allen Inertialsystemen die Vakuumlichtgeschwindigkeit gleich groß ist.”

    Wolf: Wenn ein Körper (z. B. eine Raumfähre) und ein Lichtstrahl in die gleiche Richtung reisen, wird man bei Experimenten aufgrund der Zeitdilatation immer den Lichtgeschwindigkeitswert messen.

    Sind aber die Raumfähre und ein Lichtstrahl in entgegengesetzte Richtung unterwegs, wird man nur innerhalb der Raumfähre für das Licht den Lichtgeschwindigkeitswert erhalten. Außerhalb der Raumfähre misst man dagegen einen Wert höher als c, womit das Prinzip der Konstanz der Lichtgeschwindigkeit widerlegt ist.

  902. #905 MartinB
    10. Juli 2017

    @Wolf
    Das ist schlicht falsch – und eine bloße Behauptung ohne Evidenz ist auch keine Widerlegung von irgendwas.

  903. #906 Wolf
    12. Juli 2017

    Man muss das Experiment nur nachmachen, dann sieht man, dass die angebliche Behauptung stimmt. Neben den unzähligen Millionen und Milliarden Euro, die oft für die Forschung vergeudet werden, wird man wohl für ein paar tausend Euro dieses simple Experiment an der Außenseite eines Satelliten, den man ohnehin in den Weltraum geschossen hätte, durchführen können. Wie wäre es, wenn sich ein Martin Bäker für diesen Versuch einsetzen würde? Dann hätte er seine Evidenz.

    Sie können auch genauere Informationen erhalten (die sogar einer Ihrer Kollegen ‘übernommen’ hat).

  904. #907 Spritkopf
    12. Juli 2017

    @Wolf

    wird man wohl für ein paar tausend Euro dieses simple Experiment an der Außenseite eines Satelliten, den man ohnehin in den Weltraum geschossen hätte, durchführen können.

    Man könnte doch dieses Experiment auch auf der Erde durchführen. Diese bewegt sich schließlich mit 31 km/s um die Sonne. Also einfach einen Lichtstrahl entgegen ihrer augenblicklichen Flugrichtung aussenden und dessen Geschwindigkeit messen, oder?

  905. #908 MartinB
    12. Juli 2017

    @Wolf
    Die Regeln der Geschwindigkeitsaddition sind experimentell hinreichend gut bestätigt – sowohl in Laborexperimenten als auch z.B. durch Teilchen der kosmischen Strahlung, die uns sonst mit allen möglichen Geschwindigkeiten erreichen müssten.

    Dieser Blog ist kein Forum für verwirrte und ewig-gestrige Wissenschaftsleugner, weitere Diskussionen zu deinen Behauptungen sind deshalb unerwünscht.

  906. #909 Wolf
    13. Juli 2017

    @Spritkopf
    Für die Durchführung dieses Experiments AUF DER ERDE (oder in der Erdatmosphäre) gilt dasselbe wie IN einem Raumschiff: Man wird immer wieder (entsprechend der Relativitätstheorie) den Lichtgeschwindigkeitswert c messen. Das heißt: Die Erde stellt nichts anderes als das Innere eines Raumschiffs dar.

    Zusammenfassung: Sind ein Flugkörper (z. B. ein Raumschiff, ein Satellit, eine Rakete oder die Erde) und ein Lichtstrahl in die gleiche Richtung unterwegs, wird für den Lichtstrahl immer der Lichtgeschwindigkeitswert c gemessen – egal, ob innerhalb oder außerhalb des Flugkörpers.

    Sind ein Flugkörper und ein Lichtstrahl in entgegengesetzte Richtungen unterwegs, wird innerhalb des Flugkörpers für den Lichtstrahl immer der Lichtgeschwindigkeitswert c gemessen. Aber außerhalb des Flugkörpers (sprich im Weltall) wird für einen Lichtstrahl ein höherer Wert als c ermittelt.

    @ MartinB: „Jede Theorie, die die SRT ersetzen soll, wird diese als Grenzfall (beispielsweise für hinreichend große Längen) enthalten müssen, so wie die Newtonsche Mechanik als Grenzfall in der SRT (und in anderer Weise auch in der Quantenmechanik) enthalten ist. Daran führt kein Weg vorbei – wer etwas anderes behauptet, hat eine Menge zu erklären.“

    Genau das kann ich erklären.

    Die Regeln der Geschwindigkeitsaddition scheinen hinreichend ‚gut‘ bestätigt – mit einer bestimmten Ausnahme, die nie bedacht wurde. Deshalb dieses angedachte Experiment – und zwar durch die NASA, ESA, … – um jeder Manipulation vorzubeugen.

    Aber wenn man als „verwirrter und ewig-gestriger Wissenschaftsleugner“ hingestellt wird, mit dem jede „weitere Diskussion unerwünscht“ ist, dann wird man ohne mich auskommen müssen.

  907. #910 Alderamin
    14. Juli 2017

    @Wolf

    Aber außerhalb des Flugkörpers (sprich im Weltall) wird für einen Lichtstrahl ein höherer Wert als c ermittelt.

    Relativ zum Raumschiff? Ja sicher. Wenn ich zwei Lichtstrahlen in entgegengesetzte Richtungen aussende, sähe ich, wenn ich das könnte, die Lichtstrahlen mit 2c voneinander wegfliegen.

    Das ist aber nicht die Aussage der Konstanz der Lichtgeschwindigkeit. Da geht es darum, wie ich als Beobachter einen Lichtstrahl relativ zu mir selbst messe, der wahlweise von relativ zu mir ruhender oder bewegter Quelle ausgeht. Genau dieser Wert ist immer c.

  908. #911 Wolf
    18. Juli 2017

    @Alderamin
    “Es geht darum, wie ich als Beobachter einen Lichtstrahl relativ zu mir selbst messe, der wahlweise von relativ zu mir ruhender oder bewegter Quelle ausgeht.”

    Genau. Dabei wird innerhalb eines Raumschiffs (oder auf der Erde bzw. in der Erdatmosphäre) immer der Wert c gemessen. Bei einer Messung außerhalb des Raumschiffs wird man nur dann den Wert c erhalten, wenn Raumschiff und Licht in die gleiche Richtung unterwegs sind. Sind das Raumschiff und das frei im All reisende Licht in entgegengesetzte Richtung unterwegs, wird man einen höheren Wert als c messen, was am Verhalten des Lichts liegt, welches nie bedacht wurde. Eine Messung außerhalb der Erde bzw. außerhalb eines Raumschiffs ist von Experten noch nie durchgeführt worden, weil man es nie für nötig erachtete.

    Man muss zwischen Eigen-, Einzel-, Gemeinschafts-, Brutto- und Netto-Geschwindigkeiten unterscheiden. Oft wird kein Unterschied bei diesen Geschwindigkeiten gemacht, was zu eklatanten Fehlern führt, die jedoch bei kleinen Geschwindigkeiten wie 200 km/h oder 1.600 km/h – in Bezug auf die Zeitdilatation – kaum ins Gewicht fallen.

  909. #912 MartinB
    18. Juli 2017

    @Wolf
    Welchen Teil des Wortes “unerwünscht” hast du nicht verstanden?
    Alderamin hat alles erklärt, was es dazu zu sagen gibt.

  910. #913 Kurt
    26. Juli 2017

    Stimmt das wirklich?

  911. #914 mecki
    26. Juli 2017

    ich hab da mal ne frage. ist die srt richtig oder doch falsch?

  912. #915 Kurt
    26. Juli 2017

    Das frage ich mich auch gerade.

  913. #916 Jogi
    26. Juli 2017

    Natürlich sind die Zeitdehnung und die Messung der Lichtgeschwindigkeit c hundertfach nachgewiesen, aber die Längenverkürzung sowie die Raumkrümmung (direkt noch) nie. Laut SRT sieht ein Erdbewohner einen schnell reisenden Raumfahrer in Zeitlupe, während umgekehrt derselbe Raumfahrer die Erdbewohner ebenfalls in Zeitlupe sieht. Anhand eines durchgeführten Versuchs hat man jedoch das Gegenteil festgestellt. Von der Erde aus wurde die Zeitlupe nachgewiesen, während umgekehrt vom All aus Zeitraffer nachgewiesen wurde. Jetzt kann sich jeder zusammenreimen, was das bedeutet. Oder bei Harald Lesch nachfragen oder bei dessen Doku reinschauen.

  914. #917 Alderamin
    27. Juli 2017

    @Jogi

    Natürlich sind die Zeitdehnung und die Messung der Lichtgeschwindigkeit c hundertfach nachgewiesen, aber die Längenverkürzung sowie die Raumkrümmung (direkt noch) nie.

    Man hat die Längenverkürzung noch nicht direkt messen können, da sie bei geringen Geschwindigkeiten, die große Objekte wie Flugzeuge oder Satelliten erreichen können, so klein ist, dass sie nicht hinreichend genau im Vorbeiflug gemessen werden kann, und solche Teilchen, die man in Beschleunigern auf entsprechend hohe Geschwindigkeiten bringen kann, sowieso schwer zu vermessen sind.

    Die Längenverkürzung folgt aber zwingend aus der Zeitdehnung, wenn man die Beobachterperspektive wechselt. Beispiel: Myonen entstehen in der oberen Atmosphäre, wenn kosmische Teilchen die Luftmoleküle mit hoher Geschwindigkeit treffen, und rasen selbst mit hoher Geschwindigkeit durch die Luft. Eigentlich sollten sie nur ein paar hundert Meter weit kommen, bevor sie aufgrund ihrer kurzen Halbwertszeit zerfallen. Man kann sie aber noch am Erdboden, nach über 100 km nachweisen. Für den Erdbeobachter vergeht ihre Zeit langsamer, damit wird auch die Halbwertszeit größer. Aus der Sicht der Myonen muss hingegen der Weg zur Erde verkürzt sein, damit sie in der in ihrer Eigenzeit gemessenen kurzen Halbwertszeit den Weg bis zum Erdboden schaffen können.

    Raumkrümmung ist selbstverständlich schon gemessen worden, es war sogar das erste Experiment, mit dem die allgemeine Relativitätstheorie nachgewiesen wurde. Amateure sind aufgerufen, dieses Experiment bei der kommenden totalen Sonnenfinsternis am 22. August in den USA zu wiederholen.

    Laut SRT sieht ein Erdbewohner einen schnell reisenden Raumfahrer in Zeitlupe, während umgekehrt derselbe Raumfahrer die Erdbewohner ebenfalls in Zeitlupe sieht. Anhand eines durchgeführten Versuchs hat man jedoch das Gegenteil festgestellt. Von der Erde aus wurde die Zeitlupe nachgewiesen, während umgekehrt vom All aus Zeitraffer nachgewiesen wurde.

    Hast Du eine Quelle für den Versuch? Man muss genau betrachten, was da wie gemessen wurde. Zweierlei ist zu beachten:

    1) Die Symmetrie gilt nur in der speziellen Relativitätstheorie für gleichförmige Bewegung ohne Beschleunigungen und Richtungsänderungen (Inertialsysteme). Sie gilt nicht in der allgemeinen Relativitätstheorie, etwa im Schwerefeld der Erde. Aus der Sicht eines GPS-Satelliten vergeht die Zeit auf der Erde langsamer, weil er (neben seiner Bewegung im Orbit) weiter draußen im Schwerefeld der Erde ist. Aus unserer Sicht am Boden vergeht die Zeit auf dem Satelliten schneller, obwohl er sich im Orbit bewegt, denn der Effekt der Raumkrümmung aus der allgemeinen Relativitätstheorie überwiegt den der speziellen um etwa das Doppelte (dieses Verhältnis gilt für die Geschwindigkeit und Entfernung in Bezug zur Erde auf diesen Orbits, etwa 20000 km über der Erde). D.h. die Zeitverlangsamung im Schwerefeld messen beide Beobachter für denjenigen, der tiefer im Schwerefeld sitzt.

    2) das Zwillingsparadoxon, das genau dann eintritt, wenn einer der Beobachter seine Richtung ändert. Z.B. kann ein Raumfahrer sich sehr weit von der Erde wegbewegen, dann vergeht seine Zeit von der Erde aus gesehen langsamer und er sieht die Zeit auf der Erde seinerzeit langsamer vergehen. Zum Uhrenvergleich muss er jedoch umkehren und zur Erde zurück fliegen.

    Angenommen, die Geschwindigkeit relativ zur Erde sei bei Hin- und Rückflug die gleiche. Beschleunigungen lassen wir außer Acht. Aus Sicht der Erde vergeht seine Zeit beim Hin- und Rückflug um den gleichen Faktor verlangsamt.

    Aus der Sicht eines Systems in welchem der Raumfahrer auf dem Hinweg ruht, entfernt sich die Erde auf dem Hinflug mit der gleichen Geschwindigkeit, mit der er sich von der Erde aus gesehen entfernt. Am Umkehrpunkt verlässt er jedoch das Inertialsystem, in dem er ruhend erscheint und fliegt mit erhöhter Geschwindigkeit der sich noch immer entfernenden Erde hinterher, um diese einzuholen. D.h. aus dem System betrachtet, in dem er auf dem Hinweg ruhte, ist seine Zeitdilatation auf dem Rückweg viel größer als die der langsameren Erde, so dass er weniger gealtert als diese die Erde erreicht. Entsprechendes gilt, wenn man das System betrachtet, in dem der Raumfahrer auf dem Rückweg ruht – dann altert er langsamer als die Erde auf dem Hinweg. In allen Inertialsystemen betrachtet ist der Astronaut also beim Wiedereintreffen auf der Erde weniger gealtert als die Bewohner der Erde. So löst sich das Zwillingsparadoxon in Luft auf.

    Das Zwillingsparadoxon trat auf und wurde nachgewiesen, als eine Atomuhr in einem Flugzeug unterwegs war und am Ende wieder zurück zum Ausgangspunkt gebracht wurde, wo eine geringere Zeitdauer im Vergleich zu einer ruhenden Atomuhr nachgewiesen wurde.

  915. #918 Harti
    27. Juli 2017

    Warum darf man nicht annehmen, dass die biologischen Alterungsprozesse beider Zwillinge, raumzeitlich betrachtet, gleiche Distanzen darstellen; anders ausgedrückt: als tatsächliches Geschehen invariante Raumzeitintervalle sind ?

  916. #919 Alderamin
    27. Juli 2017

    @Harti

    Weil es kein absolutes Raumzeit-Koordinatensystem gibt, in dem sich alles vergleichen lässt?

  917. #920 Harti
    27. Juli 2017

    Kann nicht der Elektromagnetismus (die Lichtgeschwindigkeit) ein solches raumzeitliches Bezugssystem definieren ? Immerhin trennt der Elektromagnetismus den wahrnehmbaren Bereich unserer Welt von dem nicht mehr wahrnehmbaren Bereich. Lichtartige Vektoren haben in der Raumzeit den Betrag 0. Dies legt eine Gleichwertigkeit von Raum und Zeit bei Anwendung dieser Kategorien auf die elektromagnetische Wechselwirkung fest. Damit bestimmt der Elektromagnetismus zumindest in der SRT das Wesen der Raumzeit ???

  918. #921 Alderamin
    28. Juli 2017

    @Harti

    Kann nicht der Elektromagnetismus (die Lichtgeschwindigkeit) ein solches raumzeitliches Bezugssystem definieren ?

    Wie denn, wenn die Lichtgeschwindigkeit in allen Inertialsystemen gleich groß erscheint?

    Immerhin trennt der Elektromagnetismus den wahrnehmbaren Bereich unserer Welt von dem nicht mehr wahrnehmbaren Bereich.

    Ich nehme an, Du meinst den Lichtkegel. Verschieden bewegte Beobachter haben verschiedene Lichtkegel. Und verschiedene Definitionen von Gleichzeitigkeit / Vergangenheit / Zukunft. Da gibt’s wenig Gemeinsamkeiten zwischen den Beobachtern.

    Damit bestimmt der Elektromagnetismus zumindest in der SRT das Wesen der Raumzeit ???

    Licht breitet sich mit der maximal möglichen Geschwindigkeit aus, die masselose Teilchen in der Raumzeit haben können, aber da man mit hohen Geschwindigkeiten nicht so simpel rechnen kann, wie mit geringen (Addition von relativistischen Geschwindigkeiten) und die Lichtgeschwindigkeit eine Art Singularität bildet, die in allen Systemen gilt, taugt sie als universelles Bezugssystem nicht. Das ist ja gerade die Aussage der speziellen Relativitätstheorie.

  919. #922 MartinB
    28. Juli 2017

    @Harti
    Das ist natürlich immer Spekulation, weil wir nur ein Universum haben, aber im Prinzip kann man sichnein Universum ohne elektromagnetismus denken; in dem würde die SRT trotzdem gelten. Wären z.B. neutrinos masselos, könnte man auch an ihnen die grenzgeschw. Messen; auch in einem universum ohne em-wechselwirkung könnte man teilchen nicht auf geschwindigkeiten größer c beschleunigen.
    Raum und zeit sind grundlegender als der elektromagnetismus. (In der Frühzeit des universjms waren ja auch em und schwache kernkraft zu einer kraft vreint, da galt die srt trozdem)
    PS: bin grade verreist, deswegen nur sporadische antworten…

  920. #923 Harti
    29. Juli 2017

    @MartinB
    “Raum und Zeit sind grundlegender als der Elektromagnetismus”
    Was bedeutet “grundlegender” ?
    Raum und Zeit sind nach meiner Ansicht nichts weiter als Vorstellungen (software unseres Gehirns), das sich im Laufe der Evolution zur Beschreibung von Veränderungen in unserer Umwelt entwickelt hat.
    Tiere haben z.T. eine wesentliche bessere Fähigkeit zur Erfassung von Geschwindigkeiten, wobei Geschwindigkeit nichts anderes als eine Beziehung zwischen Raum und Zeit darstellt, traditionell räumlich eindimensional (Strecke) aus der Perspektive des Raumes definiert. Die Trennung von Raum und Zeit ist eine evolutionäre Entwicklung unserer intelektuellen Fähigkeiten aus praktischen Gründen. Der Elektromagnetismus bildet als Naturerscheinung die Grundlage unserer Wahrnehmungsmöglichkeiten. Er schränkt damit auch unsere Wahrnehmungsmöglichkeiten in Bezug auf Raum (ca. 13,7 Milliarden Lj) und Zeit (13,7 Milliarden Jahre) ein.
    Die Anwendung der Kategorien von Raum und Zeit auf den Elektromagnetismus unter Verwendung der herkömmlichen Definition von Geschwindigkeit und der allgemein verwendeten Einheiten lässt die Beschränkung unserer Wahrnemungsmöglichkeit als Höchstgeschwindigkeit mit einem bestimmten Wert erscheinen. Der Wert hängt von den verwendeten Einheiten ab.
    In diesem Sinne halte ich den Elektromagnetismus für “grundlegender” als Raum und Zeit.

  921. #924 MartinB
    30. Juli 2017

    @Harti
    “Grundlegender” bedeutet, dass es wesentlich einfacher ist, sich vorzustellen, wie unsere Welt aussehen würde, wenn es keine em-Wechselwirkung gäbe, als sich vorzustellen, wie unsere Welt aussehen würde, wenn es keine Raumzeit gäbe.
    Elektromagnetismus ohne Raum und Zeit ist nicht vorstellbar – wie soll man die Maxwellgleichungen ohne räumlche und zeitlice Änderungen fassen?

  922. #925 Harti
    30. Juli 2017

    Klingt so wie: Was ist grundlegender, die Henne oder das Ei ?
    Ich bin der Meinung, dass der Blitz auch da ist, wenn ihn keiner misst. Du weißt, was gemeint ist.

  923. #926 MartinB
    30. Juli 2017

    @Harti
    Nein, weiß ich nicht. Ich habe Argumente angeführt, warum in meinen Augen Raum und Zeit grundlegender sind als der Elektromagnetismus:
    1. Die Theorie des Elektromagnetismus ist ohne Felder, also Größen, die von Ort und Zeit abhängen, nicht denkbar.
    2. Raum ist auch ohne em-Felder denkbar – selbst wenn du argumentierst, dass Felder den raum konstituieren (wie auch immer das gehen würde), gibt es immer noch hinreichend viele andere Felder, die das tun könnten (schwache und starke Wechselwirkung, Neutrino-Felder etc.)

    Da sind Allgemeinplätze als gegenargument wenig tauglich.

  924. #927 Harti
    31. Juli 2017

    @MartinB
    “Die Theorie des Elektromagnetismus ist ohne Felder, also Größen, die von Raum und Zeit abhängen , nicht denkbar”

    Den Elektromagnetismus gibt es allerdings auch unabhängig davon, ob ihn irgendjemand mit Hilfe der Kategorien von Raum und Zeit in Form einer Theorie beschreibt.
    Die Kategorien Raum und Zeit sind Vorstellungen, die die Menschen zur Beschreibung von Naturerscheinungen und Veränderungen entwickelt haben.
    Ein Gewitterblitz existiert nunabhängig davon, wie lang er ist (Raum) und wie lange er dauert (Zeit).

  925. #928 MartinB
    31. Juli 2017

    @Harti
    Es geht nicht um die Beschreibung oder die Kategorien – es geht darum, dass Elektromagnetismus ohne räumliche und zeitliche Änderungen schlicht nicht möglich ist.

  926. #929 AB
    28. August 2018

    Dass die SRT falsch ist, ist Fakt.
    Wie ich vorher (siehe einige Beiträge vorher) dargelegt habe, ist das Herz der SRT die Lorentz-Transformation. Und dieser liegt nichts anderes als der Doppler-Effekt zugrunde.

    Nunmehr bin ich zu dem Schluss gekommen, dass ich mich vorher geirrt hatte.

    Ich dachte, der Fehler der Lorentz-Transformation läge in der Annahme der Konstanz der Lichtgeschwindigkeit, dass der Beobachter immer nur Lichtgeschwindigkeit sieht, egal, wie schnell sich die Signalquelle auch immer bewegt.
    (Siehe einige Kommentare in diesem Blog vorher).

    Jetzt habe ich gelernt, dass nicht das der Fehler ist, sondern dass der Fehler darin liegt, dass man mit “Raum x” und “Zeit t” rechnet anstatt mit “Wellenlänge lambda” und “Periodendauer T” – wie beim Doppler-Effekt.
    Das Szenario der Lorentz-Transformation ist dasselbe, wie das des Doppler-Effekts.
    Bevor wir auf den optischen Dopplereffekt kommen, sprechen wir erst einmal vom akustischen Doppler-Effekt:
    Und zwar von demjenigen des Rettungswagens, der auf den ruhenden Beobachter zufährt (also bewegte Signalquelle und ruhender Beobachter):
    Hierzu steht in jedem Mittelstufen-Physik-Buch: Die vom Beobachter wahrgenommene Signalgeschwindigkeit ist immer die Schallgeschwindigkeit, egal, wie schnell sich der Rettungswagen bewegt.
    Diese Tatsache wird in jedem Mittelstufen-Physik-Buch errechnet.
    Also: Egal, wie schnell der Rettungswagen fährt, der ruhende Beobachter nimmt immer die Schallgeschwindigkeit wahr.

    Das jedoch ist genau das Prinzip der Konstanz der Signalgeschwindigkeit: Egal, wie schnell sich das Signal bewegt, die vom Beobachter wahrgenommene Signalgeschwindigkeit ist immer dieselbe.
    Für den Schall funktioniert das und zwar ohne Zeitdilatation!
    Warum sollte das Prinzip der Konstanz der Signalgeschwindigkeit dann nicht auch für Licht ebenfalls ohne Zeitdilatation funktionieren? Das Phänomen “Zeitdilatation” ist somit hinfällig, eine “Zeitdilatation” gibt es somit nicht.
    Und wenn es keine “Zeitdilatation” gibt, gibt es auch kein “Zwillings-Paradoxon”.
    Damit ist dann auch die SRT für immer und ewig widerlegt:
    Es bedarf keiner Infragestellung der Zeit oder/und des Raumes, um die Konstanz der Signalgeschwindigkeit sicherzustellen: Mit der Formel c = lambda mal f reichen die Änderung der Wellenlänge und der Frequenz vollkommen aus, um eine konstante Signalgeschwindigkeit (Schall oder Licht) sicherzustellen.
    Damit ist die von Einstein geforderte Konstanz der (hier: Lichtgeschwindigkeit) erfüllt. Und zwar gänzlich, ohne Zeitdilatation und Lorentz-Kontraktion bemühen zu müssen.

  927. #930 AB
    28. August 2018

    Nachtrag zu vorigem Beitrag:
    Die Zeitdilatation, die fester Bestandteil der Speziellen Relativitätstheorie ist – ebenso die “Lorentz-Kontraktion” – ist eine Beleidigung des gesunden Menschenverstands.
    Was für ein grober Unsinn zur Herleitung der Lorentz-Transformation:
    Indem man ein ruhendes Koordinatensystem (kartesisch) und ein bewegtes Koordinatensystem (ebenso kartesisch) aufmalt, glaubt Einstein, zwei verschiedene Räume (sogenannte “Inertialsysteme”) geschaffen zu haben, die vollständig unabhängig voneinander existieren.
    Welch kompletter Irrsinn ist das denn, bitteschön?
    Die Wahrheit ist: Natürlich befinden sich diese beiden Koordinatensysteme (“Inertialsysteme”) in einem und zwar in ein- und demselben Raum!
    Dasgleiche trifft selbstverständlich auch für die “Zeit” zu:
    Es gibt nur einen und denselben Raum und nur ein- und dieselbe Zeit.
    Alles andere ist blanker Unsinn!

  928. #931 MartinB
    29. August 2018

    Ganz fest mit dem Fuß aufstampfen und sagen “Ich halte die Luft an bis die Welt tut, was mein Verstand für richtig hält” hilft bestimmt ganz doll….

  929. #932 AB
    29. August 2018

    Ja, richtig. Nachdenken hilft immer!
    Und wenn es auch – zugegebenerweise – sehr lange gedauert hat.
    Aber diese Lösung ist diejenige, nach der ich immer schon intuitiv gesucht habe.
    Dass ich da nicht schon früher draufgekommen bin!
    Also nochmal: Die Schallgeschwindigkeit ist konstant und zwar ohne Zeitdilatation!
    Dann ist mit Sicherheit auch die Lichtgeschwindigkeit konstant und zwar ebenfalls ohne Zeitdilatation!
    Ist eigentlich ziemlich einfach und logisch, finden Sie nicht?

  930. #933 MartinB
    29. August 2018

    @AB
    Natürlich, noch nie hat in den letzten 120 Jahren jemand darüber nachgedacht, dass es auch eine Schallgeschwindigkeit gibt. Der nächste Physik-Nobelpreis ist dir sicher.
    [Ironie aus]

    War das jetzt der Versuch, den dümmsten Kommentar des Monats zu hinterlassen?
    Kleiner Hinweis: Die Schallgeschwindigkeit ist überraschenderweise nicht für alle Beobachter konstant, man kann sogar schneller als der Schall unterwegs sein und eine Schallwelle überholen…

  931. #934 AB
    29. August 2018

    Der Hinweis auf mehrere Beobachter spielt keine Rolle, ebenso nicht die Tatsache des natürlichen Tempolimits der Lichtgeschwindigkeit.
    Wenn beim akustischen Dopplereffekt bei einem einzigen Beobachter bei nur zwei unterschiedlichen Fahrzeuggeschwindigkeiten je dieselbe Schallgeschwindigkeit ankommt – und das tut es ja offenbar -, ist bereits das “Relativitäts”-Prinzip der Konstanz der Signalgeschwindigkeit erfüllt. Das funktioniert beim Schall gänzlich ohne Zeitdilatation!

  932. #935 MartinB
    29. August 2018

    “Der Hinweis auf mehrere Beobachter spielt keine Rolle, ebenso nicht die Tatsache des natürlichen Tempolimits der Lichtgeschwindigkeit.”
    … weil ich das sage, fuß-aufstampf?
    “ist bereits das “Relativitäts”-Prinzip der Konstanz der Signalgeschwindigkeit erfüllt”
    Nein. Tatsächlich kann man relativ zu einer Schallwelle stillstehen oder sie überholen, also ist das Prinzip nicht erfüllt – so ein Prinzip muss nämlich immer gelten, nicht nur manchmal. Und wenn ich mich mit Schallgeschwindigkeit von einer Schallquelle entferne, dann holt mich die Welle auch nicht ein.

    Weitere Kommentare, die deine Ahnungslosigkeit deutlich machen, werde ich je nach Laune löschen oder verspotten…

  933. #936 AB
    2. September 2018

    Fazit:
    Der Schall beweist es uns:
    Die Konstanz der Signalgeschwindigkeit ist eine Eigenschaft der Welle selber – und zwar der Schallwelle und damit auch der Lichtwelle .
    Und ist keine Eigenschaft von Raum und Zeit.
    So ein Quatsch.

  934. #937 MartinB
    2. September 2018

    @AB
    Natürlich. Ganz klar. In den letzten 110 Jahren hat noch keine Physikerin gemerkt, dass es auch Schall gibt. Und natürlich kann man einem Schallsignal nicht davonlaufen oder schneller als der Schall sein, das ist ja ganz klar. Deswegen ist die Schallgeschwindigkeit immer und unter allen Umständen für alle dieselbe.
    [Sarkasmus aus]

  935. #938 AB
    2. September 2018

    Z.B. von Wikipedia entnimmt man:

    Als Beispiel soll angenommen werden, dass das Martinshorn des Krankenwagens Schallwellen mit einer Frequenz von 1000 Hz aussendet. Dieses bedeutet, dass genau 1/1000 Sekunde nach dem ersten Wellenberg ein zweiter Wellenberg nachfolgt. Die Wellen breiten sich mit der Schallgeschwindigkeit
    c ≈ 340 m / s bei 20 °C aus.
    Solange der Krankenwagen steht, ist die Wellenlänge
    λ_S des Schalls, also der Abstand der Wellenberge:
    λ_S = 340 m/s ⋅ 1/1000 s = 0,34 m
    Für einen Beobachter an der Straße kommen diese Wellenberge zwar je nach Entfernung etwas zeitverzögert an. Die Zeit zwischen zwei Wellenbergen ändert sich jedoch nicht. Die Grundfrequenz f_S des wahrgenommenen Tons ist für jeden Abstand von Beobachter und Krankenwagen gleich.

    Die Situation ändert sich, wenn der Krankenwagen mit der Geschwindigkeit
    v auf den Beobachter zufährt. Da sich der Wagen in der Zeit zwischen den beiden Wellenbergen weiterbewegt, verkürzt sich der Abstand zwischen ihnen etwas. Er verkürzt sich um den Weg, den der Wagen in der Zeit von 1/1000 Sekunde zurücklegt:
    λ_B = λ_S − v_S/f_S
    Die Indizes S und B verweisen auf den Sender beziehungsweise Beobachter der Welle. Da sich beide Wellenberge mit derselben Schallgeschwindigkeit c zum Beobachter bewegen, bleibt der verkürzte Abstand zwischen ihnen erhalten, und der zweite Wellenberg kommt nicht erst 1/1000 Sekunde nach dem ersten an, sondern schon ein wenig früher. Bezogen auf obiges Beispiel verkürzt sich die Wellenlänge bei einer Geschwindigkeit von
    36 km/h = 10 m/s:
    λ_B = 0,34 m − 10 m/s / 1000 1/s = 0,33 m

    Dadurch erscheint dem Beobachter die Frequenz (also die Tonhöhe) des Martinshornes höher
    (f_B > f S).

    f_B = c/λ_B = 340 m/s / 0,33 m ≈ 1030,3 Hz

    Quantitativ erhält man die Frequenzänderung einfach durch Einsetzen der Beziehung
    λ ⋅ f = c in obige Formel für λ_B.
    Für die vom Beobachter wahrgenommene Frequenz
    f_B ergibt sich somit:
    f_B = f_S / (1 − v/c)

    Dabei bedeuten f_S die Frequenz der Schallquelle,
    c die Ausbreitungsgeschwindigkeit des Schalls und
    v die Geschwindigkeit der Schallquelle (also des Krankenwagens).

    Für die Geschwindigkeit von v = 20 m/s ergeben sich für die vom Beobachter empfangene Frequenz und Wellenlänge die folgenden Werte:
    λ_B = 0,32 m
    f_B = 1062,5 Hz

    Wie man sieht, verringert sich bei doppelter Geschwindigkeit des Krankenwagens die Wellenlänge der beim Beobachter ankommenden Schallwelle, gleichzeitig erhöht sich deren Frequenz ein wenig.

    Wichtig in diesem Zusammenhang ist jedoch, dass die beim Beobachter ankommende Schallgeschwindigkeit konstant c = 340 m/s bleibt, welche das Produkt aus λ_B und f_B ist:
    c = λ_B x f_B
    Also nochmal:
    Zwei unterschiedliche Krankenwagengeschwindigkeiten ergeben dieselbe Schallgeschwindigkeit beim Beobachter.
    Das ist genau die von Einstein – allerdings für elektromagnetische Welle (Licht) – aufgestellte Forderung der Konstanz der Signalgeschwindigkeit. Diese Konstanz der Signalgeschwindigkeit ist die Eigenschaft einer Welle.
    Da es sich bei Licht genau wie bei Schall um eine Welle handelt, ist auch bei Licht die Konstanz der Lichtgeschwindigkeit gegeben und zwar genau wie bei Schall auch: Ohne Zeitdilatation und Lorentz-Kontraktion!

    Quod erat demonstrandum!

  936. #939 MartinB
    3. September 2018

    @AB
    Wir wissen, dass du es nicht verstanden hast, das wird auch durch Abschreiben bei Wikipedia nicht besser…

  937. #940 Harti
    Münster
    3. September 2018

    Hallo zusammen,
    wie wäre es mit folgender Vorstellung:
    Die absolut vorgestellte Grundlage von Newtons Bewegungslehre waren der absolute Raum und die absolute Zeit. Die absolut vorgestellte Grundlage von Einsteins SRT ist die Vakuumlichtgeschwindigkeit (nicht die tatsächliche Lichtgeschwindigkeit, die sowohl einen räumlichen wie einen zeitlichen Anteil hat, nämlich z.B. ca. 300000 km und 1 sec). Auf dieser Grundlage wird Gleichzeitigkeit im Sinne der SRT durch die Lichtgeschwindigkeit festgelegt, indem solche Ereignisse gleichzeitig sind, deren Kenntnis durch das Licht (nach meiner Meinung durch den Elektromagnetismus ) vermittelt wird. Ein Ereignis, das jetzt auf der Sonne stattfindet nehme ich nach ca. 8,3 Minuten (Lichtlaufzeit) wahr. Damit steht in Einklang, dass lichtartige Vektoren in Raumzeitmodellen den Wert 0 haben.
    Die Längenkontraktion ist ein perspektivischer Effekt, der daraus resultiert, dass Geschwindigkeit in einem räumlich eindimensionalen Modell (Strecke) definiert wird und deshalb eine Richtungsänderung nicht darstellbar ist (Gamov`s Radfahrer). In einem vierdimensionalen Raumzeitmodell gibt es keine Längenkontraktion.
    Falls allles falsch is, wäre ich für eine Richtigstellung dankbar.

  938. #941 MartinB
    3. September 2018

    @Harti
    Es gibt hier nichts vorzuschlagen, jedenfalls nict in dem Sinne, dass man überlegen müsste, wie die SRT genau funktioniert.
    Gleichzeitigkeit wird nicht so definiert, wie du es sagst, da wird immer die Signallaufzeit herausgerechnet. Die Längenkontraktion ist ein Beobachtungseffekt, richtig, ob man das “perspektivisch” nennen will, ist wohl Geschmackssache. Auch da ist es aber so, dass die Längenkontraktion erst dann messbar wird, wenn man Lichtlaufzeiten herausrechnet, wenn der Radfahrer an dir vorbeifährt, würdest du ihn gedreht, aber nicht kontrahiert sehen:
    https://www.spacetimetravel.org/ueberblick/ueberblick1.html

  939. #942 Harti
    3. September 2018

    @MartinB
    Wie soll man den Radfahrer in einem räumlich eindimensionalen Modell (Strecke) gedreht sehen ? Die Darstellung mag zeichnerisch möglich sein, dazu braucht man dann aber eine weitere räumliche Dimension, in die der Radfahrer sich dreht (Tiefenschärfe). Von mir aus können wir es dabei belassen.

  940. #943 MartinB
    3. September 2018

    @Harti
    Nein, das drehen funktioniert natürlich so nur in 2 oder 3 Dimensionen. In eienr Dimension kann man aber auch keinen Radfahrer sehen, da sieht man nur einen Strich, oder?
    Tiefenschärfe hat damit auch wenig zu tun, die ist ne Eigenschaft von optischen Linsen.

  941. #944 Anonym_2018
    3. September 2018

    @AB (2. September 2018) #938

    Zwei unterschiedliche Krankenwagengeschwindigkeiten ergeben dieselbe Schallgeschwindigkeit beim Beobachter.

    Die Schallgeschwindigkeit von ca. 340 m/s gilt nur relativ zum Ausbreitungs-Medium (den Molekülen der umgebenden Luft). Relativ zu 2 unterschiedlich schnell bewegten Beobachtern ergeben sich daher unterschiedliche Schallgeschwindigkeiten.

    Die Vakuum-Lichtgeschwindigkeit von ca. 300.000 km/s gilt relativ zu jedem gradlinig-gleichmäßig bewegten Beobachter und damit auch relativ zu 2 unterschiedlich schnell bewegten Beobachtern.

    Das ist der Unterschied.

    Das ist genau die von Einstein – allerdings für elektromagnetische Welle (Licht) – aufgestellte Forderung der Konstanz der Signalgeschwindigkeit.

    nein (s.o.)

  942. #945 Harti
    3. September 2018

    Und wenn ich auf diesem Strich eine Strecke von z.B. enem Meter auftrage und diese Strecke bewegt sich, dann erscheint diese Strecke in Bewegungsrichtung verkürzt, weil sich die Strecke teilweise gedreht in eine andere Richtung bewegt, die auf dem ursprünglichen eindimensionalen Strich nur teilweise (verkürzt) darstellbar ist. Bei Erreichen der Lichtgeschwindigkeit verläuft die Strecke senkrecht zu dem ursprünglichen Strich und ist auf dem Strich nicht mehr darstellbar.

  943. #946 MartinB
    4. September 2018

    @Harti
    Bin mir nicht sicher, ob ich dir folgen kann. Wenn man Lichtlaufzeiten berücksichtigt, dann erscheint die Strecke irgendwann verkürzt und rotiert, ja(Terrell-Penrose-Rotation). Siehe Die Animation hier:
    https://en.wikipedia.org/wiki/Terrell_rotation

  944. #947 AB
    4. September 2018

    @Anonym_2018
    Vielen Dank für Ihre konstruktive Erläuterung!
    Sie haben recht:
    “Die Schallgeschwindigkeit von ca. 340 m/s gilt nur relativ zum Ausbreitungs-Medium (den Molekülen der umgebenden Luft). Relativ zu 2 unterschiedlich schnell bewegten Beobachtern ergeben sich daher unterschiedliche Schallgeschwindigkeiten.
    Die Vakuum-Lichtgeschwindigkeit von ca. 300.000 km/s gilt relativ zu jedem gradlinig-gleichmäßig bewegten Beobachter und damit auch relativ zu 2 unterschiedlich schnell bewegten Beobachtern.
    Das ist der Unterschied.”

    In der Tat gibt es diesen Unterschied zwischen Schall- und Licht-Welle.

    Jedoch ist dieser Unterschied immer noch kein Grund, die Konstanz der Lichtgeschwindigkeit als eine Eigenschaft von Zeit und Raum zu bewerten. Dennoch bleibt die Konstanz der (Vakuum-) Lichtgeschwindigkeit eine Eigenschaft der (Licht-)Welle an sich.

    Beim akustischen Doppler-Effekt wird unterschieden zwischen den folgenden beiden Szenarien:
    1. Beobachter in Ruhe, Signalquelle bewegt
    2. Beobachter bewegt, Signalquelle in Ruhe

    Beim Schall sind diese beiden Szenarien unterschiedlich, beim Licht nicht.
    Daher mein Vorschlag:
    Bei Licht beim Übergang vom 1. Szenario zum 2. Szenario einfach die Indizes “S” (Sender) und “B” (Beobachter) vertauschen.

    So erhält man für “Licht” die Konstanz der Lichtgeschwindigkeit auch für das Szenario “Beobachter bewegt, Signalquelle in Ruhe”.
    Was halten Sie davon?

  945. #948 AB
    4. September 2018

    Was ich noch zu meinem vorherigen Kommentar einfügen wollte:
    Der Unterschied zwischen Schall- und Lichtwelle berechtigt nicht zur Infragestellung von Raum und Zeit.
    Mit anderen Worten:
    Die Konstanz der Lichtgeschwindigkeit kommt zustande durch Änderung der Wellenlänge und der Frequenz.
    Die Konstanz der Lichtgeschwindigkeit kommt jedenfalls nicht zustande durch Zeitdilatation und Lorentz-Kontraktion!
    Hier ist Einstein mächtig seine Phantasie durchgegangen!

  946. #949 PS
    4. September 2018

    @AB
    Es ist unmöglich, als älterer Mensch aus diesem Meer des Irrtums aufzutauchen.
    Man sollte den Kindern bereits in der Volksschule erklären, dass die „Lichtgeschwindigkeit“ das Gegenteil von Geschwindigkeit eines Materiepunktes ist. Die „Lichtgeschwindigkeit“ ist der Rahmen, in dem die Begriffe Zeit, Raum und Geschwindigkeit entwickelt werden können. Die absolute Wirkungsausbreitung des Lichts ist ein Maß dafürt, wie die Zeit vergeht und wie weit der vom Licht durchmessene Raum ist. Man kann den Kleinen erklären, dass die Länge und die Zeit letztlich nur mit dieser Lichtausbreitung gemessen werden kann. Die Geschwindigkeit zB eines Autos ist ein Ausdruck dafür, welchen Weg es in welcher Zeit zurücklegt. Wenn aber die Länge des Wegs mit der Lichtausbreitung gemessen wird und die Zeit auch, dann ist die (Koordinaten)Geschwindigkeit ein Ausdruck dafür, wie schnell das Auto in Bezug auf die Lichtausbreitung ist, also in welchem Maß, zu welchem Prozentsatz es die Lichtausbreitung erreicht.
    Dann kann man ja zu m und sek zurückkehren und Newtonsche Physik betreiben. Aber es muss von Kindesbeinen an in Fleisch und Blut übergehen, dass es bei v nicht um m/sec, sondern um % von c geht. Damit wird später die relativistische Geschwindigkeitsaddition zum Kinderspiel.

  947. #950 Anonym_2018
    4. September 2018

    So erhält man für “Licht” die Konstanz der Lichtgeschwindigkeit auch für das Szenario “Beobachter bewegt, Signalquelle in Ruhe”.
    Was halten Sie davon?

    Beim Doppler-Effekt des Schalls gilt das “in Ruhe” und das “bewegt” relativ zum Ausbreitungs-Medium (den Molekülen der umgebenden Luft). Das mit der Vertauschung habe ich leider nicht verstanden.

    Jedenfalls wird die Lichtgeschwindigkeit im Vakuum durch die Maxwell’schen Gleichungen vorausgesagt. Diese Gleichungen sind invariant bei einer Lorentz-Transformation, nicht aber bei einer Galilei-Transformation.

    Das Fizeau-Experiment hat (nach einigem Hin und Her) bestätigt, dass die Newton’sche Geschwindigkeitsaddition ( V = v1 + v2 ) durch das relativistische Additionstherem für Geschwindigkeiten ersetzt werden muss:
    V = (v1 + v2) / (1 + (v1 * v2) / c²), mit c = Vakuum-Lichtgeschwindigkeit.

    Wenn sich eine Welle mit einer Geschwindigkeit v1 (z.B. ca. 340 m/s oder z.B. c) auf einen Beobachter zubewegt, und wenn sich der Beobachter seinerseits mit der Geschwindigkeit v2 (z.B. 0,9 * c) auf die ihm entgegenkommende Welle zubewegt, dann hat die Welle vom Beobachter aus gesehen relativ zu ihm die Geschwindigkeit V gemäß dem relativistischen Additionstheorem für Geschwindigkeiten (s.o.).

    Viel Spaß beim Rechnen!

  948. #951 MartinB
    https://scienceblogs.de/hier-wohnen-drachen
    5. September 2018

    @PS
    “Man kann den Kleinen erklären, dass die Länge und die Zeit letztlich nur mit dieser Lichtausbreitung gemessen werden kann”
    Das kann man tun, das wäre aber irreführend bis falsch. Wenn ich mein Geodreieck nehme oder auf ne Uhr schaue, dann geht das ohne Lichtaubsreitung. Man kann auch nicht sowohl Längen als auch Zeiten mit einer Geschwindigkeit festlegen (wie man auch mathematisch daran sieht, dass Längen und Zeiten nicht einheitenlos werden, wenn man c=1 wählt, man misst dann Zeiten in Metern oder Längen in Lichtsekunden). Für eins von beiden braucht man ein unabhängiges Messverfahren, dann kann man c verwenden, um das andere festzulegen, wenn man das will.

  949. #952 AB
    5. September 2018

    @Anonym_2018
    Wie bitte erklärt man sich dann das in meinem Beitrag 938 angesprochene Szenario des akustischen Doppler-Effekts “Schallquelle bewegt, Beobachter in Ruhe” ich meine jetzt: Geschwindigkeitstechnisch gesehen?
    In diesem Beispiel ergeben sich die folgenden Werte:
    10 m/s + 340 m/s = 340 m/s
    20 m/s + 340 m/s = 340 m/s
    Die Galilei-Transformation gilt hier offensichtlich nicht,
    die Lorentz-Transformation jedoch kann hier offensichtlich auch nicht greifen, weil Geschwindigkeit zu gering.
    Diesen Fall dürfte es in der aktuellen Physik eigentlich gar nicht geben, richtig?

  950. #953 MartinB
    5. September 2018

    @AB
    Wirklich jede Interneterklärung des Dopplereffekts erklärt, wie und warum das funktioniert (selbst bei Leifi-Physik sieht man das in der Animation sofort) – hör bitte auf, diesen Blogartikeln mit wiederholten Demonstrationen deiner Unkenntnis vollzuspammen.

  951. #954 PS
    5. September 2018

    @MartinB
    Ja, ja, die liebgewonnen Denkweisen.
    Das “unabhängige Messverfahren” beruht auf der Lichtausbreitung (Wirkungsausbreitung; hat mit dem Begriff der Koordinatengeschwindigkeit eines Materiepunkts nichts zu tun) in einer Lichtuhr, deren Länge frei festgelegt wird (wir haben uns auf eine Sekunde geeinigt). Das GeoDreieck und die Armbanduhr bauen darauf auf. Zeit und Raum werden mit einer “Geschwindigkeit” (Wirkungsausbreitung) festgelegt, darin besteht die innige Beziehung, die wir Raumzeit nennen.

  952. #955 MartinB
    5. September 2018

    @PS
    Wie gesagt, du kannst mit einer Geschwindigkeit nicht sowohl Längen als auch Zeiten festlegen. Erklär mal genau, wie das mit deiner Lichtuhr gehen soll. Wenn du die Länge der Lichtuhr festlegst und das “Sekunde” nennst, dann hast du einen zweiten Maßstab eingebracht, aus dem kannst du dann die Zeiteinheit festlegen.

  953. #956 PS
    5. September 2018

    @Martin B #955
    Es gibt (auch bei der Lichtuhr) nur eine Akt freier Festlegung über die Maßeinheit, nicht zwei.
    Um eine materiell stabile Konstante (um die Konstruktion der Uhr) kommen wir nicht herum (hier: Länge der Lichtuhr).
    Damit sind sowohl die Maßeinheit für den Raum (Länge der Lichtuhr) als auch die Maßeinheit für die Zeit (die Hälfte der Laufzeit des Lichtstrahls bis zu seiner Rückkehr) in unübertrefflicher Exaktheit bestimmt. Oder etwa nicht?
    Die Lichtuhr ist ein Gedankengebilde, das aber auf Grund seiner Einfachheit und der nach Newton paradoxen “Halbzeitreflexion” aus der Sicht aller Bezugssysteme allen anderen grundsätzlichen Überlegungen zur SRT überlegen ist. Es spricht deutlich aus, von welcher Zeit und von welchem Raum wir hier reden. Dass Zeit das ist, was die Uhr anzeigt ist mE zu wenig, weil der Paradigmenwechsel nicht erkannt wird, wenn nur dazu aufgefordert wird, auf seine Armbanduhr zu blicken.
    Mit der Begrifflichkeit der Lichtuhr sollte man im Kindergarten beginnen. Sonst erben sich die Missverständnisse um die SRT bestimmt noch weitere 100 Jahre fort.

  954. #957 AB
    5. September 2018

    @MartinB
    Würden Sie bitte kurz erläutern, welche Transformation hier zuständig ist?

  955. #958 MartinB
    5. September 2018

    @PS
    “Um eine materiell stabile Konstante (um die Konstruktion der Uhr) kommen wir nicht herum (hier: Länge der Lichtuhr).”
    Sag ich ja, das ist dann quasi dein “urmeter”. Wenn das festliegt, liegt auch die Zeiteinheit fest. Aber in der Schule zu erklären, dass man *nur* mit der Lichtgeschwindigkeit Längen- und Zeiteinheiten festlegen kann, so wie von dir oben vorgeschlagen, ist nicht korrekt.

    @AB
    Man muss jeweils das korrekte Bezugssystem wählen; auch wenn sich die Schallquelle oder der Empfänger bewegen, bewegt sich deswegen die Luft nicht mit. Wenn der Krankenwagen mit 300m/s fährt, dann entfernt sich das Schallsignal von ihm nicht mit Schallgeschwindigkeit, sondern die Wellenfront ist von ihm aus gesehen langsamer, das sieht man auf jeder Animation zum Thema und ist vollkommen offensichtlich. Genau deswegen funktionieren Äthertheorien ja auch nicht.
    Und jetzt bitte aufhören zu trollen.

  956. #959 PS
    5. September 2018

    @Martin B
    “Du kannst mit einer Geschwindigkeit nicht sowohl Längen als auch Zeiten festlegen.”

    Mit einer “Geschwindigkeit” nicht, aber mit der Ausbreitung des Lichts.

    Die Wahl des “Urmeters” ist eine freie Konstante. Der Urmeter determiniert über ein- und denselben Lichtstrahl sowohl die Zeiteinheit als auch die Raumeinheit. Wir brauchen keine Zeitmessgeräte und keine Längenmessgeräte. Darauf bezieht sich meine Behauptung, und die scheint mir richtig zu sein. Wir bauen nur auf die Ausbreitung des Lichts. Längen und Zeitmessung ist ein und derselbe Vorgang. Mit einer Messung mit einem Lichtstrahl stehen zeitlicher und räumlicher Abstand eines (Reflexions)Ereignisses fest, und zwar auch in allen anderen Bezugssystemen, L-T gemäß.
    Das ist etwas ganz anderes, als im Kopf immer mit starren Maßstäben und Uhren in verschiedenen Systemen zu hantieren.

  957. #960 MartinB
    5. September 2018

    @PS
    Erklär mir mal bitte, wie genau das gehen soll, ich kann das nicht nachvollziehen. Wie legst du die Längen- und die zeiteinheit beide über die Lichtgeschwindigkeit oder meintewegen die Ausbreitung des Lichts fest? Wie genau sieht der zugehörige experimentelle Aufbau aus, der beides kann?
    ” Mit einer Messung mit einem Lichtstrahl stehen zeitlicher und räumlicher Abstand eines (Reflexions)Ereignisses fest”
    O.k., ich habe 2 Spiegel in einem bestimmten (aber nicht in irgendeiner Einheit gemessenen) Abstand. Wie bekomme ich jetzt draus ne Längen und ne Zeiteinheit, die jemand anders nachvollziehen kann?

    (Tipp: Wenn das gehen würde, würde man das auch so machen, die Tatsache, dass die Längeneinheit Meter nicht über c festgelegt wird, sollte dir schon zeigen, dass das nicht geht, da man ja generell alle Einheiten wenn möglich über Naturkonstanten definiert…)

  958. #961 Anonym_2018
    5. September 2018

    die Tatsache, dass die Längeneinheit Meter nicht über c festgelegt wird …

    Die Längeneinheit Meter wird seit 1983 offiziel über c und “1 Sekunde” festgelegt:

    Ein Meter ist definiert als die Länge der Strecke, die das Licht im Vakuum während der Dauer von 1/299 792 458 Sekunde zurücklegt.

    Die aktuelle Definition gilt seit 1983.

    Quelle:
    https://de.wikipedia.org/wiki/Meter

    Seit 1967 ist eine Sekunde das 9.192.631.770-Fache der Periodendauer der Strahlung, die dem Übergang zwischen den beiden Hyperfeinstrukturniveaus des Grundzustandes von Atomen des Nuklids 133Cs entspricht.

    Quelle:
    https://de.wikipedia.org/wiki/Sekunde

  959. #962 MartinB
    5. September 2018

    @Anonym_2018
    Ja, dann muss aber die Sekunde anders festgelegt werden – ich sage ja nur, dass *beides* nicht geht, du kannst nicht sowohl Meter als auch Sekunde über c festlegen.

  960. #963 MartinB
    5. September 2018

    PS: Ich dachte, es wäre andersherum, aber das spielt für das Argument keine Rolle, es geht ja nur dasrum, dass c allein nicht ausreicht, um alle Maßstäbe in Raum und Zeit festzulegen.

  961. #964 PS
    5. September 2018

    @MartinB #960
    Ich wähle in einem System S eine beliebige Länge einer dort ruhenden Lichtuhr. Ich sende einen Lichtstrahl aus, lasse ihn am anderen Ende der Lichtuhr reflektieren und zum Ausgangspunkt zurückkehren. Die Hälfte der Gesamtprozessdauer nenne ich “1 Sekunde” und “1 Meter”.

    Wenn ich aus der Sicht S feststellen will, wann ein Ereignis wo eingetreten ist (zB die Position eines Fahrzeugs), so sende ich einen “Tastlichtstrahl” aus, der bei dem Ereignis (bei der Radarpistole zB bei zwei verschiedenen Ereignissen) reflektiert wird. Ich vergleiche die Prozessdauer bis zur Rückkehr des “Tastlichtstrahls” (das ist mein Messinstrument) mit dem Stand meiner nach obiger Vorschrift gewonnenen Uhr und kann präzise sagen, wann aus meiner Sicht das betreffende Ereignis wo eingetreten ist (also den zeitlichen und räumlichen Abstand von einem gewählten Ausgangsereignis ermitteln). Ich benötige nur einen Messvorgang, nicht zwei, um Ort und Zeit des Ereignisses festzustellen. Die Messung ist das Pendant zur Eichung: auch dort gab es nur einen Prozess.

    Die Eichung anderer Uhren auf die frei festgelegte “Urmetersekunde” kann sowohl bei ruhenden als auch bei gleichförmig bewegten anderen Lichtuhren erfolgen.

    Auch in anderen Systemen erfolgt die Feststellung des räumlichen und zeitlichen Abstandes des selben Ereignisses mit derselben Methode. Die Ergebnisse folgen der L-T.

    Hilft das weiter?

  962. #965 MartinB
    5. September 2018

    @PS
    “Die Hälfte der Gesamtprozessdauer nenne ich “1 Sekunde” und “1 Meter”. ”
    Ich hake etwas dabei, eine Dauer gleichzeitig in “Meter” und in Sekunden anzugeben, aber whatever floats your boat. Ich sprechen im folgenden von “einheit”, damit’s nicht verwirrend wird.

    Jetzt nehmen wir ein Ereignisse S1, deren Abstand du wissen willst. Du sendest jede Menge Lichtstrahlen aus und stellst fest, wann genau das wieder zurückkommt, das bei S1 reflektiert wurde. Nehmen wir an, das dauert 7 einheiten, dann weißt du, dass das Ereignis 3,5 einheiten entfernt ist. Habe ich soweit richtig verstanden?

    Soweit so kein Problem. Die Definition einer Längen- oder Zeiteinheit erfordert aber ja, dass du eine Vorschrift angibst, die es mir ermöglicht, dieselben Einheiten festzulegen, ohne dass ich deine Lichtuhr zur Verfügung habe. (Deswegen hat man ja mal das Urmeter abgeschafft.) Du kannst natürlich Signale mit mir austauschen, aber das ist dann nur eine Weiterleitung deiner “ur-Einheit”, dann sind wir wieder beim Urmeter. Eine Vorschrift, die ich zum Beispiel einfach nachlesen kann, um damit deine Einheit nachzubauen, funktioniert nur auf der Basis dieses Experiments nicht.

    Und genau deswegen brauchen wir eine zweite Definition, in diesem Fall (wie Anonym2018 richtig schrieb und ich nicht…) über ne Frequenz, um sowohl Sekunden als auch Meter zu definieren. Diese Definition könnte ch nach Alpha Centauri schicken und die Aliens dort könnten dann wissen, wie lange eine Sekunde ist. Mit deine Spiegelaufbau funktioniert das nicht.
    Und deswegen ist es irreführend zu behaupten, man könne (oder gar müsse) Längen und Zeiten nur über Lichtausbreitung festlegen.

  963. #966 PS
    5. September 2018

    @MartinB #965
    Wenn ich aus Sicht von S eine Frequenz definiere, ermittle ich an Hand meiner nach obigen Vorschriften geschaffenen Uhr einen (konstanten) zeitlichen Abstand zweier (und aller weiteren) Ereignisse, die keinen räumlichen Abstand voneinander haben.

    Alpha Centauri kann mit der bloßen Mitteilung über die numerische Höhe dieser Frequenz nichts anfangen, so lange sie dort keine nach denselben Vorschriften gewonnene und geeichte Uhr besitzen.

    Wenn Alpha Centauri in S ruht, dann kann ich die Dauer eines Ticks meiner Lichtuhr korrekt übermitteln, sodass die dort mit irgendeiner Lichtuhr feststellen können, wie lange in S eine definierte Sekunde ist. Jetzt haben sie eine gemeinsame Basis und können Frequenzen richtig zuordnen.

    Die Eichung von Uhren, die sich in Bezug auf S mit v bewegen, erfolgt so, dass in der Standardsituation (bei Ursprungsdeckung) die Beobachter S und S’ zwei Lichtstrahlen zB in Bewegungsrichtung aussenden, die an demselben (bewegten oder nicht bewegten) Spiegel reflektiert werden. Jeder Beobachter ermittelt die in seinem System auftretende halbe Prozessdauer. Aus dem Verhältnis dieser Prozessdauern, die der L-T folgt, kann die Uhr synchronisiert werden, wenn ich die Relativgeschwindigkeit (in % von c) kenne. Diese kann ich zB mit der Radarmethode (siehe oben) von S aus feststellen und Alpha Centauri mitteilen.

  964. #967 AB
    5. September 2018

    @MartinB
    Irrtum, hören Sie bitte endlich auf zu trollen von wegen Zeitdilatation und ähnlichem Schwachsinn!

  965. #968 MartinB
    6. September 2018

    @PS
    Ja, natürlich, so kann man es machen, das erfüllt aber gerade nicht die Anforderung an die Definition von Längen- und Zeiteinheiten, weil du ein Zeitsignal übertragen musst, damit die Leute auf Alpha Centauri deinen Maßstab kennen. Letztlich ist es dasselbe, als ob du ein Urmeter hinschickst.
    Und damit ist es eben nicht möglich, allein mit Hilfe der Lichtausbreitung Längen und Zeiten zu definieren, wie du eingangs gesagt hast. Man muss zusätzlich eine Definition entweder einer Längeneinheit (Abstand der Spiegel) oder einer Zeiteinheit (Frequenz des Signals) festlegen. Und das ist das, was ich die ganze Zeit gesagt habe.

  966. #969 Harti
    6. September 2018

    @MartinB
    Nach m.M sollte man der fundamentalen Bedeutung der Lichtgeschwindigkeit für die SRT in einer anderen Darstellung Rechnung tragen.
    Bei der Lichtgeschwindigkeit handelt es sich um die Anwendung der Kategorien von Raum (Strecke) und Zeit auf den Elektromagnetismus. Es ergeben sich unterschiedliche Werte für die Lichtgeschwindigkeit, je nachdem welche Einheiten man verwendet (z.B. Meter, Yard u.s.w.) Da es sich um eine Beziehung zwischen Raum und Zeit handelt, kann man mit Hilfe der Lichtgeschwindigkeit Raum in Zeit und und Zeit in Raum umrechnen (z.B. Lichtsekunde = Strecke)
    In einem vereinfachten Raumzeitmodell (eine Zeitachse und eine Raumachse) erscheint die Bewegung elektromagnetischer Wellen in einem 45°-Winkel, weil der zeitliche Anteil und der räumliche Anteil der Bewegung gleich sind. Vektoriell (als Weltlinie, Raumzeitintervall) betrachtet haben solche lichtartigen Vektoren den Betrag 0, als Beziehung zwischen Raum und Zeit betrachtet hat die Bewegung von Lichtwellen (Photonen) den einheitenfreien Wert 1 .Dies entspricht m.E. besser der Gleichwertigkeit von elektrischer und magnetischer Wirkung und macht die fundamentale Bedeutung der Lichtgeschwindigkeit (bzw. des Elektromagnetismus) deutlich.
    Falls ich was Falsches geschrieben habe, lasse ich mich gerne korrigieren.

  967. #970 MartinB
    6. September 2018

    @Harti
    “Bei der Lichtgeschwindigkeit handelt es sich um die Anwendung der Kategorien von Raum (Strecke) und Zeit auf den Elektromagnetismus”
    Nein. Auch wenn es keinen Elektromagnetismus gäbe, wäre die Lichtgeschwindigkeit die obere Grenze, die würde dann nur anders heißen.

    “Da es sich um eine Beziehung zwischen Raum und Zeit handelt, kann man mit Hilfe der Lichtgeschwindigkeit Raum in Zeit und und Zeit in Raum umrechnen (z.B. Lichtsekunde = Strecke)”
    Ja, natürlich.

  968. #971 PS
    6. September 2018

    @AB #967
    Ohne zu trollen: wenn Du willst, kann ich Schritt für Schritt die SRT mit Dir entdecken. Du hast die Möglichkeit, zu widersprechen, wenn Du einen der Schritte nicht für richtig hältst.

  969. #972 MartinB
    6. September 2018

    @PS
    Macht das aber bitte nicht hier in den Kommentarspalten….

  970. #973 PS
    6. September 2018

    Hier wohnen Drachen?

  971. #974 MartinB
    7. September 2018

    @PS
    Was soll mir das jetzt sagen?
    Ich möchte auf meinem Blog nicht unbedingt extrem lange Kommentarwechsel haben, bei denen der Eindruck entsteht, an der Relativitätstheorie sei irgendwas zweifelhaft oder problematisch, schon gar nicht mit so argumentresistenten Leuten wie AB. Das Internet ist groß genug, da findet ihr sicher ein Plätzchen, wo ihr euch austauschen könnt.

  972. #975 PS
    7. September 2018

    Ich verstehe Dich. AB hat sowieso kein Interesse.

  973. #976 Harti
    8. September 2018

    @MartinB
    “Auch wenn es keinen Elektromagnetismus gäbe, wäre die die Lichtgeschwindigkeit die obere Grenzenze, die würde dann nur anders heißen.”
    ??? Was wäre die Ursache dieser Grenze ? Woraus besteht diese Grenze ? Warum können wir prinzipiell nur ca. 13 Milliarden Jahre in die Vergangenheit und ca. 13 Milliarden Lichtjahre weit “sehen” ?

  974. #977 AB
    8. September 2018

    @MartinB
    Entschuldigen Sie bitte, aber ich bin argumentresistent?
    Mit Verlaub, glauben Sie wirklich, Ihr Hinweis auf v=300 m/s wäre ein Argument, was die Lorentz-Transformation inklusive Zeitdilatation – letzten Endes die SRT – rechtfertigt?
    Inwieweit das Szenario “Signalquelle bewegt, Beobachter in Ruhe” des akustischen Doppler-Effekts das Verhalten des Schalls richtig abbildet, spielt hier in diesem Zusammenhang eigentlich keine Rolle, ist hier eigentlich völlig irrelevant.
    Entscheidend ist hier im Zusammenhang mit der SRT doch nur die Tatsache, dass die Formeln zu besagtem Szenario “Signalquelle bewegt, Beobachter in Ruhe” (siehe mein Beitrag 938 in diesem Blog) das Szenario von der “Konstanz der Signalgeschwindigkeit” wiedergibt.

    Sehen Sie denn nicht, sehr geehrter MartinB, dass genau dieses Szenario dasjenige ist, welches Einstein für sein Postulat der “Konstanz der Lichtgeschwindigkeit” benötigte, wir also dieses Szenario 938 einfach nur auf das Licht übertragen müssen, um die Konstanz der Lichtgeschwindigkeit zu erreichen?

    Die Frage ist doch jetzt: Wie erreichen wir die Konstanz der Lichtgeschwindigkeit auch für das zweite Szenario “Signalquelle in Ruhe, Beobachter bewegt”?

    Für diesen Fall mein Vorschlag: “Tausch der Indizes S und B”

    @Anonym_18
    1. Szenario: “Signalquelle bewegt, Beobachter in Ruhe”:
    λ_S = c/f_S
    λ_B = λ_S − v_S/f_S
    f_B = c/λ_B

    Um jetzt zum zweiten Szenario: “Signalquelle in Ruhe, Beobachter bewegt” zu kommen, müssen lediglich die Indizes “S” und “B” getauscht werden. Man erhält:
    λ_B = c/f_B
    λ_S = λ_B − v_B/f_B
    f_S = c/λ_S

    Auf diese Art erhält man für beide Szenarien “Signalquelle oder Beobachter bewegt” die Konstanz der Signalgeschwindigkeit und zwar gänzlich ohne Lorentz-Transformation inklusive Zeitdilatation.
    Quod erat demonstrandum!

  975. #978 MartinB
    8. September 2018

    @Harti
    Die Ursache dieser Grenze ist die Struktur der Raumzeit. Auch ungeladene materielle Teilchen können die Lichtgeschwindigkeit nicht erreichen (oder bewegen sich, wenn sie masselos sind, mit Lichtgeschwindigkeit).
    Die SRT ist keine Theorie des Elektromagnetismus, sie wurde nur auf diesem Wege entdeckt.

    “Warum können wir prinzipiell nur ca. 13 Milliarden Jahre in die Vergangenheit und ca. 13 Milliarden Lichtjahre weit “sehen” ?”
    Das ist zunächst mal falsch, siehe
    https://scienceblogs.de/hier-wohnen-drachen/2010/09/19/wie-gross-ist-das-beobachtbare-universum/

    Im Übrigen sage ich ja nicht, dass Licht sich nicht mit Lichtgeschwindigkeit beweget aber die Lichtgeschwindigkeit ist eine absolute Signalgeschwindigkeit, die für jede Art Signal gilt.

    @AB
    O.k., ich sperre gern wunschgemäß auch diese mailadresse, so wie die letzte. Irgendwan ist das Maß an Dummheit, das ich in meinen Kommentarspalten akzeptiere, erreicht.

  976. #979 Anonym_2018
    8. September 2018

    Das ist schon falsch:

    @Anonym_18
    1. Szenario: “Signalquelle bewegt, Beobachter in Ruhe”:
    λ_S = c/f_S

    Im (bewegten) Koordinatensystem des mit 10 m/s fahrenden Krankenwagens hat der in Fahrtrichtung abgestrahlte Teil des Schalls nicht die Geschwindigkeit c=340 m/s, sondern 340 m/s – 10 m/s = 330 m/s.

  977. #980 MartinB
    8. September 2018

    @Anonym_2018
    Die Diskussion mit AB bitte beenden, ich habe ihn hier gesperrt.

  978. #981 Anonym_2018
    8. September 2018

    O.K.

  979. #982 Anonym_2018
    8. September 2018

    Auch ungeladene materielle Teilchen können die Lichtgeschwindigkeit nicht erreichen (oder bewegen sich, wenn sie masselos sind, mit Lichtgeschwindigkeit).

    Diese Teilchengeschwindigkeit ist die Gruppengeschwindigkeit v_Gr der über die Wellenlänge
    λ = h/p = h/(m * v_Gr) zugeordneten De-Broglie-Materiewelle.
    Mit v_Ph = Phasengeschwindigkeit gilt dann:
    v_Gr = h/(m*λ) = (h*f)/(m * v_Ph) = mc² / (m * v_Ph) = c² / v_Ph.
    =>
    v_Ph * v_Gr = c².
    Die Elektromagnetischen Wellen sind ein Spezialfall (für die ungeladenen und Ruhemasse-losen Photonen) der (Lorentz-invarianten) Materiewellen. Aus den Maxwell’schen Gleichungen für das Vakuum folgt:
    c = 1 / √ (ε0 * µ0). Daraus ergibt sich eine Abhängigkeit von der Permittivität (Polarisierbarkeit) und Permeabilität des Vakuums.

    Diese Abhängigkeit von den Vakuum-Eigenschaften sollte dann auch für den Zusammenhang mit der Lichtgeschwindigkeit
    v_Ph * v_Gr = c²
    bei den allgemeineren Materiewellen gelten. D.h. die “Teilchen” (in Wirklichkeit Wellen-Gruppen) bewegen sich immer schneller als c (hypothetische Tachyonen) oder immer gleich c (z.B. Photonen oder Gluonen) oder immer langsamer als c (z.B. Elektronen).

  980. #983 MartinB
    8. September 2018

    @Anonym_2018
    Habe keine Ahnung, was mir das sagen soll. Mir geht es darum, dass ungeladene teilchen keinerlei Wechselwirkung mit Photonen haben. Wenn für die c relevant ist, sieht man schon, dass c nicht einfach eine Eigenschaft des Elektromagnetismus ist.
    (Man kann natürlich ganz generell zeigen, dass eine Geschwindigkeit, die für alle Beobachterinnen gleich ist, ziemlich zwingend eine Maximalgrenze der Geschwindigkeit ist.)

  981. #984 Anonym_2018
    8. September 2018

    Habe keine Ahnung, was mir das sagen soll.

    Ich wollte nur das folgende infrage stellen:

    Die Ursache dieser Grenze ist die Struktur der Raumzeit.

    • Eine Grenze in der Newton’schen Theorie ist der Wert ∞,
    • Eine Grenze wird in der SRT selbst nicht als Wert festgelegt.

    Die SRT legt keinen Wert für c in dem Term
    γ = 1 / √ (1-v²/c²) fest (weder ca. 300.000 km/s, noch ∞), sondern beschreibt in dem zweiten der beiden Postulate nur eine Voraussage der Maxwell’sche Theorie.

    Die Maxwell’sche Theorie wiederum sagt zusätzlich
    c = 1 / √ (ε0 * µ0) = ca. 300.000 km/s, basierend auf indirekten Messergebnissen für ε0 und µ0 von Faraday (D=ε0 * E und B= µ0 * H).

    Bei Naturkonstanten ist es schwierig, die Ursachen für deren Wert zu benennen. Aber bei c scheinen mir das die (konstanten) Eigenschaften des Vakuums (ε0 und µ0) zu sein, von denen wiederum die Eigenschaften der Geometrie (Wert für c) abhängen.

    Das könnte über Geschwindigkeits-/Magnetfeld-abhängige Eigenschaften der Messgeräte (z.B. Caesium-Uhr, früher: Ur-Meter) geschehen.

  982. #985 Anonym_2018
    8. September 2018

    Mir geht es darum, dass ungeladene teilchen keinerlei Wechselwirkung mit Photonen haben.

    Falls es das hypothetische “Graviton” als Austauschteilchen geben sollte, könnten darüber ungeladene Teilchen (z.B. Photonen) schon mit Photonen wechselwirken.

    Zumindest, falls bei Wikipedia der Eintrag “Gravitation” nicht genauso irreführend ist, wie der Eintrag “elektromagnetisch”:

    Photon

    Wechselwirkungen elektromagnetisch
    Gravitation

    Quelle:
    https://de.wikipedia.org/wiki/Photon

  983. #986 MartinB
    8. September 2018

    @Anonym
    Alle masselosen Teilchen bewegen sich mit einer bestimmten Geschwindigkeit, Auch Photonen. Für Photonen kann man diese Geschwindigkeit auch über sowas wie die Diel-Konstante des Vakuums festlegen, aber ich glaube, du wirst keine Physikerin finden, die annimmt, dass diese Größe fundamentaler ist als c. Umkehrt wird ein Schuh draus

    Es ist ein Universum ohne Elektromagnetismus denkbar, auch in dem würde die SRT problemlos gelten. c ist eine fundamentale Umrechnungskonstante zwischen räumlichen und zeitlichen Distanzen; dass sich Licht (wie alle masselosen Teilchen) mit dieser Geschwindigkeit bewegt, ist eine Folgerung aus dieser fundamentalen Struktur der Raumzeit.

    Und dass es indirekte Wechselwirkungen zwischen ungeladenen teilchen und Photonen geben kann, ist einerseits offensichtlich, ändert aber nichts daran, dass diese indirekten Wechselwirkungen wohl kaum so fundamental sind, dass sie die Maximalgeschwindigkeit dieser Teiclhen festlegt (das wäre seltsam, wenn unterschidliche Teilchen unterschiedliche Wechselwirkung mit Photonen haben, warum sollte für alle dieselbe Max-Geschwindigkeit herauskommen?)

  984. #987 MartinB
    8. September 2018

    @Anonym
    Nachtrag:
    Auch die Gravitation breitet sich mit c aus, unabhängig davon, ob es Gravitonen gibt oder nicht. Gravitation hat nun wirklich sehr wenig Wechselwirkung mit Photonen, warum sollte für sie trotzdem die Dielektrizitätsknstante des Vakuums irgendwie relevant sein?

  985. #988 Anonym_2018
    8. September 2018

    Gravitation hat nun wirklich sehr wenig Wechselwirkung mit Photonen, warum sollte für sie trotzdem die Dielektrizitätsknstante des Vakuums irgendwie relevant sein?

    Dazu führe ich folgendes Gedankenexperiment aus:

    Ich schicke eine Laserstrahl zum Mond. Dort wird er von einem Laserreflektor reflektiert, den Apollo 11-Astronauten (angeblich) 1969 dort aufgestellt haben.
    Ich messe die Zeit, bis der Strahl wieder zurück gekommen ist. Als Uhr verwende ich einen L/C-Schwingkreis und zähle die Schwingungen der Frequenz:
    f0 = 1 / (2π √(L*C)).
    Die Kapazität des Plattenkondensators im Vakuum ist
    C= ε0 * A/d.
    D.h. die Periodendauer der Schwingung ist proportional zu √(L * ε0 * A/d). Ich nehme an, dass L unverändert bleibt und “definiere”, dass die räumlichen Abmessungen von Gegenständen (z.B. A/d) unverändert bleiben.
    => Die Periodendauer der Schwingung ist nun proportional zu √ε0.

    Ich notiere mir die so gemessene Zeitdauer, die der Laserstrahl für hin und zurück gebraucht hat.
    Nun verdoppelt sich aus irgendeinem Grund das ε0 des Vakuums, ausgedrück in seinen
    ursprünglichen SI-Einheiten [(A * s)/(V * m)].
    Die Lichtgeschwindigkeit laut Maxwellgleichungen muss also um √2 langsamer geworden sein. Meine Schwingkreis-Uhr läuft aber auch um √2 langsamer, also ich messe die gleiche Laserstrahl-Laufzeit für hin und zurück, wie zuerst.

    Ich gehen davon aus, dass alle elektromagnetischen Experimente, elektromagnetischen Uhren und die Lichtausbreitung um √2 langsamer ablaufen, wenn sich ε0 verdoppelt.

    Unter welcher Voraussetzung gilt das auch für gravitative Uhren, z.B. das Fadenpendel?
    Periodendauer T = 2π √(l/g)
    Die Gravitationsbeschleunigung g auf der Erdoberfläche ist proportional zur Gravitationskonstante. Damit T um √2 größer wird, müsste sich die Gravitationskonstante halbieren. Dann würde auch der Mond um √2 länger brauchen, um die Erde zu umkreisen.

    Ich kann das alles wieder kompensieren, wenn ich die Einheit der Zeit s (1 Sekunde) durch die um √2 größere Zeit-Einheit s_neu ersetze.

    D.h. ε0 und damit c würden immer als konstant erscheinen, weil ihre Veränderung alle physikalischen Vorgänge und Uhren so beeinflussen würde, dass diese Veränderung nicht bemerkt werden kann.

    Allerdings müsste dafür, wie oben angedeutet, vorausgesetzt werden, dass die Gravitationskonstante G von ε0 wie folgt abhängt:
    G * ε0 * µ0 = G/c² = konst.
    Dann würde auch der Schwarzschild-Radius bei verändertem ε0 konstant bleiben.

    Die Polarisation des Vakuums würde dann nicht nur die elektromagnetische, sondern auch die gravitative Wechselwirkung abschirmen. Das würde wiederum voraussetzen, dass die virtuellen Antiteilchen eine negative gravitative Masse haben. Das würde auch erklären, warum das Vakuum mit vielen virtuellen Teilchen-/Antiteilchenpaaren nicht sehr schwer ist.

  986. #989 Harti
    9. September 2018

    @Martin B
    Wenn Photonen masselose Teilchen sind, dürften sie eigentlich in einem Gravitationsfeld nicht abgelenkt werden. Oder ?

  987. #990 MartinB
    9. September 2018

    @Harti
    Sie haben Masse, aber keine Ruhemasse. Alles, was Energie hat, unterliegt der Gravitation.

  988. #991 MartinB
    9. September 2018

    @Anonym_2018
    Ich habe keine Ahnung, was das soll, und ehrlich gesagt auch keine Zeit/Lust, nachzuvollziehen, warum du meinst, mit irgendwelcher Formeltrickserei die Dielektrizitätskonstante irgendwie für fundamentaler als c zu erklären (falls es das ist, was du willst). Ich habe auch keine Idee, warum die Gravitationskonstante was damit zu tun haben sollte – wenn du das annehmen musst damit deine Idee stimmt, dann spricht das gegen deine Idee, nicht dafür.

  989. #992 Anonym_2018
    9. September 2018

    warum du meinst, mit irgendwelcher Formeltrickserei die Dielektrizitätskonstante irgendwie für fundamentaler als c zu erklären (falls es das ist, was du willst)

    Sorry, dass mein letzter Kommentar etwas länglich geworden ist!

    Der Grund, warum aus meiner Sicht ε0 fundamentaler als c sein könnte und mit der (bisher wenig verstandenen) Gravitation zusammenhängen könnte, ist: Ich verstehe nicht, wie eine in der Raumzeit “eingebaute” Maximalgeschwindigkeit einen bestimmten Wert für die Polarisierbarkeit des Vakuums (ε0) erzwingen kann. Wie kann man so etwas erklären?

  990. #993 MartinB
    9. September 2018

    @Anonym_2018
    PS: Vielleicht war der Kommentar eben zu unfreundlich im ton? Falls ja, tut mir Leid, war nicht böse gemeint.

    Ich habe das aber mal zum Anlass genommen, einen Blogartikel zum Thema zu schreiben, der erscheint irgendwann demnächst (vermutlich Ende der Woche). Wenn du dann noch Diksussionsbedarf siehst, kannst du ja dort nochmal posten.

  991. #994 MartinB
    9. September 2018

    Sorry, da haben sich Kommentare überkreuzt.

    Ich antworte mal mit ner Gegenfrage: Warum gilt dasselbe nicht für zum Beispiel Gluonen – warum müssen wir c nicht aus deren Feldgleichung ableiten?

    Vielleicht steckt dein problem auch in der Interpretation von epsilon_0. Ich würde das schlicht als ein Maß für die Kopplungsstärke zwischen Ladungen und Photonen ansehen und nicht als eine Größe, die das Vakuum per se beschreibt. So sieht es ja auch im Coulomb-Gesetz aus, da kommt die Konstante ja her. So wie die Gravitationskonstante angibt, wie stark eine Masse den Raum krümmt, gibt eps_0 an, wie stark eine elektrische Ladung an das Photonenfeld koppelt.

  992. #995 Harti
    9. September 2018

    @
    @MartinB
    Ich würde eher sagen: Photonen können aufgrund ihrer Natur nicht ruhend vorgestellt werden.

    Aber vorstellen kann man man sich natürlich alles, egal wie eng der Bezug zur Wirklichkeit ist.
    Im Verhältnis zu den masselosen, ruhenden Photonen bewegen sich dann sämtliche Objekte mit Lichtgeschwindigkeit (Vierergeschwindigkeit).
    Oder man stellt es sich umgekehrt vor: Alle Objekte ruhen (Blockuniversum) im Verhältnis zu den bewegten Photonen.
    Wäre es nicht sinnvoller, zwischen der tatsächlichen meßbaren Lichtgeschwindigkeit von ca. 300000 km/s und der absolut vorgestellten Vakuumlichtgeschwindigkeit zu unterscheiden ?

  993. #996 MartinB
    9. September 2018

    @Harti
    Das Ruhesystem eines Photos ergibt schlicht keinen Sinn, da wäre alles auf seiner Weltlinie am selben ort und zur selben zeit.

    “Wäre es nicht sinnvoller, zwischen der tatsächlichen meßbaren Lichtgeschwindigkeit von ca. 300000 km/s und der absolut vorgestellten Vakuumlichtgeschwindigkeit zu unterscheiden ?”
    Könnte man machen – da es aber keinen Grund gibt anzunehmen, dass die verschieden sind, macht man das nicht, Wie gesagt, dazu kommt demnächst noch nen Artikel.

  994. #997 Anonym_2018
    9. September 2018

    Warum gilt dasselbe nicht für zum Beispiel Gluonen – warum müssen wir c nicht aus deren Feldgleichung ableiten?

    1) Elektroschwache WW: Für Photonen, W- und Z-Teilchen gibt es eine Verallgemeinerung der Maxwell’schen Gleichungen, incl. c: die Proca-Gleichung.

    2) Starke WW: Die Gluonen-Geschwindigkeit wurde wegen des Confinements noch nie gemessen. Die Ruhemasse ist nur theoretisch vorhergesagt:

    Masse (theoretisch) 0 kg

    Quelle:
    https://de.wikipedia.org/wiki/Gluon

    Und innerhalb der Nukleonen gibt es wegen des Confinements ohnehin zusätzlich komplizierte Quanteneffekte, die z.B. zu “1/3 e” führen.

    Ich würde das schlicht als ein Maß für die Kopplungsstärke zwischen Ladungen und Photonen ansehen und nicht als eine Größe, die das Vakuum per se beschreibt.

    Die Vakuum-Polarisation hat aber einen schwächenden Einfluss, aber nicht bei sehr kurzer Entfernung zwischen Ladungen, siehe Lamb-Shift:

    Die Aufhebung der Entartung wird durch Vakuumfluktuationen bewirkt, bei denen ständig, in Übereinstimmung mit der Heisenbergschen Unschärferelation, virtuelle Photonen aus dem Vakuumfeld absorbiert und emittiert werden. Die dadurch hervorgerufene Bewegung (vgl. auch Zitterbewegung) verändert im zeitlichen Mittel das auf das Elektron wirkende Potential in Kernnähe.

    Quelle:
    https://de.wikipedia.org/wiki/Lamb-Verschiebung

  995. #998 MartinB
    9. September 2018

    @Anonym_2018
    Verwechselst du gerade die Vakuumpolarisation (den quantenfeldtheoretischen effekt) mit der Dielektrizitätskonstante? Die haben nur indirekt (über die Definition der Ladung mittels Renormierung) was miteinander zu tun, soweit ich sehe.

    Im Gaußschen cgs-System ist die Dielektrizitätskonstante übrigens gleich 1, dass wir da in SI-Einheiten einen anderen Wert haben, liegt nur daran, dass wir Ladungen mit ner komischen Einheit messen. Ich glaube, du verstehst epsilon_0 falsch…

    Und natürlich wurde die Ausbreitung von Gluonen noch nie gemessen. Ich zeige doch nur, dass es ohne Probleme denkbar ist, dass es zwei masselose Austauschteilchen gibt und spätestens dann stößt dein Argument an Grenzen.

  996. #999 MartinB
    9. September 2018

    Anderer Tipp: Schreib die Maxwellgleichungen so um, dass du mu_0 eliminierst (durch eps_0 und c). Dann bleibt ein DGL-System um, das zwei Konstanten enthält: c und eps_0. Das ist dann ganz analog zur Einsteinschen feldgleichung für die Gravitation, die auch zwei Konstanten enthält (G und c), G entspricht dem eps_0 (bzw dem Kehrwert davon) und gibt die kopplung zwischen “Ladung” und “Feld” an. Und c steckt schlicht drin, damit sich die Wechselwirkung korrekt lichtschnell ausbreitet.

    Und wie gesagt, ein Artikel zum Thema kommt noch.

  997. #1000 Anonym_2018
    9. September 2018

    Verwechselst du gerade die Vakuumpolarisation (den quantenfeldtheoretischen effekt) mit der Dielektrizitätskonstante?

    Das hängt zusammen. In der Formel für die Coulomb-Kraft
    F= 1 / (4π ε0 εr) * (q1 q2 / r²)
    ist die abschwächende Wirkung der Vakuum-Polarisation durch Renormierung und Verwendung einer verkleinerten “effektiven” Elementarladung, jeweils passend zum festgelegten ε0 im Einheitensystem, schon “eingepreist”. Zu niedrige Ergebnisse für F liefert die Formel nur bei extrem kleinen Abständen r (wenn sozusagen fast keine virtuellen Vakuum-Quantenobjekte mehr dazwischen passen).

    Vakuumpolarisation

    Anschaulich gesprochen polarisiert das elektrische Feld eines geladenen Teilchens das es umgebende Vakuum, so daß es von einer Wolke virtueller geladener Teilchen-Antiteilchen-Paare umgeben ist, die das Coulomb-Potential der klassischen Elektrodynamik modifizieren.

    https://www.spektrum.de/lexikon/physik/vakuumpolarisation/15044

    In einem dielektrischen Material kommt noch der Faktor εr hinzu:

    Die relative Permittivität ist ein Maß für die feldschwächenden Effekte der dielektrischen Polarisation des Mediums.

    Quelle:
    https://de.wikipedia.org/wiki/Permittivit%C3%A4t

    Wenn die (konstante) Vakuum-Polarisierbarkeit unerwarteterweise ansteigen würde, könnte man das durch Anhebung von ε0 oder Absenkung der (effektiven) Elementarladung beschreiben.

  998. #1001 MartinB
    9. September 2018

    Nachtrag zu 999:
    Wenn du das tust, siehst du folgendes:
    Die ersten beiden MG lasse ich mal unverändert. Di e letzten beiden schreibe ich so um:
    rot E = -dB/dt
    c^2 rot B = j/eps_0 + dE/dt

    Ich definiere eine neue Größe für das B-Feld und schreibe es als (cB). Das macht Sinn, weil diese Kombination z.B. auch in die Energiedichte eingeht:
    u= eps_0/2(E^2 +c^2 B^2).
    Man kann also mit einem c die Einheit von B und E angleichen.

    Damit bekomme ich jetzt
    c rot E = -dB / dt
    c rot(cB) = j/eps_0 + dE/dt

    Damit siehst du, warum das c in die Maxwell-Gleichung eingeht: Einmal schlicht, weil wir B ungeschickt definiert haben, einmal, weil wir auf der linken Seite jeweils räumliche, auf der rechten Seite zeitliche Ableitungen haben und die ineinander umrechnen müssen. (Raumzeit!) Und eps_0 taucht immer gemeinsam mit der Ladung auf, entweder als rho/eps_0 oder als j/eps_0, weil eps_0 nichts anderes angibt als die kopplung zwischen Ladung und Feld.

  999. #1002 MartinB
    9. September 2018

    Hat sich überkreuzt, ich hoffe, der letzte Kommentar klärt die Sache endgültig.

  1000. #1003 Anonym_2018
    9. September 2018

    Und wie gesagt, ein Artikel zum Thema kommt noch.

    interessant!

  1001. #1004 Anonym_2018
    9. September 2018

    weil eps_0 nichts anderes angibt als die kopplung zwischen Ladung und Feld.

    Das ε0 beschreibt, wie stark das Feld durch das Gegenfeld einer im Vakuum induzierten Polarisation abgeschwächt wird.

    Aber nicht nur das. Das ε0 ist immer noch in
    c (= 1 / √ (ε0 * µ0)) versteckt, und es bremst den Lichtstrahl aus: Bei dem tatsächlichen ε0 auf ca. 300.000 km/s.

  1002. #1005 MartinB
    9. September 2018

    @Anonym_2018
    “Das ε0 beschreibt, wie stark das Feld durch das Gegenfeld einer im Vakuum induzierten Polarisation abgeschwächt wird.”
    Nein. Nimm die erste Maxwell-Gleichung. eps_0 sagt dir, wie viel feld du für eine bestimmte Ladungsdichte bekommst. Dass da in der QED ne Renormierung dahinter steht, spielt dafür keine Rolle. (Sieht man ürbigens schon dran, dass man in der QED nur e renormiert und nicht auch noch epsilon_0, oder irre ich mich?)

    “es bremst den Lichtstrahl aus: Bei dem tatsächlichen ε0 auf ca. 300.000 km/s.”
    Nein. eps_0 ist die Kopplung zwischen Feld und Ladung, deshalb taucht es genau an diesen Stellen in den Maxwell-Gleichungen auf, wo man aus Ladunsgbezogenen Größen Feldgrößen berechnen will (bei rho und j).

    c taucht auf, weil wir räumliche und zeitliche Ableitungen vergleichen-

    Ich habe keine Ahnung, woher du das mit dem Abbremsen hast, aber es ist Blödsinn. Würde man eps_0 ändert, würde sich schlicht die Feldstärke ändern, die eine Ladung erzeugt, das sieht man an der Form der MG, die ich oben hingeschrieben habe, sofort. Du lässt dich schlicht vom Begriff “Vakuumpolarisation” für eps_0 irreführen, der einfach schlecht gewählt ist.

  1003. #1006 Anonym_2018
    9. September 2018

    Ich habe keine Ahnung, woher du das mit dem Abbremsen hast

    Das habe ich von der Analogie zwischen ε0 (polarisierbares Vakuum) und εr (polarisierbares Material, z.B. Wasser, mit Brechungsindex N= √εr). Das Material bremst Licht herunter auf c/N = c / √εr.

    Dabei ist mir klar, dass die Geschwindigkeit 1 / √ (ε0 * µ0) wegen der Lorentztransformation eine Sonderstellung einnimmt.

  1004. #1007 MartinB
    9. September 2018

    @Anonym_2018
    Dann vergiss das ganz schnel wirder, es ist wirklich nicht sinnvoll, das so zu betrachten. Von welchem Wert soll denn abgebremst werden? Unendlich?

  1005. #1008 MartinB
    9. September 2018

    PS: Im übrigen ist das mit der Abbremsung von Licht in Medien auch nur eine Vereinfachung, ein kleiner Teil des Signals breitet sich immer mit Vakuum-Lichtgeschwindigkeit aus, dazu steht was im Jackson Elektrodynamik. (Heißt im Deutschen “Sommerfeldscher Vorläufer”, englisch “first precursor”) Der ist nur extrem schwach und deswegen in jeder technischen Hinsicht irrelevant.

  1006. #1009 Anonym_2018
    9. September 2018

    Mir ist klar, das es nur eine philosphische Diskussion ist, ob ε0 oder c “fundamentaler” ist. Gibt es z.B. “Zeit”, falls es keine Uhren gibt? Da ε0 und c konstant sind, ändert sich ohnehin nichts an den Gleichungen, die sehr gute Ergebnisse liefern.

    Von welchem Wert soll denn abgebremst werden? Unendlich?

    zum Beispiel. Ich simuliere in Gedanken gerne auch unmögliche Szenarien, um ein besseres Verständnis der Zusammenhänge zu bekommen. Ich kann in einer Gedankensimulation eine Naturkonstante auf einen bestimmten Wert setzen und überlegen, wie sich das auswirken würde, z.B.:

    Maßnahme:
    h=0
    Auswirkungen: Die QM ist abgeschaltet. Die Materiewellen sind nicht quantisiert. Die Unschärferelationen sind weg. Die Vakuum-Polarisation ist verschwunden und damit ist auch ε0 = Null.

    Maßnahme:
    ε0 = 0
    Auswirkungen: Die Coulombkraft F ist = ∞, unabhängig vom Abstand r. Die Elektronen bewegen sich alle in die Protonen (Confinement). Die Vakuum-Lichtgeschwindigkeit 1 / √ (ε0 * µ0) ist = ∞. Die Lorentz-Transformation nimmt die Form der Galilei-Transformation an.

    Dann würden wir in folgender Welt leben:
    https://de.wikisource.org/wiki/Mathematische_Principien_der_Naturlehre

    Das war aber, wie gesagt, alles nur eine Simulation 🙂

  1007. #1010 MartinB
    9. September 2018

    @Anonym
    “Mir ist klar, das es nur eine philosphische Diskussion ist, ob ε0 oder c “fundamentaler” ist. ”
    Dann ist dir was falsches klar. Beide sind fundamental (siehe meine Form der Mg oben), die abgeleitete Größe in den MG ist mu_0. eps_0 ist die Feldtsärke einer Ladung, c ist der Umrechnungsfaktor zwischen Raum und Zeit.

    “zum Beispiel. ”
    Und wie würde das dann zur SRT passen und dazu, dass sich Gravitatonswellen für alle beobachter mit c ausbeiten? Das wäre logisch inkonsistent. mal abgesehen vom Vorläufer, breitet der sich dann auch mit unendlich aus und wir haben das nur noch nie gemerkt, weil … Gründe?

    Wenn eps_0 Null wäre, wären alle eltrkischen Felder unendlich groß, damit auch die Feldenergie und dann würde das Universum schneller kollabieren, als du Maxwell sagen kannst.

    In einem Universum, in dem es nur die elektromagnetische Wechselwirkung gäbe, wäre die Frage tatsächlich philosohpisch diskutierbar. Bei uns gibt es aber Gravitation und andere Wechselwirkungen, für die eps_0 irrelevant ist, c dagegen nicht.

    Tut mir Leid, aber in diesem Fall und unserem Universum gibt es wirklich nur einen richtigen und einen falschen Standpunkt…

  1008. #1011 Anonym_2018
    9. September 2018

    Bei uns gibt es aber Gravitation und andere Wechselwirkungen, für die eps_0 irrelevant ist

    Dann verstehe ich nicht, warum sich Gravitationswellen mit der Geschwindigkeit 1 / √ (ε0 * µ0), also abhängig von ε0, ausbreiten.

  1009. #1012 MartinB
    9. September 2018

    @Anonym
    Das tun sie nicht. Dein fehler ist, dass du die ganze zeit mit mu_0 arbeitest, es aber ignorierst.
    Nimm die Form der MG, die ich oben hingeschrieben habe, und du siehst, das c und eps_0 die unabhängigen größen sind, mu-0 lässt sich daraus ableiten. (Und ich habe genau das ja oben dezltich gemacht.)

    Dass man mu0 als Größe hat, ist rein historisch. Wenn man die MG sinnvoll schreibt, gibt es nur c und epsilon_0 – das eine für die Kopplung, das andere für die raumzeit, wie oben erklärt.

  1010. #1013 MartinB
    9. September 2018

    Sihest du auch daran, dass in Wahrheit E-und B-Felder ja nicht unabhängig sind, also kann es da auch nicht zwei Konstanten geben. Wie gesagt, mu_0 ist historischer Ballast. Feynman schreibt zum beispiel in den Lectures die Gleichungen auch nur mit eps_0 und c.

  1011. #1014 MartinB
    9. September 2018

    Zitat Feynman Kap. 18
    “We have called it c from the beginning, because we knew what it would turn out to be. We didn’t think it would be sensible to make you learn the formulas with a different constant and then go back to substitute c wherever it belonged. From the point of view of electricity and magnetism, however, we just start out with two constants, ϵ0 and c^2, that appear in the equations of electrostatics and magnetostatics”

  1012. #1015 MartinB
    9. September 2018

    Zu guter letzt: Dass man mu_0 und eps_0 nicht unabhängig variieren kann, sihest du auch daran, dass man das Magnetfeld eines stromdurchflossenen Drahtes über die Lorentzkontraktion der bewegten Elektronen beschreiben kann. mu_0 sagt, wie Ströme Magnetfelder erzeugen, eps_0 wie ladungen E-felder erzeugen. Da Magnetfelder aus bewegten ladungen kommen und man den Effekt der Bewegung über die lorentz-Trafo berechnen kann, hängt mu_0 zwangsläufig an eps_0

  1013. #1016 Anonym_2018
    9. September 2018

    Dein fehler ist, dass du die ganze zeit mit mu_0 arbeitest, es aber ignorierst.

    Ja. Aber meiner Ansicht nach wäre es physikalisch sinnvoller gewesen, es nicht zu ignorieren. Das wird aber durch das SI-Einheitensystem erschwert.

    Historisch hatte die Konstante μ 0 verschiedene Namen. Bis 1987 sprach man von der „magnetischen Permeabilität des Vakuums“.

    Der Wert und die Einheit der magnetischen Feldkonstanten (in SI-Einheiten) ergeben sich aus der Definition des Ampere als Einheit der Stromstärke.

    https://de.wikipedia.org/wiki/Magnetische_Feldkonstante

    Meiner Ansicht nach liegt das Problem darin, dass das SI-Einheitensystem seit 1948 das μ 0 nicht als echte Naturkonstante, die man messen könnte, einstuft, sondern deren Wert per Gesetz auf
    μ 0 := 4π * 10^-7 N/A²
    festgeschrieben hat, um mit ihm und der Einheit “Newton” die Einheit “Ampere” zu definieren.

    Entwas analoges ist 1983 mit c passiert: Das SI-Einheitensystem stuft c nicht mehr als echte Naturkonstante ein, die man messen könnte, sondern schrieb deren Wert per Gesetz auf
    c := 299 792 458 m/s
    fest, um mit ihr und der Einheit “Sekunde” die Einheit “Meter” zu definieren.

    ε0 kann man jetzt aus den gesetzlich definierten μ 0 und c berechnen und erhält auch dafür einen exakten Wert.

    Von diesen Formalitäten sollte man aber aus meiner Sicht abstrahieren, und alle 3 Konstanten weiterhin auf Abhängikeiten von der Natur hin zu untersuchen.

  1014. #1017 Anonym_2018
    9. September 2018

    Da Magnetfelder aus bewegten ladungen kommen und man den Effekt der Bewegung über die lorentz-Trafo berechnen kann, hängt mu_0 zwangsläufig an eps_0

    Nein. Grund: Dass das µ0 an dem ε0 hängt, kann man hier vermeiden, indem man in der Lorentz-Trafo anstelle von irgendeinem “c” den Term 1 / √ (ε0 * µ0) mit den verwendeten unabhängigen ε0 und µ0 einsetzt. Dann wird bei der Lorentz-Trafo in beiden Bezugssystemen das gleiche Ergebnis herauskommen.

  1015. #1018 Anonym_2018
    9. September 2018

    Ergänzung zu #1017

    Bei der Berechnung der Lorentz-Kontraktion sollte folgender Faktor “Gamma” verwendet werden, damit die Maxwell’schen Gleichungen Lorentz-invariant sind:
    γ = 1 / √ (1-(v² ε0 µ0)).

  1016. #1019 MartinB
    10. September 2018

    @Anonym_2018
    Ich gebe auf. Die Gleichungen sagen dir eindeutig, warum eps_0 und mu_0 nicht unabhängig sein können, spätestens das Argument mit der Lorentztrafo eines stromdurchflossenen Drahtes sollte die Sache eindeutig klären. Wenn das nicht reicht, dann bist du – ich kann es nicht anders sagen – argumentreistent. (Du lieferst ja auch keine Gegenargumente zur korrekten Schreibweise der Gleichungen, den Feynman-Zitaten, der Lorentztrafo eines Drahtes etc.)
    Da es keinen Sinn hat, dieselben Argumente ständig zu wiederholen, gebe ich auf.

  1017. #1020 Harti
    10. September 2018

    @MartinB
    “Die Gleichungen sagen dir eindeutig, warum eps_0 und mu_0 nicht unabhängig sein können.”

    Kann ich mir den Zusammenhang in einem Raumzeitmodell auch so vorstellen, dass : Je größer der zeitliche Anteil einer Veränderung ist, umso geringer ist der räumliche Anteil und umgekehrt. Dies wird wird auch bei der Berechnung von Raumzeitintervallen deutlich.
    Bleibt nur die Frage, warum dies so ist.
    Meine Antwort: Auf diese Weise wird die Beschränkung unserer Wahrnehmungs-/Informationsmöglichkeit durch den Elektromagnetismus erreicht. Kann man so auch die gegenseitige Abhängigkeit von eps_0 und mu_0 interpretieren ?
    Entschuldigung ! Aber die Ansicht, dass ein mathematisches Modell (die Raumzeit) zu einer physikalischen Entität wird und Wirkungen erzeugt, kann ich nicht nachvollziehen.

  1018. #1021 MartinB
    10. September 2018

    @Harti
    Es ist wirklich simpel: Es gibt nur eine Kopplungskonstante zwischen Ladung und em-Feld. Je nach Bewegung der Ladung erzeugt die elektrische oder magnetische Felder, es ist aber dieselbe Kopplungskonstante.

    “Je größer der zeitliche Anteil einer Veränderung ist, umso geringer ist der räumliche Anteil und umgekehrt. ”
    Das ist falsch (oder du meinst was anderes als du sagst): Bei einem Vierervektor werden die Raum- und Zeit-Komponenten bei der Längenbestimmung voneinander abgezogen, wenn du einen Vektor, der nur ne Zeitkomponente hat, rotierst, wird die Zeitkomponente größer (um den passenden Lorentz-Faktor), wie bei der Vierergeschwindigkeit.

    Ansonsten ist die Aufgabe der Physik, die Welt so gut zu beschreiben, wie es geht, und wenn das am einfachsten geht, indem man annimmt, dass es eine (gekrümmte) Raumzeit gibt, dann macht man das so. Ganz ehrlich, auch wenn es hart klingt: Was du (oder ich) dir vorstellen oder was du nachvollziehen kannst, ist letztlich egal, relervant ist, mit welchem Modell wir die Welt am besten beschreiben können.

  1019. #1022 PS
    10. September 2018

    @MartinB #968
    Ich habe oben (#949) behauptet, dass die „Lichtgeschwindigkeit“ (Lichtausbreitung; Wirkungsausbreitung) das Gegenteil von Geschwindigkeit eines Materiepunktes ist. Dass sie der Rahmen ist, in dem die Begriffe Zeit, Raum und Geschwindigkeit entwickelt werden können. Dass sie ein Maß dafür ist, wie die Zeit vergeht und wie weit der vom Licht durchmessene Raum ist.
    Du hast dagegen Einwendungen erhoben: während ich betont habe, dass beides – Zeit und Raum – mit der Lichtausbreitung (Wirkungsausbreitung) gemessen wird, hast Du betont, dass die Konstante (#956), auf die man sich zuvor einigen muss, nicht durch die Lichtausbreitung festgelegt werden kann.
    Ich gebe zu, dass ich zu diesem Missverständnis beigetragen haben könnte, indem ich sagte, “sie [die Lichtausbreitung] sei ein Maß dafür ….”, weil die Lichtausbreitung als solche natürlich nicht die Konstante in sich trägt, auf die wir uns einigen müssen, bevor jene uns zum Maß werden kann.

    Das Maß für Raum und Zeit ist aber mit dem Maß der absolut begrenzten Wirkungsausbreitung (mit der “Länge eines Lichtstrahls”) ident.
    Wir finden nicht zuerst “Raum” und “Zeit” vor und stellen dann fest, wie “schnell” die Wirkungsausbreitung (die “Lichtgeschwindigkeit”) ist, sondern die Wirkungsausbreitung ist die Voraussetzung dafür, dass wir “Zeit” und “Raum” als messbare Größen definieren können.
    Das ist die Quintessenz einer geistigen Entwicklung, die in der Etablierung eines Systems natürlicher Einheiten (Planck Einheiten) bzw. auch des SI Systems (#961), in dem die willkürlich festgelegte Sekunde über die “Lichtgeschwindigkeit” den Meter definiert, ihren Abschluss gefunden hat. “Warum” die Wirkungsausbreitung so ist wie sie ist, ist eine philosophische Frage, die wir nicht beantworten müssen, um Physik zu betreiben.

    Wenn wir aber Zeit und Raum nur über die Wirkungsausbreitung definieren, dann bleibt der Begriff der “Geschwindigkeit eines massebehafteten Teilchens”, nicht der, den wir in der Schule gelernt haben. “Geschwindigkeit” ist nicht mehr ein “Weg”, den der Materiepunkt in einer davon unabhängigen “Zeit” zurücklegt.
    Bei der Wirkungsausbreitung ist die räumliche Entfernung der durch sie bewirkten Ereignisse gleich ihrer zeitlichen Entfernung voneinander (“lichtartig”). Wenn wir Zeit und Raum von Ereignissen mit der eben definierten Methode messen, dann ist die räumliche Entfernung zweier Ereignisse, die in der Anwesenheit eines bewegten Materiepunktes bestehen, immer kleiner als ihre zeitliche Entfernung voneinander (“zeitartig”).

    Die (Koordinaten)Geschwindigkeit eines Materiepunktes ist nicht mehr Weg/Zeit, sondern ein Bruchteil der Wirkungsausbreitung (% von c). Damit verliert die Frage, warum ein Materiepunkt nicht schneller sein kann als das Licht, ihren Sinn.

    Die Frage,

    @Anonym #992

    Ich verstehe nicht, wie eine in der Raumzeit “eingebaute” Maximalgeschwindigkeit einen bestimmten Wert für die Polarisierbarkeit des Vakuums (ε0) erzwingen kann. Wie kann man so etwas erklären?

    trägt ihre Antwort insofern in sich, als sich solche Fragen diejenigen stellen, die unter Geschwindigkeit “Weg/Zeit” und nicht einen Bruchteil der Wirkungsausbreitung c verstehen.

    @MartinB #986 sagt richtig:

    c ist eine fundamentale Umrechnungskonstante zwischen räumlichen und zeitlichen Distanzen; dass sich Licht (wie alle masselosen Teilchen) mit dieser Geschwindigkeit bewegt, ist eine Folgerung aus dieser fundamentalen Struktur der Raumzeit.

    Die Struktur der Raumzeit wird aber – zumindest für den Bereich der SRT – durch die (nur philosophisch, und nicht durch irgendwelche Eigenschaften des Vakuums begründbare) Wirkungsausbreitung bestimmt bzw. beide sind ein- und dasselbe.

  1020. #1023 MartinB
    10. September 2018

    @PS
    Ich glaube nicht, dass ich verstehe, was du da schreibst – dass wir dank der Lichtgeschwindigkeit Strecen und Zeiten ineinander umrechnen können, sage ich ja die ganze Zeit.
    “Das Maß für Raum und Zeit ist aber mit dem Maß der absolut begrenzten Wirkungsausbreitung (mit der “Länge eines Lichtstrahls”) ident.”
    Wie du selbst schreibst, kann man nur eins von beidem mit der Lichtgeschwindigkeit festlegen, entweder das Maß von Raum oder von Zeit.
    Ich erkläre es mal mit ner Analogie: Du kannst strecken in Ost-West- und Nord-Süd-Richtung messen. Die lassen sich trivial ineinander umrechnen, man braucht also nicht zwei Maße für die beiden Richtungen, sondern nur eins. So ist es auch in der Raumzeit.

    ” die Wirkungsausbreitung ist die Voraussetzung dafür, dass wir “Zeit” und “Raum” als messbare Größen definieren können”
    Ob nun c fundamentaler ist als Zeit und Raum oder andersherum, halte ich wirklich für eine müßige Frage, weil das nichts ist, was sich irendwie sinnvoll argumentieren lässt, soweit ich sehe.

    “Wenn wir aber Zeit und Raum nur über die Wirkungsausbreitung definieren”
    Was wir wie gesagt nicht tun können. Oder du wirst mal konkret: Wie definierst du “Raum und Zeit” über die Wirkungsausbreitung? Und zwar quantitativ? Dazu brauchst du doch, wie du selbst sagst, eine Bezugsstrecke (wie deine Spiegel) – und die legst du eben nicht über die Wirkungsausbreitung fest.

    “Wenn wir Zeit und Raum von Ereignissen mit der eben definierten Methode messen”
    Welche Methode ist das? Verstehe ich nach wie vor nicht.

  1021. #1024 PS
    10. September 2018

    was Du da schreibst …

    Aus allen Deinen Stellungnahmen geht hervor, dass Du “(ein Maß) mit der Lichtgeschwindigkeit festlegen” verwechselst mit “(einen Abstand) mit der Lichtgeschwindigkeit messen”.

    Auf #951 “dass die Länge und die Zeit letztlich nur mit dieser Lichtausbreitung gemessen werden kann”, antwortest Du schon zu Beginn #951 “Man kann auch nicht sowohl Längen als auch Zeiten mit einer Geschwindigkeit festlegen”.

    Das von Dir immer ins Spiel gebrachte “mit der Lichtgeschwindigkeit (die Maßeinheit) festlegen” beschreibt den physikalischen Sachverhalt der Festlegung nicht korrekt, denn zum Festlegen braucht man zweierlei: erstens die materielle Konstante (entweder in der Zeit – Pulsar – oder im Raum – Länge der Lichtuhr -), die man für die Festlegung wählt, und zweitens einen Lichtstrahl, mit dessen Länge man diese materielle Konstante misst und zur Grundlage der Maßeinheit macht. Du hast nur die zweite Komponente der Messung im Blick und setzt dann die nur halb gedachte Festlegung mit Messung gleich.

    Mit dem Lichtstrahl wird also immer etwas gemessen (auch die materielle Konstante), nicht etwas festgelegt, wie Du immer schreibst.

    Die Behauptung

    man kann nicht Raum und Zeit mit der Lichtausbreitung messen, weil man nur entweder den Raum oder die Zeit mit einem Lichtstrahl festlegen kann

    ist infolge Gleichsetzung der Begriffe Festlegung und Messung ein Fehlschluss.

  1022. #1025 MartinB
    10. September 2018

    @PS
    Was meinst du denn mit “ein Maß festlegen”. Für mich heißt das, etwas festlegen, womit ich die Größe messen kann. Und wenn du eine “materielle Konstante” verwendest, dann legst du genau mit dieser einen Maßstab fest. Die durchquerst du dann mit dem Lichtstrahl und dann kannst du damit auch zeitliche Abstände messen. Aber zentral bei der Konstruktion ist die “materielle Konstante”, ohne die geht da nichts.

    MaW: Die materielle Konstante (der Abstand der Spiegel) legt einen räumlichen Abstand fest. Den kannst du als Urmeter verwendet, und zwar mit jeder beliebigen Methode zur Längenmessung, das muss kein Lichtstrahl sein. Der Lichtstrahl erlaubt dir dann, daraus einen Zeitmaßstab zu konstruieren.

    Un begrifflich scheint mir das hier:
    ” mit dessen Länge man diese materielle Konstante misst und zur Grundlage der Maßeinheit mach”
    ein Widerspruch zu sein, du kannst nicht gleichzeitig von “messen” und von “Grundlage der Maßeinheit” reden – zum Messen muss die Maßeinheit ja schon da sein, sonst ist es keine Messung (sondern eben eine Festlegung).

  1023. #1026 MartinB
    10. September 2018

    Um es nochmal anders zu sagen: Wenn du zum Beispiel eine Entfernung “mittels Licht” misst, dann brauchst du dazu das Lichtsignal und eine geeichte Uhr. (Oder ne geeichte Strecke, aus der du dann ne Lichtuhr baust.) Deshalb halte ich es für irreführend, zu sagen, dass du etwas “nur mit Lichtausbreitung” messen kannst, weil du eben zusätzlch noch deine Uhr oder dein Urmeter brauchst, ansonsten gibt es keine Messung. Während ich andersherum eine Länge schlicht direkt mit einem geeichten Maßstab (Lineal) messen kann, ohne Rückgriff auf etwas zweites. Und genau deshalb halte ich dein Eingangs-Statement
    “Man kann den Kleinen erklären, dass die Länge und die Zeit letztlich nur mit dieser Lichtausbreitung gemessen werden kann”
    für falsch. Es geht nicht “nur” mit der Lichtausbreitung (in beiden Bedeutungen von “nur” – weder geht es mit nichts anderem, noch geht es mit Lichtausbreitung allein und sonst nichts), du brauchst immer etwas zweites, das als Maßstab dient.

  1024. #1027 Anonym_2018
    10. September 2018

    @ MatinB (10. September 2018) #1019

    Du lieferst ja auch keine Gegenargumente zur korrekten Schreibweise der Gleichungen, den Feynman-Zitaten, der Lorentztrafo eines Drahtes etc.

    Das hole ich hiermit gernen nach.

    Die drei Größen ε0, µ0 und c hängen laut den Maxwell’schen Gleichungen zusammen über
    c = 1 / √ (ε0 * µ0). Daher können nur zwei dieser drei Größen echte Naturkonstanten sein. Die dritte Größe kann man dann aus diesen beiden Naturkonstanten ausrechnen. Wäre die dritte Größe auch eine Naturkonstante, wäre sie überbestimmt, analog zu einem Tisch mit 4 Beinen, der wackelt, im Vergleich zu einem Tisch mit 3 Beinen, der nicht wackelt.

    Laut dem o.a. Feynman-Zitat hat er das µ0 eliminiert und will das c behalten. Eine stichhaltige Begründung, warum er c behalten will, hat er in dem obigen Zitat leider nicht genannt. Ich vermute, dass auch er von einer Geometrie mit “eingebauter” Maximalgeschwindigkeit ausgeht.

    Mein Gegenargument: Die z.B. durch den Lamb-Effekt bewiesene Polarisierbarkeit des Vakuums muss laut der Gleichung
    c = 1 / √ (ε0 * µ0) die Lichtgeschwindigkeit bestimmen, von der z.B. die Stärke der Lorentz-Kontraktion des Drahtes abhängt.

    Diese Lorentz-Kontraktion des Drahtes könnte rein elektromagnetisch verursacht sein. Man braucht dann keine geometrischen Argumente für die Existenz der Lorentz-Kontraktion. Die Länge des Drahtes wird bestimmt durch die elektromagnetische Wechselwirkung zwischen den Elektronen und Atomkernen, aus denen der Draht besteht. Aus Sicht eines relativ zum Draht bewegten Beobachters erzeugen diese Elektronen und Atomkerne Magnetfelder, die den Draht entsprechend verkürzen. Wäre das denkbar?

  1025. #1028 PS
    10. September 2018

    @MartinB #1025

    Und begrifflich scheint mir das hier: ”mit dessen Länge man diese materielle Konstante misst und zur Grundlage der Maßeinheit macht” ein Widerspruch zu sein …

    Dein Einwand ist korrekt, weil das noch kein “Messen” (und noch weniger ein “Festlegen”) ist. Es hätte heißen müssen:
    “und zweitens einen Lichtstrahl, der diese materielle Konstante durchmisst …” (in dem Sinn von: “sich von einen Ende der Lichtuhr zum anderen ausbreitet”).

    Es ist ein Treppenwitz, dass Dir einen Absatz vorher die gleiche Ungenauigkeit unterlaufen sein dürfte:

    Die materielle Konstante (der Abstand der Spiegel) legt einen räumlichen Abstand fest. Den kannst du als Urmeter verwendet, und zwar mit jeder beliebigen Methode zur Längenmessung, das muss kein Lichtstrahl sein.

    Auch Du lässt den Abstand der Spiegel “mit jeder beliebigen Methode” messen, obwohl Messen die zu schaffende Messeinheit voraussetzen würde.

    Zu #1026:

    Wenn du zum Beispiel eine Entfernung “mittels Licht” misst, dann brauchst du dazu das Lichtsignal und eine geeichte Uhr.

    Das ist richtig.

    Früher wurde die Länge einer Hose durch Vergleich mit der Schneiderelle bestimmt.

    Jetzt wird die Länge eines zu messenden lichtartigen Abstands zwischen zwei Ereignissen (die “Länge des Lichtstrahls”) durch Vergleich dieser Länge mit der gewählten Maßeinheit “lichtartiger Abstand der Lichtuhr” bestimmt. Wir messen zeitliche und räumliche Abstände nicht mit der materiellen Konstante selbst (mit einem starren Maßstab), sondern mit einem frei festzulegenden lichtartigen Abstand zweier Ereignisse voneinander (dessen Festlegung einer materiellen Konstante bedarf).

    Die materielle “Länge der Hose”, die es früher zu messen galt und an die man einen materiellen Maßstab anlegte, gibt es als klar definierte Größe nicht mehr. Stattdessen gibt es Ereignisse und “die Länge eines Lichtstrahls” (die lichtartige Entfernung), die man mit einem lichtartigen Maßstab vergleicht (“man legt den Lichtstrahl in der Lichtuhr an den zur Messung verwendeten Lichtstrahl an”).

    “Mit der Lichtausbreitung messen” als irreführend zu bezeichnen, weil ich es unterlasse, die selbstverständlich dazu notwendige Lichtuhr zu erwähnen, ist spitzfindig, findest Du nicht auch?.

    Man sollte den Kleinen daher erklären, “dass die Länge und die Zeit letztlich nur mit dieser Lichtausbreitung gemessen werden kann”. Wenn Du mein “letztlich nur” als ein “nur” im Sinne von “ausschließlich” liest, veränderst Du meine Aussage und verlierst Dich in Unwesentlichem.

  1026. #1029 Anonym_2018
    11. September 2018

    @ MarintB (8. September 2018) #987

    Auch die Gravitation breitet sich mit c aus, unabhängig davon, ob es Gravitonen gibt oder nicht. Gravitation hat nun wirklich sehr wenig Wechselwirkung mit Photonen, warum sollte für sie trotzdem die Dielektrizitätsknstante des Vakuums irgendwie relevant sein?

    Der Physiker Lorentz hatte dazu eine interessante Idee:

    Der erste Versuch, auf Basis der maxwellschen Elektrodynamik die Gravitation zu erklären, stammt von Hendrik Lorentz (1900). Dabei ging er (wie vor ihm Mossotti und Zöllner) von der Vorstellung aus, dass die Anziehung zweier ungleichnamiger elektrischer Ladungen um einen Bruchteil stärker sei als die Abstoßung zweier gleichnamiger Ladungen. Das Ergebnis wäre nichts anderes als die universelle Gravitation. Lorentz konnte zeigen, dass auch diese Theorie von der laplaceschen Kritik nicht betroffen ist und nur Einflüsse in der Größenordnung v²/c² auftreten, jedoch erhielt er für die Periheldrehung einen viel zu geringen Wert. Lorentz fasste seine Bemühungen folgendermaßen zusammen:

    „Die besondere Form dieser Terme kann möglicherweise modifiziert werden. Doch was bis jetzt gesagt wurde reicht aus um zu zeigen, dass die Gravitation auf Aktionen zurückgeführt werden kann, welche sich nicht schneller als mit Lichtgeschwindigkeit ausbreiten.“

    Quelle:
    https://de.wikipedia.org/wiki/Aberration_(Gravitation)

  1027. #1030 MartinB
    11. September 2018

    @Anonym
    ” Eine stichhaltige Begründung, warum er c behalten will, hat er in dem obigen Zitat leider nicht genannt.”
    Die habe ich dir aber geliefert.
    Selbst wenn das Argument mit dem Lamb-Effekt richtig wäre (was es meiner Ansicht nach nicht ist), wäre es ein Argument, das sich nur auf eps_0 bezieht, aber kein Argument dafür, warum mu_0 unbedingt konstant sein muss. Und im Zweifel kannst du einfach die Lagrange-Funktion der QED hinschreiben, wenn du unbedingt QFT betreiben willst – da gibt es nur eine Kopplungskonstante zwischen Ladung und Feld, nicht zwei, also kann es in den klassischen MG nicht eine für das E- und eine für das B-Feld geben.

    Die Idee von Lorentz stammt soweit ich weiß aus der Zeit vor der RT – die ist wirklich nur noch historisch von Interesse.

    @PS
    “Auch Du lässt den Abstand der Spiegel “mit jeder beliebigen Methode” messen, obwohl Messen die zu schaffende Messeinheit voraussetzen würde.”
    Richtig. Aber ich darf das – denn im Gegensatz zu dir gehe ich davon aus, dass ich einen Längenmaßstab (Urmeter) definiert habe (oder einen Zeitmaßstab, um daraus den Längenmaßstab über c zu bekommen).

    “mit einem frei festzulegenden lichtartigen Abstand zweier Ereignisse voneinander (dessen Festlegung einer materiellen Konstante bedarf). ”
    Genau das sage ich doch die ganze Zeit, du brauchst ein Urmeter, mit dem du vergleichst.

    “letztlich nur” ist doppeldeutig, insofern ist das deine sprachliche Ungenauigkeit:
    Es kann entweder heißen, dass es keine andere Methode gibt oder dass man nicht außer der Lichtausbreitung zum Messen braucht. Da beides falsch ist (es gibt Geodreiecke und man braucht ne materielle Konstante) ist das aber letztlich egal.

    Wie war denn das “letztlich nur” gemeint?

  1028. #1031 Harti
    11. September 2018

    @MartinB und PS
    Ich nehme an, ihr seid euch darüber im Klaren, dass eure Diskussion nicht auf der Grundlage eines Raumzeitmodells erfolgt. Zeit (Uhren) und Raum (Streckenmaßstab) sind in der Raumzeit nicht getrennt darstellbar. Sie sind nur Schatten der Raumzeit (Minkowski). Es gibt daher auch keine Beziehung zwischen beiden (Geschwindigkeit). Eine Lichtgeschwindigkeit ist daher nur in einem Weg-/Zeitdiagramm darstellbar. Die Vermengung der beiden unterschiedlichen Modelle ist einer der Hauptgründe für Missverständnisse.

  1029. #1032 PS
    11. September 2018

    @MartinB

    Zwei materielle Konstanten (Urmeter als Teil des Erdumfangs und Sekunde als Teil der Erdumdrehung) bzw. deren beliebige Duplikate waren die Maßstäbe, mit denen man durch direktes “Anlegen” (Vergleich der Länge bzw. des Prozesses mit der dazugehörigen materiellen Konstante) Messungen vornahm.

    Dieses Modell verteidigst Du immer noch. Für mich ist bewundernswert, wie Du mit diesen Mühlsteinen um den Hals als Physiker so weit gekommen bist.

    Diese beiden materiellen Konstanten, die Schneiderelle und die Pendeluhr, haben nichts miteinander zu tun. Sie standen und stehen für den absoluten Raum und für die absolute Zeit. Ihre weitere Verwendung bzw. die Nichtbeachtung, dass die Maßstäbe innerlich verknüpft sein müssen, sorgt für eine Lawine von Missverständnissen.

    @Harti

    Zeit (Uhren) und Raum (Streckenmaßstab) sind in der Raumzeit nicht getrennt darstellbar. Sie sind nur Schatten der Raumzeit (Minkowski).

    Ganz genau.
    In dieser Raumzeit haben materielle Konstanten eine ganz andere Funktion. Sie sind nicht mehr die beiden Maßstäbe, die wir zur Hand nehmen, um mit ihnen selbst (die absolute Zeit und den absoluten Raum) zu messen.
    Sie bilden jetzt vielmehr das unverzichtbare Bindeglied zwischen materiellen Welt und der Lichtausbreitung (allgemeiner: Wirkungsausbreitung). Wir definieren mit irgendeiner materiellen Konstante einen lichtartigen Abstand zwischen zwei Ereignissen. Es ist egal, ob sie in einer räumlichen Ausdehnung (Länge) oder in einer zeitlichen Ausdehnung (Prozessdauer) besteht. Würde man irgendeine andere Anbindung vornehmen (eine zweite materielle Konstante heranziehen), schüfe man einen anderen lichtartigen Abstand, der linear in den ersten umgerechnet werden kann.

    Der gewonnene lichtartige Abstand steht dann für den Raum und für die Zeit. (Das siehst Du ja – ohne Dir darüber Rechenschaft abzulegen – genau so, weist Du Du doch mehrfach darauf hin, dass man aus einem Zeitmaßstab über c einen Längenmaßstab bekommt).

    Der lichtartige Abstand ist nun der neue Maßstab (siehe oben). Wir benötigen nur mehr einen. In ihm sind Raum und Zeit untrennbar verknüpft. In dieser intrinsischen Verknüpfung besteht die Überlegenheit der Messung mit lichtartigen Abständen gegenüber der althergebrachten Messung 1. mit Schneiderelle und 2. mit Pendeluhr.

    Man sollte schon von Kindesbeinen an darauf vorbereitet werden, relativistisch zu denken. Das beginnt mit der Einsicht, dass Zeit und Raum “letztlich nur” (dh, wenn man der Sache auf den Grund geht) mit einem Lichtstrahl gemessen werden. Heute bekommt man Lasermessgeräte im Baumarkt zu kaufen. Was ist denn die Messung mit diesem Instrument anderes, als lichtartige Abstände miteinander zu vergleichen?

    Das Potential dieser Denkweise bzw. Wissen um die Grundlagen der Messung vom Raum und Zeit entfaltet sich, wenn Ereignisse aus der Sicht zweier zueinander bewegter Systeme gemessen werden. Da fällt die L-T von selbst vom Baum.

  1030. #1033 MartinB
    11. September 2018

    @Harti
    Natürlich sind zeiten nd Strecken was anderes, auch in der Raumzeit. Dass man beide vereinen kann, heißt nicht, dass es nicht sinnvoll ist, z.B. Strecken mit einem Lineal oder Zeitenmit einer Uhr zu messen (so hat Einstein immerhin die Zeit definiert).

    @PS
    “Dieses Modell verteidigst Du immer noch. ”
    Nein, Aber ich sage, dass man *einen* Maßstab braucht, der nicht die Lichtausbreitung ist, wie du ja auch. Dass man aus ner Uhr nen Längenmaßstab ableiten kann und umgekehrt, heit nicht, dass man beide nicht mehr braucht. (Schau mal, wie so ein Lasermessgerät funktioniert, da wird eine Laufzeit gemessen und dafür braucht man eine geeichte Uhr, keinen weiteren Lichtstrahl.)

    Es hat aber keinen Sinn immer dieselben Argumente zu wiederholen, wenn darauf nicht eingegangen wird.

    Ist aber auch egal – langsam wird mir meine Zeit zu schade dafür.

  1031. #1034 PS
    11. September 2018

    Du verdrehst mein Argument in das Gegenteil. Ich sagte, die Lichtausbreitung (ein durch zwei Ereignisse einer materiellen Konstante definierter lichtartiger Abstand) ist der Maßstab. Die Schneiderelle kannst Du in den Schrank der Physikgeschichte stellen, ebenso die Pendeluhr, auf deren kleinen Zeiger Einstein blickte.

    Für das Lasermessgerät wird eine geeichte Uhr und kein weiterer Lichtstrahl benötigt? Jede Uhr ist eine unvollkommene Nachbildung der geeichten Lichtuhr. Jede Pendeluhr reproduziert den definierten lichtartigen Abstand mit periodischen Vorgängen (mit einer Abfolge von Ereignissen am selben Ort). Im Lasermessgerät wird daher im Grunde die Länge des Signalstrahls mit dem genormten lichtartigen Abstand verglichen, auch wenn dies (unvollkommen) mit Hilfe einer Uhr geschieht.

    Ja bitte, möge Dir Deine Zeit zu schade dafür sein, mit dieser Art von Argumenten hartnäckig am anderen vorbeizureden.

  1032. #1035 MartinB
    11. September 2018

    @PS
    Ich verdrehe es nicht, ich verstehe es schlicht nicht.
    Ich verstehe dich so:
    Wir bauen an einem Ort zwei Spiegel in einem willkürlichen, aber festen Abstand. Deren Abstand nennen wir 1 (oder irgendne andere Zahl). Daraus können wir dann eine Zeiteinheit definieren, über die Zeit, die ein Lichtstrahl braucht, um von einem Spiegel zum anderen zu kommen. Und damit können wir dann (beispielsweise über Lichtsignale) diese Zeiteinheit jemand anderes mitteilen (der relativ zu uns in Ruhe ist, sonst geht’s na klar nicht), der daraus seine Zeiteinheit baut und dann bei sich zwei entsprechende Spiegel konstruieren kann, deren Abstand dann auch gleich 1 ist. Ist das das, was du meinst?

  1033. #1036 PS
    11. September 2018

    Die zwei Spiegel sind ein Beispiel für eine materielle Konstante, mit der sich im Prinzip zwei Ereignisse mit demselben lichtartigen Abstand verlässlich reproduzieren lassen. Die Zeiteinheit ist der lichtartige Abstand zwischen diesen Ereignissen. Die Raumeinheit ist der lichtartige Abstand zwischen diesen Ereignissen.

    Damit haben wir einen für Raum und Zeit gleichermaßen geltenden Maßstab (eine Lichtuhr bestimmter Länge) geschaffen.

    Wie man diesen Maßstab vervielfältigt und wie man mit ihm praktisch hantiert, ist für die Theorie von untergeordneter Bedeutung. Die von Dir vorgeschlagene Signalübertragung funktioniert im Prinzip ebenso wie der Nachbau der geeichten Lichtuhr (entsprechend einer Kopie des “Urmeters”).

    In der Praxis wird man den zur Messeinheit auserkorenen lichtartigen Abstand zwischen zwei Ereignissen durch einen zeitartigen Abstand zwischen zwei Ereignissen, die aus der Sicht des betreffenden Bezugssystems am selben Ort stattfinden, substituieren. Diese zwei Ereignisse werden durch einen reflektierten Lichtstrahl verbunden, dessen Gesamtlänge einen lichtartigen Abstand bildet. Mit dem 9.192.631.770-fachen der Periode einer Welle, die mit einem ausgewählten Übergang zwischen zwei Energieniveaus im Caesiumatom in Resonanz ist, hat man einen hinreichend exakten und stabilen lichtartigen Abstand definiert, der für die Messung von zeitlichen und räumlichen Abständen gleichermaßen verwendet werden kann.

  1034. #1037 MartinB
    11. September 2018

    @PS
    “Wie man diesen Maßstab vervielfältigt und wie man mit ihm praktisch hantiert, ist für die Theorie von untergeordneter Bedeutung. ”
    Nein, das ist genau das entscheidende.

    “Mit dem 9.192.631.770-fachen der Periode einer Welle, die mit einem ausgewählten Übergang zwischen zwei Energieniveaus im Caesiumatom in Resonanz ist, hat man einen hinreichend exakten und stabilen lichtartigen Abstand definiert,”
    Und das ist doch genau, was ich die ganze zeit sage: man braucht einen externen Maßstab, mit dem man vergleicht. Allein mit Lichtsignalen geht es nicht.

    Und das ist nicht von untergeordneter Bedeutung, sondern entscheidend. Genauso wie man auf der Ebene problemlos strecken vergleichen kann, aber um sie zu messen (ihnen eine vergleichbare Länge zuzuweisen) braucht man einen Maßstab. (Und spätestens wenn du dich fragst, wie du den Abgleich mit einem bewegten Beobachter machen willst, siehst du, dass es alles andere als einfach ist.)

    Und genau das und nichts anderes sage ich die ganze Zeit. Das Problem einfach als “von untergeordneter Bedeutung ” wegzuerklären, ist zu einfach. Und wenn du dann – um auf deine Eingangsaussage zurückzukommen – den Kindern erklärst “Ja, wir können alles nur mit Licht messen” und dann auf Nachfrage dazu sagst” “Naja, wir baruchen natürlich noch ein Cäsium-Atom als Vergleichsmaßstab”, dann ist das erstens wenig erhellend und zweitens kannst du dann genausogut gleich das Geodreieck nehmen, von dem ich die ganze Zeit rede.

  1035. #1038 PS
    12. September 2018

    Ich brauche das Cäsium-Atom nicht als “Vergleichsmaßstab”, sondern als eine materiell stabile Konstante. So weit waren wir schon (#956; #968: “Und damit ist es eben nicht möglich, allein mit Hilfe der Lichtausbreitung Längen und Zeiten zu definieren, wie du eingangs gesagt hast. Man muss zusätzlich eine Definition entweder einer Längeneinheit (Abstand der Spiegel) oder einer Zeiteinheit (Frequenz des Signals) festlegen.”

    Einen einzigen (“entweder … oder …”) reproduzierbaren Vorgang schaffen heißt nicht, ihn auch als Maßstab zu verwenden. Das kannst Du machen, musst Du aber nicht und wirst es auch nicht, wenn es Dein Ziel ist, einen für Raum und Zeit gleichermaßen verwendbaren Maßstab zu schaffen.

    Was uns unterscheidet ist, dass Du weiterhin zwei materielle Konstanten selbst (Uhr und Geodreieck) als Maßstäbe ohne innere Verbindung verwendest bzw. zu verwenden behauptest (siehe oben die Umrechnung mit c), während ich aus einer einzigen materiellen Konstante, die selbst nicht der gesuchte Maßstab ist, einen lichtartigen Abstand als Maßstab für Zeit und Raum ableite.

    Wenn ich Kindern erkläre, “Ja, wir können alles nur mit Licht messen”, dann stimmt das, auch wenn ich ihnen den Vorgang der Eichung nicht im Detail erkläre. Für die Kinder genügt es, sich einen Lichtstrahl mit bestimmter Länge in einer Lichtuhr vorzustellen, mit dem sie durch Vergleich mit der Länge eines Signallichtstrahls zeitliche und räumliche Abstände zwischen Ereignissen messen können.

    Die Vorteile dieses einheitlichen Maßstabs sind vielfältig. Das verwirrende c2 fällt aus den Formeln heraus. Man kann neben dem Minkowski Diagramm zur besseren Anschaulichkeit ein Bezugssystem nur mit räumlichen Koordinaten verwenden, in dem der räumliche und der zeitliche Abstand zwischen Ereignissen in der Länge eines Lichtstrahls zum Ausdruck kommt, der sie verbindet.

    Der Lichtstrahl ist Ortsvektor und zeitliche Koordinate zugleich. Jedem Lichtstrahl des Systems S in beliebiger Richtung (jedem Abstand zwischen Ereignissen aus Sicht des System S) entspricht einem einzigen Lichtstrahl des Systems S’, mit einer anderen Richtung (Aberration des Lichts) und einem anderen räumlich-zeitlichen Abstand (einem anderen Abstand zwischen diesen Ereignissen aus Sicht des System S’).

    Die Gemeinsamkeit der beiden Lichtstrahlen in S und S’ besteht darin, dass sich zwei Kugelwellen aus Licht, die bei Ursprungsdeckung in S und S’ ausgesendet werden, gemeinsam ausbreiten. Aus diesem Grundsatz folgt die L-T. Dann wird es für die Kinder selbstverständlich, dass der Stand einer synchronisierten Uhr des Systems S vom Stand der synchronisierten Uhr des Systems S’, die ihr begegnet, abweichen muss, usw, usw.

    Unsere Diskussion ist sicher auch für Dich ermüdend, das meine ich nicht als Kritik, aber wir drehen uns im Kreis, leider jeder in seinem eigenen. Vielleicht müssen wir es dabei bewenden lassen.

  1036. #1039 Anonym_2018
    12. September 2018

    MartinB (11. September 2018) #1030

    Die habe ich dir aber geliefert.

    Das stimmt. Besonders gut gefällt mir die Umschreibung der Maxwell’schen Gleichungen in #999 und #1001. Dass Feyman von unabhängigen Naturkonstanten c und ε0 ausgeht, liegt wahrscheinlich daran, dass er in seiner QED das Licht als Teilchen beschreibt. In diesem Blickwinkel sieht man kein Magnetfeld, sondern nur die Wechselwirkungsstärke (Feinstrukturkonstante mit Abhägigkeit von ε0) und die Teilchengeschwindigkeit.

    Die Idee von Lorentz stammt soweit ich weiß aus der Zeit vor der RT – die ist wirklich nur noch historisch von Interesse.

    Das sehe ich anders. Meiner Ansicht nach war Lorentz genial. Er hat z.B. – wenn auch in einem nicht-relativistischen Kontext – die “Lorentz-Kontraktion” gefunden und damit eine wichtige Vorarbeit geleistet, auf die Einstein bei der Entwicklung der SRT aufsetzen konnte. Seien Idee von einer elektromagnetischen Ursache der Gravitation sollte man aus meiner Sicht aufgreifen und prüfen, ob sich das in die ART integrieren lässt.

  1037. #1040 MartinB
    12. September 2018

    @PS
    “Ich brauche das Cäsium-Atom nicht als “Vergleichsmaßstab”, sondern als eine materiell stabile Konstante.”
    die du verwendest, um die Zeit, die dein Lichtstrahl gebraucht hat, zu messen, also als Maßstab.

    “Was uns unterscheidet ist, dass Du weiterhin zwei materielle Konstanten selbst (Uhr und Geodreieck) als Maßstäbe ohne innere Verbindung verwendest bzw. zu verwenden behauptest (siehe oben die Umrechnung mit c)”
    Dann hast du mich total missverstanden. Eins reicht (sage ich ja auch die ganze Zeit), aber es ist vollkommen egal, ob das die Uhr oder das Geodreieck ist.

    “Für die Kinder genügt es, sich einen Lichtstrahl mit bestimmter Länge in einer Lichtuhr vorzustellen, mit dem sie durch Vergleich mit der Länge eines Signallichtstrahls zeitliche und räumliche Abstände zwischen Ereignissen messen können.”
    Und wie wir die Länge der Lichtuhr festlegen, fragen die Kleinen besser nicht.

    Wir sind uns doch sachlich einig, dass wir den Lichtstrahl und das brauchen, was du “materielle Komponente” nennst. Wir brauchen beides (jedenfalls, wenn wir mit Licht messen wollen – einfacher gehts, Strecken mit nem Geodreieck zu messen…). Und wenn man für irgendetwas zwei Dinge braucht, damit es klappt, ist es ziemlich verwegen zu sagen “naja, ich brauche zwei Dinge, aber eins davon ist eigentlich unwichtig (obwohl ich es brauche), deswegen darf ich sagen, dass ich nur ein Ding brauche.”

    ” Vielleicht müssen wir es dabei bewenden lassen.”
    Scheint so.

    @Anonym_2018
    “Seien Idee von einer elektromagnetischen Ursache der Gravitation sollte man aus meiner Sicht aufgreifen und prüfen, ob sich das in die ART integrieren lässt.”
    Hat man längst versucht, hat nicht geklappt, und ist inzwischen (wo wir wissen, dass die Theorie des Elektromagnetismus ein Teil der elektroschwachen Wechselwirkung ist) auch ziemlich obsolet.

  1038. #1041 PS
    13. September 2018

    @MartinB
    Wir sind uns einig, dass man für die Festlegung einer Einheit zur Messung von Zeit und Raum einen Lichtstrahl und eine materielle Konstante (entweder Caesiumatom oder Urmeter) benötigt.
    Wir sind uns einig, dass wir für das Messen sowohl von Zeit als auch für Raum die geeichte Länge eines Lichtstrahls verwenden können, auch wenn es im Alltag mit Geodreieck bzw. Uhr leichter geht.
    Unter Berücksichtigung der vielen Diskussionen, die Du hier und anderswo simultan führst, danke ich für Deine Geduld.

  1039. #1042 MartinB
    13. September 2018

    @PS
    “Wir sind uns einig, dass wir für das Messen sowohl von Zeit als auch für Raum die geeichte Länge eines Lichtstrahls verwenden könne”
    Wenn du mit der “geeichten Länge” den Abgleich mit der “materiellen Konstante” meinst, dann ja.

  1040. #1043 PS
    13. September 2018

    Ja, das meine ich.

  1041. #1044 hiroji kurihara
    1. November 2018

    Speed of Light : Reexamination

    Plane waves of light (wavelength is constant) are coming from just above. An observer is moving horizontally at different speed. Speed relative to the waves does not vary. But speed relative to photons or light ray will vary (both will be real existence). With the formula : light speed = f λ, speed of waves can be shown. However, speed of photon and light ray will not be shown. Because of large speed of light, this problem is not noticeable.

    In outer space, plane waves of a star light are coming. An observer is at a standstill. Speed of light waves and photons (light ray) relative to the observer will not be the same (in general). By the way, speed of light waves and of photons (light ray) relative to the aether frame will be the same (as a physical constant : not c, maybe).

    Sorry, I cannot receive E-mail. I do not have PC.

    https://www.geocities.co.jp/Technopolis/2561/eng.html

  1042. #1045 MartinB
    1. November 2018

    @hiroji
    Oh my, another deluded one.
    You are simply wrong, but I will not discuss this here, my blog is not for science-deniers.

  1043. #1046 rolak
    1. November 2018

    science-denier

    Not only science, MartinB, logic, too:

    I cannot receive E-mail. I do not have PC

    is a non sequitur. hiroji can post here→hiroji can receive email.

  1044. #1047 Alderamin
    1. November 2018

    @rolak

    Rentner im Internetcafé? Oder beim Enkel zu Besuch?

  1045. #1048 rolak
    1. November 2018

    ?Oder?

    Was auch immer, Alderamin, doch nichts davon hindert ihn daran, emails zu empfangen. Können von mir aus erst Weihnachten abgeholt werden, beim nächsten IC/E-Besuch.

  1046. #1049 Lulu
    1. November 2018

    Ein interessantes Experiment wäre, zum 1. April einen Artikel zu schreiben: “Warum die SRT DOCH falsch ist.”

    Ob die sich dann auch alle melden würden?

  1047. #1050 MartinB
    1. November 2018

    @Lulu
    Das hat ja damals mit den Schwarzen Löchern am CERN schon u Chaos geführt, und in Zeiten von fake news schreibe ich keine Aprilscherzartikel mehr.

  1048. #1051 Hiroji Kurihara
    30. April 2019

    Lorentz contraction

    Plain waves of light (wavelength is constant) are coming from the upper right 45 degrees. Two bars of the same length are moving to the right and the left at the same speed. The number of waves hitting the bars is the same. Lorentz contraction is unthinkable.

  1049. #1052 nnq
    19. Oktober 2021

    Ja, die SRT ist falsch!

  1050. #1053 MartinB
    19. Oktober 2021

    Klarer Fall für Hitchen’s Razor…

  1051. #1054 hitch
    lmaa
    25. November 2021

    Die RT ist komplett falsch.

  1052. #1055 Dackel
    NL
    25. November 2021

    Richtig

  1053. #1056 MartinB
    26. November 2021

    Begründungslose Behauptungen hinschreiben ist für sich schon eine eher peinliche Sache. Sich dann unter nem anderen Namen einloggen (aber mit derselben IP-Adresse), und sich selbst zu applaudieren, ist aber schon wirklich extra-peinlich, Herr Sockenpuppe.

  1054. #1057 Schüppler Hartmut
    Münster
    26. November 2021

    Die SRT ist nicht falsch, sie wird von den Experten nur schlecht erklärt.
    Ein Beispiel dafür:
    Haben Photonen Masse ?
    Die Erklärung lautet in aller Regel: Photonen haben keine Ruhemasse, als bewegte Objekte haben sie also sehr wohl Masse.
    Nun kann man sich allerdings Photonen nicht ruhend vorstellen, da es nun mal bewegte Objekte sind.
    Eine bessere Erklärung für die Masselosigkeit von Photonen im Rahmen der SRT wäre m.E.:
    Die SRT ist eine reduzierte, idealisierte Theorie. Sie lässt die Gravitation und in der Folge Masse und beschleunigte Bewegungen außen vor.
    In der wahrnehmbaren Realität (der ART) haben Photonen Masse, in der SRT nicht, weil dies so vorgestellt wird.

  1055. #1058 MartinB
    27. November 2021

    @Schüppler
    Das finde ich nur bedingt sinnvoll, aus drei Gründen:

    1. Masse taucht ja in der SRT als träge Masse durchaus auf (darüber kann man natürlich streiten, weil für Objekte mit Ruhemasse bei hohen Geschwindigkeiten die Trägheit von der Richtung der Beschleunigung abhängt, manche Leute sprechen ja auch von transversaler und logitudinaler Masse)

    2. In der SRT gilt E=mc², wenn Photonen Energie haben, müssen sie also auch Masse besitzen.

    3. Die SRT ist in der ART ja enthalten (im Grenzfall z.B. G gegen Null), insofern müssen die Theorien sozusagen fließend ineinander übergehen können, das ist bei einer so strikten Trennung nicht möglich.

    Ich könte auch noch 4. anführen, dass wir nicht wissen, dass Photonen masselos sind; es gibt eine experimentelle Obergrenze, die zwar unglaublich klein ist, aber nicht Null, im Detail hier erklärt:
    https://scienceblogs.de/hier-wohnen-drachen/2018/09/15/hat-die-lichtgeschwindigkeit-den-falschen-namen/

    Ich glaube, entscheidend ist in solchen Fällen nicht, die “Masselosigkeit” zu erklären, entscheidend ist erstens, den Begriff Masse zu hinterfragen:
    https://scienceblogs.de/hier-wohnen-drachen/2012/07/25/die-vielen-gesichter-der-masse/?all=1

    und zweitens, deutlich zu machen, dass die Begriffe der Physik selten ganz scharf definiert sind – Physik ist eben keine Mathematik. Entscheidend in der Physik sind die Zusammenhänge, die in Gleichungen ausgedrückt werden, und man muss dann eben verstehen, in welchen Fällen man bei “m” in einer Gleichung die Ruhemasse einsetzen muss und in welchen nicht.

    Das ist ja auch das große Problem aller populärwissenschaftlichen Darstellungen – man liest die Erklärungen und denkt, alle Definitionen etc. seien unanfechtbar, unendlich scharf und immer zu 100% eindeutig. Und dann versuchen Leute, diese Erklärungen in Worten weiterzudenken, kommen auf Widersprüche und glauben, die RT widerlegt zu haben (siehe die gut 1000 Kommentare hier…)

  1056. #1059 Anonym_2021
    28. November 2021

    @MartinB (27. November 2021) #1058

    “verstehen, in welchen Fällen man bei “m” in einer Gleichung die Ruhemasse einsetzen muss und in welchen nicht.”

    Man muss “m” in der SRT überhaupt nicht verwenden.

    Den Viererimpuls kann man schreiben als:
    P = E/c² * d/dt (ct, x, y, z)

  1057. #1060 MartinB
    29. November 2021

    @Anonym_2021
    Ich sprach doch von der Ruhemasse…?

  1058. #1061 Schüppler Hartmut
    Münster
    29. November 2021

    @MartinB
    zu Deinen Argumenten für die Beibehaltung der allgemein üblichen Sicht des Verhältnisses von SRT zur ART:
    zu1) die träge Masse taucht in der SRT auf, weil man nicht konsequent die Gravitation (entsprechend Masse von sämtlichen Objekten) unberücksichtigt lässt. Wie können Photonen sich mit Lichtgeschwindigkeit bewegen, wenn sie Masse haben ? Die übliche Argumentation, sie hätten keine Ruhemasse, ist unverständlich, da sie nicht ruhend vorgestellt werden können. Falls man sie doch als ruhend vorstellen kann, müsste die “Lichtgeschwindigkeit”, besser der Elektromagnetismus, auch als Bezugssystem vorgestellt werden können. Es entstehen daher Widersprüche, wenn man einerseits die Gravitation in der SRT unberücksichtigt lässt und andererseits annimmt, dass Objekte Masse haben.
    Der Lorentz-Faktor wird ja auch grundsätzlich ohne Berücksichtigung der Masse von Objekten dargestellt.
    zu 2) Meines Wissens hat Einstein bei der Herleitung des Zusammenhangs von E=mc² angenommen, dass Photonen Energie und folglich Masse haben. Nach meinem Vorschlag zur geänderten Sicht der Dinge wäre die Herleitung dieser Formel daher nicht im Rahmen der SRT erfolgt.
    zu 3)
    Die strikte Trennung von SRT und ART ist gerade der Sinn meines Vorschlags, um Widersprüche aufzulösen. Die SRT ist nur insofern ein Spezialfall der ART als in der SRT die Gravitation und damit zwangläufig die Massen sämtlicher Objekte unberücksichtigt bleiben.
    Ich erwarte nicht unbedingt eine Antwort auf meinen Vorschlag, da man ja durchaus unterschiedlicher Ansicht sein kann. Ich versuche lediglich Erklärungen für Widersprüchlichkeiten zu finden.

  1059. #1062 MartinB
    29. November 2021

    @Hartmut
    Was hat die *träge* Masse mit der Gravitation zu tun? Dass sie gleich der schweren Masse ist, ist ja vor der ART einfach nur eine beobachtete Tatsache ohne jeden physikalischen Grund.

    Die Logik von 2 verstehe ich nicht, die SRT ist nur in sich konsistent, wenn E=mc² gilt, und zwar für alles, was Energie trägt. Siehe die Artikelserie, die ich zu E=mc² mal geschrieben habe.

    zu 3) Siehe auch oben: Die SRT braucht natürlich ein Konzept der trägen Masse, sie ignoriert nur die schwere Masse, d.h. die Anziehung zwischen Massen.

    Wo du Widersprüchlichkeiten siehst, weiß ich nach wie vor nicht…

  1060. #1063 Anonym_2021
    29. November 2021

    @MartinB (29. November 2021)

    “Ich sprach doch von der Ruhemasse…?”

    Die Minkowski-Norm des Viererimpulses ||P|| muss auch nicht umbenannt werden in “m*c”. Eine “Ruhemasse” kann man auch ” E₀/c² ” nennen. Die “Masse” ist in der SRT redundant zu ||P||/c und damit überflüssig.

  1061. #1064 MartinB
    29. November 2021

    @Anonym_2021
    Halte ich schlicht für absolut nicht hilfreich.
    Masse ist ja ein Begriff mit vielen Facetten, und die SRT steht nicht allein da sondern im Zusammenhang mit anderen Theorien. Es hilft wirklich niemandem, wenn man den Begriff der Masse rausnimmt, nur um spätestens dann, wen man über konkrete Objekte wie Elektronen redet, immer wieder sagen muss “Und das ist jetzt eigentlich die Masse”.
    Physik ist keine axiomatische Disziplin, und Teile der Physik schon gar nicht, weil sie immer mit allen anderen verbunden sind. Klar kann das verwirrend sein, weswegen ich ja auch Artikel wie “viele Gesichter der Masse” schreibe, aber anders geht es kaum, weil Physikbegriffe immer auch unscharf sind.

  1062. #1065 Schüppler Hartmut
    Münster
    29. November 2021

    Ich beantworte mal einfach die von mir ausgangs gestellte Frage, vielleicht wird meine Ansicht ja dann deutlicher:
    Haben Photonen Masse ?

    In der SRT haben Photonen keine Masse, weil in dieser Theorie Gravitation unberücksichtigt bleibt. Photonen sollten konsequenterweise masselos vorgestellt werden. Photonen können sich deshalb mit Lichtgeschwindigkeit bewegen. Die SRT ist insoweit eine idealisierte Theorie.
    In der physikalischen Wirklichkeit, die durch die ART beschrieben wird, haben Photonen Masse und unterliegen deshalb der gravitativen Wirkung.

  1063. #1066 MartinB
    29. November 2021

    @Hartmut
    So ist es in meinen Augen schlicht falsch….

  1064. #1067 Anonym_2021
    29. November 2021

    @MartinB (29. November 2021)

    “nur um spätestens dann, wen man über konkrete Objekte wie Elektronen redet, immer wieder sagen muss “Und das ist jetzt eigentlich die Masse”.

    Auch das ist nicht notwendig. Elektronen speichern (aufgrund der Wechselwirkung mit dem Higgsfeld) eine Energie E₀, die sie bei der Paarerzeugung bekommen haben. Bei einer Zerstrahlung mit einem Positron geben sie diese Energie wieder ab an Photonen. Energie is träge, weil der Viererimpuls eines isolierten Systems eine Erhaltungsgröße ist.

    Ein Grund, warum an dem Begriff “Masse” festgehalten wird, ist meiner Ansicht nach, weil man im nicht-relativistischen Grenzfall die Begriffe rückwärts-kompatibel zur Newton’schen Theorie halten will.

  1065. #1068 MartinB
    30. November 2021

    @Anonym_2021
    Ja, sicher ist das ein Aspekt. Weil die Physik eben ein ganzes ist, nicht einzelne Theorien, die nix miteinander zu tun haben. Und natürlich ist in auch in der ART/SRT die Ruhemasse ein wichtiges Konzept, für das es sinnvoll ist, einen namen zu haben.